You are on page 1of 483

CIVIL LAW Answers to the BAR as Arranged by Topics (Year 1990-2006)

Is judge-made law. Civil Law judges are merely


supposed to apply laws and not interpret them.
GENERAL PRINCIPLES
Civil law vs. Common Law (1997)
How would you compare the Civil Law system in
Effect of Obiter & Dissenting Opinion; SC Decisions (1994)
its governance and trend with that of the
23 What are the binding effects of an
Common Law system?
obiter dictum and a dissenting opinion? 3)
SUGGESTED ANSWER: How can a decision of the Supreme Court be
As regards "governance": Governance in Civil set aside?
Law is codal, statutory and written law. It is ALTERNATIVE ANSWERS:
0 None. Obiter dictum and opinions are not necessary to the
additionally derived from case law. Common determination of a case. They are not binding and
law is basically derived from case law.

As regards "trend": Civil law is now tending


to rely more and more on decisions of the
courts explaining the laws. Common law is
now codifying laws more and more. So they
are now merging towards similar systems.

Additional Answers:
1. COMMON LAW refers to the traditional part
of the law as distinct from legislation; it refers
to the universal part of law as distinct from
particular local customs (Encyclopedia
Americana, Vol. 7). On the other hand, CIVIL
LAW is understood to be that branch of law
governing the relationship of persons in
respect of their personal and private interests
as distinguished from both public and
international laws.

In common law countries, the traditional


responsibility has for the most part been
with the judges; in civil law countries, the
task is primarily reposed on the
lawmakers. Contemporary practices,
however, so indicate a trend towards
centralizing that function to professional
groups that may indeed, see the gradual
assimilation in time of both systems.
[Vitug, Civil. Law and Jurisprudence, p.
XX)

23 In Civil Law, the statutes theoretically


take precedence over court decisions
interpreting them; while in Common Law, the
court decisions resolving specific cases are
regarded as law rather than the statutes
themselves which are, at the start, merely
embodiments of case law. Civil Law is code
law or written law, while Common Law is case
law. Civil Law adopts the deductive method -
from the general to the particular, while the
Common Law uses the inductive approach
from the particular to the general. Common
Law relies on equity. Civil Law anchors itself
on the letter of the law. The civilists are for
the judge-proof law even as the Common Law
cannot have the force of official precedents. It is as if the Court
were turning aside from the main topic of the case to collateral
subjects: a dissenting opinion affirms or overrules a claim, right
or obligation. It neither disposes nor awards anything it merely
expresses the view of the dissenter. (Civil Code, Paras]

A decision of a division of the Supreme Court


maybe set aside by the Supreme Court sitting
en banc, a Supreme Court decision may be set
aside by a contrary ruling of the Supreme
Court itself or by a corrective legislative act of
Congress, although said laws cannot
adversely affect those favored prior to the
Supreme Court decision. [Civil Code, Paras).

Effectivity of Laws (1990)


After a devastating storm causing widespread
destruction in four Central Luzon provinces,
the executive and legislative branches of the
government agreed to enact a special law
appropriating P1 billion for purposes of relief
and rehabilitation for the provinces. In view of
the urgent nature of the legislative enactment,
it is provided in its effectivity clause that it
shall take effect upon approval and after
completion of publication in the Official
Gazette and a newspaper of general circulation
in the Philippines. The law was passed by the
Congress on July 1, 1990. signed into law by
the President on July 3, 1990, and published in
such newspaper of general circulation on July
7, 1990 and in the Official Gazette on July 10,
1990.
0 As to the publication of said legislative
enactment, is there sufficient observance or
compliance with the requirements for a valid
publication? Explain your answer.
1 When did the law take effect? Explain
your
answer(c). Can the executive branch
start releasing and disbursing funds
appropriated by the said law the day
following its approval? Explain your
answer.
SUGGESTED ANSWER:
0 Yes, there is sufficient compliance. The law
itself prescribes the requisites of publication for
its effectivity, and all requisites have been
complied with. (Article 2, Civil Code)

1 The law takes effect upon compliance with


all the conditions for effectivity, and the last
condition was complied with on July 10, 1990.
Hence, the" law became effective on that date.
2 No. It was not yet effective when it was
approved by Congress on July 1, 1990 and
approved by the President on July 3, 1990.
The other requisites for its effectivity were
not yet complete at the time.
Equity follows the Law (2003)
It is said that ―equity follows the law‖
What do you
understand by this phrase, and what are its
basic implications?
5%
SUGGESTED ANSWER:
―Equity Follows the law‖ means that courts
exercising equity
jurisdiction are bound by rules of law and have
no arbitrary
discretion to disregard them. (Arsenal v IAC,
143 SCRA 40
[1986]). Equity is applied only in the
absence of
Page 10 of 119
CIVIL LAW Answers to the BAR as Arranged by Topics (Year 1990-2006)
but never against statutory law. (Toyota Motor 1. The civil action involves an issue similar or
Phil. V CA intimately
related to the issue raised in the criminal
216 SCRA 236 [1992]). action, and
2. the resolution of such issue determines
whether or not the
Ignorance of the Law vs. Mistake of Fact (1996) criminal action may proceed.
Is there any difference in their legal effect
between ignorance
(c) Consequences The criminal case must be
of the law and ignorance or mistake of fact? suspended.
Thus, in a criminal case for damages to one's
SUGGESTED ANSWER: property, a civil
Yes, there is a difference. While ignorance of action that involves the ownership of said
the law is not an property should
excuse for not complying with it, ignorance of first be resolved (De Leon vs. Mabanag. 38
fact eliminates Phil. 202)
criminal intent as long as there is no negligence
(Art, NCC).
In addition, mistake on a doubtful or difficult
question of law
may be the basis of good faith (Art. 526. NCC).
Mistake of
fact may, furthermore, vitiate consent in a
contract and make PERSONS
it voidable (Art. 1390. NCC).
ALTERNATIVE ANSWER: 0 concept, (b) requisites and (c)
Yes. ignorance of the law differs in legal consequences of a prejudicial question.
effect from Ignorance or mistake of fact. The SUGGESTED ANSWER:
former does not excuse a party from the 0 Concept A prejudicial question is one which
legal consequences of his conduct while the must be decided first before a criminal action
latter does constitute an excuse and is a may be instituted or may proceed because a
legal defense. decision therein is vital to the judgment in the
criminal case. In the case of People vs. Adelo
Inferior Courts Decisions (1994) Aragon (L5930, Feb. 17, 1954), the Supreme
Are decisions of the Court of Appeals Court defined it as one which arises in a case,
considered laws? the resolution of which question is a logical
ALTERNATIVE ANSWERS: antecedent of the issues involved in said case
0 a) No, but decisions of the Court of and the cognizance of which pertains to
Appeals may serve as precedents for inferior another tribunal (Paras, Vol. 1, Civil. Code
courts on points of law not covered by any Annotation, 1989 ed. p, 194).
Supreme Court decision, and a ruling of the
Court of Appeals may become a doctrine.
(Miranda vs..
Imperial 77 Phil. 1066).

0.0 No. Decisions of the Court of Appeals


merely have persuasive, and therefore no
mandatory effect. However, a conclusion or
pronouncement which covers a point of law
still undecided may still serve as judicial guide
and it is possible that the same maybe raised
to the status of doctrine. If after it has been
subjected to test in the crucible of analysis,
the Supreme Court should find that it has
merits and qualities sufficient for its
consideration as a rule of jurisprudence (Civil
Code, Paras).

Prejudicial Questions (1997)


In the context that the term is used in Civil
Law, state the
Change of Name; Under RA 9048 (2006) petition with the Office of the Local Civil Registrar
Zirxthoussous delos Santos filed a petition for to change his first name to "Roberto." He claimed
change of name with the Office of the Civil that the change is warranted because it will
Registrar of Mandaluyong City under the eradicate all vestiges of the infamy of Mary Grace's
administrative proceeding provided in father.
Republic Act No. 9048. He alleged that his
Will the petition for change of name of Jesus
first name sounds ridiculous and is extremely
delos Santos to Roberto delos Santos under
difficult to spell and pronounce. After Republic Act No. 9048 prosper? Explain. (10%)
complying with the requirements of the law, SUGGESTED ANSWER: No, under the law, Jesus
the Civil Registrar granted his petition and may only change his name once. In addition,
changed his first name Zirxthoussous to the petition for change of name may be denied
"Jesus." His full name now reads "Jesus delos on the following grounds:
Santos." 0 Jesus is neither ridiculous, nor tainted
with dishonor nor extremely difficult to write
Jesus delos Santos moved to General Santos City to or pronounce.
work in a multi-national company. There, he fell in 1 There is no confusion to be avoided or
love and married Mary Grace delos Santos. She created with the use of the registered first
requested him to have his first name changed name or nickname of the petitioner.
because his new name "Jesus delos Santos" is the 2 The petition involves the same entry in
same name as that of her father who abandoned the same document, which was previously
her family and became a notorious drug lord. She corrected or changed under this Order [Rules
wanted to forget him. Hence, Jesus filed another and Regulations Implementing RA 9048].
What entries in the Civil Registry may be
changed or corrected
without a judicial order? (2.5%)
Only clerical or
SUGGESTED ANSWER: typographical errors
and first or nick names may be changed or
(b) corrected without
Requisites1The prejudicial question must be determinative of the case
before the court. a judicial order under RA 9048.
2 Jurisdiction to try said question must be lodged in another
Clerical or typographical errors refer to mistakes committed

tribunal. in the performance of clerical work in


writing, copying,
ADDITIONAL ANSWER: transcribing or typing an entry in the civil
register. The
mistake is harmless and innocuous, such as
errors in
CIVIL LAW Answers to the BAR as Arranged by Topics (Year 1990-2006)
spelling, visible to the eyes or obvious to the absolute community amounting to 1 Million
understanding, Pesos. His
and can be corrected or changed only by
reference to other wife, will, therefore, inherit O.25 Million Pesos and his parents will
existing records. Provided, however, that no
correction must inherit 0.25 Million Pesos. When Mrs. Cruz died, she was
involve the change of nationality, age, status or succeeded by her parents as her intestate heirs. They will inherit all
sex of the of her estate consisting of her 0.5 Million half share in the absolute
petitioner. community and her 0.25 Million inheritance from her husband, or a
total of 0.750 Million Pesos.

Death; Effects; Simultaneous Death (1998)


Jaime, who is 65, and his son, Willy, who is 25,
died in a
plane crash. There is no proof as to who died In sum, the parents of Mr. Cruz will inherit
first. Jaime's 250,000 Pesos
only surviving heir is his wife, Julia, who is while the parents of Mrs. Cruz will inherit
also Willy's 750,000 Pesos.
mother. Willy's surviving heirs are his mother,
Julia and his (b) This being a case of succession, in the
wife, Wilma. absence of proof
1. In the settlement of Jaime's estate, can Wilma as to the time of death of each of the spouses,
successfully it is presumed
claim that her late husband, Willy had a they died at the same time and no transmission
hereditary share since of rights
he was much younger than his father and, from one to the other is deemed to have
therefore, should taken place.
Therefore, each of them is deemed to have an
be presumed to have survived longer? [3%] estate valued
at P500,000,00, or one-half of their conjugal
property of P1
2. Suppose Jaime had a life insurance policy million. Their respective parents will thus
with his wife, inherit the entire
Julia, and his son, Willy, as the beneficiaries. P1 Million in equal shares, of P500,000.00 per
Can Wilma set of parents.
successfully claim that one-half of the
proceeds should
belong to Willy's estate?
|2%J
SUGGESTED ANSWER: Death; Effects; Simultaneous Death (2000)
1. No, Wilma cannot successfully claim that b) Cristy and her late husband Luis had two
Willy had a children, Rose
hereditary share in his father's estate. Under and Patrick, One summer, her mother-in-law,
Art. 43, Civil aged 70, took
Code, two persons "who are called to succeed the two children, then aged 10 and 12, with
each other" her on a boat trip
are presumed to have died at the same time, in to Cebu. Unfortunately, the vessel sank en
the absence of route, and the
proof as to which of them died first. This bodies of the three were never found. None of
presumption of the survivors
simultaneous death applies in cases involving ever saw them on the water. On the
the question of settlement of her
succession as between the two who died, who in mother-in-law's estate, Cristy files a claim for a
this case are share of her
estate on the ground that the same was
mutual heirs, being father and son. inherited by her
children from their grandmother in
SUGGESTED ANSWER: representation of their
2. Yet, Wilma can invoke the presumption of
survivorship father, and she inherited the same from them.
and claim that one-half of the proceeds should Will her action
belong to
prosper? (2%)
Willy's estate, under par. 5 Rule 131,
Sec. 3 (jj) Rules of
SUGGESTED ANSWER:
Court, as the dispute does not involve No, her action will not prosper. Since there
succession. Under this was no proof as
presumption, the person between the ages of 15 to who died first, all the three are deemed to
and 60 years have died at the
is deemed to have survived one whose age was same time and there was no transmission of
over 60 at the rights from one
time of their deaths. The estate of Willy to another, applying Article 43 of the New Civil
endowed with Code.
juridical personality stands in place and stead of
Willy, as ALTERNATIVE ANSWER:
No, her action will not prosper. Under Article
beneficiary. 43 of the New
Civil Code, inasmuch as there is no proof as to
who died first,
all the three are presumed to have died at the
Death; Effects; Simultaneous Death (1999) same time and
Mr. and Mrs. Cruz, who are childless, met with a there could be no transmission of rights among
serious them. Her
motor vehicle accident with Mr. Cruz at the children not having inherited from their
wheel and Mrs. grandmother. Cristy
Cruz seated beside him, resulting in the instant has no right to share in her mother-inlaw's
death of Mr. estate. She cannot
Cruz. Mrs. Cruz was still alive when help came share in her own right as she is not a
but she also legal heir of her
died on the way to the hospital. The couple mother-in-law. The survivorship provision of
acquired Rule 131 of the
properties worth One Million (P1 ,000,000.00) Rules of Court does not apply to the problem.
Pesos during It applies only
their marriage, which are being claimed by the to those cases where the issue involved is not
parents of succession.
both spouses in equal shares. Is the claim of
both sets of
parents valid and why?
(3%)
(b) Suppose in the preceding question, both Mr.
and Mrs. Juridical Capacity vs. Capacity to Act (1996)
Cruz were already dead when help came, so Distinguish juridical capacity from capacity to
that no-body act,
could say who died ahead of the other, would
your answer be SUGGESTED ANSWER:
the same to the question as to who are JURIDICAL CAPACITY is the fitness to be the
entitled to the subject of
legal relations while CAPACITY TO ACT is the
properties of the deceased couple? (2%) power or to
do acts with legal effect. The former is
SUGGESTED ANSWER: inherent in every
natural person and is lost only through death
(a) No, the claim of both parents is not valid. When Mr. Cruz died, while the latter
is merely acquired and may be lost even before
he was succeeded by his wife and his parents as his intestate heirs death (Art.
who will share his estate equally. His estate was 0.5 Million
pesos which is his half share in the 37, NCC).
ALTERNATIVE ANSWER;
CIVIL LAW Answers to the BAR as Arranged by Topics (Year 1990-2006)
Juridical capacity, as distinguished from conditions detrimental to the moral well-being
capacity to act: (a) of their
the former is passive while the latter is active,
(b) the former children acting in the movies is in violation of the Family Code and
is inherent in a person while the latter is merely
acquired, (c) Labor laws. Thus, the waiver is invalid and not binding.
the former is lost only through death while the
latter may be
lost through death or restricted by causes other
than death,
and Id) the former can exist without capacity to The Child Labor Law is a mandatory and
act while the prohibitory law and
latter cannot exist without the rights of the child cannot be waived as it
juridical capacity. is contrary to
law and public policy.
Juridical Capacity; Natural Persons (1999)
Elated that her sister who had been married for
five years
was pregnant for the first time, Alma donated
P100,000.00 to CONFLICT OF LAWS
the unborn child. Unfortunately, the baby
died one hour
after delivery. May Alma recover the
P100.000.00 that she
had donated to said baby before it was born
considering that Appilicable Laws; laws governing contracts (1992)
the baby died? Stated otherwise, is the donation X and Y entered into a contract in Australia,
valid and whereby it was
binding? Explain. agreed that X would build a commercial
(5%) building for Y in the
Philippines, and in payment for the
SUGGESTED ANSWER: construction, Y will
The donation is valid and binding, being an act transfer and convey his cattle ranch located in
favorable to the United
the unborn child, but only if the baby had an States in favor of X. What law would govern:
intra-uterine life a) The
of not less than seven months and pro-vided validity of the contract? b) The performance of
there was due the contract?
acceptance of the donation by the proper person
representing c) The consideration of the contract?
said child. If the child had less than
seven months of
intra-uterine life, it is not deemed born since it
died less than
24 hours following its delivery, in which ease the
donation SUGGESTED ANSWER:
never became effective since the donee (a) The validity of the contract will be
never became a governed by Australian
person, birth being determinative of law, because the validity refers to the element
personality. of the making
ALTERNATIVE ANSWER: of the contract in this case.
Even if the baby had an intra-uterine life of (Optional Addendum:"... unless the parties
more than seven agreed to be
months and the donation was properly
accepted, it would be bound by another law".}
void for not having conformed with the proper
form. In
order to be valid, the donation and acceptance
of personal
property exceeding five thousand pesos should (b) The performance will be governed by the
be in writing. law of the
Philippines where the contract is to be
(Article 748, par. 3) performed.
Waiver of Rights (2004) (c) The consideration will be governed by
the law of
theUnited States where the ranch is located.
(Optional Addendum:
B. DON, an American businessman, secured In the foregoing cases, when the foreign law would
parental consent apply, the
for the employment of five minors to play absence of proof of that foreign law would render
certain roles in two Philippine law
movies he was producing at home in Makati.
They worked at applicable under the "eclectic theory".)
odd hours of the day and night, but always
accompanied by
parents or other Th producer paid childre
adults. e the n
Applicable Laws; Arts 15, 16 & 17 (1998)
talent fees at rates better than
Juan is a Filipino citizen residing in Tokyo,
adult wages.
Japan. State what
But a social worker, DEB, reported to OSWD laws govern:
that these His capacity to contract marriage in
children often missed going to school. They 1 Japan, [ 1%]
sometimes drank His successional rights as regards his
wine, aside from being exposed to In some 2 deceased
drugs. scenes,
they were filmed o revealing Filipino father's property in Texas, U.S.A. [1%]
naked r in costumes. In his The extrinsic validity of the last will
defense, contend these part artisti 3 and testament
DON ed all were of c which Juan executed while sojourning in
freedom cultural Non of parent Switzerland. [2%]
and creativity. e the s
complained, said DON. He also said they signed 4 The intrinsic validity of said will. (1%)
a contract
containing a waiver of their right to file any
complaint in any
office or tribunal concerning the working
conditions of their SUGGESTED ANSWER:
children acting in the Juan's capacity to contract marriage
movies. 1. is
governed by Philippine law -i.e., the Family
Code -pursuant
Is the waiver valid and to Art. 15, Civil Code, which provides that our
binding? Why or why not? laws relating
Explain. to, among others, legal capacity of persons
(5%) are binding upon
citizens of the Philippines even though living
SUGGESTED ANSWER: abroad.
The waiver is not valid. Although the contracting parties may
establish such stipulations, clauses, terms and conditions as they
may deem convenient, they may not do so if such are contrary to SUGGESTED ANSWER:
2. By way of exception to the general rule of
law, morals, good customs, public order, or public policy (Article lex rei sitae
prescribed by the first paragraph of Art. 16.
1306, Civil Code). The parents' waiver to file a complaint concerning Civil Code, a
person's successional rights are governed by
the working the national law
of the decedent (2nd par.. Art. 16). Since
Juan's deceased
Page 13 of 119
CIVIL LAW Answers to the BAR as Arranged by Topics (Year 1990-2006)
father was a Filipino citizen, Philippine law governs Juan's
(2). With respect to Felipe the divorce is
successional rights. valid, but with
respect to Felisa it is not. The divorce will not
capacitate
Felisa to remarry because she and Felipe were
ANOTHER ANSWER: both Filipinos
2. Juan's successional rights are governed by at the time of their marriage. However, in DOJ
Philippine law, Opinion No.
pursuant to Article 1039 and the second 134 series of 1993, Felisa is allowed to remarry
paragraph of Article because the
16, both of the Civil Code. Article 1039, Civil injustice sought to be corrected by Article 26
Code, provides also obtains in
that capacity to succeed shall be governed by her
the "law of the case.
nation" of the decedent, i.e.. his national
law. Article 16
provides in paragraph two that the amount of
successional SUGGESTED ANSWER:
rights, order of succession, and intrinsic B. The foreigner who executes his will in the
validity of Philippines may
testamentary succession shall be governed by
the "national observed the formalities described in:
law" of the decedent who is identified as a 1. The Law of the country of which he is a
Filipino in the citizen under
present problem. Article 817 of the New Civil Code, or
2. the law of the Philippines being the law of
the place of
execution under Article 17 of the New Civil
SUGGESTED ANSWER: Code.
3. The extrinsic validity of Juan's will is
governed by (a)
Swiss law, it being the law where the will was
made (Art. 17. SUGGESTED ANSWER:
1st par. Civil Code), or (b) Philippine law, by C. Philippine law will not govern the intrinsic
implication validity of the
from the provisions of Art. 816, Civil Code, will. Article 16 of the New Civil Code provides
which allows that intrinsic
even an alien who is abroad to make a will in validity of testamentary provisions shall be
conformity governed by the
National Law of the person whose
with our Civil Code. succession is under
consideration. California law will govern the
intrinsic validity
SUGGESTED ANSWER: of the will.
4. The intrinsic validity of his will is governed by
Philippine
law, it being his national law. (Art. 16,
Civil Code) Applicable Laws; Capacity to Act (1998)
Francis Albert, a citizen and resident of New
Jersey, U.S.A.,
under whose law he was still a minor, being
only 20 years of
age, was hired by ABC Corporation of Manila to
Applicable Laws; Arts 15, 16, 17 (2002) serve for two
Felipe and Felisa, both Filipino citizens, were years as its chief computer programmer. But
married in after serving for
Malolos, Bulacan on June 1, 1950. In 1960 only four months, he resigned to join XYZ
Felipe went to the Corporation,
United States, becoming a U.S. citizen in 1975. which enticed him by offering more
In 1980 they advantageous terms. His
obtained a divorce from Felisa, who was duly first employer sues him in Manila for damages
notified of the arising from the
proceedings. The divorce decree became breach of his contract of employment. He sets
final under up his minority
California Law. Coming back to the Philippines as a defense and asks for annulment of the
in 1982, contract on that
Felipe married Sagundina, a Filipino Citizen. In
2001, Filipe, ground. The plaintiff disputes this by alleging
then domiciled in Los Angeles, California, died, that since the
leaving one contract was executed in the Philippines under
child by Felisa, and another one by Sagundina. whose law the
He left a will age of majority is 18 years, he was no longer a
which he left his estate to Sagundina and his minor at the
two children and
nothing to Felisa. Sagundina files a petition for time of perfection of the contract.
the probate of
Felipe’s will. Felisa questions the intrinsic
validity of the will, 1 Will the suit prosper? [3%]
arguing that her marriage to Felipe subsisted Suppose XYZ Corporation is impleaded
despite the 2 as a co-
divorce obtained by Felipe because said defendant, what would be the basis of its
divorce is not liability, if any?
recognized in the Philippines. For this reason,
she claims that [2%]
the properties and that Sagundina has no
successional rights. SUGGESTED ANSWER:
1. The suit will not prosper under Article 15,
Civil Code,
the divorce secured by Felipe in New Jersey law governs Francis Albert's
A. Is California capacity to act, being
recognizable and valid in the Philippines? How his personal law from the standpoint of both his
does it affect nationality
Felipe’s marriage to Felisa? Explain. and his domicile. He was, therefore, a minor at
(2%). the time he
B. What law governs the formalities of the will?
Explain. entered into the contract.
(1%) ALTERNATIVE ANSWER:
C. Will Philippine law govern the intrinsic 1. The suit will not prosper. Being a U.S.
validity of national, Albert's
capacity to enter into a contract is determined
the will? Explain. (2%) by the law of
the State of which he is a national, under which
SUGGESTED ANSWER: he to still a
A. (1.) The divorce secured by Felipe in minor. This is in connection with Article 15 of
California is the Civil Code
recognizable and valid in the Philippines which embodies the said nationality principle of
because he was no lex patriae.
longer a Filipino at that time he secured it, While this principle intended to apply to
Aliens may obtain Filipino citizens
divorces abroad which may be recognized in the under that provision, the Supreme Court in
Philippines Recto v. Harden
provided that they are valid according to their is of the view that the status or capacity of
national law foreigners is to be
(Van Dorn V. Romillo, Jr., 139 SCRA 139 [1985]; determined on the basis of the same provision
Quita v. or principle,
Court of Appeals, 300 SCRA 406 [1998];
Llorente v. Court of i.e., by U.S. law in the present problem.
Appeals, 345 SCRA 595 [2000] ).
CIVIL LAW Answers to the BAR as Arranged by Topics (Year 1990-2006)
persons is governed by the law of his
nationality, capacity
Plaintiffs argument does not hold true, concerning transactions involving property is
because status or an exception.
capacity is not determined by lex loci contractus Under Article 16 of the NCC the capacity of
but by lex persons in
transactions involving title to property is
patriae. governed by the law
of the country where the property is situated.
ANOTHER ANSWER: Since the
Article 17 of the Civil Code provides that property is in the Philippines, Philippine law
1. the forms and governs the
solemnities of contracts, wills and other public
instruments capacity of the seller.
shall be governed by the laws of the country in
which they are
executed. Applicable Laws; capacity to succeed (1991)
Jacob, a Swiss national, married Lourdes, a
Filipina, in Berne,
Since the contract of employment was executed Switzerland. Three years later, the couple
in Manila, decided to reside in
Philippine law should govern. Being over 18 the Philippines. Jacob subsequently acquired
years old and no several
longer a minor according to Philippine Law, properties in the Philippines with the money
Francis Albert he inherited
can be sued. Thus, the suit of ABC Corporation from his parents. Forty years later. Jacob died
against him intestate, and is
survived by several legitimate children and
for damages will prosper. duly recognized
illegitimate daughter Jane, all residing in the
SUGGESTED ANSWER: Philippines.
2. XYZ Corporation, having enticed Francis Albert
to break
his contract with the plaintiff, may be held liable
for damages
under Art. 1314, Civil Code. (a) Suppose that Swiss law does not allow
illegitimate children
ALTERNATIVE ANSWER:
to inherit, can Jane, who is a recognized
2. The basis of liability of XYZ Corporation
would be illegitimate child,
Article 28 of the Civil Code which states that: inherit part of the properties of Jacob under
"Unfair Philippine law?
competition in agricultural, commercial, or (b) Assuming that Jacob executed a will
industrial leaving certain
enterprises or in labor through the use of force, properties to Jane as her legitime in
accordance with the law
intimidation,
of succession in the Philippines, will such
deceit, machination or any other unjust,
testamentary
oppressive or
highhanded method shall give rise to a right of disposition be valid?
action by the
SUGGESTED ANSWER:
person who thereby suffers damage." A. Yes. As stated in the problem. Swiss law
does not allow
illegitimate children to inherit Hence, Jane
cannot inherit the
ANOTHER ANSWER: property of Jacob under Philippine law.
2. No liability arises. The statement of the
problem does not
in any way suggest intent, malice, or even
knowledge, on the
SUGGESTED ANSWER:
part of XYZ Corporation as to the B. The testamentary disposition will not be
contractual relations valid if it would
between Albert and ABC Corporation. contravene Swill law; otherwise, the
disposition would be
valid. Unless the Swiss law is proved, it would
be presumed
to be the same as that of Philippine law under
Applicable Laws; Capacity to Buy Land (1995) the Doctrine of
3. What law governs the capacity of the Filipino
to buy the Processual Presumption.
land? Explain your answer and give its legal
basis.
SUGGESTED ANSWER: Applicable Laws; contracts contrary to public policy (1996)
Philippine law governs the capacity of the Alma was hired as a domestic helper in
Filipino to buy the land. In addition to the Hongkong by the Dragon Services, Ltd.,
principle of lex rei sitae given above. Article through its local agent. She executed a
15 of the NCC specifically provides that standard employment contract designed by
Philippine laws relating to legal capacity of the Philippine Overseas Workers
persons are binding upon citizens of the Administration (POEA) for overseas Filipino
Philippines no matter where they are. workers. It provided for her employment for
one year at a salary of US$1,000.00 a month.
It was submitted to and approved by the
Applicable Laws; Capacity to Contract (1995)
POEA. However, when she arrived in
0 What law governs the capacity of the Hongkong, she was asked to sign another
Japanese to sell the land? Explain your answer contract by Dragon Services, Ltd. which
and give its legal basis. reduced her salary to only US$600.00 a
SUGGESTED ANSWER:
month. Having no other choice, Alma signed
Japanese law governs the capacity of the
the contract but when she returned to the
Japanese to sell the land being his personal
Philippines, she demanded payment of the
law on the basis of an interpretation of Art.
salary differential of US$400.00 a month. Both
15, NCC.
ALTERNATIVE ANSWERS; Dragon Services, Ltd. and its local agent
0 Since capacity to contract is governed by claimed that the second contract is valid under
the personal law of an individual, the Japanese the laws of Hongkong, and therefore binding
seller's capacity should be governed either by on Alma. Is their claim correct? Explain.
his national law (Japanese law) or by the law SUGGESTED ANSWER:
of his domicile, depending upon whether
Their claim is not correct. A contract is the
Japan follows the nationality or domiciliary law between the parties but the law can
theory of personal law for its citizens. disregard the contract if it is contrary to
1 Philippine law governs the capacity of the public policy. The provisions of the 1987
Constitution on the protection of labor and on
Japanese owner in selling the land. While as a
social justice (Sec. 10. Art II) embody a
general rule capacity of public policy of the Philippines. Since the
application of Hongkong law in this case is in
violation of
CIVIL LAW Answers to the BAR as Arranged by Topics (Year 1990-2006)
that public policy, the application shall be Court of Appeals (G.R No. 104235, Nov. 10,
disregarded by 1993) the
Supreme Court applied Philippine law in
our Courts. (Cadalin v. POEA. 238 SCRA 762) recovery of damages
for breach of contract of carriage for the
ALTERNATIVE ANSWERS; reason that it is the
a) Their claim is not correct. Assuming that the law of the place where the contract was
second executed.
contract is binding under Hongkong law, such
second
contract is invalid under Philippine law which
recognizes as ALTERNATIVE ANSWER:
valid only the first contract. Since the case is If the violation of the contract was attended
being litigated in with bad faith,
the Philippines, the Philippine Court as the there is a ground to recover moral damages.
forum will not But since there
enforce any foreign claim obnoxious to the was a federal regulation which was the
forum's public basis of the act
policy. There is a strong public policy enshrined complained of, the airline cannot be in bad
in our faith. Hence, only
Constitution on the protection of labor. actual damages can be recovered. The
Therefore, the same is true with
second contract shall be disregarded and the
first contract will regards to exemplary damages.
be enforced. (Cadalin v. POEA, 238 SCRA 762).
Applicable Laws; Labor Contracts (1991)
b) No, their claim is not correct. The second A. The Japan Air Lines (JAL), a foreigner
contract corporation
executed in Hongkong, partakes of the nature of licensed to do business in the Philippines,
a waiver that executed in Manila
is contrary to Philippine law and the public a contract of employment with Maritess Guapa
policy governing under which
Filipino overseas workers. Art. 17, provides that the latter was hired as a stewardess on the
our aircraft flying the
prohibitive laws concerning persons, their acts, Manila-Japan-Manila route. The contrast
or their specifically provides
property or which have for their object public that (1) the duration of the contract shall be
order, public two (2) years, (2)
policy and good customs shall not be rendered notwithstanding the above duration, JAL may
ineffective by terminate the
laws or conventions agreed upon in a foreign agreement at any time by giving her notice in
country. Besides, writing ten (10)
Alma's consent to the second contract was days in advance, and (3) the contract shall be
vitiated by undue construed as
influence, being virtually helpless and under governed under and by the laws of Japan and
financial distress only the court
in a foreign country, as indicated by the given in Tokyo, Japan shall have the jurisdiction to
fact that she consider any
signed because she had no choice. Therefore,
the defendants matter arising from or relating to the contract.
claim that the contract is valid under Hongkong
law should be
rejected since under the DOCTRINE OF
PROCESSUAL
PRESUMPTION a foreign law is deemed similar JAL dismissed Maritess on the fourth month of
or identical her
to Philippine law in the absence of proof to the employment without giving her due notice.
contrary, and Maritess then filed
such is not mentioned in the problem as having a complaint with the Labor Arbiter for
been adduced. reinstatement,
backwages and damages. The lawyer of JAL
contends that
neither the Labor Arbiter nor any other agency
or court in the
Philippines has jurisdiction over the case in
Applicable Laws; Contracts of Carriage (1995) view of the above
On 8 December 1991 Vanessa purchased from provision (3) of the contract which Maritess
the Manila voluntarily
office of Euro-Aire an airline ticket for its Flight signed. The contract is the law between her
No. 710 and JAL. Decide
from Dallas to Chicago on 16 January 1992. Her
flight the issue.
reservation was confirmed. On her scheduled
departure
Vanessa checked in on time at the Dallas
airport. However, at
the check-in counter she discovered that she B. Where under a State's own conflicts rule
was waitlisted that domestic law
with some other passengers because of of another State should apply, may the courts
intentional of the former
overbooking, a Euro-Aire policy and practice. nevertheless refuse to apply the latter? If so,
Euro-Alre under what
admitted that Vanessa was not advised of such
policy when circumstance?
she purchased her plane ticket. Vanessa was
only able to fly SUGGESTED ANSWER:
two days later by taking another airline. A, Labor Legislations are generally intended as
expressions of
Vanessa sued Euro-Aire in Manila for breach of public policy on employer-employee relations.
contract and The contract
damages. Euro-Aire claimed that it cannot be therefore, between Japan Air Lines (JAL) and
held liable for Maritess may
damages because its practice of overbooking apply only to the extent that its provisions are
passengers was not inconsistent
allowed by the U.S. Code of Federal Regulations. with Philippine labor laws intended
Vanessa on particularly to protect
the other hand contended that assuming that
the U.S. Code employees.
of Federal Regulations allowed Intentional
overbooking, the
airline company cannot invoke the U.S. Code on Under the circumstances, the dismissal of
the ground Maritess without
that the ticket was purchased in Manila, hence, complying with Philippine Labor law would be
Philippine law invalid and
should apply, under which Vanessa can recover any stipulation in the contract to the contrary
damages for is considered
breach of contract of carriage. Decide. Discuss void. Since the law of the forum in this case is
fully. the Philippine
law the issues should-be resolved in
accordance with
SUGGESTED ANSWER: Philippine law.
Vanessa can recover damages under Philippine
law for breach
of contract of carriage, Philippine law should B. The third paragraph of Art. 17 of the Civil
govern as the Code provides
law of the place where the plane tickets were
bought and the that:
contract of carriage was executed. In Zalamea v.
CIVIL LAW Answers to the BAR as Arranged by Topics (Year 1990-2006)
Maris then returned to the Philippines and in a
civil ceremony celebrated in Cebu City
according to the formalities of Philippine law,
she married her former classmate Vincent
likewise a Filipino citizen. a) Was the marriage
of Maris and Johnson valid when celebrated? Is

SUGGESTED ANSWER:
(a) The marriage of Mans and Johnson was
valid when celebrated because all
marriages solemnized outside the
Philippines (Tokyo) in accordance with the
laws in force in the country where they are
"Prohibitive laws concerning persons, their
acts or property, and those which have for
their object public order, public policy and
good customs shall not be rendered
ineffective by laws or judgments
promulgated, or by determinations or
conventions agreed upon in a foreign
country."

Accordingly, a state's own conflict of laws rule


may, exceptionally be inapplicable, given public
policy considerations by the law of the forum.

Going into the specific provisions of the contract in question,


I would rule as follows:
0 The duration of the contract is not opposed to Philippine
law and it can therefore be valid as stipulated; Their marriage no longer validly subsists,
because it has been
0 The second provision to the effect that notwithstanding dissolved by the absolute
divorce validly obtained by Johnson duration, Japan Air Lines (JAL) may terminate her
employment is which capacitated Maris to remarry (Art. 26. Family Code). invalid, being
inconsistent with our Labor laws;
0 That the contract shall be construed as governed under and
by the laws of Japan and only the courts of Tokyo, Japan shall haveApplicable Laws; laws governing
marriages (2003) jurisdiction, is invalid as clearly opposed to the aforecited third Gene and Jane,
Filipino, met and got married in England paragraph of Arts. 17 and 1700 of the Civil Code, which
provides:while both were taking up post-graduate courses there. A few
that the contract is governed by the laws of the
foreign country. Given this generally accepted
principle of international law, the contract
"Art. 1700. The relations between between Maritess and JAL is valid and it should
capital and labor are not merely therefore be enforced.
contractual. They are so impressed
with public interest that labor Applicable Laws; laws governing marriages (1992)
contracts must yield to the common In 1989, Maris, a Filipino citizen, married her
good. Therefore, such contracts are boss Johnson, an American citizen, in Tokyo in
subject to the special laws on labor a wedding ceremony celebrated according to
unions, collective bargaining, strikes Japanese laws. One year later, Johnson
and lockouts, closed shop, wages, returned to his native Nevada, and he validly
working conditions, hours of labor and obtained in that state an absolute divorce from
similar subjects." his wife Maris.
ALTERNATIVE ANSWER;
A. When a contract has a foreign element such After Maris received the final judgment of
as in the factual setting stated in the problem divorce, she married her childhood sweetheart
where one of the parties is a foreign Pedro, also a Filipino citizen, in a religious
corporation, the contract can be sustained as ceremony in Cebu City, celebrated according to
valid particularly the stipulation expressing the formalities of Philippine law. Pedro later
left for the United States and became years after their graduation, they decided to
naturalized as an American citizen. Maris annul their marriage. Jane filed an action to
followed Pedro to the United States, and after annul her marriage to Gene in England on the
a serious quarrel, Maris filed a suit and ground of latter’s sterility, a ground for
obtained a divorce decree issued by the court annulment of marriage in England. The English
in the state of Maryland. court decreed the marriage annulled.
Returning to the Philippines, Gene asked you
whether or not he would be free to marry his
former girlfriend. What would your legal advice
be? 5%
SUGGESTED ANSWER:
No, Gene is not free to marry his former
girlfriend. His marriage to Jane is valid
according to the forms and solemnities of
British law, is valid here (Article 17, 1st par.,
NCC). However, since Gene and Jane are still
Filipinos although living in England, the
dissolution of their marriage is still governed
by Philippine law (Article 15, NCC). Since,
sterility is not one of the grounds for the
annulment of a marriage under Article 45 of
the Family Code, the annulment of Gene’s
marriage to Jane on that ground is not valid in
the Philippines (Article 17, NCC)
ALTERNATIVE ANSWER:
Yes, Gene is free to marry his girlfriend
because his marriage was validly annulled in
England. The issue of whether or not a
marriage is voidable, including the grounds
therefore, is governed by the law of the place
where the marriage was solemnized (lex loci
celebrationis). Hence, even if sterility is not a
ground to annul the marriage under the
Philippine law, the marriage is nevertheless
voidable because sterility makes the marriage
voidable under English law. Therefore,
annulment of the marriage in England is valid
in the Philippines.
Applicable Laws; Sale of Real Property (1995)
While in Afghanistan, a Japanese by the name
of Sato sold to Ramoncito, a Filipino, a parcel
of land situated in the Philippines which Sato
inherited from his Filipino mother.
What law governs the formality in the
execution of the contract of sale? Explain your
answer and give its legal basis.

SUGGESTED ANSWER:
CIVIL LAW Answers to the BAR as Arranged by Topics (Year 1990-2006)
Under Art. 16 par. 1, NCC, real property is 3. The distribution of the personal properties in
subject to the Germany
law of the country where it is situated. Since the shall be governed by French law. The legal
property is basis is Art. 16,
situated in the Philippines, Philippine law
applies. The rule of NCC).
lex rei sitae in Article 16 prevails over lex loci
contractu in
Article 17 of the NCC. Applicable Laws; Wills executed abroad (1993)
A, a Filipino, executed a will in Kuwait
ALTERNATIVE ANSWER: while there as a
Afghanistan law governs the formal contract worker. Assume that under the laws of
requirements of the Kuwait, it is
contract since the execution is in Afghanistan. enough that the testator affix his signature to
Art. 17 of the the presence of
Civil Code provides that the forms and two witnesses and that the will need not be
solemnities of acknowledged
contracts, wills, and other public instruments before a notary public. May the will be
shall be probated in the
governed by the laws of the country in Philippin
which they are es?
executed. However, if the contract was executed
before the SUGGESTED ANSWER:
diplomatic or consular officials of the Yes. Under Articles 815 and 17 of the Civil
Republic of the Code, the
Philippines in Afghanistan, Philippine law shall formality of the execution of a will is governed
apply. by the law of
the place of execution. If the will was
executed with the
formalities prescribed by the laws of Kuwait
Applicable Laws; Succession; Intestate & Testamentary and valid there
as such, the will is valid and may be
(2001) probated in the
Alex was born a Filipino but was a Philippin
naturalized Canadian es.
citizen at the time of his death on December 25,
1998. He left Definition; Cognovit; Borrowing Statute;
behind a last will and testament in which he
bequeathed all Characterization(1994)
his properties, real and personal, in the In Private International Law (Conflict of Laws)
Philippines to his what is:
acknowledged illegitimate Fillpina daughter and 1} Cognovit? 2) A borrowing
nothing to statute? 3)
his two legitimate Filipino sons. The sons
sought the Characterization?
annulment of the last will and testament on the
ground that it
deprived them of their legitimes but the
daughter was able to SUGGESTED ANSWER:
prove that there were no compulsory heirs or 1) a) COGNOVIT is a confession of judgment
legitimes under whereby a
portion of the complaint is confessed by the
Canadian law. Who should prevail? Why? (5%) defendant who
denies the rest thereof (Philippine law
SUGGESTED ANSWER: Dictionary, 3rd Ed.)
The daughter should prevail because Article 16 (Ocampo v. Florenciano, L-M 13553,
of the New 2/23/50).
Civil Code provides that intestate and
testamentary succession
shall be governed by the national law of the b) COGNOVIT is a "statement of confession"
person whose Oftentimes, it
is referred to as a "power of attorney" or simply
succession is under consideration. as a "power",
it is the written authority of the debtor and his
direction to the
clerk of the district court, or justice of the
Applicable Laws; Sucession of Aliens (1995) peace to enter
Michelle, the French daughter of Penreich, a judgment against the debtor as stated
German therein. (Words and
national, died in Spain leaving real Phrases, vol. 7, pp.
properties in the 115-166).
Philippines as well as valuable personal
properties in c) COGNOVIT is a plea in an action which
Germany. acknowledges
1. What law determines who shall succeed the that the defendant did undertake and promise
deceased? as the plaintiff
in its declaration has alleged, and that it cannot
Explain your answer and give its legal basis. deny that it
2. What law regulates the distribution of the owes and unjustly detains from the plaintiff the
real properties sum claimed
in the Philippines? Explain your answer and give by him in his declaration, and consents that
its legal judgment be
entered against the defendant for a certain
basis. sum. [Words and
3. What law governs the distribution of the Phrases, vol. 7, pp.
personal 115-166).
properties in Germany? Explain your answer
and give its
d) COGNOVIT is a note authorizing a lawyer for
legal basis. confession
of judgment by
SUGGESTED ANSWER: defendant.
Assuming that the estate of the decedent is
being settled in
2 "BORROWING STATUTE" -Laws of the
the Philippines) ) state or
1. The national law of the decedent (French law) jurisdiction used by another state in
shall govern deciding conflicts
in determining who will succeed to his estate. questioned involved in the choice of law
The legal basis (Black's Law
Dictionary, 5th ed.
is Art. 16 par. 2, NCC. 1979).
ALTERNATIVE ANSWER:
French law shall govern the distribution of his
real properties 3 a) otherwi
in the Philippines except when the real property ) "CHARACTERIZATION" is se called
is land which "classification" or "qualification." It is the
may be transmitted to a foreigner only by process of assigning
hereditary disput question to correc lega categor (Priva
a ed its t l y te
succession. International Law,
SUGGESTED ANSWER:
Salonga).
2. The distribution of the real properties in the
Philippines b) "CHARACTERIZATION" is a process in
shall be governed by French law. The legal determining
basis is Art. 16, under what category a certain set of facts or
rules fall. (Paras,
NCC).
Conflict of Laws, p. 94. 1984 ed.)
SUGGESTED ANSWER:
CIVIL LAW Answers to the BAR as Arranged by Topics granted, U.S. citizenship. In 1989, Mario,
Definition; forum non-conveniens; long-arm statute (1994) claiming to have been abandoned by Clara, was
23 What is the doctrine of Forum non able to secure a decree of divorce in Reno,
conveniens? Nevada, U.S.A.
24 What is a "long arm statute"?
SUGGESTED ANSWER: In 1990, Mario returned to the Philippines and
512⤀ĀᜀĀᜀĀᜀĀᜀĀᜀĀᜀĀᜀĀᜀĀĀȀ⤀ĀᜀĀᜀĀᜀĀᜀĀᜀĀ married Juana who knew well Mario's past life.
ᜀĀᜀĀᜀĀĀȀ⤀ĀᜀĀᜀĀᜀĀᜀĀᜀĀᜀĀᜀĀᜀĀĀȀ⸀ĀᜀĀᜀĀᜀĀ (a) Is the marriage between Mario and Juana
ᜀĀᜀĀᜀĀᜀ a) FORUM NON CONVENIENS valid?(b)Would the renvoi doctrine have any
is a principle in Private International Law that relevance to the case?
where the ends of justice strongly indicate
SUGGESTED ANSWER:
that the controversy may be more suitably
tried elsewhere, then jurisdiction should be 0 Yes, because Phil law recognizes the divorce
declined and the parties relegated to relief to between Mario and Clara as valid.
SUGGESTED ANSWER:
be sought in another forum. (Moreno. No, The renvoi doctrine is relevant in cases where one country
Philippine Law Dictionary, p. 254, 1982 ed.). applies the domiciliary theory and the other the

0 Where in a broad sense the ends of justice


strongly indicate that the controversy may be
more suitably tried elsewhere, then
jurisdiction should be declined and the parties
relegated to relief to be sought in another
forum. (Handbook on Private International
Law, Aruego).

1 FORUM NON CONVENIENS means simply


that a court may resist imposition upon its
jurisdiction even when jurisdiction is
authorized by the letter of a general venue
statute. (Salonga. Private International Law. p,
51. 1967 ed.)

2 Forum non conveniens is a doctrine


whereby a court of law having full Jurisdiction
over a case brought in a proper venue or
district declines to determine the case on its
merits because Justice would be better served
by the trial over the case in another
jurisdiction. (Webster's Dictionary)
SUGGESTED ANSWER:
(2} a) LONG ARM STATUTE is a legislative act
which provides for personal jurisdiction, via
substituted service or process, over persons or
corporations which are nonresidents of the
state and which voluntarily go into the state,
directly or by agent or communicate with
persons in the state for limited purposes,
inactions which concern claims relating to
performance or execution of those purposes
(Black's Law Dictionary, 5th Ed. 1979).

23 Long arm statute refers simply to


authorized substituted service.

Divorce; effect of divorce granted to former Filipinos;


Renvoi Doctrine (1997)
In 1977, Mario and Clara, both Filipino
citizens, were married in the Philippines.
Three years later, they went to the United
States of America and established their
residence in San Francisco, California. In
1987, the couple applied for, and were
(Year 1990-2006) C. Should the Philippine court assume
nationality theory, and the issue involved is which of the laws of jurisdiction over the case? Explain. (5%)
the two countries should apply to determine the order of SUGGESTED ANSWER:
succession, the amount of successional rights, or, the intrinsic A. LEX LOCI CONTRACTUS may be
validity of testamentary provisions. Such issue is not involved in
this case. understood in two senses, as follows:

0 It is the law of the place where


ALTERNATIVE ANSWER: contracts, wills, and other public
Yes. "Renvoi" - which means "referring back" instruments are executed and governs
is relevant because here, we are applying their
U.S. law to Mario, being already its citizen, ―forms and solemnities‖, pursuant to the first paragraph,
although the formalities of the second Article(2)Itis 17theofpropertheNewlaw
marriage will be governed by Philippine law CiviloftheCode;contract;or e.i., the system of law
under the principle of lex loci celebrationis. intended to govern the entire contract, including
its essential requisites, indicating the law of the
Domiciliary theory vs. Nationality Theory (2004)
place with which the contract has its closest
Distinguish briefly but clearly between:
connection or
Domiciliary theory and nationality theory of
personal law. (5%)
SUGGESTED ANSWER:
DOMICILIARY THEORY posits that the
personal status and rights of a person are
governed by the law of his domicile or the
place of his habitual residence. The
NATIONALITY THEORY, on the other hand,
postulates that it is the law of the person's
nationality that governs such status and
rights

Forum Non Conveniens & Lex Loci Contractus (2002)


Felipe is a Filipino citizen. When he went to
Sydney for vacation, he met a former
business associate, who proposed to him a
transaction which took him to Moscow.
Felipe brokered a contract between Sydney
Coals Corp. (Coals), an Australian firm, and
Moscow Energy Corp. (Energy), a Russian
firm, for Coals to supply coal to Energy on a
monthly basis for three years. Both these
firms were not doing, and still do not do,
business in the Philippines. Felipe shuttled
between Sydney and Moscow to close the
contract. He also executed in Sydney a
commission contract with Coals and in
Moscow with Energy, under which contracts
he was guaranteed commissions by both
firms based on a percentage of deliveries for
the three-year period, payable in Sydney and
in Moscow, respectively, through deposits in
accounts that he opened in the two cities.
Both firms paid Felipe his commission for
four months, after which they stopped
paying him. Felipe learned from his contacts,
who are residents of Sydney and Moscow,
that the two firms talked to each other and
decided to cut him off. He now files suit in
Manila against both Coals and Energy for
specific performance.
A. Define or explain the principle of ―lex loci
contractus‖. (2%)
B. Define or explain the rule of ―forum non
conveniens‖ (3%)
CIVIL LAW Answers to the BAR as Arranged by Topics (Year 1990-2006)
where the main elements of the contract country of which they are citizens. Since
converge. As their marriage is
illustrated by Zalamea v. Court of Appeals
(228 SCRA 23 valid under Hong Kong law, it shall be valid and respected in the
[1993]), it is the law of the place where the
airline ticket Philippines.
was issued, where the passengers are
nationals and
residents of, and where the defendant airline
company Naturalization (2003)
maintained its Miss Universe, from Finland, came to the
office. Philippines on a
tourist visa. While in this country, she fell in love
with and
married a Filipino doctor. Her tourist visa
ALTERNATIVE ANSWER: having been
A. Under the doctrine of lex loci contractus, as a expired and after the maximum extension
general rule, allowed therefore,
the law of the place where a contract is made or the Bureau of Immigration and Deportation
entered into (BID) is
governs with respect to its nature and validity, presently demanding that she immediately leave
obligation and the country
interpretation. This has been said to be the rule but she refuses to do so, claiming that she is
even though already a Filipino
the place where the contract was made is Citizen by her marriage to a Can the
different from the Filipino citizen. BID
place where it is to be performed, and still order the deportation of Miss Universe?
particularly so, if the Explain. 5%
place of the making and the place of
performance are the
same (United Airline v. CA, G.R. No. 124110,
April 20, 2001). SUGGESTED ANSWER:
Yes, the BID can order the deportation of Miss
Universe. The
marria a alie t Filipi
ge of n n woman o a no does not
SUGGESTED ANSWER:
automatic mak he a Citizen She must
B. FORUM NON CONVENIENS means that a
ally e r Filipino . first
court has
prove in an appropriate proceeding that she does
discretionary authority to decline jurisdiction
not have
over a cause of
an disqualifica Philippi citizensh (Yung
action when it is of the view that the action may
y tion for ne ip. Uan
be justly and
Chu v. Republic of the Philippines, 158 SCRA 593
[1988]). Since
effectively adjudicated elsewhere. Miss Universe is still a foreigner, despite her
marriage to a
Filipin doctor can deported expiry
SUGGESTED ANSWER: o , she be upon of her
C. No, the Philippine courts cannot acquire allowable stay in the
jurisdiction over Philippines.
ANOTHER SUGGESTED ANSWER:
the case of Felipe. under the rule of
Firstly, forum non No, the Bureau of Immigration cannot order her
conveniens, the Philippine court is not a deportation.
convenient forum as alie marryi Filipino native-
all the incidents of the case occurred outside the An n woman ng a , born or
Philippines. naturaliz becom ipso Filipin she
Neither are both Coals and Energy doing ed, es facto a o if is not
business inside the disqualified to be a citizen of the Philippines (Mo
Philippines. Secondly, the contracts were not Ya Lim v
perfected in the 41
of
Commissi Immigration SCR 29 [1971]),
Philippines. Under the principle of lex loci
on , A 2 (Sec 4,
contractus, the law
Naturalization All that she has to do is prove
of the place where the contract is made shall
Law). in the
apply. Lastly, the
deportation proceeding the fact of her marriage
Philippine court has no power to fact
and that she is
determine the s not disqualified to become a
surrounding the execution of said contracts. And Filipino Citizen.
even if a ANOTHER SUGGESTED ANSWER:
proper decision could be reached, such would It If she is disqualified to be a Filipino
have no biding depends. citizen, she
effect on Coals and Energy as the court may be If she is not disqualified to be a
was not able to deported. Filipino
acquire jurisdiction over the said corporations. citizen, she may not be alie woman
(Manila Hotel deported. An n who
marri Filipincitizen on Th marria
Corp. v. NLRC. 343 SCRA 1, 1314[2000]) es a obecomes e. e ge of
Miss Universe to the Filipino doctor did not
automatically
make her a Filipino citizen. She still has to prove
that she is
Nationality Theory (2004) not disqualified to become a
PH and LV are HK Chinese. Their parents are citizen.
now Filipino
citizens who live in While still students in Theory; significant relationships theory
Manila. MNS (1994)
State, they got married although they are Able, a corporation domiciled in State A, but,
first cousins. It doing business
appears that both in HK and in MNS State first in the Philippines, hired Eric, a Filipino
cousins could engineer, for its
project in State B. In the contract of employment
marry legally. executed by
the parties in State B, it was stipulated that the
They plan to reside and set up business in the contract could
Philippines. be terminated at the company's will, which
But they have been informed, however, that the stipulation is
marriage of allowed in State B. When Eric was summarily
first cousins here is considered void from the dismissed by
beginning by Able, he sued Able for damages in the
reason of public policy. They are in a dilemma. Philippines. Will the
They don’t Philippine court apply the contractual
want to break Philippine law, much less their stipulation?
marriage vow.
They seek your advice on whether their civil
status will be SUGGESTED ANSWER:
adversely affected by Philippine domestic law? a) Using the "SIGNIFICANT RELATIONSHIPS
What is your THEORY",
there are contacts significant to the Philippines.
advice? (5%) Among these are
tha th place business is Philippin the
SUGGESTED ANSWER: t e of the es, employee
My advise is as follows: The civil status of' PH and LV will not be concerned is a Filipino and the suit was filed in
adversely affected by Philippine law because they are nationals of
the Philippines,
thereby justifying the application of Philippine
Hong Kong and not Filipino citizens.Being foreigners, their status, law. In the
American Airlines case the Court held that
conditions and legal capacity in the Philippines are governed by the when what is
involved is PARAMOUNT STATE INTEREST such
law of Hong Kong, the as the
CIVIL LAW Answers to the BAR as Arranged by Topics (Year 1990-2006)
protection of the rights of Filipino laborers, the natural mother as her middle name. The Court
court can has ruled
disregard choice of forum and choice of law. that there is no law prohibiting an illegitimate
Therefore the child adopted by
Philippine Court should not apply the stipulation her natural father to use, as middle name, her
in question. mother's
surname. What is not prohibited is allowed.
After all, the use
of the maternal name as the middle name is in
ALTERNATIVE ANSWER: accord with
b) No, lex fori should be applied because the Filipino culture and customs and adoption is
suit is filed in intended for the
Philippine courts and Eric was hired in the benefit of the adopted [In re: Adoption of
Philippines. The Stephanie Nathy
Philippine Constitution affords full protection to Astorga Garcia, G.R. No. 148311, March 31,
labor and 2005; Rabuya, The
the stipulation as to summary dismissal runs
counter to our Law on Persons and Family Relations, p. 613].
fundamental and statutory
laws.

Torts; Prescriptive Period (2004) Inter-Country Adoption; Formalities (2005)


In a class suit for damages, plaintiffs claimed Hans Berber, a German national, and his
they suffered Filipino wife,
injuries from torture during The suit was Rhoda, are permanent residents of Canada.
martial law. filed They desire so
upon President EM’s arrival on exile in HI, a much to adopt Magno, an 8-year old orphaned
U.S. state. The boy and a
court in HI awarded plaintiffs the equivalent of baptismal godson of Rhoda. Since the
P100 billion accidental death of
under the U.S. law on alien tort On appeal, Magno's parents in 2004, he has been staying
claims. EM’s with his aunt
Estate raised the issue of prescription. It who, however, could hardly afford to feed her
argued that since own family.
said U.S. law is silent on the matter, the court Unfortunately, Hans and Rhoda cannot come to
should apply: the
(1) HI’s law setting a two-year limitation on tort Philippines to adopt Magno although they
claims; or (2) possess all the
the Philippine law which appears to require that
claims for qualifications as adoptive parents.
personal injury arising from martial law be Is there a possibility for them to adopt
brought within Magno? How
one year. should they go about it? (5%)
The US Court will apply US law, the law of the
Plaintiffs countered that provisions of the most Jorum, in determining the applicable
analogous federal statute, the Torture Victims prescriptive period. While US law is silent on
Protection Act, should be applied. It sets ten this matter, the US Court will not apply
years as the period for prescription. Moreover, Philippine law in determining the prescriptive
they argued that equity could toll the statute period. It is generally affirmed as a principle in
of limitations. For it appeared that EM had private international law that procedural law is
procured Constitutional amendments granting one of the exceptions to the application of
himself and those acting under his direction foreign law by the forum. Since prescription is
immunity from suit during his tenure. a matter of procedural law even in Philippine
jurisprudence, (Codaltn v. POEA/ JVLRC/Broum
and Root International, 238 SCRA 721 [1994]),
In this case, has prescription set in or not? the US Court will apply either HI or Federal
Considering the differences in the cited laws, law in determining the applicable prescriptive
which prescriptive period should be applied: period and not Philippine law. The
one year under Philippine law, two years under Restatement of American law affirms this
HI’s law, ten years under U.S. federal law, or principle.
none of the above? Explain. (5%)
SUGGESTED ANSWER:
SUGGESTED ANSWER:
ADOPTION Yes, it is possible for Hans and Rhoda to adopt
Adoption; Use of Surname of her Natural Mother (2006) Magno. Republic Act No. 8043 or the Inter-
May an illegitimate child, upon adoption by her Country Adoption Act, allows aliens or
Filipinos permanently residing abroad to
natural father, use the surname of her natural
apply for inter-country adoption of a Filipino
mother as the middle name?
(2.5%) child. The law however requires that only
SUGGESTED ANSWER: Yes,an illegitimate child, legally free child, or one who has been
upon adoption by her natural father, can use voluntarily or involuntarily committed to the
the surname of her DSWD or any of its accredited agencies, may
be subject of intercountry adoption. The law
further requires that aside from possessing all
the qualifications, the adoptive parents must
come from a country where the Philippines
has diplomatic relations and that the
government maintains a similarly accredited
agency and that adoption is allowed under the
national law of the alien. Moreover, it must be
further shown that all possibilities for a
domestic adoption have been exhausted and
the inter-country adoption is best for the
interest of the child.

Hans and Rhoda have to file an application to


adopt Magno, either with the Regional Trial
Court having jurisdiction over Magno or with
the Inter-Country Adoption Board in Canada.
Hans and Rhoda will then undergo a trial
custody for six (6) months from the time of
placement. It is only after the lapse of the trial
custody that the decree of adoption can be
issued.

Parental Authority; Rescission of Adoption (1994)


In 1975, Carol begot a daughter Bing, out of
wedlock. When Bing was ten years old, Carol
gave her consent for Bing's legal adoption by
Norma and Manuel, which was granted by the
court in 1990. In 1991, Carol learned that
Norma and Manuel were engaged in a call-
girl-ring that catered to tourists. Some of the
girls lived with Norma and Manuel. Carol got
Bing back, who in the first place wanted to
return to her natural mother. 1) Who has a
better right to the custody of Bing, Carol or
Norma? 2) Aside from taking physical custody
of Bing, what legal actions can Carol take to
protect Bing?
CIVIL LAW Answers to the BAR as Arranged by Topics petition on the ground that the petitioners, being
SUGGESTED ANSWER: both foreigners, are disqualified to adopt Vicky.
0 a) It depends on whether or not Bing was 0 Is the government's opposition tenable?
at least 18 years old at the time Carol asserts Explain. (2%)
the prerogative to take custody of Bing. If she SUGGESTED ANSWER:
was at least 18 years old, then she is no The government's position is untenable. Under
longer under parental authority and neither paragraph 3, Article 184 of the Family Code, an
Carol nor Norma can assert the prerogative to alien, as a general rule cannot adopt. However,
take custody. However, if she was less than 18 an alien who is a former Filipino citizen and who
years old, then Norma has a better right since seeks to adopt a relative by consanguinity is
the adoption by Norma of Bing terminates the qualified to adopt, (par. 3[a], Art. 184, Family
parental authority of Carol over Bing. Code)

0 The natural mother, Carol, should have the


better right in light of the principle that the
child's welfare is the paramount consideration
in custody rights. Obviously, Bing's continued
stay in her adopting parents' house, where
interaction with the call girls is inevitable,
would be detrimental to her moral and
spiritual development. This could be the
reason for Bing's expressed desire to return to
her natural mother. It should be noted,
however, that Bing is no longer a minor, being
19 years of age now. It is doubtfu1 that a court
can still resolve the question of custody over
one who is sui juris and not otherwise
incapacitated.
SUGGESTED ANSWER:
0 a) On the assumption that Bing is still a
minor or otherwise incapacitated, Carol may
petition the proper court for resolution or
rescission of the decree of adoption on the
ground that the adopting parents have
exposed, or are exposing, the child to corrupt
influence, tantamount to giving her corrupting
orders or examples. She can also ask for the
revesting in her of parental authority over
Bing. If However, Bing is already 19 years of
age and therefore no longer a minor, it is not
Carol but Bing herself who can petition the
court for judicial rescission of the adoption,
provided she can show a ground for
disinheritance of an ascendant.

Carol may file an action to deprive Norma of


parental authority under Article 231 of the
Family Code or file an action for the rescission
of the adoption under Article 191 in relation
to Article 231 (2) of the Family Code.
Qualification of Adopter (2005)
In 1984, Eva, a Filipina, went to work as a
nurse in the USA. There, she met and fell in
love with Paul, an American citizen, and they
got married in 1985. Eva acquired American
citizenship in 1987. During their sojourn in the
Philippines in 1990, they filed a joint petition
for the adoption of Vicky, a 7-year old daughter
of Eva's sister. The government, through the
Office of the Solicitor General, opposed the
(Year 1990-2006) A German couple filed a petition for adoption
sister. Thus, under the above-cited of a minor Filipino child with the Regional
provision, Eva is qualified to adopt Vicky. Trial Court of Makati under the provisions of
the Child and Youth Welfare Code which
23Would your answer be the same if they allowed aliens to adopt. Before the petition
sought to adopt Eva's illegitimate could be heard, the Family Code, which
daughter? Explain. (2%) repealed the Child and Youth Welfare Code,
SUGGESTED ANSWER: came into effect. Consequently, the Solicitor
My answer will still be the same. Paragraph General filed a motion to dismiss the petition,
3(a) of Article 184 of the Family Code does on the ground that the Family Code prohibits
not make any distinction. The provision aliens from adopting. If you were the judge,
states that an alien who is a former Filipino how will you rule on the motion? (5%)
citizen is qualified to adopt a relative by
consanguinity. SUGGESTED ANSWER:
The motion to dismiss the petition for adoption
0 Supposing that they filed the petition should be denied. The law that should govern
to adopt Vicky in the year 2000, will your the action is the law in force at the time of
answer be the same? Explain. (2%) filing of the petition. At that time, it was the
SUGGESTED ANSWER: Child and Youth Welfare Code that was in
Yes, my answer will still be the same. Under effect, not the Family Code. Petitioners have
Sec. 7(b), Art. III of the New Domestic already acquired a vested right on their
Adoption Act, an alien who possesses all the qualification to adopt which cannot be taken
qualifications of a Filipino national who is away by the Family Code. (Republic v. Miller
qualified to adopt may already adopt provided G.R. No. 125932, April 21,
that his country has diplomatic relations with 1999, citing Republic v. Court of Appeals,
the Philippines, that he has been living in the 205 SCRA 356)
ALTERNATIVE ANSWER:
Philippines for at least three (3) continuous
The motion has to be granted. The new law
years prior to the filing of the application for
shall govern their qualification to adopt and
adoption and maintains such residence until
under the new law, the German couple is
the adoption decree is entered, that he has
disqualified from adopting. They cannot claim
been certified by his diplomatic or consular
that they have already acquired a vested right
office or any appropriate government agency
because adoption is not a right but a mere
that he has the legal capacity to adopt in his
privilege. No one acquires a vested right on a
country, and that his government allows the
privilege.
adoptee to enter his country as his adopted [Note: If the examinee based his answer on the
child. current law, RA 8552, his answer should be
considered correct. This question is based on the
repealed provision of the Family Code on
Qualification of Adopter; Applicable Law (2001) Adoption.]
In the given problem, Eva, a naturalized Qualifications of Adopter
American citizen (2000)
would like to adopt Vicky, a 7-year old
daughter of her
CIVIL LAW Answers to the BAR as Arranged by Topics (Year 1990-2006)
Sometime in 1990, Sarah, born a Filipino but under Sec. 7(b) of RA8552. The Supreme Court
by then a has held in
naturalized American citizen, and her several cases that when husband and wife are
American husband required to
Tom, filed a petition in the Regional Trial Court adopt jointly, each one of them must be
of Makati, qualified to adopt in
for the adoption of the minor child of her sister, a his or her own right (Republic v. Toledano, 233
Filipina. SCRA 9 (1994).
However, the American husband must comply
Can the petition be granted? (5%) with the
requirements of the law including the
SUGGESTED ANSWER: residency requirement
(per dondee) It depends. Rules on Adoption of three (3) years. Otherwise, the adoption will
effective August not be allowed.
22, 2002 provides the following; SEC. 4. Who may
adopt. –
The following may adopt: Any Filipino Citizen
Successional Rights of Adopted Child (2004)
of legal age,
A Filipino couple, Mr. and Mrs. BM, Jr., decided
in possession of full civil capacity and legal
rights,
to adopt
YV, an orphan from St. Claire’s orphanage in
of good moral character, New York City.
has not been convicted of any crime They loved and treated her like a legitimate
involving moral child for they
have none of their very own. However, BM, Jr.,
turpitude; died in an
who is emotionally and psychologically accident at sea, followed to the grave a year
capable of caring later by his sick
father, BM, Sr. Each left a sizable estate
for children, consisting of bank
at least sixteen (16) years older than the deposits, lands and buildings in Manila. May
adoptee, the adopted
and who is in a position to support and care for his children
child, YV, inherit from BM, Jr.? May she also inherit from
in keeping with the means of the family. BM, Sr.? Is there a difference? Why?
Explain. (5%)
0The requirement of a 16-year difference within the fourth (4th) degree of consanguinity
between the age of the adopter and adoptee or affinity; or b) one who seeks to adopt the
may be waived when the adopter is the legitimate child of his Filipino spouse; or
biological parent of the adoptee or is the spouse
0 one who is married to a Filipino citizen
of the adoptee’s parent;
and seeks to adopt jointly with his spouse a
Any Alien possessing the same qualifications relative within the fourth (4th) degree of
as above-stated for Filipino nationals: consanguinity or affinity of the Filipino spouse.
Provided, a) That his country has diplomatic
relations with the
Republic of the Philippines, Qualifications of Adopter (2003)
0 that he has been living in the Lina, a former Filipina who became an
Philippines for at least three (3) continuous American citizen shortly after her marriage to
years prior to the filing of the petition for an American husband, would like to adopt in
adoption and maintains such residence until the Philippines, jointly with her husband, one
the adoption of her minor brothers. Assuming that all the
decree is entered,
0 that he has been certified by his required consents have been obtained, could
diplomatic or consular office or any the contemplated joint adoption in the
appropriate government agency to have Philippine prosper? Explain.
the legal capacity to adopt in his country, SUGGESTED ANSWER:
Yes, Lina and her American husband can jointly adopt a minor
0 and that his government allows the
brother of Lina because she and her husband are both qualified to
adoptee to enter his country as his adopted adopt. Lina, as a former Filipino citizen, can adopt her minor
child. brother under Sec. 7(b)(i) of RA 8552 (Domestic Adoption Act of
1998), or under Art. 184 (3)(1) of the Family Code. The alien
Provided, further, That the requirements on husband can now adopt
residency and certification of the alien’s
qualification to adopt in his country may be
waived for the following: a) a former Filipino
citizen who seeks to adopt a relative
SUGGESTED ANSWER:
YV can inherit from BM, Jr. The succession to
the estate of BM, Jr. is governed by
Philippine law because he was a Filipino
when he died (Article 16, Civil Code). Under
Article 1039 of the Civil Code, the capacity
of the heir to succeed is governed by the
national law of the decedent and not by the
national law of the heir. Hence, whether or
not YV can inherit from BM, Jr. is determined
by Philippine law. Under Philippine law, the
adopted inherits from the adopter as a
legitimate child of the adopter.

YV, however, cannot inherit, in his own right,


from the father of the adopter, BM, Sr.,
because he is not a legal heir of BM, Sr. The
legal fiction of adoption exists only between
the adopted and the adopter. (Teotico v. Del
Val 13 SCRA 406 [1965]). Neither may he
inherit from BM, Sr. by representing BM, Jr.
because in representation, the representative
must be a legal heir not only of the person he
is representing but also of the decedent from
whom the represented was supposed to
inherit (Article 973, Civil Code).

FAMILY CODE
Emancipation (1993)
Julio and Lea, both 18 years old, were
sweethearts. At a party at the house of a
mutual friend. Lea met Jake, also 18 years
old, who showed interest in her. Lea seemed
to entertain Jake because she danced with
him many times. In a fit of jealousy, Julio shot
Jake with his father's 38 caliber revolver
which, before going to the party he was able
to get from the unlocked drawer inside his
father's bedroom. Jake died as a result of the
lone gunshot wound he sustained. His parents
sued Julio's parents for damages arising from
quasi-delict. At the time of the incident, Julio
was 18 years old living with his parents.
Julio's parents moved to dismiss the
complaint against them claiming that since
Julio was already of majority age, they were
no longer liable for his acts. 1) Should the
motion to dismiss be granted? Why? 2) What
is the liability of Julio's parents to Jake's
parents? Explain your answer.
Page 23 of 119
CIVIL LAW Answers to the BAR as Arranged by Topics transferred to another house of his in Pasig. Can
SUGGESTED ANSWER: the two family homes be the subject of execution
0 No, the Motion to Dismiss should not be on a judgment against Victor's wife for non-
granted. Article 236 of the Family Code as payment of the purchase in 1992 of household
amended by Republic Act 6809, provides in appliances?
the third paragraph that "nothing in this Code
shall be construed to derogate from the duty SUGGESTED ANSWER:
or responsibility of parents and guardians for The two (2) so-called family homes can be the
children and wards below twenty-one years of subject of execution. Neither of the abodes are
age mentioned in the second and third considered family homes because for purposes of
paragraphs of Article 2180 of the Civil Code". availing the benefits under the Family Code,
2) The liability of Julio's parents to Jake's there can only be one (1) family home which is
parents arises from quasi-delict (Arts. 2176 defined as the "dwelling house" where the
and 2180 Civil Code) and shall cover husband and the wife and their family actually
specifically the following: "reside" and the land on which it is situated.
(Arts. 152 and 161, Family Code)
0 P50,000.00 for the death of the son;
1 such amount as would correspond to
lost earning capacity; and
2 moral damages.

Family Code; Retroactive Application; Vested Rights


(2000) On April 15, 1980, Rene and Angelina
were married to each other without a
marriage settlement. In 1985, they acquired a
parcel of land in Quezon City. On June 1,
1990, when Angelina was away in Baguio,
Rene sold the said lot to Marcelo. Is the sale
void or voidable? (2%)
SUGGESTED ANSWER:
The sale is void. Since the sale was executed
in 1990, the Family Code is the law
applicable. Under Article 124 of the FC, the
sale of a conjugal property by a spouse
without the consent of the other is void.
ALTERNATIVE ANSWER:
The sale is voidable. The provisions of the
Family Code may apply retroactively but only
if such application will not impair vested
rights. When Rene and Angelina got married
in 1980, the law that governed their property
relations was the New Civil Code. Under the
NCC, as interpreted by the Supreme Court in
Heirs of Felipe v. Aldon, 100 SCRA 628 and
reiterated
in Heirs of Ayuste v. Malabonga, G.R No,
118784, 2 September 1999, the sale executed
by the husband without the consent of the
wife is voidable. The husband has already
acquired a vested right on the voidable nature
of dispositions made without the consent of
the wife. Hence, Article 124 of the Family
Code which makes the sale void does not
apply.

Family Home; Dwelling House (1994)


In 1991, Victor established judicially out of
conjugal property, a family home in Manila
worth P200.000.00 and extrajudicially a
second family home in Tagaytay worth
P50.000.00. Victor leased the family home in
Manila to a foreigner. Victor and his family
(Year 1990-2006) must be complied with before he can
Family; Constitutional Mandates; Divorce (1991) remarry? What advice would you give H?
0 How does the 1987 Constitution 1 Suppose that children were born from
strengthen the family as an Institution? the union of H and W, what would be the
1 Do the Constitutional policy on the status of said children? Explain your
family and the provision that marriage is answer.
the foundation of the family and shall be 2 If the subsequent marriage of H to S was
protected by the State bar Congress from contracted before compliance with the
enacting a statutory condition for its validity, what are
law allowing divorce in the Philippines? the rights of the children of the first marriage
SUGGESTED ANSWER: (i.e., of H and W) and of the children of the
A. Sec, 2, Article II of the Constitution subsequent marriage (of H and S)?
provides that: The State recognizes the SUGGESTED ANSWER:
sanctity of family life and shall protect and ̀̀⠀⤀ĀᜀĀᜀĀᜀĀᜀĀᜀĀᜀĀᜀĀᜀЀ̀Ȁ̀⠀⤀ĀᜀĀᜀĀᜀĀᜀĀᜀĀᜀ
strengthen the family as a basic ĀᜀĀᜀ̀̀Ȁ̀⠀⤀ĀᜀĀᜀĀᜀĀᜀĀᜀĀᜀĀᜀĀᜀ̀̀Ȁ̀⠀⤀ĀᜀĀ砀Āᜀ
autonomous social institution. It shall ĀᜀĀᜀĀᜀĀᜀ0 H, or either spouse for that matter, can
equally protect the life of the mother and marry again after complying with the provisions of Article
the life of the unborn from conception. The 52 of the Family Code, namely, there must be a partition
natural and primary right and duty of and distribution, of the properties of the spouses, and the
parents in the rearing of the youth for civic delivery of the
efficiency and the development of moral
character shall receive the support of the
Government.

Section I, Article XV, further provides that:


The State recognizes the Filipino family as
the foundation of the nation. Accordingly, it
shall strengthen its solidarity and actively
promote its total development.
(Note: The Committee recommends that
a citation of either one of the provisions
be credited as a complete answer).
SUGGESTED ANSWER:
B, No, the Constitutional policy, as well as
the supporting provision, does not amount
to a prohibition to Congress to enact a law
on divorce. The Constitution only meant to
help the marriage endure, to "strengthen its
solidarity and actively promote its total
development."
ALTERNATIVE ANSWER:
B. Yes. Congress is barred from enacting a
law allowing divorce, since Section 2 of
Article XV provides: "Sec. 2. Marriage, as an
inviolable social institution, is the foundation
of the family and shall be protected by the
State." Since marriage is "Inviolable", it
cannot be dissolved by an absolute divorce.
Marriage; Annulment; Effects; Requisites Before
Remarriage (1990)
The marriage of H and W was annulled by
the competent court. Upon finality of the
judgment of nullity. H began looking for his
prospective second mate. He fell in love
with a sexy woman S who wanted to be
married as soon as possible, i.e., after a few
months of courtship. As a young lawyer, you
were consulted by H,
0 How soon can H be joined in lawful
wedlock to his girlfriend S? Under existing
laws, are there certain requisites that
CIVIL LAW Answers to the BAR as Arranged by Topics (Year 1990-2006)
children's presumptive legitimes which should sexually-transmissible disease, found to be
be recorded serious and
in the appropriate civil registry and registries of
property. H appears incurable. Two (2) years after their marriage, which took
should be so
advised. place on 10 October 1988, Bethel discovered that her husband
ALTERNATIVE ANSWER: for
(a) James has a sexually-transmissible disease which he contracted
The following are the requisites prescribed
by law and I even prior to their marriage although James did not know it
advice to H is to comply with
them, namely: himself until he was examined two [2) years later when a child
If either spouse contracted the marriage
1) in bad faith, was already born to them. Bethel sues James for annulment of
his or her share of the net profits of the
community property : their marriage. James opposes the annulment on the ground that
or conjugal partnership property shall be
forfeited in favor of he did not even know that he had such a disease so that there
the common children or, if there are none, the
children of the was no fraud or bad faith on his part. Decide.
guilty spouse by a previous marriage or, in
default of children, B. Suppose that both parties at the time of
the innocent spouse; their marriage
were similarly afflicted with sexually-
transmissible diseases,
Donations by reason of marriage shall serious and incurable, and both knew of their
2) remain valid respective
except that if the donee contracted the marriage infirmities, can Bethel or James sue for
in bad faith, annulment of their
such donations made to said donee are revoked
by operation marriage?
of law; SUGGESTED ANSWER:
The spouse who contracted the A. The marriage can be annulled, because good
3) subsequent marriage faith is not a
in bad faith shall be disqualified to inherit from
the innocent defense when the ground is based upon
spouse by testate and intestate sexually-transmissible disease on the part of
succession; either party.
If both spouses of the subsequent
4) marriage acted in SUGGESTED ANSWER:
bad donatio reaso marriag B. Yes, the marriage can still be annulled
because the fact
faith all ns by n of e and
that both of them are afflicted with sexually-
testamentary dispositions made by one in favor
transmissible
of the other
are revoked by diseases does not efface or nullity the ground.
operation of law.
Th judgment annulment th marria Alternative Answer:
5) e of of e ge, the B. No, the marriage can no longer be annulled,
partiti an distribution of the o spouse because the
on d properties f the s, fact that both were afflicted and that both
and the delivery of the children's presumptive knew of their
legitimes shall respective infirmities constitutes a waiver of
be recorded in the appropriate civil registry and that ground.
registers of
property, (Articles 53.
52, 43.
Marriage; Annulment; Judicial Declaration (1993)
44. Family
Maria and Luis, both Filipinos, were married
Code).
by a Catholic
priest in Lourdes Church, Quezon City in 1976,
SUGGESTED ANSWER:
Luis was
(b) The children born from the union of H and W drunk on the day of his wedding. In fact, he
would be slumped at the
legitimate children if conceived or born before altar soon after the ceremony. After marriage,
the decree of Luis never had
annulme o the marriage 4 of a steady job because he was drunk most of the
nt f (under Art. 5 the Family time. Finally,
Code) become executo (Art. he could not get employed at all because of
has final and ry 54, Family drunkenness.
Hence, it was Maria who had to earn a living to
Code}
support
.
herself and her child begotten with Luis. In
1986, Maria filed a
petition in the church matrimonial court in
SUGGESTED ANSWER: Quezon City to
(c) The children of the first marriage shall annul her marriage with Luis on the ground of
be considered psychological
legitimate children if conceived or born before incapacity to comply with his marital
the Judgment obligation. Her petition
of annulment of the marriage of H and W has was granted by the church matrimonial court.
become final 1) Can Maria
and executory. Children conceived or born of the now get married legally to another man under
subsequent Philippine laws
marriage shall likewise be legitimate even if the after her marriage to Luis was annulled by the
marriage of H church
and S be null and void for failure to comply with matrimonial court? Explain. 2) What must
the requisites Maria do to
of Article 52 of the Family Code (Article 53, enable her to get married lawfully to another
Family Code). As man under
legitimate children, they have the
following rights; Philippine laws?
bear surnames of fathe an
a) To the the r d the SUGGESTED ANSWER:
mother in conformity with the provisions of 1) No, Maria cannot validly contract a
the Civil subsequent marriage
without a court declaration of nullity of the
Code on
first marriage.
Surnames;
The law does not recognize the church
To receive support from their
b) parents, their declaration of nullity
ascendants, and in proper cases, their
brothers and of a marriage.
sisters, in conformity with the provisions of 2) To enable Maria to get married lawfully to
this Code on another man.
Support; and
To be entitled to the legitime and she must obtain a judicial declaration of nullity
c) other of the prior
successio rights to b the Civil
nal granted them y Code marriage under Article 36 Family Code.
(Article 174, Family Code). Marriage; Annulment; Legal Separation; Prescription of
Actions (1996)
Marriage; Annulment; Grounds (1991) 2) Bert and Baby were married to each other
on December
One of the grounds for annulment of marriage is that either party, 23, 1988. Six months later, she discovered
at the time of their marriage was afflicted with a that he was a
CIVIL LAW Answers to the BAR as Arranged by Topics (Year 1990-2006)
drug addict. Efforts to have him rehabilitated In 1989, Maris, a Filipino citizen, married her
were boss Johnson,
unsuccessful. Can Baby ask for annulment of an American citizen, in Tokyo in a wedding
marriage, or ceremony
celebrated according to Japanese laws. One
legal separation? Explain. year later, Johnson
returned to his native Nevada, and he validly
obtained in that
state an absolute divorce from his wife Maris.
SUGGESTED ANSWER: After Maris
No, Baby cannot ask for annulment of her received the final judgment of divorce, she
marriage or for married her
legal separation because both these actions had childhood sweetheart Pedro, also a Filipino
already citizen, in a
religious ceremony in Cebu City, celebrated
prescribed. according to the
formalities of Philippine law. Pedro later left for
While concealment of drug addiction existing at the the United
States and became naturalized as an American
time of marriage constitutes fraud under Art. 46 of the citizen. Maris
followed Pedro to the United States, and after
FC which makes the marriage voidable under Art. 45 a serious
quarrel, Marts filed a suit and obtained a
of the FC, the action must, however, be brought within divorce decree issued
by the court in the state of Maryland. Maris
5 years from the discovery thereof under Article 47(3), then returned to
the Philippines and in a civil ceremony
FC, Since the drug addiction of Bert was discovered celebrated in Cebu
City according to the formalities of Philippine
by Baby in June 1989, the action had already law, she
married her former classmate Vincent likewise
prescribed in June of 1994. Although drug addiction is a Filipino
citizen. b) Was the marriage of Maris and
a ground for legal separation under Art. 55(5) and Art. Pedro valid when
celebrated? Is their marriage still valid existing
57 of the FC requires that the action must be brought now? Reasons.
c) Was the marriage of Marts and Vincent valid
within 5 years from the occurrence of the cause. when
celebrated? Is their marriage still validly
Since Bert had been a drug addict from the time of the existing now?
Reasons. d) At this point in time, who is the
celebration of the marriage, the action for legal lawful husband
separation must have been brought not later than 23 of Marts? Reasons.
December 1993. Hence, Baby cannot, now, bring the
action for legal separation.
Marriage; Annulment; Proper Party (1990)
D and G, age 20 and 19, respectively, and both
single, eloped
and got married to each other without parental
consent in the
case of G, a teenaged student of an exclusive
college for girls. SUGGESTED ANSWER:
Three years later, her parents wanted to seek (b) The marriage of Maris and Pedro was valid
judicial when
annulment on that ground. You were consulted celebrated because the divorce validly
and asked to obtained by Johnson in
prepare the proper complaint. What advice Manila capacitated Maris to marry Pedro. The
would you give marriage of
Maris and Pedro is still validly existing,
G's parents? Explain your answer. because the marriage
SUGGESTED ANSWER: has not been validly dissolved by the Maryland
divorce [Art.
G himself should file the complaint under Article
45 of the 26, Family Code).
Family Code, and no longer the parents because
G is already
(c) The marriage of Maris and Vincent is void
22 years of age. ab initio
because it is a bigamous marriage contracted
by Maris during
the subsistence of her marriage with Pedro
Marriage; Annulment; Proper Party (1995) (Art 25 and 41,
Yvette was found to be positive for HIV virus, Family Code). The marriage of Maris and
considered Vincent does not
sexually transmissible, serious and incurable. validly exist because Article 26 does not apply.
Her boyfriend Pedro was not
Joseph was aware of her condition and yet a foreigner at the time of his marriage with
married her. After marts and the
two (2) years of cohabiting with Yvette, and in divorce abroad (in Maryland) was initiated and
his belief that obtained not
she would probably never be able to bear him a by the alien spouse, but by the Filipino spouse.
healthy child, Hence, the
Joseph now wants to have his marriage with Maryland divorce did not capacitate Marts to
Yvette annulled. marry Vincent.
Yvette opposes the suit contending that Joseph
is estopped
from seeking annulment of their marriage since
he knew even
before their marriage that she was afflicted with
HIV virus.
Can the action of Joseph for annulment of his (d) At this point in time, Pedro is still the lawful
marriage with husband of
Maris because their valid marriage has not
Yvette prosper? Discuss fully. been dissolved by
any valid cause (Art. 26. Family Code)
SUGGESTED ANSWER:
No, Joseph knew that Yvette was HIV positive Marriage; Divorce Decrees; Filiation of Children (2005)
at the time of the marriage. He is, therefore, In 1985, Sonny and Lulu, both Filipino
not an injured party. The FC gives the right to citizens, were married in the Philippines. In
annul the marriage only to an injured party. 1987, they separated, and Sonny went to
[Art. 47 (5), FC] Canada, where he obtained a divorce in the
ALTERNATIVE ANSWER:
same year. He then married another Filipina,
The action for annulment can prosper because
Auring, in Canada on January 1,1988. They
the prescriptive period of five (5) years has not
had two sons, James and John. In 1990, after
yet lapsed. [Art. 45 (6), FC].
failing to hear from Sonny, Lulu married Tirso,
Marriage; Divorce Decree; Void Marriages (1992) by whom she had a daughter, Verna. In 1991,
Sonny visited the Philippines where he
succumbed to heart attack..
CIVIL LAW Answers to the BAR as Arranged by Topics Sonny's heirs include James, John, and Lulu.
23 Discuss the effect of the divorce Article 887 of the Civil Code provides that the
obtained by Sonny and Lulu in Canada. compulsory heirs of the deceased are among
(2%) others, his widow and his illegitimate children.
SUGGESTED ANSWER: The widow referred to in Article 887 is the legal
The divorce is not valid. Philippine law does wife of the deceased. Lulu is still a compulsory
not provide for absolute divorce. Philippine heir of Sonny because the divorce obtained by
courts cannot grant it. A marriage between Sonny in Canada cannot be recognized in the
two (2) Filipinos cannot be dissolved by a Philippines. The legitime of each illegitimate
divorce obtained abroad. (Garcia v. Redo, G.R. child shall consist of one-half of the legitime of
No. 138322, October 2, 2001). Philippine laws a legitimate child. (Art. 176, Family Code)
apply to Sonny and Lulu. Under Article 15 of
the New Civil Code, laws relating to family Marriage; Divorce Decrees; Filipino Spouses becoming Alien
rights and duties, status, and capacity of (1996)
persons are binding upon citizens of the
Philippines wherever they may be. Thus, the
marriage of Sonny and Lulu is still valid and
subsisting.

23 Explain the status of the marriage


between Sonny and Auring. (2%)
SUGGESTED ANSWER:
Since the decree of divorce obtained by Lulu
and Sony in Canada is not recognized here in
the Philippines, the marriage between Sonny
and Auring is void. (Art. 35, Family Code) Any
marriage subsequently contracted during the
lifetime of the first spouse shall be illegal and
void, subject only to the exception in the cases
of absence or where the prior marriage was
dissolved or annulled. (Ninal
v. Bayadog, G.R. No. 133778, March 14, 2000)
The marriage of Sonny and Auring does not
fall within the exception.

0 Explain the status of the marriage


between Lulu and Tirso. (2%)
SUGGESTED ANSWER:
The marriage of Lulu and Tirso is also void.
Mere absence of the spouse does not give rise
to a right of the present spouse to remarry.
Article 41 of the Family Code provides for a
valid bigamous marriage only where a spouse
has been absent for four consecutive years
before the second marriage and the present
spouse had a well-founded belief that the
absent
spouse is already dead. (Republic
v. Nolasco, G.R. No. 94053,
March 17, 1993)

0 Explain the respective filiation of James,


John and Verna. (2%)
SUGGESTED ANSWER:
James, John and Verna are illegitimate
children since their parents are not validly
married. Under Article 165 of the Family
Code, children conceived and born outside a
valid marriage are illegitimate, unless
otherwise provided in this Code.

e) Who are the heirs of Sonny? Explain.


(2%) Suggested answer:
(Year 1990-2006) Later, he returned to the Philippines with his
Flor and Virgillo were married to each other new wife. Eva now wants to know what action
in Roxas City in 198O. In 1984, Flor was or actions she can file against Ben. She also
offered a teaching Job in Canada, which she wants to know if she can likewise marry again.
accepted. In 1989, she applied for and was What advice can you give her? {5%)
granted Canadian citizenship. The following
SUGGESTED ANSWER:
year, she sued for divorce from Virgilio in a
Canadian court. After Virgilio was served Considering that Art. 26(2nd par.)
with summons, the Canadian court tried the contemplates a divorce between a foreigner
case and decreed the divorce. Shortly and a Filipino, who had such respective
thereafter, Flor married a Canadian. Can nationalities at the time of their marriage, the
Virgilio marry again in the Philippines? divorce in Europe will not capacitate the
Explain. Filipino wife to remarry. The advice we can
give her is either to file a petition for legal
SUGGESTED ANSWER: separation, on the ground of sexual infidelity
No, Virgilio cannot validly remarry. His case and of contracting a bigamous marriage
is not covered by Article 26 of the Family abroad, or to file a petition to dissolve the
Code, For said Article to be applicable, the conjugal partnership or absolute community of
spouse who filed for divorce must be a property as the case maybe.
foreigner at the time of the marriage. Since
both of them were Filipinos at the time of the
marriage, the divorce obtained by Flor did
not capacitate Virgilio to remarry. The fact
that Flor was already an alien at the time she
obtained the divorce does not give Virgilio
the capacity to remarry under Philippine
Law.
ALTERNATIVE ANSWERS:
23 Yes, Virgilio can validly remarry. Art. 26
of the FC, merely States the alien spouse
without taking into consideration his or her
nationality at the time of the marriage. While
his case is not covered by the letter of Article
26 FC, it is, however, covered by the spirit of
said Article, the injustice to the Filipino
spouse sought to be cured by said Article is
present in this case. (Department of Justice
Opinion No. 134 Series of 1993).

24 Although the marriage originally


involved Filipino citizens, it eventually
became a marriage between an alien and a
Filipino after Flor became a Canadian
citizen. Thus, the divorce decree was one
obtained by an alien spouse married to a
Filipino. Although nothing is said about
whether such divorce did capacitate Flor to
remarry, that fact may as well be assumed
since the problem states that she married a
Canadian shortly after obtaining the divorce.
Hence, Virgillo can marry again under
Philippine law, pursuant to Art. 26. FC which
applies because Flor was already an alien at
the time of the divorce.

Marriage; Divorce Decrees; Filipino Spouses becoming


Alien (1999)
Ben and Eva were both Filipino citizens at
the time of their marriage in 1967, When
their marriage turned sour, Ben went to a
small country in Europe, got himself
naturalized there, and then divorced Eva in
accordance with the law of that country,
CIVIL LAW Answers to the BAR as Arranged by Topics Since the problem states that the marriage was
ALTERNATIVE ANSWER: annulled and there is no intimation of bad faith
Eva may file an action for legal separation on on the part of the donee Digna, the conclusion is
the grounds of sexual infidelity of her husband that the donor cannot revoke the donation.
and the contracting by her husband of a ALTERNATIVE ANSWER:
bigamous marriage abroad. 0 Yes, the donation can be revoked. The ground used in dissolving
the marriage was the psychological immaturity of George, which is
not a ground for annulment of marriage. If this term is equated with
She may remarry. While a strict interpretation psychological incapacity as used in Art. 36 of the Family Code, then it
of Article 26 of the Family Code would is a ground for declaration of nullity of the marriage. Consequently,
capacitate a Filipino spouse to remarry only par. (1) of Art. 86, FC, is the applicable law. Since Art. 86 of the FC
makes no qualification as to who furnished the ground or who was in
when the other spouse was a foreigner at the bad faith in connection with the nullification of
time of the marriage, the DOJ has issued an
opinion (Opinion 134 s. of 1993) that the same
injustice sought to be cured by Article 26 is
present in the case of spouses who were both
Filipino at the time of the marriage but one
became an alien subsequently. Said injustice is
the anomaly of Eva remaining married to her
husband who is no longer married to her.
Hence, said Opinion makes Article 26
applicable to her case and the divorce
obtained abroad by her former Filipino
husband would capacitate her to remarry. To
contract a subsequent marriage, all she needs
to do is present to the civil registrar the
decree of divorce when she applies for a
marriage license under Article 13 of the Family
Code.

Marriage; Donations by Reason of Marriage; Effect of


Declaration of Nullity (1996)
0 On the occasion of Digna's marriage to
George, her father gave her a donation
propter nuptias of a car. Subsequently, the
marriage was annulled because of the
psychological immaturity of George. May
Digna's father revoke the donation and get
back the car? Explain.
SUGGESTED ANSWER:
No, Digna's father may not revoke the
donation because Digna was not in bad faith,
applying Art. 86(3) of the Family Code.
ALTERNATIVE ANSWER:
0 Yes, the donation is revocable. Since the
ground for the annulment of the marriage is
the psychological immaturity of George, the
judgment was in the nature of a declaration of
nullity under Art. 36 of the FC and, therefore,
the donation may be revoked under Art. 86( 1)
of the FC for the reason that the marriage has
been judicially declared void ab initio.
ALTERNATIVE ANSWER:
23 No, the donation cannot be revoked.
The law provides that a donation by reason of
marriage may be revoked by the donor if
among other cases, the marriage is judicially
declared void ab initio [par. (1) Art. 86. Family
Code], or when the marriage is annulled and
the donee acted in bad faith [par. (3), Id.].
(Year 1990-2006)
the marriage, the conclusion is that Digna's father may revoke
the donation and get back the car.

Marriage; Grounds; Declaration of Nullity: Annulment:


Legal
Separation: Separation of Property (2003)
Which of the following remedies, i.e., (a)
declaration of nullity of marriage, (b)
annulment of marriage, (c) legal separation,
and/or (d) separation of property, can an
aggrieved spouse avail himself/herself of-

0 If the wife discovers after the marriage


that her
husband(ii) hasIfthe―AIDS‖wifegoes. (to)
abroad to work as a nurse and refuses to
come home after the expiration of her
three-year contract there.
23 If the husband discovers after the
marriage that his wife has been a prostitute
before they got married.
24 If the husband has a serious affair
with his secretary and refuses to stop
notwithstanding advice from relatives and
friends.
25 If the husband beats up his wife
every time he
comes home drunk. 5%
SUGGESTED ANSWER:
(i) Since AIDS is a serious and incurable
sexually-transmissible disease, the wife may
file an action for annulment of the marriage on
this ground whether such fact was concealed
or not from the wife, provided that the
disease was present at the time of the
marriage. The marriage is voidable even
though the husband was not aware that he
had the disease at the time of marriage.

0 If the wife refuses to come home for three


(3) months from the expiration of her
contract, she is presumed to have abandoned
the husband and he may file an action for judicial
separation of property . If the refusal continues for
more than one year from the expiration of her
contract, the husband may file the action for
legal separation under Art. 55 (10) of the Family
Code on the ground of abandonment of
petitioner by respondent without justifiable
cause for more than one year. The wife is
deemed to have abandoned the husband
when she leaves the conjugal dwelling
without any intention of returning (Article
101, FC). The intention not to return cannot
be presumed during the 30year period of her
contract.
If the husband discovers after the marriage
that his wife was a prostitute before they got
married, he has no remedy. No
misrepresentation or deceit as to character,
health, rank, fortune or chastity shall
constitute fraud as legal ground for an action
for the annulment of marriage (Article 46 FC).

The wife may file an action for legal separation .


The husband’s sexual infidelity is a ground
for legal separation
9Article 55, FC). She may also file an action for
judicial separation of property for failure of her
husband to comply with his martial duty of
fidelity (Article 135 (4), 101, FC).
The wife may file an action for legal separation on
the ground of repeated physical violence on
her person (Article 55 (1), FC). She may also
file an action for judicial
CIVIL LAW Answers to the BAR as Arranged by Topics marriage, they: a) Will not constitute as ground
separation of property for failure of the husband to for declaration of nullity
comply with his marital duty of mutual respect (Art. 36, Family Code); b) Will constitute as grounds for
(Article 135 (4), Article 101, FC). She may also legal separation (Art. 56, FC) and c) will not constitute
file an action for declaration of nullity of the marriage as grounds to render the marriage voidable (Art.45and
if the husband’s behavior constitute 46, FC)
psychological incapacity existing at the time of
the celebration of marriage.
Marriage; Legal Separation; Grounds; Prescriptive Period
Marriage; Grounds; Nullity; Annulment; Legal (1994)
Separation (1997) Rosa and Ariel were married in the Catholic Church of Tarlac,
Under what conditions, respectively, may Tarlac on January 5. 1988. In 1990, Ariel went to
drug addiction be a ground, if at all, (a) for a
declaration of nullity of marriage,
for an annulment of the marriage contract,
and (c) for legal separation between the
spouses?
SUGGESTED ANSWER:
Declaration of nullity of marriage:
The drug addiction must amount to
psychological incapacity to comply with
the essential obligations of marriage;
It must be antecedent (existing at the
time of marriage), grave and incurable:
The case must be filed before August 1,
1998. Because if they got married before
August 3, 1998, it must be filed before
August 1, 1998.

Annulment of the Marriage Contract: 1) The


drug
addiction must be concealed; 2) It must exist
at the time of marriage; 3) There should be
no cohabitation with full

knowledge of the drug addiction; 4)


The case is filed within five (5) years
from discovery.

Legal Separation; 1) There should be no


condonation or consent to the drug
addiction; 2) The action must be filed within
five (5) years from the occurrence of the
cause.

Drug addiction arises during the


marriage and not at the time of marriage.

Marriage; Legal Separation; Declaration of Nullity (2002) If


drug addiction, habitual alcoholism,
lesbianism or homosexuality should occur only
during the marriage, would this constitute
grounds for a declaration of nullity or for legal
separation, or would they render the marriage
voidable? (1%).
SUGGESTED ANSWER:
In accordance with law, if drug addiction,
habitual alcoholism, lesbianism or
homosexuality should occur only during the
(Year 1990-2006)
Saudi Arabia to work. There, after being converted into Islam, If you were the judge, how will you
Ariel married Mystica, Rosa learned of the second marriage of decide the case? (5%)
Ariel on January 1, 1992 when Ariel returned to the Philippines SUGGESTED ANSWER:
with Mystica. Rosa filed an action for legal separation on If I were the judge, I will dismiss the action on
February 5, 1994, 1) Does Rosa have legal grounds to ask for the ground of mutual guilt of the parties. The
legal separation? 2) Has the action prescribed? Philippine Constitution protects marriage as
an inviolable social institution (Art. XV, Sec. 2,
SUGGESTED ANSWER: 1987 Constitution). An action for legal
a) Yes, the abandonment of Rosa by Ariel for separation involves public interest and no such
more than one (1) year is a ground for legal decree should be issued if any legal obstacle
separation unless upon returning to the thereto appears on record. This is in line with
Philippines, Rosa agrees to cohabit with the policy that in case of doubt,
Ariel which is allowed under the Muslim
Code. In this case, there is condonation. b)
Yes. The contracting of a subsequent
bigamous marriage whether in the
Philippines or abroad is a ground for legal
separation under Article 55(7) of the Family
Code. Whether the second marriage is valid
or not, Ariel having converted into Islam, is
immaterial.
SUGGESTED ANSWER:
No. Under Article 57 of the Family Code, the
aggrieved spouse must file the action within
five (5) years from the occurrence of the
cause. The subsequent marriage of Ariel
could not have occurred earlier than 1990,
the time he went to Saudi Arabia. Hence,
Rosa has until 1995 to bring the action under
the Family Code.

Marriage; Legal Separation; Mutual guilt (2006)


Saul, a married man, had an adulterous
relation with Tessie. In one of the trysts,
Saul's wife, Cecile, caught them in flagrante.
Armed with a gun, Cecile shot Saul in a fit of
extreme jealousy, nearly killing him. Four (4)
years after the incident, Saul filed an action
for legal separation against Cecile on the
ground that she attempted to kill him.
If you were Saul's counsel, how will
you argue his case? (2.5%)
SUGGESTED ANSWER:
As the counsel of Saul, I will argue that an
attempt by the wife against the life of the
husband is one of the grounds enumerated
by the Family Code for legal separation and
there is no need for criminal conviction for
the ground to be invoked (Art. 55, par. 9,
Family Code).

If you were the lawyer of Cecile, what


will be your defense? (2.5%)
SUGGESTED ANSWER:
As the counsel of Cecile, I will invoke the
adultery of Saul. Mutual guilt is a ground
for the dismissal of an action for legal
separation (Art. 56, par. 4, Family Code).
The rule is anchored on a well-established
principle that one must come to court with
clean hands.
CIVIL LAW Answers to the BAR as Arranged by Topics (Year 1990-2006)
the court shall uphold the validity and sanctity of marriage the preceding Article, only the
properties acquired by both
(Brown v. Yambao, G.R. No. L-10699, October A. No. The contention of James is not correct.
18, 1957). Art. 40, Family Code, provides that the
"absolute nullity of a previous marriage may
Marriage; Non-Bigamous Marriages (2006) be invoked for purposes of remarriage on the
Marvin, a Filipino, and Shelley, an American, basis solely of a final judgment declaring such
both residents of California, decided to get previous marriage void." It can be said,
married in their local parish. Two years after therefore, that the marriage of James to
their marriage, Shelley obtained a divorce in Shirley is void since his previous marriage to
California. While in Boracay, Marvin met Ophelia, although itself void, had not yet been
Manel, a Filipina, who was vacationing there. judicially declared void,
Marvin fell in love with her. After a brief ALTERNATIVE ANSWER:
courtship and complying with all the A. No. The contention of James is not
requirements, they got married in Hongkong correct. He cannot set up as a defense his
to avoid publicity, it being Marvin's second own criminal act or wrongdoing-
marriage. Is his marriage to Manel valid?
Explain. (5%)
SUGGESTED ANSWER: SUGGESTED ANSWER:
Yes. The marriage will not fall under Art. 35(4) B. The provisions of Art 148 of the Family Code, shall govern:
Art. 148. In cases of cohabitation not falling under
of the Family Code on bigamous marriages,
provided that Shelley obtained an absolute
divorce, capacitating her to remarry under
her national law. Consequently, the marriage
between Marvin and Manel may be valid as
long as it was solemnized and valid in
accordance with the laws of Hongkong [Art.
26, paragraphs 1 and 2, Family Code].

Marriage; Property Relations; Void Marriages (1991)


In June 1985, James married Mary. In
September 1988, he also married Ophelia
with whom he begot two (2) children, A and
B. In July 1989, Mary died. In July 1990, he
married Shirley and abandoned Ophelia,
During their union. James and Ophelia
acquired a residential lot worth
P300,000.00.

Ophelia sues James for bigamy and prays that


his marriage with Shirley be declared null and
void. James, on the other hand, claims that
since his marriage to Ophelia was contracted
during the existence of his marriage with
Mary, the former is not binding upon him, the
same being void ab initio he further claims
that his marriage to Shirley is valid and
binding as he was already legally capacitated
at the time he married her. a) Is the contention
of James correct? b) What property Relations
governed the union of James

and Ophelia? c) Is the estate of Mary entitled


to a share in the

residential lot acquired by James and


Ophelia?
SUGGESTED ANSWER:
A. Yes. His marriage to Ophelia is void ab
initio because of his subsisting prior marriage
to Mary. His marriage to Shirley, after Mary's
death, is valid and binding.
ALTERNATIVE ANSWER:
of the parties through their actual joint contribution of money, Trial was conducted and Marina neither
property, or industry shall be owned by them in common in appeared nor presented evidence in her favor.
proportion to their respective contributions. In the absence, of proof
to the contrary, their contributions and corresponding shares are If you were the judge, will you grant the
presumed to be equal. The same rule and presumption shall apply annulment. Explain.
to joint deposits of money and evidences of credit.

SUGGESTED ANSWER:
As judge, I will not grant the annulment. The
SUGGESTED ANSWER: facts do not show any taint of personality
It should be distinguished when the disorder on the part of the wife Marina so as
property was acquired. to lend substance to her husband's averment
If it was acquired before Mary's death, of psychological incapacity within the
the estate of Mary is entitled to 1/2 of the meaning of Art 36 of the Family Code. In
share of James. Santos vs. CA (240 SCRA 20) , this particular
If it was acquired after Mary's death, ground for nullity of marriage was held to be
there will be no share at all for the limited only to the most serious cases of
estate of Mary. personality disorders (clearly demonstrative
of utter sensitivity or inability to give meaning
Marriage; Psychological Incapacity (1996) and significance to the marriage. Marina's
On April 15, 1983, Jose, an engineer, and refusal to come home to her husband unless
Marina, a nurse, were married to each other in he agreed not to work overseas, far from
a civil ceremony in Boac. Marinduque. Six being indicative of an insensitivity to the
months after their marriage, Jose was meaning of marriage, or of a personality
employed in an oil refinery in Saudi Arabia for disorder, actually shows a sensitive awareness
a period of three years. When he returned to on her part of the marital duty to live together
the Philippines, Marina was no longer living in as husband and wife. Mere refusal to rejoin
their house, but in Zamboanga City, working in her husband when he did not accept the
a hospital. He asked her to come home, but she condition imposed by her does not furnish any
refused to do so, unless he agreed not to work basis for concluding that she was suffering
overseas anymore because she cannot stand from psychological incapacity to discharge the
living alone. He could not agree as in fact, he essential marital obligations.
had signed another three year contract. When
he returned in 1989, he could not locate Mere intention to live apart does not fall
Marina anymore. In 1992, Jose filed an action under Art. 36, FC. Furthermore, there is no
served by publication in a newspaper of proof that the alleged psychological
general circulation. Marina did not file any incapacity existed at the time of the
answer, a possible collusion between the marriage.
parties was ruled out by the Public Prosecutor.
Page 30 of 119
CIVIL LAW Answers to the BAR as Arranged by Topics ground of psychological incapacity. She alleged
that after 2 months of their marriage, Arnell
showed signs of disinterest in her, neglected her
and went abroad. He returned to the Philippines
after 3 years but did not even get in touch with
her. Worse, they met several times in social
functions but he snubbed her. When she got sick,
he did not visit her even if he knew of her
confinement in the hospital. Meanwhile, Arnell
met an accident which disabled him from
reporting for work and earning a living to
support himself. Will Gemma's suit prosper?
Explain. (5%)

SUGGESTED ANSWER:
No, Gemma's suit will not prosper. Even if taken as
true, the grounds, singly or collectively, do not
constitute "psychological incapacity." In Santos v.
CA, G.R. No. 112019, January 4, 1995, the Supreme
Court clearly explained that "psychological
incapacity must be characterized by (a) gravity, (b)
juridical antecedence, and (c) incurability"
(Ferraris v. Ferraris, G.R. No. 162368, July 17,
2006; Choa v. Choa, G.R. No. 143376, November
26, 2002). The illness must be shown as downright
incapacity or inability to perform one's marital
obligations, not a mere refusal, neglect, difficulty or
much less, ill will. Moreover, as ruled in Republic v.
Molina, GR No. 108763, February 13, 1997, it is
essential that the husband is capable of meeting his
marital responsibilities due to psychological and not
physical illness
(Antonio v. Reyes, G.R. No. 155800, March 10,
2006; Republic
Quintero-Hamano, G.R. No. 149498, May 20,
2004). Furthermore, the condition
complained of did not exist at the time of the
celebration of marriage.

Marriage; Psychological Incapacity (2006)


Article 36 of the Family Code provides that a marriage
contracted by any party who, at the time of the celebration,
was psychologically incapacitated to comply with the
essential marital obligations of marriage, shall be void.
Choose the spouse listed below who is psychologically
incapacitated. Explain. (2.5%) a) Nagger b) Gay or Lesbian
c)
Congenital sexual pervert d) Gambler e) Alcoholic
SUGGESTED ANSWER: The best answers are B and C. To be
sure, the existence and concealment of these conditions
at the inception of marriage renders the marriage
contract voidable (Art. 46, Family Code). They may serve
as indicia of psychological incapacity, depending on the
(Chi Ming
degree and severity of the disorder (Santos v. CA, G.R.
No. 112019, Jan. 4, 1995). Hence, if the condition of
homosexuality, lesbianism or sexual perversion, existing
at the inception of the marriage, is of such a degree as to
prevent any form of sexual intimacy, any of them may
qualify as a ground for psychological incapacity. The law
provides that the husband and wife are obliged to live
together, observe mutual love, respect and fidelity (Art.
68, Family Code). The mandate is actually the
Marriage; Psychological Incapacity (2006) spontaneous, mutual affection between the spouses. In
Gemma filed a petition for the declaration of the natural order it is sexual intimacy which brings the
nullity of her marriage with Arnell on the spouses wholeness and oneness
(Year 1990-2006) plane landed in Libya Irma gave birth.
ALTERNATIVE ANSWER: However, the baby died a few minutes after
None of them are necessarily psychologically complete delivery. Back in the Philippines
incapacitated. Being a nagger, etc. are at best Irma immediately filed a claim for inheritance.
only physical manifestations indicative of The parents of Isidro opposed her claim
psychological incapacity. More than just contending that the marriage between her and
showing the manifestations of incapacity, the Isidro was void ab initio on the following
petitioner must show that the respondent is grounds: (a) they had not given their consent
incapacitated to comply with the essential to the marriage of their son; (b) there was no
marital obligations of marriage and that it is marriage license; (c) the solemnizing officer
also essential that he must be shown to be had no authority to perform the marriage;
incapable of doing so due to some and, (d) the solemnizing officer did not file an
psychological, not physical illness affidavit of marriage with the proper civil
(Republic v. Quintero-Hamano, G.R. No. registrar.
149498, May 20, 2004).
ALTERNATIVE ANSWER:
A congenital sexual pervert may be Resolve each of the contentions ([a] to [d])
psychologically incapaci-tated if his
raised by the parents of Isidro. Discuss fully.
perversion incapacitates him from SUGGESTED ANSWER:
discharging his marital obligations. For (a) The fact that the parents of Isidro and of
instance, if his perversion is of such a nature Irma did not give their consent to the
as to preclude any normal sexual activity marriage did not make the marriage void ab
with his spouse. initio. The marriage is merely voidable
Marriage; Requisites (1995) under Art 45 of the FC.
Isidro and Irma, Filipinos, both 18 years of Absence of marriage license did not make
age, were passengers of Flight No. 317 of the marriage void ab initio. Since the
Oriental Airlines. The plane they boarded marriage was solemnized in articulo mortis,
was of Philippine registry. While en route it was exempt from the license requirement
from Manila to Greece some passengers under Art. 31 of the FC.
hijacked the plane, held the chief pilot
hostage at the cockpit and ordered him to fly On the assumption that the assistant pilot
instead to Libya. During the hijacking Isidro was acting for and in behalf of the airplane
suffered a heart attack and was on the verge chief who was under disability, and by reason
of death. Since Irma was already eight of the extraordinary and exceptional
months pregnant by Isidro, she pleaded to circumstances of the case [ie. hostage
the hijackers to allow the assistant pilot to situation), the marriage was solemnized by an
solemnize her marriage with Isidro. Soon authorized officer under Art. 7 (3) and Art.
after the marriage, Isidro expired. As the 31. of the FC.

v. CA, G.R. No. 119190, January 16,1997).


CIVIL LAW Answers to the BAR as Arranged by Topics enumerated therein, hence, it is void even
Failure of the solemnizing officer to file the though valid in Spain where it was celebrated.
affidavit of marriage did not affect the validity
of the marriage. It is merely an irregularity By reason of Art. 15 in relation to Article 38 of
which may subject the solemnizing officer to the Civil Code, which applies to Filipinos
sanctions. wherever they are, the marriage is void.
ALTERNATIVE ANSWER: SUGGESTED ANSWER:
Considering that the solemnizing officer has no It depends. If the marriage before the notary
authority to perform the marriage because public is valid under Hongkong Law, the
under Art. 7 the law authorizes only the marriage is valid in the Philippines. Otherwise,
airplane chief, the marriage is void, hence, a, the marriage that is invalid in Hongkong will be
c, and d are immaterial. invalid in the Philippines.

Marriage; Requisites (1999)


What is the status of the following marriages
and why?
A marriage between two 19-year olds
without parental consent, (2%)
A marriage between two 21-year olds
without parental advice. (2%)
A marriage between two Filipino first
cousins in Spain where such marriage is
valid. (2%)
A marriage between two Filipinos in
Hongkong before a notary public. (2%)
A marriage solemnized by a town mayor three
towns away from his jurisdiction, (2%)
SUGGESTED ANSWER:
The marriage is voidable. The consent of the
parties to the marriage was defective. Being
below 21 years old, the consent of the parties
is not full without the consent of their parents.
The consent of the parents of the parties to
the marriage is indispensable for its validity.
SUGGESTED ANSWER:
(b) Between 21-year olds, the marriage is valid
despite the absence of parental advice,
because such absence is merely an irregularity
affecting a formal requisite i.e., the marriage
license and does not affect the validity of the
marriage itself. This is without prejudice to the
civil, criminal, or administrative liability of the
party responsible therefor.

SUGGESTED ANSWER:
By reason of public policy, the marriage
between Filipino first cousins is void [Art. 38,
par. (1), Family Code], and the fact that it is
considered a valid marriage in a foreign
country in this case, Spain— does not validate
it, being an exception to the general rule in
Art. 96 of said Code which accords validity to
all marriage solemnized outside the Philippine
x x x and valid there as such.
ALTERNATIVE ANSWER
The marriage it void. Under Article 96 of the
Family Code, a marriage valid where
celebrated is valid in the Philippines except
those marriages enumerated in said Article
which marriages will remain void even though
valid where solemnized. The marriage between
first cousins is one of those marriages
(Year 1990-2006) The marriage is valid. The irregularity in the
ALTERNATIVE ANSWER: issuance of a valid license does not adversely
If the two Filipinos believed in good faith affect the validity of the marriage. The
that the Notary Public is authorized to marriage license is valid because it was in fact
solemnize marriage, then the marriage is issued by a Civil Registrar (Arts. 3 and 4. FC).
valid. ALTERNATIVE ANSWER:
It depends. If both or one of the parties was a member of
SUGGESTED ANSWER:
the religious sect of the solemnizing officer, the marriage is
Under the Local Government Code, a town valid. If none of the parties is a member of the sect and
mayor may validly solemnize a marriage but both of them were aware of the fact, the marriage is void.
said law is silent as to the territorial limits
They cannot claim good faith in believing that the
for the exercise by a town mayor of such
solemnizing officer was authorized because the scope of
authority. However, by analogy, with the
the authority of the solemnizing officer is a matter of law.
authority of members of the Judiciary to
solemnize a marriage, it would seem that the If, however, one of the parties believed in good faith that
mayor did not have the requisite authority to the other was a member of the sect, then the marriage is
solemnize a marriage outside of his valid
territorial jurisdiction. Hence, the marriage
is void, unless it was contracted with either
or both parties believing in good faith that
the mayor had the legal authority to
solemnize this particular marriage (Art 35,
par 2 Family Code).
ALTERNATIVE ANSWER:
The marriage is valid. Under the Local
Government Code, the authority of a mayor
to solemnize marriages is not restricted
within his municipality implying that he has
the authority even outside the territory
thereof. Hence, the marriage he solemnized
outside his municipality is valid. And even
assuming that his authority is restricted
within his municipality, such marriage will
nevertheless, be valid because solemnizing
the marriage outside said municipality is a
mere irregularity applying by analogy the
case of Navarro v Domagtoy, 259 Scra 129. In
this case, the Supreme Court held that the
celebration by a judge of a marriage outside
the jurisdiction of his court is a mere
irregularity that did not affect the validity of
the marriage notwithstanding Article 7 of the
Family Code which provides that an
incumbent member of the judiciary is
authorized to solemnize marriages only
within the court’s jurisdiction.

Marriage; Requisites; Marriage License (1996)


On Valentine's Day 1996, Ellas and Fely,
both single and 25 years of age, went to the
city hall where they sought out a fixer to
help them obtain a quickie marriage. For a
fee, the fixer produced an ante-dated
marriage license for them, Issued by the
Civil Registrar of a small remote
municipality. He then brought them to a
licensed minister in a restaurant behind
the city hall, and the latter solemnized
their marriage right there and then. 1) Is
their marriage valid, void or voidable?
Explain.
SUGGESTED ANSWER:
CIVIL LAW Answers to the BAR as Arranged by Topics (Year 1990-2006)
under Article 35 (2), FC. In that case, the party This is different from the case of Nināl V.
in good faith Bayadog, (328
is acting under a mistake of fact, not a mistake SCRA 122 [2000]). In the said case, the
of law, situation occurred
during the Relations of the new Civil Code
where Article 76
2) Would your answer be the same if it should thereof clearly provides that during the five-
turn out that year cohabitation,
the parties must be unmarried. This is not so
the marriage license was spurious? Explain. anymore in the
Family Code. The Change in the Family Code is
SUGGESTED ANSWER: significant. If
No, the answer would not be the same. The the second marriage occurred before the
marriage would effectivity of the
be void because of the absence of a formal Family Code, the answer would that be that the
requisite. In such marriage is
a case, there was actually no valid marriage voi
license. d.
B. Does Sotero have the personality to seek the
Marriage; Requisites; Marriage License (2002) declaration
On May 1, 1978 Facundo married Petra, by of nullity of the marriage, especially now that
whom he had a Facundo is
son Sotero. Petra died on July 1, 1996, while
Facundo died on already deceased? Explain. (3%)
January 1, 2002. Before his demise, Facundo
had married, on SUGGESTED ANSWER:
July 1, 2002, Quercia. Having lived together as B. A void marriage may be questioned by any
husband and interested party
wife since July 1, 1990, Facundo and Quercia in any proceeding where the resolution of the
did not secure a issue is material.
marriage license but executed the requisite Being a compulsory heir, Soterro has the
affidavit for the personality to
purpose. To ensure that his inheritance rights question the validity of the marriage of Facundo
are not and Quercia.
adversely affected by his father second Otherwise, his participation in the estate on
marriage, Sotero now Facundo would
brings a suit to seek a declaration of the nullity be (Ninãl V. Bayadog, 328 SCRA 122
of the affected. [2000] ).
marriage of Facundo and Quercia, grounded on
the absence
of a valid marriage license. Quercia contends
that there was
no need for a marriage license in view for her
having lived Marriage; Requisites; Solemnizing Officers (1994)
continuously with Facundo for five years before 1} The complete publication of the Family Code
their was made
marriage and that has Sotero has no legal on August 4, 1987. On September 4, 1987,
personality to seek a Junior Cruz and
declaration of nullity of the marriage since Gemma Reyes were married before a municipal
Facundo is now mayor. Was
the marriage valid? 2) Suppose the couple got
deceased. married on
September 1, 1994 at the Manila Hotel before
the Philippine
A. Is the marriage of Facundo and Quercia Consul General to Hongkong, who was on
valid, despite the vacation in
Manila. The couple executed an affidavit
absence of a marriage license? Explain. (2%) consenting to the
celebration of the marriage at the Manila Hotel.
SUGGESTED ANSWER: Is the
A. The marriage with Quercia is void. The marriage
exemption from valid?
the requirement of a marriage license under SUGGESTED ANSWER:
Art, 34, Family
Code, requires that the man and woman must a) Yes, the marriage is valid. The Family
have lived 1) Code took
together as husband and wife for at least five effect on August 3, 1988. At the time of the
years and marriage on
without any legal impediment to marry each September 4, 1987, municipal mayors were
other during empowered to
those five years. The cohabitation of Facundo solemnize marriage under the Civil Code of
and Quercia 1950.
for six years from 1990 to July 1, 1996 when
Petra died was
one with a legal impediment hence, not in a) The marriage is not valid. Consuls and
compliance with 2) vice-consuls
the requirement of law. On other hand, the are empowered to solemnize marriages
cohabitation between Philippine
thereafter until the marriage on July 1, 2000, citizens abroad in the consular office of the
although free foreign country to
from legal impediment, did not meet the 5-year which they were assigned and have no power to
cohabitation solemnize
requirement. marriage on Philippine soil.
ALTERNATIVE ANSWER:
The marriage of Facundo and Quercia is VALID. A Philippine consul is authorized by law to
The second marriage was solemnized on July 1, solemnize marriages abroad between Filipino
2000, when the Family code was already citizens. He has no authority to solemnize a
affective. The family code took effect on August marriage in the Philippines. Consequently, the
3, 1988. Under the Family Code, no marriage marriage in question is void, unless either or
license is required if the parties have been both of the contracting parties believed in
cohabiting for the period of five years and good faith that the consul general had
there is no legal impediment. There must no authority to solemnize their marriage in which
legal impediment ONLY AT THE TIME OF THE case the marriage is valid.
SOLEMNIZATION OF THE MARRIAGE, and not
the whole five years period. This is clearly the Marriage; Requisites; Void Marriage (1993)
intent of the code framers (see Minutes of the A and B, both 18 years old, were sweethearts
150th joint Civil Code of the Family Law studying in Manila. On August 3, 1988, while
Committees held on August 9, 1986). Also, in in first year college, they eloped. They stayed
Manzano V. Sanchez, AM NO. MT –00-129, in the house of a mutual friend in town X,
March 8, 2001, the Supreme Court said that, as where they were able to obtain a marriage
one of the requisites for the exception to apply, license. On August 30, 1988, their marriage
there must be no legal impediment at the time was solemnized by the town mayor of X in his
of the marriage. The Supreme Court did not office. Thereafter, they returned to Manila and
say that the legal impediment must exist all continued to live separately in their respective
throughout the five-year period. boarding houses, concealing from their
parents, who were living in the province what
they had done. In 1992, after graduation
CIVIL LAW Answers to the BAR as Arranged by Topics (Year 1990-2006)
from college, A and B decided to break their Philippine Law, his marriage to Anne is void
relation and because of a
parted ways. Both went home to their
respective towns to prior existing marriage which was not dissolved by the divorce
live and work. 1) Was the marriage of A and B
solemnized decreed in Oslo. Divorce obtained abroad by a Filipino is not
on August 30, 1988 by the town mayor of X in recognized.
his office a
valid marriage? Explain your Can either or
answer. 2) both
of them contract marriage with another person If Boni was no longer a Filipino citizen, the
without divorce is valid.
committing bigamy? Explain your Hence, his marriage to Anne is valid if
answer. celebrated in
accordance with the law of the place where it
was celebrated.
Since the marriage was celebrated aboard a
vessel of
Norwegian registry, Norwegian law applies. If
SUGGESTED ANSWER: the Ship
1) The marriage of A and B is void because the Captain has authority to solemnize the
solemnizing marriage aboard his
officer had no legal authority to solemnize the ship, the marriage is valid and shall be
marriage. But recognized in the
if either or both parties believed in good
faith that the Philippines.
solemnizing officer had the legal authority
to do so, the
marriage is voidable because the marriage As to the second question, if Boni is still a
between the Filipino, Anne can
parties, both below 21 years of age, was file an action for declaration of nullity of her
solemnized without marriage to
the consent of the parents. (Art. 35, par. (2) and
Art. 45 par. him.
(1), Family Code)
Marriage; Void Marriages (2006)
2) Either or both of the parties cannot contract Gigi and Ric, Catholics, got married when they
marriage in were 18 years
the Philippines with another person without old. Their marriage was solemnized on August
committing 2, 1989 by
bigamy, unless there is compliance with the Ric's uncle, a Baptist Minister, in Calamba,
requirements of Laguna. He
Article 52 Family Code, namely: there must be a overlooked the fact that his license to
judgment of solemnize marriage
annulment or absolute nullity of the marriage, expired the month before and that the parties
partition and do not belong
distribution of the properties of the spouses and to his congregation. After 5 years of married
the delivery life and blessed
of their children's presumptive legitimes, with 2 children, the spouses developed
which shall be irreconcilable
recorded in the appropriate Civil Registry and differences, so they parted ways. While
Registry of separated, Ric fell in
Property, otherwise the same shall not affect love with Juliet, a 16 year-old sophomore in a
third persons local college
and the subsequent marriage shall be null and and a Seventh-Day Adventist. They decided to
void. (Arts. 52 get married
with the consent of Juliet's parents. She
and 53. Family Code) presented to him a
birth certificate showing she is 18 years old.
ALTERNATIVE ANSWER: Ric never
2) Yes, they can. The subsequent marriage contracted
by one doubted her age much less the authenticity of
her birth
of the parties will not give rise to bigamy even
in the absence certificate. They got married in a Catholic
of a court declaration of nullity of the first church in Manila. A
marriage. The year after, Juliet gave birth to twins, Aissa and
subsistence of a prior valid marriage is an Aretha.
indispensable
element of the crime of bigamy. The prior court
declaration (1) What is the status of the marriage
of nullity of the first marriage is required by the between Gigi and
Family Code Ric — valid, voidable or void? Explain.
only the purpose of the subseque (2.5%)
for validity of the nt SUGGESTED ANSWER: Even if the Minister's license
expired, the
marriage, not as an element of the crime of marriage is valid if either or both Gigi and Ric
bigamy. believed in good
faith that he had the legal authority to solemnize
marriage. While
the authority of the solemnizing officer is a
formal requisite of
Marriage; Void Marriages (2004) marriage, and at least one of the parties must
A. BONI and ANNE met while working belong to the
overseas. They solemnizing officer's church, the law provides
became sweethearts and got engaged to be that the good faith
married on New of the parties cures the defect in the lack of
Year’s Eve aboard a cruise ship in the authority of the
Caribbean. They took
the proper license to marry in New York City, solemnizing officer
where there is (Art. 35 par. 2, Family Code; Sempio-Diy, p. 34;
a Filipino consulate. But as planned the Rabuya, The
wedding ceremony
was officiated by the captain of the Law on Persons and Family Relations, p. 208).
Norwegian-registered
vessel in a private suite among The absence of parental consent despite their
selected friends. having married
at the age of 18 is deemed cured by their
continued
Back in Manila, Anne discovered that Boni had cohabitation beyond the age of 21. At this
been married point, their
in Bacolod City 5 years earlier but divorced in
Oslo only last marriage is valid (See Art. 45, Family Code).
year. His first wife was also a Filipina but now
based in
Swede Boni himself is a resident of Norway (2) What is the status of the marriage
n. where he and between Ric and
Anne plan to live permanently. Juliet — valid, voidable or void? (2.5%)
SUGGESTED ANSWER: The marriage between Juliet
and Ric is
Anne retains your services to advise her on void. First of all, the marriage is a bigamous
whether her marriage not
marria to Boni is valid under falling under Article 41 [Art. 35(4)Family
ge Philippine law? Is there Code], A subsisting
anything else she should do under the marriage constitutes a legal impediment to
circumstances? (5%) remarriage.
Secondly, Juliet is below eighteen years of age.
SUGGESTED ANSWER: The marriage is
If Boni is still a Filipino citizen, his legal capacity is governed by
Philippine Law (Art. 15 Civil Code). Under void even if consented to by her parents
CIVIL LAW Answers to the BAR as Arranged by Topics (Year 1990-2006)
[Art. 35(1), Family Code]. The fact that Ric was not aware Under Article 213 of the Family
Code, no child under 7
of her real age is immaterial. years of age shall be separated from the mother
unless the
court finds compelling reasons to order
otherwise.
Suppose Ric himself procured the falsified trust and commitment. It must be
birth certificate to persuade Juliet to marry characterized by Juridical antecedence, gravity
him despite her minority and assured her and incurability and its root causes must be
that everything is in order. He did not clinically identified or examined. (Santos v. CA,
divulge to her his prior marriage with Gigi. 240 SCRA 20 [1995]).
What action, if any, can Juliet take against
B. In the case of Santos v. Court of Appeals, 240
him? Explain.
(2.5%) SCRA 20 (1995), the Supreme Court held that
SUGGESTED ANSWER: Juliet can file an action for being of unsound mind, drug addiction,
the declaration of nullity of the marriage on habitual alcoholism, lesbianism or
the ground that he willfully caused loss or homosexuality may be indicia of psychological
injury to her in a manner that is contrary to incapacity, depending on the degree of severity
morals, good customs and public policy [Art. of the disorder. However, the concealment of
21, New Civil Code]. She may also bring drug addiction, habitual alcoholism, lesbianism
criminal actions for seduction, falsification, or homosexuality is a ground of annulment of
illegal marriage and bigamy against Ric. marriage.

If you were the counsel for Gigi, what Parental Authority; Child under 7 years of age (2006)
action/s will you take to enforce and
protect her interests? Explain.
(2.5%)
SUGGESTED ANSWER: I would file an action to
declare the marriage between Juliet and Ric
null and void ab initio and for Ric's share in
the co-ownership of that marriage to be
forfeited in favor and considered part of the
absolute community in the marriage between
Gigi and Ric [Arts. 148
147, Family Code]. I would also file an action
for damages against Ric on the grounds that
his acts constitute an abuse of right and they
are contrary to law and morals, causing
damages to Gigi (See Arts 19, 20, 21, New
Civil Code).

Marriage; Void Marriages; Psychological Incapacity (2002)


A. Give a brief definition or explanation of the
term
―psychological incapacity‖ as a ground for the
declaration of nullity of a marriage. (2%)
B. If existing at the inception of marriage,
would the state of being of unsound mind or
the concealment of drug addiction, habitual
alcoholism, homosexuality or lesbianism be
considered indicia of psychological
incapacity? Explain. (2%).
SUGGESTED ANSWER:
A.‖ PSYCHOLOGICAL INCAPACITY‖ is a mental
disorder of the most serious type showing the
incapability of one or both spouses to comply
the essential marital obligations of love,
respect, cohabitation, mutual help and support,
Explain the rationale of this provision. corridor during the class period when the
(2.5%) stabbing incident occurred. The incident could
SUGGESTED ANSWER: have been prevented had the teacher been
The rationale of the 2nd paragraph of Article 213 inside the classroom at that time. The guilty boy’s
of the Family Code is to avoid the tragedy of a PARENTS are subsidiarily liable under Article
mother who sees her baby torn away from her. It 219 of the Family Code.
is said that the maternal affection and care during
the early years of the child are generally needed
by the child more than paternal care Parental Authority; Substitute vs. Special (2004)
(Hontiveros v. IAC, G.R. No. 64982, October 23,
1984; Tolentino, Commentaries and
Jurisprudence on the Civil Code, Volume One,
pp. 718-719). The general rule is that a child
below 7 years old shall not be separated from
his mother due to his basic need for her loving
care (Espiritu v. C.A., G.R. No. 115640, March
15,1995).
Give at least 3 examples of "compelling
reasons" which justify the taking away
from the mother's custody of her child
under 7 years of age. (2.5%)
SUGGESTED ANSWER:
The mother is insane (Sempio-Diy,
Handbook on the Family Code of the
Philippines, pp. 296-297);
The mother is sick with a disease that is
com-municable and might endanger the
health and life of the child;
The mother has been maltreating the child;
The mother is engaged in prostitution;
The mother is engaged in
adulterous relationship;
The mother is a drug addict;
The mother is a habitual drunk or
an alcoholic;
Parental• Authority;ThemotherSpecialis inParentaljailor
servingAuthority;sentenceLiability. of

Teachers (2003)
If during class hours, while the teacher was
chatting with other teachers in the school
corridor, a 7 year old male pupil stabs the eye of
another boy with a ball pen during a fight,
causing permanent blindness to the victim, who
could be liable for damages for the boy’s injury: the
teacher, the school authorities, or the guilty boy’s
parents? Explain.
SUGGESTED ANSWER:
The school, its administrators, and teachers
have special parental authority and
responsibility over the minor child while under
their supervision, instruction or custody
(Article 218, FC). They are principally and
solidarily liable for the damages caused by the
acts or omissions of the unemancipated minor
unless they exercised the proper diligence
required under the circumstances (Article 219,
FC). In the problem, the TEACHER and the
SCHOOL AUTHORITIES are liable for the
blindness of the victim, because the student
who cause it was under their special parental
authority and they were negligent. They were
negligent because they were chatting in the
CIVIL LAW Answers to the BAR as Arranged by Topics (Year 1990-2006)
Distinguish briefly but clearly between: sperm. After a series of test, Andy's sperm was
Substitute parental medically
authority and special parental authority. introduced into Beth's ovary. She became pregnant and 9 months
SUGGESTED ANSWER: later, gave birth to a baby boy, named Alvin.
In substitute parental authority, the parents lose
their parental (1) Who is the Father of Alvin? Explain.
authority in favor of the substitute who acquires (2.5%)
it to the SUGGESTED ANSWER:
Andy is the biological father of Alvin being the
exclusion of the parents. source of the
sperm. Andy is the legal father of Alvin
because there was
In special parental authority, the parents or neither consent nor ratification to the artificial
anyone exercising insemination.
parental authority does not lose parental Under the law, children conceived by artificial
authority. Those who insemination
are charged with special parental authority are legitimate children of the spouses,
exercise such provided, that both of
authority only during the time that the child is in them authorized or ratified the insemination
their custody in a written
instrument executed and signed by both of
or supervision. them before the
Substitute parental authority displaces parental birth of the child (Art. 164, Family Code).
authority while
special parental authority concurs with parental
authority. (2) What are the requirements, if any, in
order for Ed to
establish his paternity over Alvin. (2.5%)
Paternity & Filiation (1999) SUGGESTED ANSWER:
(a) Two (2) months after the death of her The following are the requirements for Ed to
husband who was establish his
shot by unknown criminal elements on his way
home from paternity over Alvin:
office, Rose married her childhood boyfriend, The artificial insemination has been
and seven (7) • authorized or
months after said marriage, she delivered a ratified by the spouses in a written instrument
baby. In the executed and
absence of any evidence from Rose as to who is signed by them before the birth of the child;
her child's and
father, what status does the law give to said The written instrument is recorded in
child? Explain. • the civil
registry together with the birth certificate of
(2%) the child (Art.
SUGGESTED ANSWER: 164, 2nd paragraph, Family Code).
(a) The child is legitimate of the second
marriage under
Article 168(2) of the Family Code which
provides that a
"child born after one hundred eighty days
following the Paternity & Filiation; Common-Law Union (2004)
celebration of the subsequent marriage is A. RN and DM, without any impediment to
considered to have marry each
been conceived during such marriage, even other, had been living together without
though it be born benefit of church
within three hundred days after the termination blessings. Their common-law union resulted in
of the former the birth of
ZMN. Two years later, they got married in a
marriage." civil ceremony.
Could ZMN be legitimated? Reason. (5%)
Paternity & Filiation; Proofs (1999) SUGGESTED ANSWER:
(b) Nestor is the illegitimate son of Dr. Perez. ZMN was legitimated by the subsequent
When Dr. marriage of RN and
Perez died, Nestor intervened in the settlement DM because at the time he was conceived, RN
of his father's and DM
estate, claiming that he is the illegitimate son of coul have validly married each other. Under
said deceased, d the Family
but the legitimate family of Dr. Perez is denying Code children conceived and born outside of
Nestor's wedlock of
claim. What evidence or evidences should parents who, at the time of the former's
Nestor present so conception, were not
that he may receive his rightful share in his disqualified by any impediment to marry
father's estate? each other are
legitimated by the subsequent marriage of the
(3%) parents.
SUGGESTED ANSWER:
To be able to inherit, the illegitimate Paternity & Filiation; Proofs; Limitations; Adopted Child
filiation of Nestor must have been admitted (1995)
by his father in any of the following: Abraham died intestate on 7 January 1994
the record of birth appearing in the civil survived by his son Braulio. Abraham's older
register, son Carlos died on 14 February 1990. Danilo
a final judgment, who claims to be an adulterous child of Carlos
a public document signed by the father, or intervenes in the proceedings for the
a private handwritten document signed by the settlement of the estate of Abraham in
lather (Article 17S in relation to Article 172 of representation of Carlos. Danilo was legally
the Family Code). adopted on 17 March 1970 by Carlos with the
consent of the " latter's wife.
Paternity & Filiation; Artificial Insemination; 1. Under the Family Code, how may an
Formalities(2006) illegitimate filiation be proved? Explain.
Ed and Beth have been married for 20 years without children. As lawyer for Danilo, do you have to
Desirous to have a baby, they consulted Dr. Jun Canlas, a , prove Danilo's illegitimate filiation?
prominent medical specialist on human fertility. He advised Beth Explain.
to undergo artificial insemination. It was found that Ed’s sperm Can Danilo inherit from Abraham in
count was inadequate to induce pregnancy Hence, the couple representation of his father Carlos? Explain.
looked for a willing donor. Andy the brother of Ed, readily SUGGESTED ANSWER:
consented to donate his Under Art. 172 in relation to Art. 173 andArt. 175 of the
FC, the filiation of illegitimate children may be established
CIVIL LAW Answers to the BAR as Arranged by Topics (Year 1990-2006)
in the same way and by the same evidence recognition of an illegitimate child can be
as legitimate brought at any
children. Art. 172 provides that the filiation
of legitimate time during the lifetime of the child. However, if the action is based
children is established by any of the following:
(1) the record on "open and continuous possession of the status of an illegitimate
child, the same can be filed during the lifetime of the putative
of birth appearing in the civil register or a final
Judgment; or
(2) an admission of legitimate filiation in a father."
public document
or a private handwritten instrument and signed
by the parent
concerned. In the absence of the foregoing In the present case, the action for compulsory
evidence, the recognition was
legitimate filiation shall be proved by: (1) filed by Joey's mother, Dina, on May 16,1994,
the open and after the death
continuous possession of the status of a of Steve, the putative father. The action will
legitimate child; or prosper if Joey
(2) any other means allowed by the Rules can present his birth certificate that bears the
of Court and signature of his
special putative father. However, the facts clearly state
laws. that the birth
certificate of Joey did not indicate the father's
SUGGESTED ANSWER: name. A birth
2. No. Since Danilo has already been adopted certificate not signed by the alleged father
by Carlos, he cannot be taken as a
ceased to be an illegitimate child. An adopted record of birth to prove recognition of the
child acquires child, nor can said
all the rights of a legitimate child under Art, birth certificate be taken as a recognition in a
189 of the FC. public
instrument. (Reyes v. Court of Appeals, G.R. No.
39537, March
19, 1985) Consequently, the action filed by
SUGGESTED ANSWER: Joey's mother has
3. No, he cannot. Danilo cannot represent
Carlos as the
latter's adopted child in the inheritance of already prescribed.
Abraham because
adopti not Danilo a legitimate
on did make grandchild of b) Are the defenses set up by Tintin
Abraham. Adoption is personal between Carlos tenable? Explain.
and Danilo.
cannot represent th latte illegitima (2%)
He also Carlos as e r's te SUGGESTED ANSWER:
child because in such case he is barred by Art. Yes, the defenses of Tintin are tenable. In
992 of the Tayag v. Court of
NCC from inheriting from his illegitimate Appeals (G.R. No. 95229, June 9,1992), a
grandfather complaint to compel
recognition of an illegitimate child was brought
Abraham. before
effectivity of the Family Code by the mother of
ALTERNATIVE ANSWER: a minor child
An adopted child's successional rights do not
include the right based on "open and continuous possession of
to represent his deceased adopter in the the status of an
inheritance of the illegitimate child." The Supreme Court held
latter's legitimate parent, in view of Art. 973 that the right of
which provides action of the minor child has been vested by
th orde tha representat tak place the filing of the
at in r t ion may e , the complaint in court under the regime of the
representa mus himself be o succeedi Civil Code and
tive t capable f ng the
prior to the effectivity of the Family Code. The
decedent. Adoption by itself did not render ruling in Tayag
Danilo an heir of v. Court of Appeals finds no application in the
th adopte legitim parent. his being instant case.
e r's ate Neither does a Although the child was born before the
effectivity of the
grandchild Abraha render heir of the Family Code, the complaint was filed after its
of m him an latter effectivity.
because as an illegitimate child of Carlos, who
was a legitimate Hence, Article 175 of the Family Code should
child of Abraham, Danilo is incapable of apply and not
succeeding Abraham
Article 285 of the Civil Code.
under Art. 992 of the Code.
Supposing that Joey died during the
Paternity & Filiation; Recognition of illegitimate Child
(2005) Steve was married to Linda, with whom pendency of the action, should the action
he had a daughter, Tintin. Steve fathered a be dismissed? Explain. (2%)
son with Dina, his secretary of 20 years, whom
SUGGESTED ANSWER:
Dina named Joey, born on September 20,
If Joey died during the pendency of the action,
1981. Joey's birth certificate did not indicate
the action should still be dismissed because
the father's name. Steve died on August 13,
the right of Joey or his heirs to file the action
1993, while Linda died on December 3, 1993,
has already prescribed. (Art. 175, Family
leaving their legitimate daughter, Tintin, as
Code)
sole heir. On May 16, 1994, Dina filed a case
on behalf of Joey, praying that the latter be Paternity & Filiation; Rights of Legitimate Children (1990)
declared an acknowledged illegitimate son of B and G (college students, both single and not
Steve and that Joey be given his share in disqualified to marry each other) had a
Steve's estate, which is now being solely held romantic affair, G was seven months in the
by Tintin. Tintin put up the defense that an family way as of the graduation of B. Right
action for recognition shall only be filed after graduation B went home to Cebu City.
during the lifetime of the presumed parents Unknown to G, B had a commitment to C (his
and that the exceptions under Article 285 of childhood sweetheart) to marry her after
the Civil Code do not apply to him since the getting his college degree. Two weeks after B
said article has been repealed by the Family marriage in Cebu City, G gave birth to a son E
Code. In any case, according to Tintin, Joey's in Metro Manila. After ten years of married life
birth certificate does not show that Steve is in Cebu, B became a widower by the sudden
his father. death of C in a plane crash. Out of the union of
B and C, two children, X and Y were born.
Does Joey have a cause of action against Unknown to C while on weekend trips to
Tintin for recognition and partition? Manila during the last 5 years of their
Explain. (2%) marriage, B invariably visited G and lived at
SUGGESTED ANSWER:
No, Joey does not have a cause of action against Tintin for her residence and as a result of which, they
recognition and partition. Under Article 175 of the Family Code, as renewed their relationship. A baby girl F was
a general rule, an action for compulsory born to B and G two years
CIVIL LAW Answers to the BAR as Arranged by Topics (Year 1990-2006)
before the death of C. Bringing his family later Paulita left the conjugal home because of the
to Manila, B excessive
finally married G. Recently. G died. What are the drinking of her husband, Alberto. Paulita, out
rights of of her own
B's four children: X and Y of his first marriage; endeavor, was able to buy a parcel of land
and E and F, which she was able
to register under her name with the addendum
his children with G? Explain your answer. "widow." She
also acquired stocks in a listed corporation
registered in her
name. Paulita sold the parcel of land to Rafael,
SUGGESTED ANSWER: who first
Under the facts stated, X and Y are legitimate examined the original of the transfer
children of B certificate of title. 1) Has
and C. E is the legitimate children of B and G. E Alberto the right to share in the shares of
is the stock acquired by
legitimated child of B&G. F is the illegitimate Paulita? 2) Can Alberto recover the land from
child of B and Rafael?
C. As legitimate children of B and C, X and Y
have the
following rights: 1) To bear the surnames of the
father and
the mother, in SUGGESTED ANSWER:
conformity with the provisions of the Civil 1. a) Yes. The Family Code provides that all
Code on property acquired
Surnames; 2) To receive support from their during the marriage, whether the acquisition
parents, their appears to have
been made, contracted or registered in the
ascendants, name of one or
and in proper cases, their brothers and both spouses, is presumed to be absolute
sisters, in community
conformity with the provisions of the Family
Code on property unless the contrary is proved.
Support; and
To be entitled to the legitime and other b) Yes. The shares are presumed to be absolute
3) successional community
rights granted to them by the Civil Code. property having been acquired during the
(Article 174, Family marriage despite
Code the fact that those shares were registered only
). in her name.
Alberto's right to claim his share will only
E is the legitimated child of B and G. Under Art. arise, however, at
177 of the dissolution.
Family Code, only children conceived and born
outside of
wedlock of parents who, at the time of the c) The presumption is still that the shares
conception of the of stock are
former, were not disqualified by any impediment owned in common. Hence, they will form
to marry part of the
each other may be legitimated. E will have the absolute community or the conjugal
same rights partnership depending
as X and Y. on what the property Relations is.
F is the illegitimate child of B and G. F has the d) Since Paulita acquired the shares of stock
right to use by onerous title
the surname of G, her mother, and is entitled to during the marriage, these are part of the
support as conjugal or absolute
well as the legitime consisting of 1/2 of that of community property, as the case maybe
each of X, Y (depending on
and whether the marriage was celebrated prior to.
E. (Article 176, Family Code) or after, the
effectivity of the Family Code). Her physical
separation from
her husband did not dissolve the community of
Presumptive Legitime (1999) property.
What do you understand by "presumptive Hence, the husband has a right to share in the
legitime", in what shares of
case or cases must the parent deliver such
legitime to the stock.
children, and what are the legal effects in each
case if the
parent fails to do so? (5%) SUGGESTED ANSWER:
2) a) Under a community of property, whether
SUGGESTED ANSWER: absolute or
PRESUMPTIVE LEGITIME is not defined in the relative, the disposition of property
law. Its belonging to such
definition must have been taken from Act community is void if done by just one spouse
2710, the Old without the
Divorce Law, which required the delivery to the consent of the other or authority of the
legitimate proper court.
children of "the equivalent of what would have However, the land was registered in the name
been due to of Paulita as
them as their legal portion if said spouse had "widow". Hence, the buyer has the right to rely
died intestate upon what
immediately after the dissolution of the appears in the record of the Register of Deeds
community of and should,
property." As used in the Family Code, consequently, be protected. Alberto cannot
presumptive legitime recover the land
is understood as the equivalent of the legitimate from Rafael but would have the right of
children's recourse against his
legitimes assuming that the spouses had
died immediately wife
after the dissolution of the community of
property. b) The parcel of land is absolute community
property having
been acquired during the marriage and
through Paulita's
Presumptive legitime is required to be industry despite the registration being only in
delivered to the the name of
common children of the spouses when the Paulita. The land being community property,
marriage is its sale to Rafael
annulled or declared void ab initio and possibly, without the consent of Alberto is void.
when the However, since the
conjugal partnership or absolute community is land is registered in the name of Paulita as
dissolved as in widow, there is
the case of legal separation. Failure of the nothing in the title which would raise a
parents to deliver suspicion for Rafael
the presumptive legitime will make their to make inquiry. He, therefore, is an innocent
subsequent marriage purchaser for
null and void under Article 53 of the Family value from whom the land may no longer be
Code. recovered.
Property Relations; Absolute Community (1994)
CIVIL LAW Answers to the BAR as Arranged by Topics (Year 1990-2006)
c) No. Rafael is an innocent purchaser in good 1. Since Bob and Sofia got married In 1970,
faith who, then the law
upon relying on the correctness of the that governs is the New Civil Code (Persons), in
certificate of title, which case,
acquires rights which are to be protected by the the property relations that should be applied as
courts. regards the
property of the spouses is the system of relative
community
Under the established principles of land or conjugal partnership of gains (Article 119,
registration law, the Civil Code). By
presumption is that the transferee of registered conjugal partnership of gains, the husband and
land is not the wife place
aware of any defect in the title of the property in a common fund the fruits of their separate
he purchased. property and
(See Tojonera v. Court of Appeals, 103 SCRA the income from their work or Industry (Article
467). Moreover, 142, Civil
the person dealing with registered land may Code). In this instance, the lot inherited by Bob
safely rely on the in 1975 is his
correctness of its certificate of title and the law own separate property, he having acquired
will in no way the same by
oblige him to go behind the certificate to lucrative title (par. 2, Art. 148, Civil Code).
determine the However, the
condition of the property. [Director of Lands v. house constructed from his own savings in 1981
Abache, et al. during the
73 Phil. 606). No strong considerations of public subsistence of his marriage with Issa is
policy have conjugal property and
been presented which would lead the Court to not exclusive property in accordance with the
reverse the principle of
established and sound doctrine that the buyer in "reverse accession" provided for in Art. 158,
good faith of Civil Code.
a registered parcel of land does not have to look
beyond the
Torrens Title and search for any hidden defect
or inchoate ANOTHER ANSWER:
right which may later invalidate or diminish his Sofia, being her deceased son's legal heir
right to what 1. concurring
wit his surviving spouse 986 and 997,
he purchased. (Lopez v. Court of Appeals. 189 h (Arts. 985, Civil
SCRA 271)
Code), may rightfully claim that the house and
lot are not
conjugal but belong to the hereditary estate of
Bob. The value
d) The parcel of land is absolute community of the land being more than the cost of the
property having improvement
been acquired during the marriage and
through Paulita's (Art. 120, Family Code).
industry despite registration only in the name of
Paulita. The
land being community property, its sale to
Rafael without the SUGGESTED ANSWER:
2. Yes, the answer would still be the same.
consent of Alberto is void. Since Bob and Issa
contracted their marriage way back in 1970,
then the property
Property Relations; Ante Nuptial Agreement relations that will govern is still the relative
(1995) community or
Suppose Tirso and Tessie were married on 2 conjugal partnership of gains (Article 119, Civil
August 1988 Code). It will
without executing any ante nuptial not matter if Bob died before or after
agreement. One year August 3. 1988
after their marriage, Tirso while supervising the (effectivity date of the Family what matters is
clearing of Code], the
Tessie's inherited land upon the latter's request, date when the marriage was contracted. As
accidentally Bob and Issa
found the treasure not in the new river bed contracted their marriage way back in
but on the 1970. the property
property of Tessie. To whom shall the relation that governs them is still the conjugal
treasure belong? partnership of
Explain. gains. (Art. 158, Civil Code)
SUGGESTED ANSWER: ANOTHER ANSWER:
Since Tirso and Tessie were married before the 2. If Bob died be fore August 3, 1988. which is
effectivity of the date the
the Family Code, their property relation is Family Code took effect, the answer will not be
governed by the same.
conjugal partnership of gains. Under Art. 54 Art. 158. Civil Code, would then apply. The land
of the Civil would then
Code, the share of the hidden treasure which be deemed conjugal, along with the house,
the law awards since conjugal
to the finder or the proprietor belongs to funds were used in constructing it. The
the conjugal husband's estate
partnership of gains. The one-half share would be entitled to a reimbursement of the
pertaining to Tessie value of the land
as owner of the land, and the one-half share from conjugal partnership
pertaining to funds.
Tirso as finder of the belong th conjuga
Property Relations; Marriage Settlement; Conjugal
treasure, to e l
Partnership
partnership of gains.
of Gains (2005)
Gabby and Mila got married at Lourdes Church
Property Relations; Conjugal Partnership of Gains (1998) in Quezon
In 1970, Bob and Issa got married without City on July 10, 1990. Prior thereto, they
executing a executed a marriage
marriage settlement. In 1975, Bob inherited settlement whereby they agreed on the regime
from his father a of conjugal
residential lot upon 1981, constructed partnership of gains. The marriage settlement
which, in he a was registered
two-room bungalow with savings from his own in the Register of Deeds of Manila, where Mila
earnings. At is a resident.
that time, the lot was worth P800.000.00 while In 1992, they jointly acquired a residential
the house, house and lot, as
when finished cost P600,000.00. In 1989 Bob well as a condominium unit in Makati. In 1995,
died, survived they decided
only by his wife, Issa and his mother, Sofia. to change their property relations to the regime
Assuming that the of complete
relative values of both assets remained at separation of property. Mila consented, as
the same she was then
engaged in a lucrative business. The spouses
proportion: then signed a
1. State whether Sofia can rightfully claim that private document dissolving their conjugal
the house and partnership and
lot are not conjugal but exclusive property of
her deceased agreeing on a complete separation of property.
son. [3%]
Will your answer be the same if Bob died
before August 3, 1988? [2%]
SUGGESTED ANSWER:
CIVIL LAW Answers to the BAR as Arranged by Topics (Year 1990-2006)
Thereafter, Gabby acquired a mansion in Baguio Bar Candidates Patricio Mahigugmaon and
City, and a Rowena Amor
5-hectare agricultural land in Oriental Mindoro, decided to marry each other before the last
which he day of the 1991
registered exclusively in his name. In the year Bar Examinations. They agreed to execute a
2000, Mila's Marriage
business venture failed, and her creditors sued Settlement. Rowena herself prepared the
her for document in her
P10,000,000.00. After obtaining a favorable own handwriting. They agreed on the
judgment, the following: (1) a conjugal
creditors sought to execute on the spouses' partnership of gains; (2) each donates to the
house and lot and other fifty
condominium unit, as well as Gabby's mansion percent (50%) of his/her present property, (3)
and Rowena shall
administer the conjugal partnership property;
agricultural land. and (4) neither
a) Discuss the status of the first and may bring an action for the annulment or
the amended declaration of
nullity of their marriage. Both signed the
marriage settlements. (2%) agreement in the
presence of two (2) witnesses. They did not,
SUGGESTED ANSWER: however,
The marriage settlement between Gabby and
Mila adopting acknowledge it before a notary public.
the regime of conjugal partnership of gains still As to form, is the Marriage Settlement
subsists. It is A. valid? May it
not dissolved by the mere agreement of the be registered in the registry of property? If
spouses during not, what steps
the marriage. It is clear from Article 134 of the
Family Code must be taken to make it registerable?
that in the absence of an express declaration in
the marriage B. Are the stipulations valid?
settlement, the separation of property between If the Marriage Settlement is valid as to
the spouses C. form and
during the marriage shall not take place except the above stipulations are likewise valid, does
by judicial it now follow
that said Marriage Settlement is valid and
order. enforceable?
claiming against the properties. (Ayala
Discuss the effects of the said settlements Investment v. Court of Appeals,
on the properties acquired by the spouses. G.R. No. 118305, February 12,1998, reiterated
(2%) in
SUGGESTED ANSWER: Homeowners Savings & Loan Bank v. Dailo,
The regime of conjugal partnership of gains G.R. No. 153802, March 11, 2005)
governs the properties acquired by the spouses. ALTERNATIVE ANSWER:
All the properties acquired by the spouses after Except for the residential house which is the
the marriage belong to the conjugal partnership. family home, all other properties of Gabby
Under Article 116 of the Family Code, even if and Mila may be held answerable for Mila's
Gabby registered the mansion and 5 -hectare obligation. Since the said properties are
agricultural land exclusively in his name, still conjugal in nature, they can be held liable for
they are presumed to be conjugal properties, debts and obligations contracted during the
unless the contrary is proved. marriage to the extent that the family was
benefited or where the debts were contracted
by both spouses, or by one of them, with the
What properties may be held answerable consent of the other.
for Mila's obligations? Explain. (2%)
ALTERNATIVE ANSWER:
Since all the properties are conjugal, they can A family home is a dwelling place of a person
be held answerable for Mila's obligation if the and his family. It confers upon a family the
obligation redounded to the benefit of the right to enjoy such property, which must
family. (Art. 121 [3], Family Code) However, remain with the person constituting it as a
the burden of proof lies with the creditor family home and his heirs. It cannot be seized
by creditors except in special cases. (Taneo, Jr. SUGGESTED ANSWER:
v. Court of Appeals, G.R. No. 108532, Yes, it is valid as to form because it is in
March 9, 1999) writing. No, it cannot be registered in the
registry of property because it is not a
Property Relations; Marriage Settlements (1991) public document. To make it registerable, it
must be reformed and has to be notarized.
SUGGESTED ANSWER:
B. Stipulations (1) and (3) are valid because
they are not contrary to law. Stipulation (4) is
void because it is contrary to law. Stipulation
(2) is valid up to 1/5 of their respective
present properties but void as to the excess
(Art 84, Family Code).
SUGGESTED ANSWER:
C. No. on September 15, 1991, the marriage
settlement is not yet valid and enforceable
until the celebration of the marriage, to take
place before the last day of the 1991 bar
Examinations.
Property Relations; Marriage Settlements (1995)
On 10 September 1988 Kevin, a 26-year old
businessman, married Karla, a winsome lass
of 18. Without the knowledge of their parents
or legal guardians, Kevin and Karla entered
into an ante-nuptial contract the day before
their marriage stipulating that conjugal
partnership of gains shall govern their
marriage. At the time of their marriage
Kevin's estate was worth 50 Million while
Karla's was valued at 2 Million. A month after
their marriage Kevin died in a freak
helicopter accident. He left no will, no debts,
no obligations. Surviving Kevin, aside from
Karla, are his only relatives: his brother Luis
and first cousin Lilia. 1) What property
Relations governed the marriage of

Kevin and Karla? Explain. 2) Determine the


value of the estate of Kevin, 3) Who are
Kevin's heirs? 4) How much is each of
Kevin's heirs entitled to

inherit?
SUGGESTED ANSWER:
Since the marriage settlement was entered
into without the consent and without the
participation of the parents (they did not sign
the document), the marriage settlement is
invalid applying Art. 78, F.C. which provides
that a minor
CIVIL LAW Answers to the BAR as Arranged by Topics (Year 1990-2006)
who according to law may contract marriage together, Rico was a salaried employee and
may also enter Mabel kept
into marriage settlements but they shall be house for Rico and did full-time household
valid only if the chores for him.
person who may give consent to the During their cohabitation, a parcel of coconut
marriage are made land was
parties to the agreement. (Karla was still a
minor at the time acquired by Rico from his savings.
the marriage settlement was executed in
September 1988
because the law, R.A. 6809, reducing the age of After living together for one (1) year, Rico and
majority to Mabel
18 years took effect on 18 December 1989). The separated. Rico then met and married Letty, a
marriage single woman
settlement being void, the property Relations twenty-six (26) years of age. During the
governing the marriage of Rico and
marriage is, therefore, absolute community Letty, Letty bought a mango orchard out of her
of property, own personal
earnings. a) Who would own the riceland, and
under Art. 75 of the FC. what
property Relations governs the ownership?
Explain. b) Who
2. All the properties which Kevin and Karla would own the coconut land, and what
owned at the property Relations
time of marriage became community property governs the ownership? Explain. c) Who
which shall be would own the
divided equally between them at dissolution. mango orchard, and what property Relations
Since Kevin governs the
owned 50 Million and Karla. 2 Million, at the
time of the ownership? Explain.
marriage, 52 Million constituted their
community property.
Upon the death of Kevin, the community was
dissolved and SUGGESTED ANSWER:
half of the 52 Million or 26 Million is his share (a) Rico and Cora are the co-owners of the
in the riceland. The
community. This 26 Million therefore is his Relations is that of co-ownership (Art. 147,
estate. Family Code,
first paragraph).
Karla and Luis are the Intestate heirs of Kevin. The levy is not proper there being no showing that
the surety agreement executed by the husband
They are entitled to share the estate equally redounded to the benefit of the family. An obligation
under Article 1001 of the NCC. Therefore. contracted by the husband alone is chargeable
Karla gets 13 Million and Luis gets 13 against the conjugal partnership only when it was
Million. contracted for the benefit of the family. When the
obligation was contracted on behalf of the family
Property Relations; Obligations; Benefit of the Family business the law presumes that such obligation will
(2000) redound to the benefit of the family. However, when
the obligation was to guarantee the debt of a third
As finance officer of K and Co., Victorino party, as in the problem, the obligation is presumed
arranged a loan of P5 Million from PNB for the for the benefit of the third party, not the family.
corporation. However, he was required by the Hence, for the obligation under the surety
bank to sign a Continuing Surety Agreement agreement to be chargeable against the partnership
to secure the repayment of the loan. The it must be proven that the family was benefited and
corporation failed to pay the loan, and the that the benefit was a direct result of such
bank obtained a judgment against it and agreement,
Victorino, jointly and severally. To enforce the
(Ayala Investment v. Ching, 286 SCRA 272)
judgment, the sheriff levied on a farm owned
by the conjugal partnership of Victorino and Property Relations; Unions without Marriage (1992)
his wife Elsa. Is the levy proper or not? (3%)
In 1989, Rico, then a widower forty (40) years
SUGGESTED ANSWER: of age, cohabited with Cora, a widow thirty
(30) years of age. While living together, they
acquired from their combined earnings a (Optional Addendum: However, after Rico's
parcel of riceland. marriage to Letty, the half interest of Rico
in the riceland will then become absolute
After Rico and Cora separated, Rico lived community property of Rico and Letty.)
together with Mabel, a maiden sixteen (16)
Rico is the exclusive owner of the coconut
years of age. While living land. The Relations is a sole/single
proprietorship (Art. 148. Family Code, first
paragraph is applicable, and not Art. 147
Family Code).
(Optional Addendum: However, after Rico's
marriage to Letty, the coconut land of Rico
will then become absolute community
property of Rico and Letty.)

Rico and Letty are the co-owners. The


Relations is the Absolute Community of
Property (Arts, 75,90and9l, Family Code).

Property Relations; Unions without Marriage (1997) Luis


and Rizza, both 26 years of age and single, live
exclusively with each other as husband and
wife without the benefit of marriage, Luis is
gainfully employed, Rizza is not employed,
stays at home, and takes charge of the
household chores. After living together for a
little over twenty years, Luis was able to save
from his salary earnings during that period the
amount of P200,000.00 presently deposited in
a bank. A house and lot worth P500,000.00
was recently purchased for the same amount
by the couple. Of the P500.000.00 used by the
common-law spouses to purchase the property,
P200.000.00 had come from the sale of palay
harvested from the hacienda owned by Luis
and P300,000.00 from the rentals of a building
belonging to Rizza. In fine, the sum of
P500.000.00 had been part of the fruits
received during the period of cohabitation
from their separate property, a car worth
P100.000.00. being used by the common-law
spouses, was donated Just months ago to
Rizza by her parents. Luis and Rizza now
decide to terminate their cohabitation, and
they ask you to give them your legal advice
on the following:

How, under the law should the bank deposit


of P200,000.00 the house and lot valued at
P500.000.00 and the car worth P100.000.00
be allocated to them?
Page 41 of 119
CIVIL LAW Answers to the BAR as Arranged by Topics (Year 1990-2006)
(b) What would your answer be (to the above a) Who will be entitled to the house and lot?
question) had (3%)
Luis and Rizza been living together all the time,
ie., since SUGGESTED ANSWER:
Tony and Susan are entitled to the house and
twenty years ago, under a valid marriage? lot as coowners
in equal shares. Under Article 147 of the
SUGGESTED ANSWER: Family Code, when a
a) Art. 147 of the Family Code provides in part man and a woman who are capacitated to
that when a marry each other
man and a woman who are capacitated to marry lived exclusively with each other as husband
each other, and wife, the
live exclusively with each other as husband and property acquired during their cohabitation
wife without are presumed to
the benefit of marriage or under a void have been obtained by their joint efforts, work
marriage, their wages or industry
and salaries shall be owned by them in equal and shall be owned by them in equal shares.
shares and the This is true even
property acquired by both of them through their though the efforts of one of them consisted
work or merely in his or
industry shall be governed by the rules of her care and maintenance of the family and of
coownership. In the the household.
absence of proof to the contrary, properties
acquired while
they lived together shall be presumed to have
been obtained
by their Joint efforts, worker Industry, and shall b) Would it make any difference if Tony could
be owned by not marry
them in equal shares. A party who did not Susan because he was previously married
participate in the to Alice from
acquisition by the other party of any property
shall be deemed whom he is legally separated? (2%)
to have contributed jointly in the acquisition
thereof if the SUGGESTED ANSWER:
former's efforts consisted in the care and Yes, it would make a difference. Under Article
maintenance of the 148 of the
family and of the household. Thus: 1) the Family Code, when the parties to the
wages and salaries cohabitation could not
of Luis in the amount of P200,000.00 shall be marry each other because of an
divided equally impediment, only those
between Luis and Rizza. 2) the house and lot properties acquired by both of them through
valued at their actual joint
P500.000.00 having been acquired by both of contribution of money, property, or Industry
them through shall be owned
work or industry shall be divided between them by them in common in proportion to their
in proportion respective
to their respective contribution, in consonance contributions. The efforts of one of the parties
with the rules in maintaining
on co-ownership. Hence, Luis gets 2\5 while the family and household are not
Rizza gets 3\5 considered adequate
of P500.000.00. 3) the car worth P100,000.00 contribution in the acquisition of the
shall be properties.
exclusively owned by Rizza, the same having
been donated to Since Susan did not contribute to the
her by her parents. acquisition of the house
and lot, she has no share therein. If Tony
cohabited with
Susan after his legal separation from Alice, the
house and lot
is his exclusive property. If he cohabited with
Susan before
his legal separation from Alice, the house and
lot belongs to
SUGGESTED ANSWER: his community or partnership with Alice.
(b) The property relations between Luis and
Rizza, their
marriage having been celebrated 20 years ago
(under the Civil
Code) shall be governed by the conjugal
partnership of gains, SUCCESSION
under which the husband and wife place in a
common fund
the proceeds, products, fruits and income from
their separate Amount of Successional Rights (2004)
properties and those acquired by either or both Mr. XT and Mrs. YT have been married for
spouses 20 years.
through their efforts or by chance, and upon Suppose the wife, YT, died childless, survived
dissolution of only by her
the marriage or of the partnership, the net gains husband, XT. What would be the share of
or benefits XT from her
obtained by either or both spouse shall be estate as
divided equally inheritance? Why? Explain. (5%)
between them (Art. 142. Civil Code). Thus: 1)
The salary of SUGGESTED ANSWER:
Luis deposited in the bank in the amount of Under the Civil Code, the widow or widower is
P200.000.00 and a legal and
the house and lot valued at P500,000.00 shall be compulsory heir of the deceased spouse. If the
divided widow is the
equally between Luis and Rizza. 2) However, the only surviving heir, there being no
car worth legitimate ascendants,
P100.000,00 donated to Rizza by her parents descendants, brothers, and sisters, nephews
shall be and nieces, she
considered to her own paraphernal property, gets the entire
having been estate.
acquired by lucrative title (par. 2, Art. 148, Civil
Code).
Barrier between illegitimate & legitimate relatives (1993)
A is the acknowledged natural child of B who
died when A
was already 22 years old. When B's full blood
brother, C,
died he (C) was survived by his widow and four
Property Relations; Unions without Marriage (2000) children of
For five years since 1989, Tony, a bank Vice- his other brother D. Claiming that he is
president, and entitled to inherit
Susan, an entertainer, lived together as husband from his father's brother C. A brought suit to
and wife obtain his
without the benefit of marriage although they share in the estate of C. Will his action
were prosper?
capacitated to many each other. Since Tony's
salary was more
than enough for their needs, Susan stopped
working and SUGGESTED ANSWER:
merely "kept house". During that period, Tony No, the action of A will not prosper. On the
premise that B,
was able to
C and D are legitimate brothers, as an
buy a lot and house in a plush subdivision.
illegitimate child of B,
However, after five
A cannot intestacy from C who is a
inherit in legitimate
years, Tony and Susan decided to separate.
brother of B. Only the wife of C in her own
right and the
CIVIL LAW Answers to the BAR as Arranged by Topics (Year 1990-2006)
legitimate relatives of C (i.e. the children of D How will you rule on Jorge's opposition to the
as C's probate of
legitimate nephews inheriting as collateral
relatives) can Maria's will. If you were the Judge?
inherit in intestacy. (Arts. 992, 1001, 1OO5 and
975, Civil SUGGESTED ANSWER:
As Judge, I shall rule as follows: Jorge's
Code) opposition should be
sustained in part and denied in part.
ALTERNATIVE ANSWER: Jorge's omission as
The action of A will not prosper. Being an spouse of Maria is not preterition of a
illegitimate, he is compulsory heir in the
barred by Article 992 of the Civil Code from direct line. Hence, Art. 854 of the Civil Code
inheriting ab does not apply,
intestato from the legitimate relatives of his and the institution of Miguela as heir is valid,
father. but only to the
extent of the free portion of one-half. Jorge is
still entitled to
one-half of the estate as his legitime. (Art.
Barrier between illegitimate & legitimate relatives (1996) 1001, Civil Code)
Cristina the illegitimate daughter of Jose and
Maria, died
intestate, without any descendant or ascendant.
Her valuable ALTERNATIVE ANSWERS:
estate is being claimed by Ana, the legitimate a) As Judge, I shall rule as follows: Jorge's
daughter of opposition should
Jose, and Eduardo, the legitimate son of Maria. be sustained in part and denied in part. This is
Is either, a case of
both, or neither of them entitled to inherit? ineffective disinheritance under Art, 918 of the
Explain. Civil Code,
because the omission of the compulsory heir
Jorge by Maria
was intentional. Consequently, the institution
SUGGESTED ANSWER: of Miguela as
Neither Ana nor Eduardo is entitled to inherit of heir is void only insofar as the legitime of Jorge
ab intestato is prejudiced.
from Cristina. Both are legitimate relatives of Accordingly, Jorge is entitled to his legitime of
Cristina's one-half of the
illegitimate parents and therefore they fall
under the estate, and Miguela gets the other half.
prohibition prescribed by Art. 992, NCC
(Manuel v. Ferrer,
242 SCRA 477; Diaz v. Court of Appeals, 182 father. Will the action against Joaquina Roxas
SCRA prosper?
427).
SUGGESTED ANSWER:
Collation (1993) Yes, because there is a presumed donation
Joaquin Reyes bought from Julio Cruz a in favor of Joaquina under Art. 1448 of the
residential lot of 300 square meters in Quezon Civil Code (De los Santos v. Reyes, 27
City for which Joaquin paid Julio the amount of January 1992, 206 SCRA 437). However, the
P300,000.00, When the deed was about to be donation should be collated to the
prepared Joaquin told Julio that it be drawn in hereditary estate and the legitime of the
the name of Joaquina Roxas, his acknowledged other heirs should be preserved.
natural child. Thus, the deed was so prepared
and executed by Julio. Joaquina then built a
house on the lot where she, her husband and
children resided. Upon Joaquin's death, his
legitimate children sought to recover
possession and ownership of the lot, claiming
that Joaquina Roxas was but a trustee of their
As Judge, I shall rule as follows: Jorge's that the will was made without his
opposition should be sustained. This is a case knowledge;
of preterition under Article 854 Civil Code, the 2that the will was made without his
result of the omission of Jorge as compulsory consent; and
heir having the same right equivalent to a that it has the effect of depriving him of
legitimate child "in the direct line" is that total his legitime, which is a ground that goes
intestacy will arise, and Jorge will inherit the into the intrinsic
entire estate. validity of the will and need not be
resolved during the probate proceedings.
As Judge, I shall rule as follows: the However, the opposition may be
opposition should be denied since it is entertained for, the purpose of securing to
predicated upon causes not recognized the husband his right to the legitime on
by law as grounds for disallowance of a wll, to the theory that the will constitutes an
wit: ineffective disinheritance under Art. 918
of the Civil Code,
ALTERNATIVE ANSWER:
As Judge, I shall rule as follows: Jorge is
Yes, the action against Joaquina Roxas will
entitled to receive his legitime from the estate
prosper, but only to the extent of the aliquot
of his wife. He was not disinherited in the will
hereditary rights of the legitimate children as
even assuming that he gave ground for
heirs. Joaquina will be entitled to retain her
disinheritance, hence, he is still entitled to his
own share as an illegitimate child, (Arts.
legitime. Jorge, however, cannot receive
1440 and 1453. Civil Code; Art. 176, F. C.)
anything from the free portion. He cannot
Disinheritance vs. Preterition (1993) claim preterition as he is not a compulsory heir
Maria, to spite her husband Jorge, whom she in the direct line. There being no preterition,
suspected was having an affair with another the institution of the sister was valid and the
woman, executed a will, unknown to him, only right of Jorge is to claim his legitime.
bequeathing all the properties she inherited Disinheritance; Ineffective (1999)
from her parents, to her sister Miguela. Upon
Mr. Palma, widower, has three daughters D, D-
her death, the will was presented for probate.
l and D-2. He executes a Will disinheriting D
Jorge opposed probate of the will on the ground
because she married a man he did not like,
that the will was executed by his wife without
and instituting daughters D -1 and D-2 as his
his knowledge, much less consent, and that it
deprived him of his legitime. After all, he had
heirs to his entire estate of P 1,000,000.00,
given her no cause for disinheritance, added
Upon Mr, Palma's death, how should his estate
Jorge in his opposition. be divided? Explain. (5%)

SUGGESTED ANSWER:
CIVIL LAW Answers to the BAR as Arranged by Topics (Year 1990-2006)
This is a case of ineffective disinheritance 1028 for being in consideration of her
because marrying adulterous relation
a man that the father did not approve of is not a
ground for with the testator. She is, therefore, disqualified to receive the
disinheritiD. Therefore, the institution of D-
ng l and D-2 legacy. Ernie will receive the legacy in his favor because it is not
shall be annulled insofar as it prejudices the inofficious. The institution of Baldo, which applies only to the free
legitime of D, portion, will be respected. In sum, the estate of Lamberto shall be
and the institution of D-l and D-2 shall only
apply on the distributed as follows:
free portion in the amount of P500,000.00.
Therefore, D, D-l
and D-2 will get their legitimes of P500.000.00
divided into
three equal parts and D-l and D-2 will get
a reduced Heir Legitime Legacy Institution TOTAL
testament disposition of P250,000.00
ary each. Hence, the Baldo 500,000 200.000 700,000 Elvira 250,000
shares will be: 250,000 Ernie
50,000 50,000 TOTAL 750,000 50,000 200,000
D P166,666.66 1,000,000
D-l P166,666.66 + P250.000.00
D-
2 P166,666.66 + P250,000.00
ANOTHER ALTERNATIVE ANSWER:
Disinheritance; Ineffective; Preterition (2000) Same answer as the first Alternative Answer
In his last will and testament, Lamberto 1) except as to distribution. Justice Jurado solved
disinherits his daughter Wilma because "she is this problem differently. In his opinion, the
disrespectful towards me and raises her voice legitime of the heir who was disinherited is
talking to me", 2) omits entirely his spouse distributed among the other compulsory heirs
Elvira, 3) leaves a legacy of P100,000.00 to his in proportion to their respective legitimes,
mistress Rosa and P50,000.00 to his driver while his share in the intestate portion. If any,
Ernie and 4) institutes his son Baldo as his is distributed among the other legal heirs by
sole heir. How will you distribute his estate of accretion under Article 1018 of the NCC in
P1,000,000.00? (5%) proportion to their respective intestate
SUGGESTED ANSWER:
shares. In sum the distribution shall be as
The disinheritance of Wilma was ineffective
follows:
because the ground relied upon by the testator
does not constitute maltreatment under Article
919(6) of the New Civil Code. Hence, the Heir Distribution
Legitime
testamentary provisions in the will shall be of Legacy TOTAL
annulled but only to the extent that her legitime Wilma’s Institution
was impaired. Legitime
the estate of Lamberto will be distributed as
The total omission of Elvira does not follows:
constitute preterition because she is not a
compulsory heir in the direct line. Only Baldo-------------- 450,0
compulsory heirs in the direct line may be the --- 00
subject of preterition. Not having been Wilma------------ 250,0
preterited, she will be entitled only to her --- 00
legitime. Elvira-------------- 250,0
--- 00
The legacy in favor of Rosa is void under Ernie-------------- 50,0
Article 1028 for being in consideration of her --- 00
adulterous relation with the testator. She is, 1,000,000
therefore, disqualified to receive the legacy of
100,000 pesos. The legacy of 50,000 pesos in ALTERNATIVE ANSWER:
The disinheritance of Wilma was effective because disrespect of,
favor of Ernie is not inofficious not having and raising of voice to, her father constitute maltreatment under
exceeded the free portion. Hence, he shall be Article 919(6) of the New Civil Code. She is, therefore, not entitled
entitled to receive it. to inherit anything. Her inheritance will go to the other legal heirs.
The total omission of Elvira is not preterition because she is not a
The institution of Baldo, which applies only to compulsory heir in the direct line. She will receive only her legitime.
The legacy in favor of Rosa is void under Article
the free portion, shall be respected. In sum,
Bald
o 250,0000 125,000 200,000 575,000
Wil
ma (250.000)
Elvi 250,000 125.000 375.000
Heirs; ra Intestate Heirs; Reserva Troncal (1995)
Isidro and Irma, Filipinos, both 18 years of
age, were passengers of Flight No. 317 of
Oriental Airlines. The plane they boarded was
of Philippine registry. While en route from
Manila to Greece some passengers hijacked
the plane, held the chief pilot hostage at the
cockpit and ordered him to fly instead to Libya.
During the hijacking Isidro suffered a heart
attack and was on the verge of death. Since
Irma was already eight months pregnant by
Isidro, she pleaded to the hijackers to allow the
assistant pilot to solemnize her marriage with
Isidro. Soon after the marriage, Isidro expired.
As the plane landed in Libya Irma gave birth.
However, the baby died a few minutes after
complete delivery. Back in the Philippines Irma
Immediately filed a claim for inheritance. The
parents of Isidro opposed her claim contending
that the marriage between her and Isidro was
void ab initio on the following grounds: (a) they
had not given their consent to the marriage of
their son; (b) there was no marriage license;
(c) the solemnizing officer had no authority to
perform the marriage; and, (d) the solemnizing
officer did not file an affidavit of marriage with
the proper civil registrar.

Does Irma have any successional rights at


all? Discuss fully.
SUGGESTED ANSWER:
2. Irma succeeded to the estate of Isidro as
his surviving spouse to the estate of her
legitimate child. When Isidro
CIVIL LAW Answers to the BAR as Arranged by Topics (Year 1990-2006)
died, he was succeeded by his surviving wife (c) X = 1/2 by representation of B C=l/2 Y =
Irma, and his legitimate unborn child. They 1/4 by representation of C
divided the estate equally between them, the
child excluding the parents of Isidro. An X - 1/3 in his own right Y- 1/3 in his own
unborn child is considered born for all right 2 - 1/3 in his own right
purposes favorable to it provided it is born
later. The child was considered born because, Article 977 of the Civil Code provides
having an intra -uterine life of more than that heirs who repudiate their share
seven months, it lived for a few minutes after cannot be represented.
its complete delivery. It was legitimate
because it was born within the valid marriage Intestate Succession (1997)
of the parents. Succession is favorable to it. "T" died intestate on 1 September 1997.He
When the child died, Irma inherited the share was survived by M (his mother), W (his
of the child. However, the share of the child in widow), A and B (his legitimate children), C
the hands of Irma is subject to reserva troncal (his grandson, being the legitimate son of B),
for the benefit of the relatives of the child D (his other grandson, being the son of E who
within the third degree of consanguinity and was a legitimate son of, and who
who belong to the line of Isidro. predeceased, "T"), and F (his grandson, being
ALTERNATIVE ANSWER: the son of G, a legitimate son who repudiated
If the marriage is void. Irma has no the inheritance from "T"). His distributable
successional rights with respect to Isidro but net estate is
she would have successional rights with P120.000.00. How should this amount be
respect to the child. shared in intestacy among the surviving
heirs?
Heirs; Intestate Heirs; Shares (2003)
Luis was survived by two legitimate children, SUGGESTED ANSWER:
two illegitimate children, his parents, and two The legal heirs are A, B, D, and W. C is
brothers. He left an estate of P1 million. Luis excluded by B who is still alive. D inherits in
died intestate. Who are his intestate heirs, and representation of E who predeceased. F is
how much is the share of each in his estate? excluded because of the repudiation of G, the
A will get P20.000.00. and P
SUGGESTED ANSWER: 13.333.33 (1/3 B will get P
The intestate heirs are the two (2) legitimate 20,000.00. and P13. 333.33 (1/3 D
children and the two (2) illegitimate children.
In intestacy the estate of the decedent is
divided among the legitimate and illegitimate predecessor. M is excluded by the legitimate
children such that the share of each children of
illegitimate child is one -half the share of each T. The answer may be premised on two
legitimate child. theories: the Theory of Exclusion and the
Their share are : For each legitimate child – Theory of Concurrence.
P333,333.33 For each illegitimate child – Under the Theory of Exclusion the
P166,666.66 legitimes of the heirs are accorded them
(Article 983, New Civil Code; Article 176, and the free portion will be given
Family Code)
exclusively to the legitimate descendants.
Hence under the Exclusion Theory:

of the free portion)


of the free portion)
of the free portion)
Who are the intestate heirs of F? What
Intestate Succession (1992) are their respective fractional
F had three (3) legitimate children: A, B, and shares?
C. B has one Who are the intestate heirs of A?
legitimate child X. C has two (2) legitimate What are their respective fractional
children: Y and Z. F and A rode together in a shares?
car and perished together at the same time If B and C both predeceased F, who are
in a vehicular accident, F and A died, each of F’s intestate heirs? What are their
them leaving substantial estates in intestacy. respective fractional shares? Do they
inherit in their own right or by
representation? Explain your answer.
If B and C both repudiated their shares in
the estate of F who are F's intestate heirs? W, the widow is limited to the legitime of
What are their respective fractional shares? P20.000.00 Under the Theory of
Do they inherit in their own right or by Concurrence. In addition to their legitimes,
representation? Explain your answer, the heirs of A, B, D and W will be given equal
SUGGESTED ANSWER: shares in the free portions:
B= P20.000.00 plus P10.000.00 (1 /4 of the free
1/2 portion)
B = 1/2 Z = 1/4 by representation of C C= 1/2 P20,000.00 plus P10.000.00 (l/4 of the free
Article 982 of the Civil Code provides that portlon)
grandchildren inherit by right of P20,000.00 plus P10.000.00 (1/4 of the free
representation. portion)
W: P20,000.00 plus P10,000.00 (l/4 of the free
portion) Alternative
Answer: Shares in Intestacy T - decedent
Estate:
P120.000.00
Survived by: M - Mother............................None W -
Widow.............................P 30,000.00 A -
Son.................................P 30,000.00 B
- Son.................................P30.000.00 C - Grandson (son of
B).............None D -
Grandson (son of E who predeceased T)................P
30,000.00 F - Grandson
(son of G who repudiated the Inheritance
from"T").......................None

Explanation:
The mother (M) cannot inherit from T
because under Art. 985 the ascendants
shall inherit in default of legitimate
children and descendants of the deceased.
The widow's share is P30.000.00 because
under Art, 996 it states that if the widow or
widower and legitimate children or
descendants are left, the surviving
CIVIL LAW Answers to the BAR as Arranged by Topics (Year 1990-2006)
spouse has in the succession the same be set aside as Mario's conjugal share from
share as that of the community
each of the children, c) C has no share because
his father is property. The other half, amounting to one million pesos, is her
still alive hence succession by representation
shall not apply conjugal share (net estate), and should be distributed to her
intestate heirs. Applying the above provision of law, Michelle and
(Art. 975).
D inherits P30.000 which is the share of Jorelle, Tessie's nieces, are entitled to one-half of her conjugal
d) his father share worth one million pesos, or 500,000 pesos, while the other
E who predeceased T by virtue of Art. 981 on
the right of one-half amounting to P500,000 will go to Mario, Tessie's surviving
representation. spouse. Michelle and Jorelle are then entitled to P250,000 pesos
F has no share because his father G
e) repudiated the each as their hereditary share.
inheritance. Under Article 977 heirs who repudiate their
share may not be represented.
Intestate Succession (1998)
Intestate Succession (1998) Tessie died survived by her husband Mario,
Enrique died, leaving a net hereditary estate and two nieces, Michelle and Jorelle, who are
of P1.2 million. He is survived by his widow, the legitimate children of an elder sister who
three legitimate children, two legitimate had predeceased her. The only property she
grandchildren sired by a legitimate child left behind was a house and lot worth two
who predeceased him, and two recognized million pesos, which Tessie and her husband
illegitimate children. Distribute the estate in had acquired with the use of Mario's savings
intestacy. [5%] from his income as a doctor. How much of the
SUGGESTED ANSWER:
property or its value, if any, may Michelle and
Under the theory of Concurrence, the shares
Jorelle claim as their hereditary shares? [5%]
are as follows: A (legitimate child) = SUGGESTED ANSWER:
P200,000 B (legitimate child) = P200,000 C Article 1001 of the Civil Code provides, "Should brothers and sisters
(legitimate child) = P200,000 D (legitimate or their children survive with the widow or widower, the latter shall
be entitled to one-half of the inheritance and the brothers and
child) = O (predeceased] E (legitimate child of sisters or their children to the other half." Tessie's gross estate
D) = P100,000 - by right of representation F consists of a house and lot acquired during her marriage, making it
(legitimate child of D) = P100,000 - by right of part of the community property. Thus, one-half of the said property
representation G (illegitimate child) = P100,000 - would have to
1/2 share of the legitimate child H (illegitimate
child) = P100,000 - 1/2 share of the legitimate
child W (Widow) = P200.000 - same share as
legitimate child
ANOTHER ANSWER:
Under the theory of Exclusion the free portion
(P300,000) is distributed only among the
legitimate children and is given to them in
addition to their legitime. All other Intestate
heirs are entitled only to their respective
legitimes. The distribution is as follows:
Legitime Free Portion Total
A [legitimate child) P150.000 + - P225.000 B
P 75,000 {legitimate
child) P150.000 + P150.000 - C (legitimate
P225.000 child)
P150.000 + P 75.000 - D (legitimate child) 0
P225.000 00E
(legitimate child P 75,000 P35.50 - P112,500
of D) + 0 F
(legitimate child P P 37.500 - P112,500
of D) 75.000 + G
(illegitimate P 75,500 H
child) 75.000 0 -P (illegitimate
child) P 75.000 - P 75,500 W
0 (Widow) P150,000 0
-P150.000
The motion to dismiss should be granted. Article 992 does
Intestate Succession (1999) not apply. Antero is not claiming any inheritance from
Mr. and Mrs. Cruz, who are childless, met Eugenio. He is claiming his share in the inheritance of his
with a serious motor vehicle accident with Mr. father consisting of his father's share in the inheritance of
Cruz at the wheel and Mrs. Cruz seated
beside him, resulting in the instant death of
Mr. Cruz. Mrs. Cruz was still alive when help
came but she also died on the way to the
hospital. The couple acquired properties
worth One Million (P1,000,000.00) Pesos
during their marriage, which are being
claimed by the parents of both spouses in
equal shares. Is the claim of both sets of
parents valid and why? (3%)
SUGGESTED ANSWER:
No, the claim of both parents is not valid.
When Mr. Cruz died, he was succeeded by his
wife and his parents as his intestate heirs who
will share his estate equally. His estate was
0.5 Million pesos which is his half share in the
absolute community amounting to 1 Million
Pesos. His wife, will, therefore, inherit O.25
Million Pesos and his parents will inherit 0.25
Million Pesos.

When Mrs. Cruz died, she was succeeded by


her parents as her intestate heirs. They will
inherit all of her estate consisting of her 0.5
Million half share in the absolute community
and her 0.25 Million inheritance from her
husband, or a total of 0.750 Million Pesos.

In sum, the parents of Mr. Cruz will inherit


250,000 Pesos while the parents of Mrs. Cruz
will inherit 750,000 Pesos.

Intestate Succession (2000)


Eugenio died without issue, leaving several
parcels of land in Bataan. He was survived by
Antonio, his legitimate brother; Martina, the
only daughter of his predeceased sister
Mercedes; and five legitimate children of
Joaquin, another predeceased brother. Shortly
after Eugenio's death, Antonio also died,
leaving three legitimate children.
Subsequently, Martina, the children of Joaquin
and the children of Antonio executed an
extrajudicial settlement of the estate of
Eugenio, dividing it among themselves. The
succeeding year, a petition to annul the
extrajudicial settlement was filed by Antero,
an illegitimate son of Antonio, who claims he
is entitled to share in the estate of Eugenio.
The defendants filed a motion to dismiss on
the ground that Antero is barred by Article
992 of the Civil Code from inheriting from the
legitimate brother of his father. How will you
resolve the motion? (5%)
SUGGESTED ANSWER:
CIVIL LAW Answers to the BAR as Arranged by Topics (Year 1990-2006)
Eugenio (Dela Merced v. Dela Merced, Gr No. 5M inherited by Mrs. Luna from Mr. Luna will
126707, 25 be inherited
February 1999). from her by her parents.
ALTERNATIVE ANSWER:
It depends. If Antero was not acknowledged by However, if the child had intra-uterine life of
Antonio, the less than 7
motion to dismiss should be granted because months, half of the estate of Mr. Luna, or 5M,
Antero is not a will be
legal heir of Antonio. If Antero was inherited by the widow (Mrs. Luna), while the
acknowledged, the other half, or
motion should be denied because Article 5M, will be inherited by the parents of Mr.
992 is not Luna. Upon the
applicable. This is because Antero is claiming death of Mrs. Luna, her estate of 5M will be
his inheritance inherited by her
from his illegitimate father, not from Eugenio. own parents.
Intestate Succession; Reserva Troncal (1999) Legitime (1997)
Mr. Luna died, leaving an estate of Ten "X", the decedent, was survived by W (his
Million (P1 widow). A (his
0,000,000.00) Pesos. His widow gave birth to a son), B (a granddaughter, being the daughter
child four of A) and C and
months after Mr, Luna's death, but the child D (the two acknowledged illegitimate children
died five hours of the
after birth. Two days after the child's death, the decedent). "X" died this year (1997) leaving a
widow of Mr. net estate of
Luna also died because she had suffered P180,000.00. All were willing to succeed,
from difficult except A who
childbirth. The estate of Mr. Luna is now being repudiated the inheritance from his father, and
claimed by his they seek your
parents, and the parents of his widow. Who is legal advice on how much each can expect to
entitled to Mr. receive as their
respective shares in the distribution of the
Luna'a estate and why? (5%) estate. Give your
SUGGESTED ANSWER:
answer.
Half of the estate of Mr. Luna will go to the
parents of Mrs. SUGGESTED ANSWER:
Luna as their inheritance from Mrs. Luna, while The heirs are B, W, C and D. A inherits nothing
the other because of his
half will be inherited by the parents of Mr. renunciation. B inherits a legitime of
Luna as the P90.000.00 as the nearest
reservatarios of the reserved property inherited and only legitimate descendant, inheriting in
by Mrs. Luna his own right not
by representation because of A's renunciation.
from her child. W gets a
legitime equivalent to one-half (1 / 2) that of B
When Mr. Luna died, his heirs were his wife and amounting to
the unborn P45.000. C and D each gets a legitime
child. The unborn child inherited because the equivalent to one-half
inheritance was (1/2) that of B amounting to P45.000.00 each.
favorable to it and it was born alive later though But since the
it lived only total exceeds the entire estate, their legitimes
would have to be
for five hours. Mrs. Luna inherited half of the 10 reduced to P22.500.00 each (Art.
Million corresponding 895. CC).
estate while the unborn child inherited the The total of all of these amounts to
other half. When P180.000.00.
the child died, it was survived by its mother,
Mrs. Luna. As
the only heir, Mrs. Luna inherited, by operation
of law, the ALTERNATIVE ANSWER:
estate of the child consisting of its 5 Million INTESTATE SUCCESSION
inheritance from
Mr. Luna. In the hands of Mrs. Luna, what she ESTATE:
inherited P180,000.00
from her child was subject to reserva troncal for W- (widow gets 1/2 share) P90.000.00 (Art. 998) A-
the benefit (son who
of the relatives of the child within the third repudiated his None Art. 977) B -
degree of inheritance) (Granddaughter)
consanguinity and who belong to the family of None C - (Acknowledged illegitimate child)
Mr. Luna, the P45.000.00 (Art.998)
D - (Acknowledged illegitimate child) P45,000.00
(Art. 998) The
line where the property came from. acknowledged illegitimate child gets 1/2 of the share
of each
legitimate child.
When Mrs. Luna died, she was survived by her
parents as her only heirs. Her parents will Legitime; Compulsory Heirs (2003)
inherit her estate consisting of the 5 Million Luis was survived by two legitimate children,
she inherited from Mr. Luna. The other 5 two illegitimate children, his parents, and two
Million she inherited from her child will be brothers. He left an estate of P1 million. Who
delivered to the parents of Mr. Luna as are the compulsory heirs of Luis, how much is
beneficiaries of the reserved property. the legitime of each, and how much is the free
portion of his estate, if any?
SUGGESTED ANSWER:
In sum, 5 Million Pesos of Mr. Luna's estate The compulsory heirs are the two legitimate
will go to the parents of Mrs. Luna, while the children and the two illegitimate children.
other 5 Million Pesos will go to the parents of The parents are excluded by the legitimate
Mr. Luna as reservatarios. children, while the brothers are not
ALTERNATIVE ANSWER: compulsory heirs at all. Their respective
If the child had an intra-uterine life of not less than 7 months, it
inherited from the father. In which case, the estate of 10M will be legitimate are: a) The legitime of the two (2)
divided equally between the child and the widow as legal heirs. legitimate children is one
Upon the death of the child, its share of 5M shall go by operation
of law to the mother, which shall be subject to reserva troncal. half (1/2) of the estate (P500,000.00) to be
Under Art. 891, the reserva is in favor of relatives belonging to the divided between them equally, or P250,000.00
paternal line and who are within 3 degrees from the child. The
parents of Mr, Luna are entitled to the reserved portion which is each. b) The legitimate of each illegitimate
5M as they are 2 degrees related from child. The child is one-half
(1/2) the legitime of each legitimate child
or P125,000.00.
CIVIL LAW Answers to the BAR as Arranged by Topics (Year 1990-2006)
c) Since the total legitime of the compulsory legitime of the legitimate children and it
heirs is follows that the
P750,000.00, the balance of P250,000.00 is legitime of one legitimate child is P100,000.
the free The legitime,
therefore, of the oldest son is P100,000.
portion. However, since the
donation given him was P100,000, he has
already received in
full his legitime and he will not receive
Legitime; Compulsory Heirs vs. Secondary Compulsory anything anymore
from the decedent. The remaining P900,000,
Heirs (2005) therefore, shall
Emil, the testator, has three legitimate children, go to the four younger children by institution
Tom, Henry in the will, to
and Warlito; a wife named Adette; parents be divided equally among them. Each will
named Pepe and receive P225,000.
Pilar; an illegitimate child, Ramon; brother,
Mark; and a sister, ALTERNATIVE ANSWER:
Nanette. Since his wife Adette is well-off, he Assuming that the donation is valid as to form
wants to leave to and substance,
his illegitimate child as much of his estate as he Juan cannot invoke preterition because he
can legally do. actually had
His estate has an aggregate net amount of received a donation inter vivos from the
Pl,200,000.00, and testatrix (III
all the above-named relatives are still living. Tolentino 188,1992 ed.). He would only have a
Emil now comes right to a
to you for advice in making a will. How will you completion of his legitime under Art. 906 of the
distribute his Civil Code.
estate according to his wishes without violating The estate should be divided equally among
the law on the five children
who will each receive P225,000.00 because the
testamentary succession? (5%) total hereditary
estate, after collating the donation to Juan (Art.
SUGGESTED ANSWER: 1061, CC),
P600,000.00 — legitime to be divided equally would be P1 million. In the actual distribution
between Tom, of the net
Henry and Warlito as the legitimate children. estate, Juan gets nothing while his will
Each will be siblings get
entitled to P200,000.00. (Art. 888, Civil Code)
P100,000.00 -- P225,000.00 each.
share of Ramon the illegitimate child.
Equivalent to 1/2 of
the share of each legitimate child. (Art. 176,
Family Code) Preterition; Compulsory Heir (1999)
P200,000.00 — Adette the wife. Her share is
equivalent to the
share of one legitimate child. (Art. 892, par. 2, Mr, Cruz, widower, has three legitimate
Civil Code) (a) children, A, B
and C. He executed a Will instituting as his
heirs to his estate
of One Million (P1,000,000.00) Pesos his two
children A and
B, and his friend F. Upon his death, how should
Mr. Cruz's
Pepe and Pilar, the parents are only secondary
compulsory estate be divided? Explain. (3%)
heirs and they cannot inherit if the primary (b) In the preceding question, suppose Mr.
compulsory heirs Cruz instituted his
(legitimate children) are alive. (Art. 887, par. 2, two children A and B as his heirs in his Will,
Civil Code) but gave a legacy
of P 100,000.00 to his friend F. How should the
estate of Mr,
Cruz be divided upon his death?
Brother Mark and sister Nanette are not Explain, (2%)
compulsory heirs
since they are not included in the enumeration
under Article SUGGESTED ANSWER:
(a) Assuming that the institution of A, B and F
887 of the Civil Code. were to the
entire estate, there was preterition of C
since C is a
The remaining balance of P300,000.00 is the compulsory heir in the direct line. The
free portion preterition will result
which can be given to the illegitimate child in the total annulment of the institution of
Ramon as an heirs. Therefore,
instituted heir. (Art. 914, Civil Code) If so given the institution of A, B and F will be set aside
by the and Mr. Cuz's
decedent, Ramon would receive a total of estate will be divided, as in intestacy, equally
P400,000.00. among A, B and
C as follows: A - P333,333.33; B - P333.333.33;
and C -
Preterition (2001) P333,333.33.
Because her eldest son Juan had been pestering
her for
capital to start a business, Josefa gave him (b) On the same assumption as letter (a), there
P100,000. Five was preterition
years later, Josefa died, leaving a last will and of C. Therefore, the institution of A and B is
testament in annulled but the
which she instituted only her four younger legacy of P100.000.00 to F shall be respected
children as her for not being
sole heirs. At the time of her death, her only inofficious. Therefore, the remainder of
properly left was P900.000.00 will be
P900,000.00 in a bank. Juan opposed the will on
the ground divided equally among A, B and C.
of preterition. How should Josefa's estate be
divided among
her heirs? State briefly the reason(s) for your
answer. (5%)
Proceedings; Intestate Proceedings; Jurisdiction (2004)
In his lifetime, a Pakistani citizen, ADIL,
SUGGESTED ANSWER: married three times
There was no preterition of the oldest son under Pakistani law. When he died an old
because the widower, he left
testatrix donated 100,000 pesos to him. This behind six children, two sisters, three homes,
donation is and an estate
considered an advance on the son's inheritance. worth at least 30 million pesos in the He
There being Philippines. was
no preterition, the institutions in the will shall born in Lahore but last resided in Cebu City,
be respected where he had a
but the legitime of the oldest son has to be mansion and where two of his youngest
completed if he children now live
and work. Two of his oldest children are
received less. farmers in Sulu,
while the two middle-aged children are
employees in
After collating the donation of P100.000 to the Zamboanga City. Finding that the deceased
remaining left no will, the
property of P900,000, the estate of the testatrix youngest son wanted to file intestate
is P1,000,000. proceedings before the
Regional Trial Court of Cebu City. Two other
Of this amount, one-half or P500,000, is the siblings
CIVIL LAW Answers to the BAR as Arranged by Topics (Year 1990-2006)
objected, arguing that it should be in Jolo before mother, in favor of another sister, with their
a Shari’a mother not
court since his lands are in Sulu. But Adil’s only giving her authority thereto but even
sisters in signing said deeds,
Pakistan want the proceedings held in Lahore there is a valid partition inter vivos between
before a the mother and
Pakistani court. Which court has jurisdiction her children which cannot be revoked by the
and is the mother. Said
proper venue for the intestate proceedings? deeds of sale are not contracts entered into
The law of with respect to
which country shall govern succession to his
estate? (5%) future inheritance.
"It would be unjust for the mother to revoke
SUGGESTED ANSWER: the sales to a
In so far as the properties of the decedent son and to execute a simulated sale in favor of
located in the a daughter
Philippines are concerned, they are governed by
Philippine who already benefited by the partition."
law (Article 16, Civil Code). Under Philippine
law, the proper
venue for the settlement of the estate is the
domicile of the SUGGESTED ANSWER:
decedent at the time of his death. Since the C. Yes, under Arts. 51 and 52 of the New
decedent last Family Code. In
resided in Cebu City, that is the proper venue case of legal separation, annulment of
for the intestate marriage, declaration
of nullity of marriage and the automatic
settlement of his estate. termination of a
subsequent marriage by the reappearance of
the absent
However, the successional rights to the estate spouse, the common or community property
of ADIL are of the spouses
governed by Pakistani law, his national law,
under Article 16 shall be dissolved and liquidated.
of the Civil Code. Art, 51. In said partition, the value of the
presumptive
legitimes of all common children, computed as
Succession; Death; Presumptive Legitime (1991) of the date of
a) For purposes of succession, when is death the final judgment of the trial court, shall be
deemed to delivered in
occur or take place? b) May succession be cash, property or sound securities, unless the
conferred by parties, by
mutual agreement, judicially approved, had
contracts or acts inter already provided
vivos? Illustrate. c) Is there any law which
allows the delivery for such matters.
to
compulsory heirs of their presumptive The children of their guardian, or the trustee
legitimes during of their
the lifetime of their parents? If so, in what property, may ask for the enforcement of the
instances? judgment.
SUGGESTED ANSWER: now requires that donations of future property
Death as a fact is deemed to occur when it be governed by the provisions on the
actually takes place. Death is presumed to testamentary succession and formalities of
take place in the circumstances under Arts. wills.
390-391 of the Civil Code. The time of death is ALTERNATIVE ANSWER:
presumed to be at the expiration of the 10year B. In the case of Coronado vs.CA(l91 SCRA81),
period as prescribed by Article 390 and at the it was ruled that no property passes under a
moment of disappearance under Article 391. will without its being probated, but may under
Article 1058 of the Civil Code of 1898, be
B. Under Art. 84 of the Family Code amending sustained as a partition by an act inter vivos
Art 130 of the Civil Code, contractual [Many-Oy vs. CA 144SCRA33).
succession is no longer possible since the law
And in the case of Chavez vs, IAC 1191
SCRA211), it was ruled that while the law The delivery of the presumptive legitimes
prohibits contracts upon future inheritance, the herein prescribed shall in no way prejudice
partition by the parent, as provided in Art. 1080 the ultimate successional rights of the
is a case expressly authorized by law. A person children accruing upon the death of either or
has two options in making a partition of his both of the parents; but the value of the
estate: either by an act inter vivos or by will. If properties already received under the decree
the partition is by will, it is imperative that such of annulment or absolute nullity shall be
partition must be executed in accordance with considered as advances on their legitime.
the provisions of the law on wills; if by an act
Art. 52. The judgment of annulment or of
inter vivos, such partition may even be oral or
absolute nullity of the marriage, the partition
written, and need not be in the form of a will,
and distribution of the properties of the
provided the legitime is not prejudiced.
spouses, and the delivery of the children's
presumptive legitimes shall be recorded in the
"Where several sisters execute deeds of sale appropriate civil registry and registries of
over their 1 /6 undivided share of the property; otherwise, the same shall not affect
paraphernal property of their third persons.

Wills; Codicil; Institution of Heirs; Substitution of


Heirs (2002)
By virtue of a Codicil appended to his will,
Theodore devised to Divino a tract of sugar
land, with the obligation on the part of Divino
or his heirs to deliver to Betina a specified
volume of sugar per harvest during Betina’s
lifetime. It is also stated in the Codicil that in
the event the obligation is not fulfilled, Betina
should immediately seize the property from
Divino or latter’s heirs and turn it over to
Theodore’s compulsory heirs. Divino failed to
fulfill the obligation under the Codicil. Betina
brings suit against Divino for the reversion of
the tract of land. a) Distinguish between modal
institution and substation

of heirs. (3%) b) Distinguish between


simple and fideicommissary
substitution of heirs. (2%) c) Does Betina
have a cause of action against Divino?
Explain (5%)
SUGGESTED ANSWER:
CIVIL LAW Answers to the BAR as Arranged by Topics (Year 1990-2006)
A. A MODAL INSTITUTION is the
institution of
an heir made for a certain purpose or cause b. In the case of a foreigner, his national law
(Arts. 871 and shall govern
882, NCC). SUBSTITUTION is the appointment substantive validity whether he executes his
of another will in the
heir so that he may enter into the inheritance in Philippines or in a foreign
default of the country.
heir originality instituted. (Art. 857, NCC).
Wills; Holographic Wills; Insertions & Cancellations (1996)
B. In a SIMPLE SUBSTITUTION of heirs, Vanessa died on April 14, 1980, leaving behind
the testator a holographic
designates one or more persons to substitute will which is entirely written, dated and signed
the heirs in her own
instituted in case such heir or heirs should die handwriting. However, it contains insertions
before him, or and cancellations
should not wish or should be incapacitated which are not authenticated by her signature.
to accept the For this reason,
inheritance. In a FIDEICOMMISSARY the probate of Vanessa's will was opposed by
SUBSTITUTION, her relatives
the testator institutes a first heir and charges who stood to inherit by her intestacy. May
him to preserve Vanessa's
and transmit the whole or part of the holographic will be probated?
inheritance to a second Explain.
heir. In a simple substitution, only one heir
inherits. In a SUGGESTED ANSWER:
fideicommissary substitution, both the first and Yes, the will as originally written may be
second heirs probated. The
insertions and alterations were void since
inherit. (Art. 859 and 869, NCC) they were not
authenticated by the full signature of Vanessa,
under Art. 814,
NCC. The original will, however, remains
valid because a
C. Betina has a cause of action against Divino. holographic will is not invalidated by the
This is a case unauthenticated
of a testamentary disposition subject to a mode insertions or alterations (Ajero v. CA, 236 SCRA
and the will 468].
itself provides the consequence if the
for mode is not ALTERNATIVE ANSWER:
complied with. enforce the mode, the will It depends. As a a holographic is adverse
rule, will not ly
To itself gives
affected by or whic were
Betina the right to compel the return of the
Insertions cancellations h not
property to the
authenticated by the full signature of the
heirs of Theodore. (Rabadilla v. Conscoluella, testator (Ajero v. CA,
334 SCRA 522
236 SCRA 468). However, when the insertion
or cancellation
[2000] GR 113725, 29 June 2000).
amounts to revocation of the will, Art.814 of
the NCC does
not apply but Art. 830. NCC. Art. 830 of the
Wills; Formalities (1990) NCC does not
(1) If a will is executed by a testator who is a require the testator to authenticate his
Filipino citizen, cancellation for the
what law will govern if the will is executed in effectivity of a revocation effected through
the Philippines? such cancellation
What law will govern if the will is executed (Kalaw v. Relova, 132 SCRA 237). In the
in another Kalaw case, the
original holographic will designated only one
country? Explain your answers. heir as the only
substantial provision which was altered by
substituting the
(2) If a will is executed by a foreigner, for original heir with another heir. Hence, if the
instance, a unauthenticated
Japanese, residing in the Philippines, what law cancellation amounted to a revocation of the
will govern if will, the will may
the will is executed in the Philippines? And not be probated because it had already been
what law will revoked.
govern if the will is executed in Japan, or some
other country,
for instance, the U.S.A.? Explain your
answers.
SUGGESTED ANSWER: Wills; Holographic Wills; Witnesses (1994)
(1) a. If the testator who is a Filipino citizen On his deathbed, Vicente was executing a will.
executes his will In the room
in the Philippines, Philippine law will govern the were Carissa, Carmela, Comelio and Atty.
formalities. Cimpo, a notary
public. Suddenly, there was a street brawl
which caught
b. If said Filipino testator executes his will in Comelio's attention, prompting him to look out
another country, the window.
the law of the country where he maybe or Cornelio did not see Vicente sign a will. Is the
Philippine law will will valid?
govern the formalities. (Article 815, Civil
Code}
SUGGESTED ANSWERS:
Yes, The will is valid. The law does not require
SUGGESTED ANSWER: a witness to actually see the testator sign the
a. If the testator is a foreigner residing in the will. It is sufficient if the witness could have
Philippines and he executes his will in the seen the act of signing had he chosen to do so
Philippines, the law of the country of which by casting his eyes to the proper direction.
he is a citizen or Philippine law will govern
the formalities. Yes, the will is valid. Applying the "test of
position", although Comelio did not actually
b. If the testator is a foreigner and executes see Vicente sign the will, Cornelio was in the
his will in a foreign country, the law of his proper position to see Vicente sign if Cornelio
place of residence or the law of the country of so wished.
which he is a citizen or the law of the place of
execution, or Philippine law will govern the Wills; Joint Wills (2000)
formalities (Articles 17. 816. 817. Civil Code). Manuel, a Filipino, and his American wife
Eleanor, executed a Joint Will in Boston,
POSSIBLE ADDITIONAL ANSWERS:
Massachusetts when they were residing in
In the case of a Filipino citizen, Philippine law
said city. The law of Massachusetts allows the
shall govern substantive validity whether he
execution of joint wills. Shortly thereafter,
executes his will in the Philippines or in a Eleanor died. Can the said Will be probated in
foreign country. the Philippines for the settlement of her
estate? (3%)
CIVIL LAW Answers to the BAR as Arranged by Topics (Year 1990-2006)
SUGGESTED ANSWER:
Yes, the will may be probated in the Wills; Probate; Notarial and Holographic Wills (1997)
Philippines insofar as the estate of Eleanor is Johnny, with no known living relatives,
concerned. While the Civil Code prohibits the executed a notarial will giving all his estate to
execution of Joint wills here and abroad, such his sweetheart. One day, he had a serious
prohibition applies only to Filipinos. Hence, altercation with his sweetheart. A few days
the joint will which is valid where executed is later, he was introduced to a charming lady
valid in the Philippines but only with respect who later became a dear friend. Soon after, he
to Eleanor. Under Article 819, it is void with executed a holographic will expressly
respect to Manuel whose joint will remains revoking the notarial will and so designating
void in the Philippines despite being valid his new friend as sole heir. One day when he
where executed. was clearing up his desk, Johnny mistakenly
ALTERNATIVE ANSWER: burned, along with other papers, the only
The will cannot be probated in the Philippines, copy of his holographic will. His business
even though valid where executed, because it associate, Eduardo knew well the contents of
is prohibited under Article 818 of the Civil the will which was shown to him by Johnny
Code and declared void under Article 819, The the day it was executed. A few days after the
prohibition should apply even to the American burning incident, Johnny died. Both wills were
wife because the Joint will is offensive to sought to be probated in two separate
public policy. Moreover, it is a single juridical petitions. Will either or both petitions
act which cannot be valid as to one testator prosper?
and void as to the other. SUGGESTED ANSWER:
The probate of the notarial will will prosper.
Wills; Probate; Intrinsic Validity (1990) The holographic will cannot be admitted to
H died leaving a last will and testament probate because a holographic will can only
wherein it is stated that he was legally be probated upon evidence of the will itself
married to W by whom he had two legitimate unless there is a photographic copy. But since
children A and B. H devised to his said forced the holographic will was lost and there was no
heirs the entire estate except the free portion other copy, it cannot be probated and
which he gave to X who was living with him at therefore the notarial will will be admitted to
the time of his death. probate because there is no revoking will.
In said will he explained that he had been As a general rule, the will should be admitted
estranged from his wife W for more than 20 in probate proceedings if all the necessary
years and he has been living with X as man requirements for its extrinsic validity have
and wife since his separation from his been met and the court should not consider
legitimate family. the intrinsic validity of the provisions of said
will. However, the exception arises when the
In the probate proceedings, X asked for the will in effect contains only one testamentary
issuance of letters testamentary in accordance disposition. In effect, the only testamentary
with the will wherein she is named sole disposition under the will is the giving of the
executor. This was opposed by W and her free portion to X, since legitimes are provided
children. by law. Hence, the trial court may consider the
Should the will be admitted in said probate intrinsic validity of the provisions of said will.
proceedings? (Nuguid v. Nuguid, etal.. No. L23445, June
Is the said devise to X valid? 23, 1966, 17 SCRA; Nepomuceno v. CA, L-
Was it proper for the trial court to consider the 62952,
intrinsic 9 October 1985. 139 SCRA 206).
validity of the provisions of said will? Explain
your answers,
SUGGESTED ANSWER:
Yes, the will may be probated if executed
according to the formalities prescribed by law.

The institution giving X the free portion is not


valid, because the prohibitions under Art. 739
of the Civil Code on donations also apply to
testamentary dispositions (Article 1028, Civil
Code), Among donations which are
considered void are those made between
persons who were guilty of adultery or
concubinage at the time of the donation.
ADDITIONAL ANSWERS:
In the case of Gan vs. Yap (104 Phil 509), the
execution and the contents of a lost or
destroyed holographic will may not be
proved by the bare testimony of witnesses
who have seen or read such will. The will
itself must be presented otherwise it shall
produce no effect. The law regards the
document itself as material proof of
authenticity. Moreover, in order that a will
may be revoked by a subsequent will, it is
necessary that the latter will be valid and
executed with the formalities required for
the making of a will. The latter should
possess all the requisites of a valid will
whether it be ordinary or a holographic
will, and should be probated in order that
the revocatory clause thereof may produce
effect. In the case at bar, since the
holographic will itself cannot be presented,
it cannot therefore be probated. Since it
cannot be probated, it cannot revoke the
notarial will previously written by the
decedent.
On the basis of the Rules of Court, Rule 76,
Sec. 6, provides that no will shall be proved
as a lost or destroyed will unless its
provisions are clearly and distinctly proved
by at least two (2) credible witnesses.
Hence, if we abide strictly by the two-
witness rule to prove a lost or destroyed
will, the holographic will which Johnny
allegedly mistakenly burned, cannot be
probated, since there is only one witness,
Eduardo, who can be called to testify as to
the existence of the will. If the holographic
will, which purportedly, revoked the earlier
notarial will cannot be proved because of
the absence of the required witness, then
the petition for the probate of the notarial
will should prosper.
Wills; Revocation of Wills; Dependent Relative
Revocation (2003)
Mr. Reyes executed a will completely valid as to
form. A week later, however, he executed
another will which expressly revoked his first
will, which he tore his first will to pieces. Upon
the death of Mr. Reyes, his second will was
presented for probate by his heirs, but it was
denied probate
CIVIL LAW Answers to the BAR as Arranged by Topics (Year 1990-2006)
due to formal defects. Assuming that a copy of the first will excluded by a legitimate son of
the decedent [Art. 887, New
is available, may it now be admitted to probate and given Civil Code]. This follows the
principle that the descendants
effect? Why? exclude the ascendants from inheritance.
SUGGESTED ANSWER:
Yes, the first will may be admitted to probate
and given effect. When the testator tore first Wills; Testamentary Intent (1996)
will, he was under the mistaken belief that the Alfonso, a bachelor without any descendant or
second will was perfectly valid and he would ascendant, wrote a last will and testament in
not have destroyed the first will had he known which he devised." all the properties of which I
that the second will is not valid. The revocation may be possessed at the time of my death" to
by destruction therefore is dependent on the his favorite brother Manuel. At the time he
validity of the second will. Since it turned out wrote the will, he owned only one parcel of
that the second will was invalid, the tearing of land. But by the time he died, he owned
the first will did not produce the effect of twenty parcels of land. His other brothers and
revocation. This is known as the doctrine of sisters insist that his will should pass only the
dependent relative revocation parcel of land he owned at the time it was
(Molo v. Molo, 90 Phil 37.) written, and did not cover his properties
ALTERNATIVE ANSWERS: acquired, which should be by intestate
No, the first will cannot be admitted to probate. succession. Manuel claims otherwise. Who is
While it is true that the first will was successfully correct? Explain.
revoked by the second will because the second
will was later denied probate, the first will was, SUGGESTED ANSWER:
nevertheless, revoked when the testator Manuel is correct because under Art. 793,
destroyed it after executing the second invalid NCC, property acquired after the making of a
will. will shall only pass thereby, as if the testator
(Diaz v. De Leon, 43 Phil 413 [1922]). had possessed it at the time of making the
will, should it expressly appear by the will that
Wills; Testamentary Disposition (2006) such was his intention. Since Alfonso's
Don died after executing a Last Will and intention to devise all properties he owned at
Testament leaving his estate valued at P12 the time of his death expressly appears on the
Million to his common -law wife Roshelle. He will, then all the 20 parcels of land are
is survived by his brother Ronie and his half- included in the devise.
sister Michelle.
Was Don's testamentary disposition of his
estate in accordance with the law on
succession? Whether you agree or not, explain Assuming he died intestate survived by his
your answer. Explain. brother Ronie, his half-sister Michelle, and his
SUGGESTED ANSWER: Yes, Don's testamentary legitimate son Jayson, how will you distribute
disposition of his estate is in accordance with his estate? Explain. (2.5%)
the law on succession. Don has no compulsory SUGGESTED ANSWER: Jayson will be entitled to
heirs not having ascendants, descendants nor the entire P12 Million as the brother and
a spouse [Art. 887, New Civil Code]. Brothers sister will be excluded by a legitimate son of
and sisters are not compulsory heirs. Thus, he the decedent. This follows the principle of
can bequeath his entire estate to anyone who proximity, where "the nearer excludes the
is not otherwise incapacitated to inherit from farther."
him. A common-law wife is not incapacitated
under the law, as Don is not married to Assuming further he died intestate, survived by
anyone. his father Juan, his brother Ronie, his half-
sister Michelle, and his legitimate son Jayson,
how will you distribute his estate?
Explain. (2.5%)
If Don failed to execute a will during his
SUGGESTED ANSWER: Jayson will still be entitled
lifetime, as his
to the entire P12 Million as the father,
lawyer, how will you distribute his estate?
Explain. (2.5%) SUGGESTED ANSWER: After paying brother and sister will be
the legal obligations of
the estate, I will give Ronie, as full -blood
brother of Don, 2/3 of the net estate, twice the
share of Michelle, the half-sister who shall
receive 1/3. Roshelle will not receive anything
as she is not a legal heir [Art. 1006 New Civil
Code].
DONATION
Donation vs. Sale (2003)
May a person sell something that does not
belong to him? Explain. b) May a person
donate something that does not belong
to him? Explain. 5%
SUGGESTED ANSWER:
Yes, a person may sell something which does not
belong to him. For the sale to be valid, the law
does not require the seller to be the owner of the
property at the time of the sale. (Article 1434,
NCC). If the seller cannot transfer ownership
over the thing sold at the time of delivery
because he was not the owner thereof, he shall
be liable for breach of contact.

As a general rule, a person cannot donate


something which he cannot dispose of at the
time of the donation (Article 751, New Civil
Code).

Donations; Condition; Capacity to Sue (1996)


Sometime in 1955, Tomas donated a parcel of
land to his stepdaughter Irene, subject to the
condition that she may not sell, transfer or
cede the same for twenty years. Shortly
thereafter, he died. In 1965, because she
needed money for medical expenses, Irene
sold the land to Conrado. The following year,
Irene died, leaving as her sole heir a son by
the name of Armando. When Armando learned
that the land which he expected to inherit had
been sold by Irene to Conrado, he filed an
action against the latter for annulment of the
sale, on the ground that it violated the
restriction imposed by Tomas. Conrado filed a
motion to dismiss, on the ground that
Armando did not have the legal capacity to
sue. If you were the Judge, how will you rule
on this motion to dismiss? Explain.
CIVIL LAW Answers to the BAR as Arranged by Topics which he had no intervention, (Teves vs. PHHC,
23 SCRA 1141).

Such detriment or prejudice cannot be shown


by Armando. As a forced heir, Armando's
interest in the property was, at best, a mere
expectancy. The sale of the land by his mother
did not impair any vested right. The fact
remains that the premature sale made by his
mother (premature because only half of the
period of the ban had elapsed) was not voidable
at all, none of the vices of consent under Art.
(Garrido u. CA, 139 of the NCC being present. Hence, the
SUGGESTED ANSWER: motion to dismiss should be granted.
As judge, I will grant the motion to dismiss.
Armando has no personality to bring the action Donations; Conditions; Revocation (1991)
for annulment of the sale to Conrado. Only an
Spouses Michael and Linda donated a 3-hectare
aggrieved party to the contract may bring the
action for annulment thereof (Art. 1397. NCC). residential land to the City of Baguio on the
While Armando is heir and successor-in- condition that the city government would build
interest of his mother (Art. 1311, NCC), he thereon a public park with a boxing arena, the
[standing in place of his mother) has no construction of which shall commence within six
personality to annul the contract. Both are not months from the date the parties ratify the
aggrieved parties on account of their own donation. The donee accepted the donation and
violation of the condition of, or restriction on, the title to the property was transferred in its
their ownership imposed by the donation. Only name. Five years elapsed but the public park
the donor or his heirs would have the with the boxing arena was never started.
personality to bring an action to revoke a
donation for violation of a condition thereof or Considering the failure of the donee to comply
a restriction thereon. with the condition of the donation, the donor-
spouses sold the
SCRA 450). Consequently, while the donor or
his heirs were not parties to the sale, they
have the right to annul the contract of sale
because their rights are prejudiced by one of
the contracting parties thereof [DBP v. CA, 96
SCRA 342; Teves vs. PHHC. 23 SCRA 114].
Since Armando is neither the donor nor heir of
the donor, he has no personality to bring the
action for annulment.
ALTERNATIVE ANSWER:
As judge, I will grant the motion to dismiss.
Compliance with a condition imposed by a
donor gives rise to an action to revoke the
donation under Art. 764, NCC. However, the
right of action belongs to the donor. Is
transmissible to his heirs, and may be
exercised against the donee's heirs. Since
Armando is an heir of the donee, not of the
donor, he has no legal capacity to sue for
revocation of the donation. Although he is not
seeking such revocation but an annulment of
the sale which his mother, the donee, had
executed in violation of the condition imposed
by the donor, an action for annulment of a
contract may be brought only by those who are
principally or subsidiarily obliged thereby (Art.
1397, NCC). As an exception to the rule, it has
been held that a person not so obliged may
nevertheless ask for annulment if he is
prejudiced in his rights regarding one of the
contracting parties (DBP us. CA. 96 SCRA 342
and other cases) and can show the detriment
which would result to him from the contract in
(Year 1990-2006) that the donor would not have made the
property to Ferdinand who then sued to donation without the mode or condition.
recover the land from the city
government. Will the suit prosper? Donations con causa onerosa is governed by
law on obligations and contracts, under which
SUGGESTED ANSWER: an impossible or Illicit condition annuls the
Ferdinand has no right to recover the land. It obligation dependent upon the condition
is true that the donation was revocable where the condition is positive and suspensive.
because of breach of the conditions. But until If the impossible or illicit condition is negative,
and unless the donation was revoked, it it is simply considered as not written, and the
remained valid. Hence, Spouses Michael and obligation is converted into a pure and simple
Linda had no right to sell the land to one. However, in order that an illegal condition
Ferdinand. One cannot give what he does not may annul a contract, the impossibility must
have. What the donors should have done first exist at the time of the creation of the
was to have the donation annulled or obligation; a supervening impossibility does
revoked. And after that was done, they could not affect the existence of the obligation.
validly have disposed of the land in favor of ADDITIONAL ANSWER:
Ferdinand.
ALTERNATIVE ANSWER:
A. Until the contract of donation has been
resolved or rescinded under Article 1191 of
the Civil Code or revoked under Art. 764 of
the Civil Code, the donation stands effective
and valid. Accordingly, the sale made by the
donor to Ferdinand cannot be said to have
conveyed title to Ferdinand, who, thereby,
has no cause of action for recovery of the
land acting for and in his behalf.

B. The donation is onerous, And being


onerous, what applies is the law on
contracts, and not the law on donation (De
Luna us. Abrigo, 81 SCRA 150). Accordingly,
the prescriptive period for the filing of such
an action would be the ordinary prescriptive
period for contacts which may either be six
or ten depending upon whether it is verbal
or written. The filing of the case five years
later is within the prescriptive period and,
therefore, the action can prosper,
Alternative Answer:
The law on donation lays down a special
prescriptive period in the case of breach of
condition, which is four years from non-
compliance thereof (Article 764 Civil Code).
Since the action has prescribed, the suit will
not prosper,
Donations; Effect; illegal & immoral conditions (1997)
Are the effects of illegal and immoral
conditions on simple donations the same as
those effects that would follow when such
conditions are imposed on donations con
causa onerosa?
SUGGESTED ANSWER:
No, they don't have the same effect. Illegal
or impossible conditions in simple and
remuneratory donations shall be considered
as not imposed. Hence the donation is valid.
The donation will be considered as simple or
pure. The condition or mode is merely an
accessory disposition, and its nullity does not
affect the donation, unless it clearly appears
CIVIL LAW Answers to the BAR as Arranged by Topics (Year 1990-2006)
No. In simple or pure donation, only the illegal irrevocable, the latter is revocable. In the
or problem given, all
impossible condition is considered not the clauses or conditions mentioned in the
written but the deed of donation,
donation remains valid and becomes free from except one, are consistent with the rule of
conditions. irrevocability and
The condition or mode being a mere accessory would have sustained the view that the
disposition. donation is inter vivos
Its nullity does not affect the donation and therefore valid. The lone exception is the
unless it clearly clause which
appears that the donor would not have made the reserves the donor's right to sell the property
donation at any time
without the mode or condition. On the other before his death. Such a reservation has been
hand, onerous held to render
donation is governed by the rules on contracts. the donation revocable and, therefore,
Under Article becomes a donation
1183, Impossible or illegal conditions shall mortis causa (Puig vs. Penqflorida, 15 SCRA
annul the 276, at p. 286).
obligation which depends upon them. In these That the right was not exercised is immaterial;
cases, both the its reservation
was an implied recognition of the donor's
obligation and the condition are void. power to nullify the
donation anytime he wished to do so.
Consequently, it should
have been embodied in a last will and
Donations; Formalities; Mortis Causa (1990) testament. The suit for
B donated to M a parcel of land in 1980. B made
the deed of nullity will thus prosper.
donation, entitled ―Donation Inter Vivos,‖ in a public
instrument and M accepted the donation in
the same
document. It was provided in the deed that the
land donated Donations; Inter Vivos; Acceptance (1993)
shall be immediately delivered to M and that M On January 21, 1986, A executed a deed of
shall have the donation inter
right to enjoy the fruits fully. The deed also vivos of a parcel of land to Dr. B who had
provided that B earlier constructed
thereon a building in which researches on the
was reserving the right to dispose of said land during his (B’s) dreaded disease
lifetime, and that M shall not register the deed AIDS were being conducted. The deed,
of donation acknowledged before
a notary public, was handed over by A to Dr. B
until after B’s death. Upon B’s death, W, B’s widow and sole who received
heir, filed an action for the recovery of the it. A few days after, A flew to Davao City.
donated land, Unfortunately, the
contending that the donation made by B is a airplane he was riding crashed on landing
donation mortis killing him. Two
causa and not a donation inter vivos. Will said days after the unfortunate accident. Dr. B,
action prosper? upon advice of a
lawyer, executed a deed acknowledged before
Explain your answer. a notary public
accepting the donation. Is the donation
SUGGESTED ANSWER: effective? Explain
Yes, the action will prosper. The donation is
a donation your answer.
mortis causa because the reservation is to
dispose of all the
property donated and, therefore, the donation is
revocable at SUGGESTED ANSWER:
will. Accordingly, the donation requires the No, the donation is not effective. The law
execution of a requires that the
valid will, either notarial or holographic (Arts separate acceptance of the donee of an
755, 728 NCC). immovable must be
done in a public document during the lifetime
of the donor
(Art. 746 & 749, Civil Code) In this case, B
executed the
deed of acceptance before a notary public
Donations; Formalities; Mortis Causa (1998) after the donor
Ernesto donated in a public instrument a parcel
of land to had already died.
Demetrio, who accepted it in the same
document. It is there
declared that the donation shall take effect
immediately, with Donations; Perfection (1998)
the donee having the right to take possession of On July 27, 1997, Pedro mailed in Manila a
the land and letter to his
receive its fruits but not to dispose of the land brother, Jose, a resident of Ilollo City, offering
while Ernesto to donate a
is alive as well as for ten years following his vintage sports car which the latter had long
death. Moreover, been wanting to
Ernesto also reserved in the same deed his right buy from the former. On August 5, 1997, Jose
to sell the called Pedro
property should he decide to dispose of it at any by cellular phone to thank him for his
time - a right generosity and to
which he did not exercise at all. After his death, inform him that he was sending by mail his
Ernesto's letter of
heirs seasonably brought an action to recover acceptance. Pedro never received that letter
the property, because it was
alleging that the donation was void as it did not never mailed. On August 14, 1997, Pedro
comply with received a telegram
the formalities of a will. Will the suit prosper? from Iloilo informing him that Jose had been
[5%] killed in a road
accident the day before (August 13, 1997)
SUGGESTED ANSWER: 1. Is there a perfected donation? [2%]
Yes, the suit will prosper as the donation did not 2. Will your answer be the same if Jose did
comply with mail his
the formalities of a will. In this instance, the fact acceptance letter but it was received by Pedro
that the in Manila days
donor did not intend to transfer ownership or
possession of after Jose's death? [3%]
the donated property to the donee until the
donor's death, SUGGESTED ANSWER:
would result in a donation mortis causa and in 1. None. There is no perfected donation.
this kind of Under Article 748
disposition, the formalities of a will should be of the Civil Code, the donation of a movable
complied with, may be made
otherwise, the donation is void. In this Instance, orally or in writing. If the value of the personal
donation property
mortis causa embodied only in a public donated exceeds five thousand pesos, the
instrument without donation and the
the formalities of a will could not have acceptance shall be made in writing.
transferred ownership Assuming that the value
of the thing donated, a vintage sports car,
of disputed property to another. exceeds P5,000.00
then the donation and the acceptance must be
ALTERNATIVE ANSWER: in writing. In
this instance, the acceptance of Jose was not
One of the essential distinctions between a donation inter vivos in writing,
and a donation mortis causa is that while the former is therefore, the donation is void. Upon the other
CIVIL LAW Answers to the BAR as Arranged by Topics (Year 1990-2006)
hand, assuming that the sports car costs less than P5,000.00 Code which requires the donation
and the acceptance
then the donation maybe oral, but still, the simultaneous thereof to be in a public
instrument in order to be valid. The
delivery of the car is needed and there being none, the acceptance not being in a public
instrument, the part which is
donation was never perfected. not onerous is void and Rosa may recover it from
Amanda.
For a donation to be onerous, the burden must
SUGGESTED ANSWER:
be imposed by the donor on the donee. In the
Yes, the answer is the same. If Jose's mail problem, there is no such burden imposed by
containing his acceptance of the donation was the donor on the donee. The donation not being
received by Pedro after the former's death, onerous, it must comply with the formalities of
then the donation is still void because under Article 749.
Article 734 of the Civil Code, the donation is ALTERNATIVE ANSWER:
perfected the moment the donor knows of the Neither Rosa nor Amanda is correct. The
acceptance by the donee. The death of Jose donation is onerous only as to the portion of the
before Pedro could receive the acceptance property corresponding to the value of the
indicates that the donation was never installments and taxes paid by Amanda.
perfected. Under Article
acceptance must be made during the lifetime
of both the donor and the donee. The portion in excess thereof is not onerous.
The onerous portion is governed by the rules
Donations; Requisites; Immovable Property on contracts which do not require the
Anastacia purchased a house and lot on acceptance by the donee to be in any form.
installments at a housing project in Quezon The onerous part, therefore, is valid. The
City. Subsequently, she was employed in portion which is not onerous must comply
California and a year later, she executed a with Article 749 of the New Civil
deed of donation, duly authenticated by the
Philippine Consulate in Los Angeles, California,
donating the house and lot to her friend
Amanda. The latter brought the deed of
donation to the owner of the project and
discovered that Anastacia left unpaid
installments and real estate taxes. Amanda
paid these so that the donation in her favor can
be registered in the project owner's office. Two
months later, Anastacia died, leaving her
mother Rosa as her sole heir. Rosa filed an
action to annul the donation on the ground that
Amanda did not give her consent in the deed of
donation or in a separate public instrument.
Amanda replied that the donation was an
onerous one because she had to pay unpaid
installments and taxes; hence her acceptance
may be implied. Who is correct? (2%)
SUGGESTED ANSWER:
Rosa is correct because the donation is void.
The property donated was an immovable. For
such donation to be valid, Article 749 of the
New Civil Code requires both the donation and
the acceptance to be in a public instrument.
There being no showing that Amanda's
acceptance was made in a public instrument,
the donation is void. The contention that the
donation is onerous and, therefore, need not
comply with Article 749 for validity is without
merit. The donation is not onerous because it
did not impose on Amanda the obligation to
pay the balance on the purchase price or the
arrears in real estate taxes. Amanda took it
upon herself to pay those amounts voluntarily.
Sons, Inc. v. Roman Catholic Bishop, G.R. No.
133705, March 31, 2005; Heirs ofRozendo
Donations; Unregistered; Effects; Non-Compliance; Sevilla v. De Leon, G.R. No.
Resolutory Condition (2006) 149570, March 12,
Spouses Alfredo and Racquel were active 2004).
members of a religious congregation. They
donated a parcel of land in favor of that
Donations; Validity; Effectivity; for
congregation in a duly notarized Deed of
Donation, subject to the condition that the Unborn Child (1999) Elated that her sister who
Minister shall construct thereon a place of had been married for five years was pregnant for the first
worship within 1 year from the acceptance of time, Alma donated P100,000.00 to the unborn child.
the donation. In an affidavit he executed on Unfortunately, the baby died one hour after delivery. May
behalf of the congregation, the Minister Alma recover the P100.000.00 that she
accepted the donation. The Deed of Donation
was not registered with the Registry of
Deeds.

However, instead of constructing a place of worship,


the Minister constructed a bungalow on the
property he used as his residence. Disappointed
with the Minister, the spouses revoked the donation
and demanded that he vacate the premises
immediately. But the Minister refused to leave,
claiming that aside from using the bungalow as his
residence, he is also using it as a place for worship
on special occasions. Under the circumstances, can
Alfredo and
Racquel evict the Minister and recover
possession of the property? If you were the
couple's counsel, what action you take to
protect the interest of your clients? (5%)

ALTERNATIVE ANSWER:
Yes, Alfredo and Racquel can bring an action for
ejectment against the Minister for recovery of
possession of the property evict the Minister and
recover possession of the property. An action for
annulment of the donation, reconveyance and
damages should be filed to protect the interests of
my client. The donation is an onerous donation and
therefore shall be governed by the rules on
contracts. Because there was no fulfillment or
compliance with the condition which is resolutory
in character, the donation may now be revoked and
all rights which the donee may have acquired
under it shall be deemed lost and extinguished

(Central Philippine University, G.R. No. 112127,


July 17,1995).
ALTERNATIVE ANSWER:
No, an action for ejectment will not prosper. I
would advice Alfredo and Racquel that the
Minister, by constructing a structure which also
serves as a place of worship, has pursued the
objective of the donation. His taking up
residence in the bungalow may be regarded as a
casual breach and will not warrant revocation of
the donation. Similarily, therefore, an action for
revocation of the donation will be denied (C. J.
Yulo
CIVIL LAW Answers to the BAR as Arranged by Topics (Year 1990-2006)
had donated to said baby before it was born not been fixed in the Deed of Donation, the
considering donee is not
that the baby died? Stated otherwise, is the yet default in his obligation until the period is
donation valid fixed by order
of the court under Article 1197 of the New
and binding? Explain. (5%) Civil Code. Since
the period has not been fixed as yet, the donee
SUGGESTED ANSWER: is not yet
The donation is valid and binding, being an act default, and therefore the donor has no cause
favorable to of action to
the unborn child, but only if the baby had an revoke the donation. (Dissenting opinion of
intra-uterine life Davide, CJ,
of not less than seven months and provided Central Philippine University v. Court of
there was due Appeals, 246 SCRA
acceptance of the donation by the proper person
representing 511 [1995])
said child. If the child had less than
seven months of
24 hours following its delivery, in which ease the donation
PROPERTY
intra-uterine life, it is not deemed born since it
died less than
never became effective since the donee
never became a
person, birth being determinative of
personality. Accretion; Alluvion (2001)
For many years, the Rio Grande river deposited
ALTERNATIVE ANSWER: soil along its
Even if the baby had an intra-uterine life of bank, beside the titled land of Jose. In time,
more than seven such deposit
months and the donation was properly reached an area of one thousand square
accepted, it would be meters. With the
void for not having conformed with the proper permission of Jose, Vicente cultivated the said
form. In area. Ten years
order to be valid, the donation and acceptance later, a big flood occurred in the river and
of personal transferred the
property exceeding five thousand pesos should 1000 square meters to the opposite bank,
be in writing. beside the land of
Agustin. The land transferred is now contested
(Article 748, par. 3) by Jose and
Agustin as riparian owners and by Vicente
who claims
ownership by prescription. Who should
Donations; with Resolutory Condition (2003) prevail,? Why? (5%)
In 1950, Dr. Alba donated a parcel of land
to Central
University on condition that the latter must
establish a SUGGESTED ANSWER:
medical college on the land to be named after Jose should prevail. The disputed area, which is
him. In the an alluvion,
year 2000, the heirs of Dr. Alba filed an action to belongs by right of accretion to Jose, the
annul the riparian owner (Art.
donation and for the reconveyance of the 457 CC). When, as given in the problem, the
property donated very same area"
to them for the failure, after 50 years, of the was "transferred" by flood waters to the
University to opposite bank, it
established on the property a medical school became an avulsion and ownership thereof is
named after retained by Jose
their father. The University opposed the who has two years to remove it (Art. 459, CC).
action on the Vicente's claim
ground of prescription and also because it had based on prescription is baseless since his
not used the possession was by
property for some purpose other than that mere tolerance of Jose and, therefore, did not
stated in the adversely affect
donation. Should the opposition of the Jose's possession and ownership (Art. 537, CC).
University to the Inasmuch as
his possession is merely that of a holder, he
action of Dr. Alba’s heirs be sustained? Explain. cannot acquire the
SUGGESTED ANSWER: disputed area by prescription.
The donation may be revoked. The non-established of
the medical
college on the donated property was a resolutory
condition imposed
on the donation by the donor. Although the Deed of
Donation did Accretion; Avulsion (2003)
not fix the time for the established of the medical Andres is a riparian owner of a parcel of
college, the failure registered land. His
of the donee to establish the medical college after land, however, has gradually diminished in
fifty (50) years area due to the
from the making of the donation should be considered current of the river, while the registered land
as occurrence of Mario on the
of the resolutory condition, and the donation may now opposite bank has gradually increased in area
be revoked. by 200square
While the general rule is that in case the period is not
fixed in the meters.
agreement of the parties, the period must be fixed
first by the court
before the obligation may be demanded, the period of (a) Who has the better right over the 200-
fifty (50) years square meter area
was more than enough time for the donee to
comply with the that has been added to Mario’s registered land, Mario or
condition. Hence, in this case, there is no more need (b)Andres?May a third person acquire said 200-
for the court to square meter land by
fix the period because such procedure with the
condition. (Central prescription?
Philippine University v. CA. 246 SCRA 511). SUGGESTED ANSWER:
a. Mario has a better right over the 200 square
meters increase in area by reason of accretion,
applying Article 457 of the New Civil Code, which
ANOTHER SUGGESTED ANSWER: provides that ―to the owners of lands adjoining the
The donation may not as yet revoked. The banks of rivers belong the accretion which
establishment of a medical college is not a they gradually received from the effects of the
resolutory or suspensive condition but a current of the waters‖.
―charge‖, obligation‖, or a ―mode‖. The non- Andres cannot claim that the increase in Mario’s land
compliance with the charge or mode will give is his own, because such is an accretion and not
the donor the right to revoke the donation result of the sudden detachment of a known
within four (4) years from the time the charge portion of his land and its attachment to Mario’s
was supposed to have been complied with, or land, a process called ―avulsion‖. He can no longer
to enforce the charge by specific performance claim ownership of the portion of his
within ten registered land which was gradually and
years from the time the cause of action naturally eroded due to the current of the
accrued. Inasmuch as the time to established river, because he
the medical college has
CIVIL LAW Answers to the BAR as Arranged by Topics (Year 1990-2006)
had lost it by operation of law. That portion of reasonable rent, if the owner of the land does
the land has not choose to
appropriate the building after proper
become part of the public domain. indemnity. The parties
shall agree upon the terms of the lease
and in case of
SUGGESTED ANSWER:
the disagreement, the court fix the terms thereof.
b. Yes, a third party may acquire by
prescription 200
square meters, increase in area, because it is
not included in Builder; Good Faith vs. Bad Faith (1999)
the Torrens Title of the riparian owner. Hence,
this does not
involve the imprescriptibility conferred by
Section 47, (a) Because of confusion as to the boundaries
P.D. No. 1529. The fact that the riparian land is of the
registered adjoining lots that they bought from the same
does not automatically make the accretion subdivision
thereto a registered company, X constructed a house on the
land. (Grande v. CA, 115 521 (1962); Jagualing v. adjoining lot of Y in
CA, 194 SCRA the honest belief that it is the land that he
bought from the
subdivision company. What are the respective
607 (1991). rights of X
Builder; Good Faith (1992)
A owns a parcel of residential land worth and Y with respect to X's house? (3%)
P500,000.00 (b) Suppose X was in good faith but Y knew
unknown to A, a residential house costing P that X was
100,000.00 is constructing on his (Y's) land but simply kept
built on the entire parcel by B who claims quiet about it,
ownership of the thinking perhaps that he could get X's house
land. Answer all the following questions based later. What are
on the premise the respective rights of the parties over X's
that B is a builder in good faith and A is a house in this
landowner in good
faith. a) May A acquire the house built by B? If case? (2%)
so, how? b) If SUGGESTED ANSWER:
the land increased in value to (a) The rights of Y, as owner of the lot, and of
P500,000.00 by reason X, as builder
of a house thereon, are governed by Art. 448
the building of the house of the Civil
of thereon, what amount Code which grants to Y the right to choose
should be paid by A in order to acquire the between two
house from remedies: (a) appropriate the house by
B indemnifying X for its
? value plus whatever necessary expenses the
Assuming that the cost of the house was latter may have
c) P90,000.00 incurred for the preservation of the land, or (b)
and not P100,000.00, may A require B to buy compel X to
the land? buy the land if the price of the land is not
If B voluntarily buys the land as desired considerably more
d) by A, under than the value of the house. If it is, then X
what circumstances may A nevertheless be cannot be obliged
entitled to have to buy the land but he shall pay reasonable
rent, and in case
the house removed? of disagreement, the court shall fix the terms
In what situation may a "forced lease" of the lease.
e) arise between
A and B. and what terms and conditions would
govern the
lease
? SUGGESTED ANSWER:
(b) Since the lot owner Y is deemed to be in
Give reasons for your answers. bad faith (Art
453), X as the party in good faith may (a)
SUGGESTED ANSWER: remove the house
(a) Yes, A may acquire the house build by B by
paying and demand indemnification for damages
indemnity to B. Article 448 of the Civil Code suffered by him, or
provides that (b) demand payment of the value of the
the owner of the land on which anything has house plus
been built, sown reparation for damages (Art 447, in relation to
or planted in good faith, shall have the right to Art 454). Y
appropriate as continues as owner of the lot and becomes,
his own the works, sowing or planting, after under the second
payment of the option, owner of the house as well, after he
indemnity provided for in Articles 546 and 546 pays the sums
of the Civil
demanded.
Code.
(b) A should pay B the sum of P50,000. Article Builder; Good Faith vs. Bad Faith (2000)
548 of the In good faith, Pedro constructed a five-door
Civil Code provides that useful expenses shall commercial
be refunded to building on the land of Pablo who was also in
the possessor in good faith with the right of good faith.
retention, the When Pablo discovered the construction, he
person who has defeated him in the possession opted to
having the appropriate the building by paying Pedro the
option of refunding the amount of the expenses cost thereof.
or of paying However, Pedro insists that he should be paid
the increase in value which the thing may have the current
acquired by market value of the building, which was much
higher because
reason thereof. The increase in value amounts of inflation. 1) Who is correct Pedro or Pablo?
to P50,000.00. (1%) 2) In the
meantime that Pedro is not yet paid, who is
entitled to the
(c) Yes, A may require B to buy the land. Article rentals of the building, Pedro or Pablo? (1%)
448 of the
Civil Code provides that the owner of the land
on which
SUGGESTED ANSWER:
anything has been built in good faith shall have Pablo is correct. Under Article 448 of the New
the right to Civil Code in
oblige the one who built to pay the price of the relation to Article 546, the builder in good
land if its faith is entitled to
value is not considerably more than that of the a refund of the necessary and useful expenses
building, incurred by
(d) If B agrees to buy land but fails to pay, A can him, or the increase in value which the
have the land may have
house removed ( Depra vs. Dumlao, 136 acquired by reason of the improvement, at the
SCRA 475). option of the
landowner. The builder is entitled to a refund
of the expenses
(e) Article 448 of the Civil Code provides that he incurred, and not to the market value of the
the builder improvement
cannot be obliged to buy the land if its value is
considerably
more than that of the building. In such case, he shall pay
CIVIL LAW Answers to the BAR as Arranged by Topics (Year 1990-2006)
The case of Pecson v. CA, 244 SCRA 407, is not square meters. Jose claims that Mike is a
applicable to builder in bad faith
the problem. In the Pecson case, the builder
was the owner because he should know the boundaries of his lot, and demands
of the land who later lost the property at a
public sale due to that the portion of the house which encroached on his land should
non-payment of taxes. The Court ruled that
Article 448 does be destroyed or removed. Mike replies that he is a builder in good
not apply to the case where the owner of the
land is the faith and offers to buy the land occupied by the building instead.
builder but who later lost the land; not being
applicable, the 1) Is Mike a builder in good faith or bad faith? Why? (3%) 2)
indemnity that should be paid to the buyer must
be the fair Whose preference should be followed? Why? (2%)
market value of the building and not just
the cost of
construction thereof. The Court opined in that
case that to SUGGESTED ANSWER:
do otherwise would unjustly enrich the new 1) Yes, Mike is a builder in good faith. There is
owner of the no showing
that when he built his house, he knew that a
land. portion thereof
encroached on Jose's lot. Unless one is versed
ALTERNATIVE ANSWER: in the science
Pedro is correct. In Pecson vs. CA, it was held of surveying, he cannot determine the precise
that Article boundaries or
546 of the New Civil Code does not specifically location of his property by merely examining
state how the his title. In the
value of useful improvements should be absence of contrary proof, the law
determined in fixing presumes that the
the amount of indemnity that the owner of the encroachment was done in good faith
land should [Technogas Phils, v.
pay to the builder in good faith. Since the
objective of the law CA, 268 SCRA 5, 15 (1997)].
is to adjust the rights of the parties in such
manner as "to
administer complete justice to both of them in 2} None of the preferences shall be followed.
such a way as The preference
neither one nor the other may enrich himself of of Mike cannot prevail because under Article
that which 448 of the Civil
does not belong to him", the Court ruled that Code, it is the owner of the land who has the
the basis of option or
reimbursement should be the fair market choice, not the builder. On the other hand,
value of the the option
belongs to Jose, he cannot demand that the
building. portion of the
house encroaching on his land be
SUGGESTED ANSWER: destroyed or removed
2) Pablo is entitled to the rentals of the because this is not one of the options given by
building. As the law to the
owner of the land, Pablo is also the owner of the owner of the land. The owner may choose
building between the
being an accession thereto. However, Pedro who appropriation of what was built after payment
is entitled to of indemnity,
retain the building is also entitled to retain the or to compel the builder to pay for the land if
rentals. He, the value of the
however, shall apply the rentals to the land is not considerably more than that of
indemnity payable to the building.
him after deducting reasonable cost of Otherwise, the builder shall pay rent for the
repair and portion of the
maintenance. land encroached.
ALTERNATIVE ANSWER: ALTERNATIVE ANSWER:
Pablo is entitled to the rentals. Pedro became a 1) Mike cannot be considered a builder
possessor in in good faith
bad faith from the time he learned that the land because he built his house without first
belongs to determining the
Pablo. As such, he loses his right to the corners and boundaries of his lot to make
building, including sure that his
construction was within the perimeter of his
the fruits thereof, except the right of retention. property. He
could have done this with the help of a
geodetic engineer as
an ordinary prudent and reasonable man
Builder; Good Faith vs. Bad Faith; Accession (2000) would do under the
a) Demetrio knew that a piece of land bordering
the beach circumstances.
belonged to Ernesto. However, since the latter
was studying
in Europe and no one was taking care of the Jose's preference should be followed. He
land, Demetrio 2) may have
occupied the same and constructed thereon the building removed at the expense of Mike,
nipa sheds with appropriate the
tables and benches which he rented out to building as his own, oblige Mike to buy the
people who want land and ask for
to have a picnic by the beach. When Ernesto damages in addition to any of the three
returned, he options. (Articles
demanded the return of the land. Demetrio
agreed to do so 449, 450, 451, CC)
after he has removed the nipa sheds. Ernesto
refused to let
Demetrio remove the nipa sheds on the ground
that these Chattel Mortgage vs. Pledge (1999)
already belonged to him by right of Distinguish a contract of chattel mortgage
accession. Who is from a contract of
correct? (3%) pledge. (2%)
SUGGESTED ANSWER: SUGGESTED ANSWER:
Ernesto is correct, Demetrio is a builder in bad In a contract of CHATTEL MORTGAGE
faith because he knew beforehand that the possession belongs to the creditor, while in a
land belonged to Ernesto, under Article 449 of contract of PLEDGE possession belongs to the
the New Civil Code, one who builds on the debtor.
land of another loses what is built without
right to indemnity. Ernesto becomes the owner A chattel mortgage is a formal contract while
of the nipa sheds by right of accession. Hence, a pledge is a real contract.
Ernesto is well within his right in refusing to
allow the removal of the nipa sheds. A contract of chattel mortgage must be
recorded in a public instrument to bind third
Builder; Good Faith vs. Bad Faith; Presumption (2001) persons while a contract of pledge must be in
Mike built a house on his lot in Pasay City. Two years later, a a public instrument containing description of
survey disclosed that a portion of the building actually stood on the thing pledged and the date thereof to bind
the neighboring land of Jose, to the extent of 40 third persons.
CIVIL LAW Answers to the BAR as Arranged by Topics (Year 1990-2006)
foreclosure sale, foreclosed the mortgage and
Chattel Mortgage; Immovables (1994) acquired X’s
Vini constructed a building on a parcel of land house and lot. Learning of the proceedings
he leased from conducted by the
Andrea. He chattel mortgaged the land to bank, Z is now demanding that the bank
Felicia. When he reconvey to him X’s
could not pay Felicia. Felicia initiated house or pay X’s loan to him plus interests. Is
foreclosure proceedings. Z’s demand
Vini claimed that the building he had against the bank valid and sustainable? Why?
constructed on the 5%
leased land cannot be validly foreclosed because
the building SUGGESTED ANSWER:
No, Z’s demand is not valid. A building is
was, by law, an immovable. Is Vini correct? immovable or real
property whether it is erected by the owner of
the land, by a
usufructuary, or by a lessee. It may be treated
SUGGESTED ANSWERS: as a movable by
a) The Chattel Mortgage is void and cannot be the parties to chattel mortgage but such is
foreclosed binding only
because the building is an immovable and between them and not on third parties
cannot be an (Evangelista v. Alto
Surety Col, inc. 103 Phil. 401 [1958]). In this
object of a chattel mortgage. case, since the
bank is not a party to the chattel mortgage, it
is not bound by
b) It depends. If the building was intended and it, as far as the Bank is concerned, the chattel
is built of mortgage, does
light materials, the chattel mortgage may be not exist. Moreover, the chattel mortgage
considered as does not exist.
valid as between the parties and it may be Moreover, the chattel mortgage is void
considered in because it was not
respect to them as movable property, since it registered. Assuming that it is valid, it does
can be removed not bind the Bank
from one place to another. But if the building is because it was not annotated on the title
of strong of the land
material and is not capable of being removed or mortgaged to the bank. Z cannot demand that
transferred the Bank pay
without being destroyed, the chattel mortgage him the loan Z extended to X, because the
is void and Bank was not
cannot be foreclosed. privy to such loan transaction.
If it was the land which Vini chattel
c) mortgaged, such ANOTHER SUGGESTED ANSWER:
mortgage would be void, or at least No, Z’s demand against the bank is not valid.
unenforceable, since he His demand
that the bank reconvey to him X’s house
was not the owner of the land. presupposes that he
If what was mortgaged as a chattel is the has a real right over the house. All that Z has
building, the chattel is a personal
mortgage is valid as between the parties only, right against X for damages for breach of the
on grounds of contract of
estoppel which would preclude the mortgagor
from assailing loan.
the contract on the ground that its subject-
matter is an
immovable. Therefore Vini's defense is The treatment of a house, even if built on
untenable, and Felicia rented land, as
can foreclose the mortgage over the movable property is void insofar as third
building, observing, persons, such as the
however, the procedure prescribed for the bank, are concerned. On the other hand, the
execution of sale Bank already
of a judgment debtor's immovable under Rule had a real right over the house and lot when
39, Rules of the mortgage
Court, specifically, that the notice of auction was annotated at the back of the Torrens title.
sale should be The bank later
became the owner in the foreclosure sale. Z
published in a newspaper of general circulation. cannot ask the
bank to pay for X’s loan plus interest. There is
no privity of
contract between Z and the bank.
The problem that Vini mortgaged the land by ALTERNATIVE ANSWER:
way of a chattel mortgage is untenable. Land The answer hinges on whether or not the bank
can only be the subject matter of a real estate is an innocent mortgagee in good faith or a
mortgage and only an absolute owner of real mortgagee in bad faith. In the former case, Z’s
property may mortgage a parcel of land. demand is not valid. In the latter case, Z’s
(Article 2085 demand against the bank is valid and
(2) Civil Code). Hence, there can be no sustainable.
foreclosure.
Under the Torrens system of land registration,
But on the assumption that what was
every person dealing with registered land may
mortgaged by way of chattel mortgage was
rely on the correctness of the certificate of
the building on leased land, then the parties
title and the law will not in any way oblige to
are treating the building as chattel. A building
him to look behind or beyond the certificate in
that is not merely superimposed on the ground
order to determine the condition of the title.
is an immovable property and a chattel
He is not bound by anything not annotated or
mortgage on said building is legally void but
reflected in the certificate. If he proceeds to
the parties cannot be allowed to disavow their
buy the land or accept it as a collateral relying
contract on account of estoppel by deed.
on the certificate, he is considered a buyer or
However, if third parties are involved such
a mortgagee in good faith. On this ground, the
chattel mortgage is void and has no effect.
Bank acquires a clean title to the land and the
Chattel Mortgage; Immovables (2003) house.
X constructed a house on a lot which he was
However, a bank is not an ordinary mortgagee.
leasing from Y. Later, X executed a chattel
Unlike private individuals, a bank is expected to
mortgage over said house in favor of Z as
exercise greater care and prudence in its
security for a loan obtained from the latter.
dealings. The ascertainment of the condition of a
Still later, X acquired ownership of the land
property offered as collateral for a loan must be
where his house was constructed, after which
a standard and indispensable part of its
he mortgaged both house and land in favor of
operation. The bank should have conducted
a bank, which mortgage was annotated on the
Torrens Certificate of Title. When X failed to further inquiry regarding the house standing on
pay his loan to the bank, the latter, being the the land considering that it was already
highest bidder at the Page 59 of 119
CIVIL LAW Answers to the BAR as Arranged by Topics (Year 1990-2006)
standing there before X acquired the title to the was then valued only at P1 Million. Lawrence
land. The was declared
bank cannot be considered as a mortgagee in
good faith. On insolvent.
this ground, Z’s demand against the Bank is
valid and
Assuming that the aircraft was sold for Pl
sustainable. Million, give the
order of preference of the creditors of
Lawrence and
Chattel Mortgage; Possession (1993) distribute the amount of P1 Million.
A, about to leave the country on a foreign
assignment, SUGGESTED ANSWER:
entrusted to B his brand new car and its Assuming that the aircraft was sold for P1
certificate of Million, there is no
registration. Falsifying A's signature. B sold A's order of preference. The P1 Million will all go
car to C for to the bank as
P200,000.00. C then registered the car in his a chattel mortgagee because a chattel
name. To mortgage under Art.
complete the needed amount, C borrowed 2241 (4) NCC defeats Art. 2244 (12) and (14}.
P100.000.00 from Art. 2241 (3)
the savings and loan association in his office, and (5) are not applicable because the aircraft
constituting a is no longer in
chattel mortgage on the car. For failure of C to
pay the the possession of the creditor.
amount owed, the savings and loan association
filed in the
RTC a complaint for collection with application
for issuance Easement vs. Usufruct (1995)
of a writ of replevin to obtain possession of the 1. What is easement? Distinguish easement
vehicle so from usufruct.
that the chattel mortgage could be foreclosed. 2. Can there be (a) an easement over a
The RTC usufruct? (b) a
issued the writ of replevin. The car was then usufruct over an easement? (c) an easement
seized from C over another
and sold by the sheriff at public auction at
which the savings easement? Explain.
and loan association was the lone bidder.
Accordingly, the car SUGGESTED ANSWER:
was sold to it. A few days later, A arrived from 1. An EASEMENT or servitude is an encumbrance
imposed
his foreign upon an immovable for the benefit of another
assignment. Learning of what happened to his immovable
car, A sought
to recover possession and ownership of it from belonging to a different owner. (Art. 613, NCC)
the savings
and loan association. Can A recover his car from
the savings USUFRUCT gives a right to enjoy the property
of another
and loan association? Explain your answer.
with the obligation of preserving its form
and substance,
unless the title constituting it or the law
SUGGESTED ANSWER: otherwise provides.
Under the prevailing rulings of the Supreme
Court, A can (Art. 562, NCC).
recover the car from the Savings and Loan
Association
provided he pays the price at which the
Association bought ALTERNATIVE ANSWER:
the car at a public auction. Under that doctrine, Easement is an encumbrance imposed upon an
immovable
there has
for the benefit of another immovable belonging
been an unlawful deprivation by B of A of his
to a different
car and,
therefore, A can recover it from any person in owner in which case it is called real or predial
possession easement, or
thereof. But since it was bought at a public for the benefit of a community or group of
auction in good persons in which
faith by the Savings and Loan Association, he case it is known as a personal easement.
must reimburse
the Association at the price for which the car
The distinctions between usufruct and
was bought.
easement are:
Usufruct includes all uses of the
ALTERNATIVE ANSWER: a) property and for all
Yes, A can recover his car from the Savings and purposes, including jus fruendi. Easement is
Loan limited to a
Association. In a Chattel Mortgage, the
mortgagor must be specific use.
the absolute owner of the thing mortgaged. Usufruct may be constituted on
Furthermore, the b) immovable or
person constituting the mortgage must have the movable property. Easement may be
free disposal constituted only on an
of the property, and in the absence thereof,
must be legally immovable property.
authorized for the purpose. In the case at bar, Easement is not extinguished by the
these essential c) death of the
requisites did not apply to the mortgagor B, owner of the dominant estate while usufruct is
hence the Chattel extinguished
by the death of the usufructuary unless a
Mortgage was not valid. contrary intention
appea
rs.
An easement contemplates two (2)
Chattel Mortgage; Preference of Creditors (1995) d) estates belonging
Lawrence, a retired air force captain, decided to to two (2) different owners; a usufruct
go into the contemplates only one
air transport business. He purchased an aircraft property (real or personal) whereby the
in cash except usufructuary uses and
for an outstanding balance of P500,000.00. He enjoys the property as well as its fruits, while
incurred an another owns the
indebtedness of P300,000.00 for repairs with an
aircraft repair naked title during the period of the usufruct.
company. He also borrowed P1 Million from a
bank for
additional capital and constituted a chattel A usufruct may be alienated
mortgage on the e) separately from the
property to which it attaches, while an
aircraft to secure the loan. easement cannot be
alienated separately from the property to which
it attaches.
While on a test flight the aircraft crashed
causing physical
injuries to a third party who was awarded NOTE: It is recommended by the
damages of Committee that any
two (2) distinctions should be given
P200,000.00. full credit.
Lawrence's insurance claim for damage to the SUGGESTED ANSWER:
aircraft was
denied thus leaving him nothing else but the
aircraft which
CIVIL LAW Answers to the BAR as Arranged by Topics (Year 1990-2006)
2. (a) There can be no easement over a usufruct. there is a degree of regularity to indicate
Since an continuity of
easement may be constituted only on a
corporeal immovable possession and that if coupled with an apparent sign, such
property, no easement may be constituted on a
usufruct which easement of way may be acquired by prescription.
is not a corporeal right ALTERNATIVE ANSWER:
(b) There can be no usufruct over an Yes, Ernie could close the pathway on his land.
easement. While a Don has not
usufruct maybe created over a right, such right acquired an easement of right of way either by
must have an agreement or
existence of its own independent of the property. by judicial grant. Neither did the buyers. Thus,
A servitude establishment
cannot be the object of a usufruct because it has of a road or unlawful use of the land of
no existence Ernie would
independent of the property to which It constitute an invasion of possessory rights
attaches. of the owner,
which under Article 429 of the Civil Code may
be repelled or
prevented. Ernie has the right to exclude any
ALTERNATIVE ANSWERS: person from the
There cannot be a usufruct over an enjoyment and disposal of the land. This is an
easement since an attribute of
easement presupposes two (2) tenements
belonging to ownership that Ernie enjoys.
different persons and the right attaches to the
tenement and ALTERNATIVE ANSWER:
not to the owner. While a usufruct gives the Yes, Ernie may close the pathway, subject
usufructuary a however, to the
right to use, right to enjoy, right to the fruits, rights of the lot buyers. Since there is no
and right to access to the public
possess, an easement gives only a limited use of road, this results in the creation of a legal
the servient easement. The lot
buyers have the right to demand that Ernie
estate. grant them a right
However, a usufruct can be constituted over a of way. In turn, they have the obligation to pay
property that the value of
has in its favor an easement or one burdened the portion used as a right of way, plus
with servitude. damages.
The usufructuary will exercise the easement
during the period
of usufruct. c) What are the rights of the lot buyers, if
any? Explain.
There can be no easement over another ALTERNATIVE ANSWER:
easement for the same reason as in (a). An No, Don did not acquire an easement of right
easement, although it is a real right over an of way. An easement of right of way is
immovable, is not a corporeal right. There is a discontinuous in nature — it is exercised only
Roman maxim which says that: There can be if a man passes over somebody's land. Under
no servitude over another servitude. Article 622 of the Civil Code, discontinuous
easements, whether apparent or not, may only
Easement; Effects; Discontinuous Easements; Permissive be acquired by virtue of a title. The Supreme
Use (2005) Court, in Abellana, Sr. v. Court of Appeals (G.R.
Don was the owner of an agricultural land with No. 97039, April 24, 1992), ruled that an
no access to a public road. He had been easement of right of way being discontinuous
passing through the land of Ernie with the in nature is not acquirable by prescription.
latter's acquiescence for over 20 years.
Further, possession of the easement by Don is
Subsequently, Don subdivided his property into
only permissive, tolerated or with the
20 residential lots and sold them to different
acquiescence of Ernie. It is settled in the case
persons. Ernie blocked the pathway and
of Cuaycong v. Benedicto (G.R. No. 9989,
refused to let the buyers pass through his land.
March 13, 1918) that a permissive use of a
Did Don acquire an easement of right of road over the land of another, no matter how
way? Explain. (2%)
long continued, will not create an easement of (2%)
way by prescription. SUGGESTED ANSWER:
ALTERNATIVE ANSWER: Prior to the grant of an easement, the buyers
Yes, Don acquired an easement of right of way. An easement that of the dominant estate have no other right than
is continuous and apparent can be acquired by prescription and to compel grant of easement of right of way.
title. According to Professor Tolentino, an easement of right of way
may have a continuous nature if
Since the properties of the buyers are
surrounded by other immovables and has no
adequate outlet to a public highway and the
isolation is not due to their acts, buyers may
demand an easement of a right of way
provided proper indemnity is paid and the
right of way demanded is the shortest and
least prejudicial to Ernie. (Villanueva v.
Velasco,
G.R. No. 130845, November 27, 2000).

Easement; Nuisance; Abatement (2002)


Lauro owns an agricultural land planted mostly
with fruit trees. Hernando owns an adjacent
land devoted to his piggery business, which is
two (2) meters higher in elevation. Although
Hernando has constructed a waste disposal
lagoon for his piggery, it is inadequate to
contain the waste water containing pig
manure, and it often overflows and inundates
Lauro’s plantation. This has increased the
acidity of the soil in the plantation, causing the
trees to wither and die. Lauro sues for
damages caused to his plantation. Hernando
invokes his right to the benefit of a natural
easement in favor of his higher estate, which
imposes upon the lower estate of Lauro the
obligation to receive the waters descending
from the higher estate. Is Hernando correct?
(5%)
SUGGESTED ANSWER:
Hernando is wrong. It is true that Lauro’s land is burdened
with the natural easement to accept or receive the water
which naturally and without interruption of man descends
from a higher estate to a lower estate. However, Hernando
has constructed a waste disposal lagoon for his piggery and
it is this waste water that flows downward to Lauro’s land.
Hernando has, thus, interrupted the flow of water and has
created and is maintaining a nuisance. Under Act. 697 NCC,
abatement of a nuisance does not preclude recovery of
damages by Lauro even for the past existence of a
nuisance.
CIVIL LAW Answers to the BAR as Arranged by Topics (Year 1990-2006)
The claim for damages may also be to time. As Tomas' business grows, the
premised in Art. 2191 need for use of
(4) NCC. modern conveyances requires widening of the easement.
ANOTHER ANSWER: the price of which he was willing to pay, to
Hernando is not correct. Article 637 of the enable him to construct a road to have access
New Civil Code provides that the owner of the to his plant nursery. Aniceta refused claiming
higher estate cannot make works which will that she had already allowed him a previous
increase the burden on the servient estate. road right of way. Is Tomas entitled to the
(Remman Enterprises, Inc. v. CA, 330 SCRA easement he now demands from Aniceta?
145 [2000]). The owner of the higher estate
may be compelled to pay damages to the SUGGESTED ANSWER:
owner of the lower estate. Art. 651 of the Civil Code provides that the width of the easement
must be sufficient to meet the needs of the dominant estate, and
Easements; Classification (1998) may accordingly change from time to time. It is the need of the
dominant estate which determines the width of the passage. These
Distinguish between: needs may vary from time
1. Continuous and discontinuous easements; |
2%]
Apparent and non-apparent easements; and
[2%]
Positive and negative easements. [1%]
SUGGESTED ANSWER:
CONTINUOUS EASEMENTS are those the
use of which is or may be incessant, without
the intervention of any act of man, while
DISCONTINUOUS EASEMENTS are those
which are used at intervals and depend upon
the acts of man. (Art. 615, Civil Code)
SUGGESTED ANSWER:
APPARENT EASEMENTS are those which are
made known and are continually kept in view
by external signs that reveal the use and
enjoyment of the same, while NON-
APPARENT EASEMENTS are those which
show no external indication of their existence.
(Art. 615, Civil Code)
SUGGESTED ANSWER:
POSITIVE EASEMENTS are those which
impose upon the owner of the servient estate
the obligation of allowing something to be
done or of doing it himself, while NEGATIVE
EASEMENTS are those which prohibit the
owner of the servient estate from doing
something which he could lawfully do if the
easement did not exist. (Art. 615. Civil Code)

Easements; Right of Way (1993)


Tomas Encarnacion's 3,000 square meter
parcel of land, where he has a plant nursery,
is located just behind Aniceta Magsino's two
hectare parcel land. To enable Tomas to have
access to the highway, Aniceta agreed to
grant him a road right of way a meter wide
through which he could pass. Through the
years Tomas' business flourished which
enabled him to buy another portion which
enlarged the area of his plant nursery. But he
was still landlocked. He could not bring in and
out of his plant nursery a jeep or delivery
panel much less a truck that he needed to
transport his seedlings. He now asked Aniceta
to grant him a wider portion of her property,
ALTERNATIVE ANSWER: ALTERNATIVE ANSWER:
The facts show that the need for a wider right Under Section 44, PD No. 1529, every
of way arose from the increased production registered owner receiving a certificate of title
owing to the acquisition by Tomas of an pursuant to a decree of registration, and every
additional area. Under Art. 626 of the Civil subsequent innocent purchaser for value, shall
Code, the easement can be used only for the hold the same free from all encumbrances
immovable originally contemplated. Hence, except those noted on said certificate. This
the increase in width is justified and should rule, however, admits of exceptions.
have been granted.
Under Act 496, as amended by Act No. 2011, and Section
Easements; Right of Way (2000) 4, Act 3621, an easement if not registered shall remain and
The coconut farm of Federico is surrounded shall be held to pass with the land until cutoff or
by the lands of Romulo. Federico seeks a right
of way through a portion of the land of
Romulo to bring his coconut products to the
market. He has chosen a point where he will
pass through a housing project of Romulo.
The latter wants him to pass another way
which is one kilometer longer. Who should
prevail? (5%)
SUGGESTED ANSWER:
Romulo will prevail. Under Article 650 of the
New Civil Code, the easement of right of way
shall be established at the point least
prejudicial to the servient estate and where
the distance from the dominant estate to a
public highway is the shortest. In case of
conflict, the criterion of least prejudice
prevails over the criterion of shortest
distance. Since the route chosen by Federico
will prejudice the housing project of Romulo,
Romulo has the right to demand that Federico
pass another way even though it will be
longer.
Easements; Right of Way; Inseparability (2001)
Emma bought a parcel of land from Equitable-
PCI Bank, which acquired the same from
Felisa, the original owner. Thereafter, Emma
discovered that Felisa had granted a right of
way over the land in favor of the land of
Georgina, which had no outlet to a public
highway, but the easement was not annotated
when the servient estate was registered
under the Torrens system. Emma then filed a
complaint for cancellation of the right of way,
on the ground that it had been extinguished
by such failure to annotate. How would you
decide the controversy? (5%)
SUGGESTED ANSWER:
The complaint for cancellation of easement of
right of way must fail. The failure to annotate
the easement upon the title of the servient
estate is not among the grounds for
extinguishing an easement under Art. 631 of
the Civil Code. Under Article 617, easements
are inseparable from the estate to which they
actively or passively belong. Once it attaches,
it can only be extinguished under Art. 631,
and they exist even if they are not stated or
annotated as an encumbrance on the Torrens
title of the servient estate. (II Tolentino 326,
1987 ed.)
CIVIL LAW Answers to the BAR as Arranged by Topics (Year 1990-2006)
extinguished by the registration of the servient consistent with this rule, where the distance to
estate. the street or
However, this provision has been suppressed in
Section 44, highway is shortest.
PD No. 1529. In other words, the registration of
the servient
estate did not operate to cut-off or extinguish 2) Is David entitled to a right of way in this
the right of case? Why or
way. Therefore, the complaint for the
cancellation of the right why not?
of way should be dismissed. SUGGESTED ANSWER:
No, David is not entitled to the right of way
being claimed.
The isolation of his subdivision was due to his
Easements; Right of Way; Requisites (1996) own act or
David is the owner of the subdivision in Sta. omission because he did not develop into an
Rosa, Laguna, access road the
without an access to the highway. When he rice field which he was supposed to purchase
applied for a according to
license to establish the subdivision, David his own representation when he applied for
represented that he a license to
will purchase a rice field located between his establish the subdivision (Floro us. Llenado,
land and the 244 SCRA713).
highway, and develop it into an access road.
But. when the
license was already granted, he did not bother
to buy the rice Ejectment Suit vs. Cancellation of Title (2005)
field, which remains unutilized until the In an ejectment case filed by Don against
present. Instead, he Cesar, can the
chose to connect his subdivision with the latter ask for the cancellation of Don's title
neighboring considering that
subdivision of Nestor, which has an access to he (Cesar) is the rightful owner of the lot?
the highway. Explain. (2%)
Nestor allowed him to do this, pending
negotiations on the SUGGESTED ANSWER:
compensation to be paid. When they failed to Cesar cannot ask for the cancellation of Don's
title even if he is
arrive at an
the rightful owner of the lot. In an action for
agreement, Nestor built a wall across the road
ejectment, the only
connecting issu involved on of possession de facto, the
with David's subdivision. David filed a complaint e is e purpose of
in court, for whic is protec own physic
the establishment of an easement of right of h merely to t the er from any al
way through the encroachment withou Th of lan
subdivision of Nestor which he claims to be the from t. e title the d or its
most ownership is not involved, for if a person is in
adequate and practical outlet to the highway. 1) actual possession
What are the thereof, he is entitled to be maintained and
requisites for the establishment of a compulsory respected in it even
easement of against the owner himself.
(Garcia
a right of way?
v. Anas, G.R. No. L-20617, May
31, 1965)
SUGGESTED ANSWER:
Art, 649, NCC. The owner, or any person who by Since the case filed by Don against Cesar is an
virtue of a ejectment case,
real right may cultivate or use any immovable the latter cannot ask for the cancellation of
which is Don's title. He
surrounded by other immovables pertaining to has to file the proper action where the issue of
other persons ownership
and without adequate outlet to a public over the property can be
highway, is entitled to raised.
demand a right of way through the neighboring Ejectment Suit; Commodatum (2006)
estates, after
payment of the property indemnity.
Alberto and Janine migrated to the United
States of America,
Should this easement be established in such a leaving behind their 4 children, one of whom is
manner that its Manny. They
use may be continuous for all the needs of the own a duplex apartment and allowed Manny to
dominant live in one of
estate, establishing a permanent passage, the the units. While in the United States, Alberto
indemnity shall died. His widow
consist of the value of the land occupied and the and all his children executed an Extrajudicial
amount of Settlement of
Alberto's estate wherein the 2door apartment
the damage caused to the servient estate. was assigned by
all the children to their mother, Janine.
Subsequently, she sold
In case the right of way is limited to the the property to George. The latter required
necessary passage for Manny to sign a
the cultivation of the estate surrounded by prepared Lease Contract so that he and
others and for the his family could
gathering of its crops through the servient continue occupying the unit. Manny refused
estate without a to sign the
permanent way, the indemnity shall consist in contract alleging that his parents allowed him
the payment of and his family
to continue occupying the
the damage cause by such encumbrance. premises.
This easement is not compulsory if the isolation If you were George's counsel, what legal
of the steps will you
immovable is due to the proprietor's own acts. take? Explain.
(564a). The (5%)
easement of right of way shall be established at
the point least SUGGESTED ANSWER:
prejudicial to the servient estate, and insofar as If I were George's counsel, I would first demand
that Manny
consistent vacate the apartment. If Manny file
with this rule, where the distance from the refuses, I will an
dominant estate to ejectment suit. When Manny was allowed by
a public highway may be the shortest (Art. 650, his parents to
NCC: Vda. de
occupy the premises, without compensation,
Baltazar v. CA. 245 SCRA 333} the contract of
commodatum was created. Upon the death of
ALTERNATIVE ANSWER: the father, the
contract was extinguished as it is a purely
The requisites for a compulsory easement of right of way are: (a) personal contract.
As the new owner of the apartment George is
the dominant estate is surrounded by other immovables and is entitled to
exercise his right of possession over
without an adequate outlet to a public street or highway; (b) proper the same.
indemnity must be paid; (c) the isolation must not be due to the acts
of the owner of the dominant estate; and (d) the right of way
claimed is at a point least prejudicial to the servient estate and, Extra-Judicial Partition; Fraud (1990)
X was the owner of a mete property.
insofar as is 10,000 square r X
married Y and out of their union. A, B and C
were born.
CIVIL LAW Answers to the BAR as Arranged by Topics (Year 1990-2006)
After the death of Y, X married Z and they begot share allotted by law to the finder since the
as phrase "by
children, D, E and F. After the death of X, the
children of chance" means "by accident", meaning an unexpected discovery.
the first and second marriages executed an
extrajudicial The liberal view, however, would sustain Tim's right to the allocated
partition of the aforestated property on May 1, share interpreting the phrase in question as meaning "by a stroke of
1970. D, E good fortune", which does not rule out deliberate or intentional
and F were given a one thousand square meter
portion of the search. It is submitted that the liberal view should prevail since in
property. They were minors at the time of the
execution of practical reality, hidden treasure is hardly ever found without
the document. D was 17 years old, E was 14 and
F was 12; conscious effort to find it, and the strict view would tend to render
and they were made to believe by A, B and C the codal provision in question illusory.
that unless they
sign the document they will not get any share. Z
was not
present then. In January 1974, D, E and F filed
an action in
court to nullify the suit alleging they discovered
the fraud
only in 1973. Hidden Treasures (1997)
(a) Can the minority of D, E and F be a basis to Marcelino, a treasure hunter as just a hobby,
nullify the has found a map
which appears to indicate the location of
partition? Explain your answer. hidden treasure. He
has an idea of the land where the treasure
(b) How about fraud? Explain your answer. might possibly be
found. Upon inquiry, Marcelino learns that the
SUGGESTED ANSWER: owner of the
(a) Yes, minority can be a basis to nullify the land, Leopoldo, is a permanent resident of
partition Canada, Nobody,
because D, E and F were not properly however, could give him Leopoldo's exact
represented by their address.
parents or guardians at the time they Ultimately, anyway, he enters the land and
contracted the extra- conducts a search.
judicial partition. (Articles 1327. 1391, Civil
Code). He succeeds.
(b) In the case of fraud, when through insidious Leopoldo learning of Marcelino's "find", seeks
words or to recover the
machinations of one party the other is induced treasure from Marcelino but the latter is not
to enter into willing to part
the contract without which he would not have with it. Failing to reach an agreement,
agreed to, the Leopoldo sues
action still prosper because under Art, 1391 of Marcelino for the recovery of the property.
the Civil Marcelino
Code, in case of fraud, the action for annulment contests the action. How would you decide the
may be case?
brought within four years from the discovery of
the fraud.
causes. To whom shall the treasure belong?
Explain.
Hidden Treasure (1995)
Tim came into possession of an old map SUGGESTED ANSWER:
showing where a purported cache of gold The treasure was found in a property of public
bullion was hidden. Without any authority dominion, the new river bed. Since Tim did
from the government Tim conducted a not have authority from the government and,
relentless search and finally found the therefore, was a trespasser, he is not entitled
treasure buried in a new river bed formerly to the one-half share allotted to a finder of
part of a parcel of land owned by spouses hidden treasure. All of it will go to the State.
Tirso and Tessie. The old river which used to In addition, under Art. 438 of the NCC in
cut through the land of spouses Ursula and order that the finder be entitled to the 1/2
Urbito changed its course through natural share, the treasure must be found by chance,
that is by sheer luck. In this case, since Tim SUGGESTED ANSWER:
found the treasure not by chance but because I would decide in favor of Marcelino since he
he relentlessly searched for it, he is not is considered a finder by chance of the hidden
entitled to any share in the hidden treasure. treasure, hence, he is entitled to one-half (1/2)
ALTERNATIVE ANSWER: of the hidden treasure. While Marcelino may
The law grants a one-half share to a finder of hidden treasure have had the intention to look for the hidden
provided he is not a trespasser and the finding is by chance. It is
submitted that Tim is not a trespasser despite his not getting treasure, still he is a finder by chance since it
authority from the government, because the new river bed where is enough that he tried to look for it. By
he found the treasure is property for public use (Art. 420 NCC), to chance in the law does not mean sheer luck
which the public has legitimate access. The question, therefore, such that the finder should have no intention
boils down to whether or not the finding was by chance in view of
the fact that Tim "conducted a relentless search" before finding the at all to look for the treasure. By chance
treasure. The strict or literal view holds that deliberate or intentional means good luck, implying that one who
search precludes entitlement to the one-half intentionally looks for the treasure is
embraced in the provision. The reason is that
it is extremely difficult to find hidden treasure
without looking for it deliberately. Marcelino
is not a trespasser since there is no
prohibition for him to enter the premises,
hence, he is entitled to half of the treasure.
ALTERNATIVE ANSWERS:
1. Marcelino did not find the treasure by
chance because he had a map, he knew the
location of the hidden treasure and he
intentionally looked for the treasure, hence,
he is not entitled to any part of the treasure.

Marcelino appears to be a trespasser and


although there may be a question of whether
he found it by chance or not, as he has found
the hidden treasure by means of a treasure
map, he will not be entitled to a finder's
share. The hidden treasure shall belong to
the owner.

The main rule is that hidden treasure belongs


to the owner of the land, building or other
property on which it is found. If it is found by
chance by a third person and he is not a
trespasser, he is entitled to one-half (1/2). If he
is a trespasser, he loses everything.

Mortgage; Pactum Commissorium (1999)


Page 64 of 119
CIVIL LAW Answers to the BAR as Arranged by Topics (Year 1990-2006)
X borrowed money from Y and gave a Are the right of redemption and the equity of
(a) piece of land redemption
as security by way of mortgage. It was given by law to a mortgagor the same?
expressly agreed Explain. (2%)
between the parties in the mortgage
contract that upon SUGGESTED ANSWER:
nonpayment of the debt on time by X, the The equity of redemption is different from
mortgaged the right of
land would already belong to Y. If X redemption. EQUITY OF REDEMPTION is the
defaulted in right of
paying, would Y now become the owner the mortgagor after judgment in a judicial
of the foreclosure to
redeem the property by paying to the court the
mortgaged land? Why? (3%) amount of the
judgment debt before the sale or confirmation
of the sale. On
Suppose in the preceding question, the the other hand, RIGHT OF REDEMPTION is
(b) agreement the right of
between X and Y was that if X failed to pay the the mortgagor to redeem the property sold at
mortgage an extra-judicial
debt on time, the debt shall be paid with the foreclosure by paying to the buyer in the
land mortgaged foreclosure sale the
by X to Y. Would your answer be the same amount paid by the buyer within one year from
as in the such sale.
preceding question? Explain. (3%)
SUGGESTED ANSWER: The assignment was a mortgage, not a cession,
No, Y would not become the owner of the land. of the leasehold rights. A cession would have
The stipulation is in the nature of pactum transferred ownership to the bank. However,
commissorium which is prohibited by law. The the grant of authority to the bank to sell the
property should be sold at public auction and leasehold rights in case of default is proof that
the proceeds thereof applied to the no such ownership was transferred and that a
indebtedness. Any excess shall be given to the mere encumbrance was constituted. There
mortgagor. would have been no need for such authority
SUGGESTED ANSWER:
had there been a cession.
No, the answer would not be the same. This is SUGGESTED ANSWER:
a valid stipulation and does not constitute No, the clause in question is not a pactum
pactum commissorium. In pactum commissorium. It is pactum commissorium
commissorium, the acquisition is automatic when default in the payment of the loan
without need of any further action. In the automatically vests ownership of the
instant problem another act is required to be encumbered property in the bank. In the
performed, namely, the conveyance of the problem given, the bank does not
property as payment (dacion en pago). automatically become owner of the property
upon default of the mortgagor. The bank has to
Mortgage; Pactum Commissorium (2001)
sell the property and apply the proceeds to the
To secure a loan obtained from a rural bank, indebtedness.
Purita assigned her leasehold rights over a
stall in the public market in favor of the bank. Mortgage; Right of Redemption vs. Equity of Redemption
The deed of assignment provides that in case (1999)
of default in the payment of the loan, the bank
shall have the right to sell Purita's rights over
the market stall as her attorney-in-fact, and to
apply the proceeds to the payment of the
loan. 1) Was the assignment of leasehold
rights a mortgage or a
cession? Why? (3%)
Assuming the assignment to be a mortgage,
does the provision giving the bank the power
to sell Purita's rights constitute pactum
commissorium or not? Why? (2%)
SUGGESTED ANSWER:
Nuisance; Family House; Not Nuisance per se (2006)
A drug lord and his family reside in a small
bungalow where they sell shabu and other
prohibited drugs. When the police found the
illegal trade, they immediately demolished the
house because according to them, it was a
nuisance per se that should be abated. Can
this demolition be sustained? Explain. (5%)
SUGGESTED ANSWER:
No, the demolition cannot be sustained. The
house is not a nuisance per se or at law as it is
not an act, occupation, or structure which is a
nuisance at all times and under any
circumstances, regardless of location or
surroundings. A nuisance per se is a nuisance in
and of itself, without regard to circumstances
[Tolentino, p. 695, citing Wheeler v. River Falls
Power Co., 215 Ala. 655, 111 So. 907].

Nuisance; Public Nuisance vs. Private Nuisance (2005)


State with reason whether each of the
following is a nuisance, and if so, give its
classification, whether public or private:
Article 694 of the Civil Code defines nuisance
as any act, omission, establishment, business,
condition or property, or anything else which
injures or endangers the health or safety of
others, or annoys or offends the senses, or
shocks, defies or disregards decency or
morality or obstructs or interferes with the
free passage of any public highway or street or
any body of water or hinders or impairs the
use of property.

It is a public nuisance if it affects a community


or neighborhood or any considerable number
of persons. It is a direct encroachment upon
public rights or property which results
injuriously to the public. It is a private
nuisance, if it affects only a person or small
number of persons. It violates only private
rights.
A squatter's hut (1%)
If constructed on public streets or riverbeds, it
is a public nuisance because it obstructs the
free use by the public of said places. (City of
Manila v. Garcia, G.R. No. L -26053, February
21,1967) If constructed on private land, it is a
private nuisance because it hinders or impairs
the use of the property by the owner.

A swimming pool (1%)


This is not a nuisance in the absence of any
unusual condition or artificial feature other
than the mere water. In
Hidalgo Enterprises v. Balandan (G.R. No. L-
3422, June 13,
1952), the Supreme Court ruled that a
swimming pool is but
CIVIL LAW Answers to the BAR as Arranged by Topics (Year 1990-2006)
a duplication of nature — thus, could not be
considered as a (b) The mortgage shall not bind the 1/3 right and interest
of A and shall be deemed to cover only the
nuisance. rights and
interests of B and C in the house and lot. The
mortgage shall
be limited to the portion (2/3) which may be allotted
c) A house of prostitution (1%) to B
Irrespective of its location and how its business
is conducted, and C in the partition (Art. 493, Civil Code).
it is a nuisance since it defies, shocks and
disregards decency
and morality. It is a public nuisance because of
its injury to the SUGGESTED ANSWER:
(c) B's sole decision to build the concrete fence is not
public. binding
upon A and C. Expenses to improve the thing owned
in
d) A noisy or dangerous factory in a private comm must be decided upon by a majority of
land (1%) on the
If the noise injuriously affects the health and co-owners who represent the controlling interest
comfort of (Arts. 489
ordinary people in the vicinity to an
unreasonable extent, it is and 492. Civil Code).
a nuisance. It is a public nuisance because there
is a tendency
to annoy the public. (Velasco v. Manila Electric
Co., G.R. No. SUGGESTED ANSWER:
(d) C's sole decision to build the grotto is not binding
upon A
L-18390, August 6, 1971)
and B who cannot be required to contribute to the
expenses
for the embellishment of the thing owned in common
e) Uncollected garbage (1%) if not
It will become a nuisance if it substantially decided upon by the majority of the coowners who
impairs the represent
comfort and enjoyment of the adjacent the controlling interest (Arts. 489 and 492, Civil
occupants. The Code).
annoyance and the smell must be substantial as
to interfere
sensibly with the use and enjoyment by persons
of ordinary
sensibilities. It is a public nuisance because of
its injury to the SUGGESTED ANSWER:
(e) The sale to X shall not bind the 1/3 share of B and shall
public. be deemed to cover only the 2/3 share of A and C in
the
land (Art. 493, Civil Code). B shall have the right to
Ownership; Co-Ownership (1992) redeem
A, B and C are the co-owners in equal shares of the 2/3 share sold to X by A and C since X is a third
a residential person
house and lot. During their co-ownership, the (Art. 1620, Civil Code).
following acts
were respectively done by the co-owners: 1) A
undertook the
repair of the foundation of the house, Ownership; Co-Ownership; Prescription (2000)
In 1955, Ramon and his sister Rosario inherited a
then tilting to one side, to prevent the house
parcel of
from
land in Albay from their parents. Since Rosario was
collapsing. 2) B and C mortgaged the house and
gainfully
lot to secure
employed in Manila, she left Ramon alone to possess
a loan. 3) B engaged a contractor to build a
and
concrete fence all
cultivate the land. However, Ramon never shared the
harvest
around the lot. 4) C built a beautiful grotto in with Rosario and was even able to sell one-half of the
the land in
garden. 5) A and C sold the land to X for a very 1985 by claiming to be the sole heir of his parents.
good Having
reache retirement age in 1990 Rosario returned
d to the
price.
province and upon learning what had transpired,
demanded
(a Is A's sole decision to repair the that the remaining half of the land be given to her as
) foundation of her
the house binding on B and C? May A share. Ramon opposed, asserting that he has already
require B and acquired
C to contribute their 2/3 share of the ownership of the land by prescription, and that
expense? Rosario is
barred by laches from demanding partition and
Reasons. reconveyance.
(b What is the legal effect of the
) mortgage Decide the conflicting claims. (5%)
contract executed by B and C? Reasons.
(c Is B's sole decision to build the SUGGESTED ANSWER:
) fence binding Ramon is wrong on both counts: prescription and
upon A and C? May B require A and C to laches. His
contribute possession as co-owner did not give rise to
acquisitive
their 2/ 3 share of the expense? Reasons. prescription. Possession by a co-owner is deemed not
adverse
to the other co-owners but is, on the contrary,
(d Is C's sole decision to build the
deemed
) grotto binding
beneficial to them (Pongon v. GA, 166 SCRA 375).
upon A and B? May C require A and B to
Ramon's
contribute
possession will become adverse only when he has
repudiated
their 2/ 3 share of the expense? Reasons. the co-ownership and such repudiation was made
known to
Rosario. Assuming that the sale in 1985 where
(e What are the legal effects of the
Ramon
) contract of
claimed he was the sole heir of his parents amounted
to a
sale executed by A. C and X? Reasons.
repudiation of the co-ownership, the prescriptive
SUGGESTED ANSWER: period
(a) Yes. A's sole decision to repair the began to run only from that time. Not more than 30
foundation is binding years
upon B and C. B and C must contribute 2/3 of having lapsed since then, the claim of Rosario has not
the expense. as yet
Each co-owner has the right to compel the other prescribed. The claim of laches is not also
co-owners meritorious. Until
to contribute to the expense of preservation of the repudiation of the co-ownership was made known
the thing (the to the
house) owned in common in proportion to their other co-owners, no right has been violated for
respective the said
co-owners to vindicate. Mere delay in vindicating the
interests (Arts. 485 and 488, Civil Code). right,
standing alone, does not constitute laches.
SUGGESTED ANSWER:
CIVIL LAW Answers to the BAR as Arranged by Topics 1940, the bank foreclosed the mortgage for non-
ALTERNATIVE ANSWER: payment of the principal obligation. As the only
Ramon has acquired the land by acquisitive bidder at the extrajudicial foreclosure sale, the
prescription, and because of laches on the part bank bought the property and was later issued a
of Rosario. Ramon's possession of the land was certificate of sale. The war supervened in 1941
adverse because he asserted sole ownership without the bank having been able to obtain
thereof and never shared the harvest actual possession of the property which
therefrom. His adverse possession having been remained with A's three children who
continuous and uninterrupted for more than 30 appropriated for themselves the income from it.
years, Ramon has acquired the land by In 1948, B bought the property from the bank
prescription. Rosario is also guilty of laches not using the money he received as back pay from
having asserted her right to the harvest for the U.
more than 40 years. S. Government, and utilized the same in
agribusiness. In 1960, as B's business
flourished, C and D sued B for partition and
Ownership; Co-Ownership; Prescription (2002)
accounting of the income of the property,
Senen and Peter are brothers. Senen migrated claiming that as heirs of their father they were
to Canada early while still a teenager. Peter co-owners
stayed in Bulacan to take care of their
widowed mother and continued to work on the
Family farm even after her death. Returning to
the country some thirty years after he had left,
Senen seeks a partition of the farm to get his
share as the only co-heir of Peter. Peter
interposes his opposition, contending that
acquisitive prescription has already set in and
that estoppel lies to bar the action for
partition, citing his continuous possession of
the property for at least 10 years, for almost
30 years in fact. It is undisputed that Peter has
never openly claimed sole ownership of the
property. If he ever had the intention to do so,
Senen was completely ignorant of it. Will
Senen’s action prosper? Explain. (5%).
SUGGESTED ANSWER:
Senen’s action will prosper. Article 494 of the
New Civil Code provides that ―no
prescription shall run in favor of a co-owner or
co-heir against his co-owners or co-heirs so
long as he expressly or impliedly recognizes
the coownership nor notified Senen of his
having repudiated the same.
ALTERNATIVE ANSWER:
Senen’s action will prosper. This is a case of
implied trust. (Art 1441, NCC) For purposes of
prescription under the concept of an owner
(Art. 540, NCC). There is no such concept
here. Peter was a co-owner, he never claimed
sole ownership of the property. He is therefore
estopped under Art. 1431, NCC.
Ownership; Co-Ownership; Redemption (1993)
In 1937, A obtained a loan of P20,000.00 from
the National City Bank of New York, an
American-owned bank doing business in the
Philippines. To guarantee payment of his
obligation, A constituted a real estate
mortgage on his 30-hectare parcel of
agricultural land. In 1939, before he could pay
his obligation. A died intestate leaving three
children. B, a son by a first marriage, and C
and D, daughters by a second marriage. In
(Year 1990-2006) Ownership; Co-Ownership; Redemption (2002)
thereof and offering to reimburse B for Antonio, Bart, and Carlos are brothers. They purchased
whatever he had paid in purchasing the from their parents specific portions of a parcel of land as
property from the bank. In brief, how will evidenced by three separates deeds of sale, each deed
you answer the complaint of C and D, if you referring to a particular lot in meter and bounds. When the
were engaged by D as his counsel?
SUGGESTED ANSWER:
deeds were presented for registration, the Register of
As counsel of B, I shall answer the complaint Deeds could not issue separate certificates of Title had to
as follows: When B bought the property, it be issued, therefore, in the names of three brothers as
was not by a right of redemption since the coowners of the entire property. The situation has not
period therefore had already expired. Hence, changed up to now, but each of the brothers has been
B bought the property in an independent receiving rentals exclusively from the lot actually
unconditional sale. C and D are not co-owners purchased by him. Antonio sells his lot to a third person,
with B of the property. Therefore, the suit of with notice to his brothers. To enable the buyer to secure a
C and D cannot prosper. new title in
ALTERNATIVE ANSWER:
As counsel of B, I shall answer the complaint
as follows: From the facts described, it would
appear that the Certificate of sale has not
been registered. The one-year period of
redemption begins to run from registration.
In this case, it has not yet even commenced.
Under the Rules of Court, the property may
be released by the Judgment debtor or his
successor in interest. (Sec. 29, Rule 27). It
has been held that this includes a joint owner.
(Ref. Magno vs.Ciola, 61 Phil. 80).

Ownership; Co-Ownership; Redemption (2000)


Ambrosio died, leaving his three daughters,
Belen, Rosario and Sylvia a hacienda which
was mortgaged to the Philippine National
Bank due to the failure of the daughters to
pay the bank, the latter foreclosed the
mortgage and the hacienda was sold to it as
the highest bidder. Six months later, Sylvia
won the grand prize at the lotto and used
part of it to redeem the hacienda from the
bank. Thereafter, she took possession of the
hacienda and refused to share its fruits with
her sisters, contending that it was owned
exclusively by her, having bought it from the
bank with her own money. Is she correct or
not? (3%)
SUGGESTED ANSWER:
Sylvia is not correct. The 3 daughters are the
co -owners of the hacienda being the only
heirs of Ambrosio. When the property was
foreclosed, the right of redemption belongs
also to the 3 daughters. When Sylvia
redeemed the entire property before the
lapse of the redemption period, she also
exercised the right of redemption of her co-
owners on their behalf. As such she is
holding the shares of her two sisters in the
property, and all the fruits corresponding
thereto, in trust for them. Redemption by
one co-owner inures to the benefit of all
(Adille v. CA.157 SCRA 455). Sylvia, however,
is entitled to be reimbursed the shares of her
two sisters in the redemption price.
CIVIL LAW Answers to the BAR as Arranged by Topics (Year 1990-2006)
his name, the deed of sale was made to Salvador, a timber concessionaire, built
refer to undivided on his lot a
interest in the property of the seller (Antonio), warehouse where he processes and stores
with the metes his timber for
and bounds of the lot sold being stated. Bart shipment. Adjoining the warehouse is a
and Carlos furniture factory
reacted by signifying their exercise of their owned by NARRAMIX of which Salvador is
right of a majority
redemption as co owners. Antonio in his behalf stockholder. NARRAMIX leased space in the
and in behalf warehouse
of his buyer, contends that they are no where it placed its furniture-making
longer coowners, machinery.
although the title covering the property has 1. How would you classify the furniture-
remained in their making machinery
names as such. May Bart and Carlos still as property under the Civil Code?
redeem the lot sold Explain.
2. Suppose the lease contract between
by Antonio? Explain. (5%) Salvador and
NARRAMIX stipulates that at the end of the
lease the
machinery shall become the property of the
SUGGESTED ANSWER: lessor, will your
No, they may not redeem because there was no answer be the same?
Coownership Explain.
among Antonio, Bart, and Carlos to start with.
Their parents SUGGESTED ANSWER:
already partitioned the land in selling separate 1. The furniture-making machinery is
portions to movable property
them. The situation is the same as in the because it was not owner of the
case Si installed by the tenement.
To become immovable under Art. 415 (5) of the
v. Court of Appeals, (342 SCRA 653 [2000]). NCC, the
machinery must be installed by the owner of
the tenement.
Possession (1998)
Using a falsified manager's check, Justine, as
the buyer, was ALTERNATIVE ANSWER:
able to take delivery of a second hand car which It depends on the circumstances of the case. If
she had just the machinery
bought from United Car Sales Inc. The sale was was attached in a fixed manner, in such a way
registered that it cannot
with the Land Transportation Office. A week be separated from the tenement without
later, the seller breaking the material
learned that the check had been dishonored, but or causing deterioration thereof, it is
by that time, immovable property
Justine was nowhere to be seen. It turned out [Art. 415 (3), NCC]. However, if the
that Justine had machinery can be
sold the car to Jerico, the present possessor transported from place to place without
who knew impairment of the
nothing about the falsified check. In a suit by tenement to which they were fixed, then it
United Car is movable
Sales, Inc. against Jerico for recovery of the property. [Art. 416 (4),
car, plaintiff NCC]
alleges it had been unlawfully deprived of
its property
through fraud and should, consequently,
be allowed to SUGGESTED ANSWER:
recover it without having to reimburse the 2. It is immovable property. When there is a
provision in the
defendant for the leas contract making the at the end of the
price the latter had paid. Should the suit e lessor, lease,
prosper? [5%] owner of the machinery installed by the
lessee, the said
machinery is considered to have been installed
SUGGESTED ANSWER: by the lessor
The suit should prosper as to the recovery through the lessee who acted merely as
of the car. his agent. Having
However, since Jerico was not guilty of any been installed by the owner of the tenement,
fraud and appears the machinery
to be an innocent purchaser for value, he became immovable .under Art. 415 of the
should be NCC. (Davao
reimbursed for the price he paid. This is without Sawmill v. Castillo 61 Phil.
prejudice to 709)
United Car Sales, Inc. right of action
against Justine. As
between two innocent parties, the party causing
the injury Property; Real vs. Personal Property (1997)
should suffer the loss. Therefore, United Car Pedro is the registered owner of a parcel of
Sales, Inc. land situated in
Malolos, Bulacan. In 1973, he mortgaged
should suffer the loss. the land to the
Philippine National Bank (PNB) to
ALTERNATIVE ANSWER: secure a loan of
Yes, the suit will prosper because the criminal P100.000.00. For Pedro's failure to pay the
act of estafa loan, the PNB
should be deemed to come within the meaning foreclosed on the mortgage in 1980, and the
of unlawful land was sold at
deprivation under Art. 559, Civil Code, as public auction to PNB for being the highest
without it plaintiff bidder. PNB
would not have parted with the possession secured title thereto in
of its car. 1987.
In the meanwhile, Pedro, who was still in
ANOTHER ANSWER: possession of the
No, the suit will not prosper. The sale is valid land, constructed a warehouse on the property.
and Jerico is a In 1988, the
PNB sold the land to Pablo, the Deed of Sale
buyer in good faith. was amended
in 1989 to include the
ANOTHER ANSWER: warehouse.
Under the law on Sales, when the thing sold is
delivered by
the seller to the buyer without reservation of
ownership, the Pedro, claiming ownership of the warehouse,
ownership is transferred to the buyer. Therefore files a complaint
in the suit of to annul the amended Deed of Sale before the
United Car Sales, Inc. against Jerico for the Regional Trial
recovery of the Cou Quezon City, resides, th
car, the plaintiff should not be allowed to rt of where he against both e
recover the car an Pablo. The PNB a motion to th
without reimbursing the defendant for the price PNB d filed dismiss e
that the latter complaint for improper venue contending that
paid. (EDCA Publishing and Distributing Corp. the warehouse
vs. Santos, 184 is real property under Article 415(1) of the
Civil Code and
therefore the action should have instead been
SCRA 614, April 26, 1990) filed in Malolos,
Bulacan. Pedro claims otherwise. The
question arose as to
Property; Real vs. Personal Property (1995) whether the warehouse should be considered
as real or as
personal property.
CIVIL LAW Answers to the BAR as Arranged by Topics (Year 1990-2006)
latter vacate the premises and deliver the
If consulted, what would your legal advice be? same to the
former. Petronila refused to vacate the place
SUGGESTED ANSWER: on the ground
The warehouse which is a construction adhered that the usufruct in her favor would expire
to the soil is only on 1 June
an immovable by nature under Art. 415 (1) and 1998 when Manuel would have reached his
the proper 30th birthday and
venue of any case to recover ownership of the that the death of Manuel before his 30th
same, which is birthday did not
what the purpose of the complaint to annul the extinguish the usufruct. Whose contention
amended should be
Deed of Sale amounts to, should be the place
where the accepted?
property is located, or the RTC of Bulacan. SUGGESTED ANSWER:
Petronila's contention is correct. Under Article
ADDITIONAL ANSWERS: 606 of the
1. Buildings are always immovable property, and Civil Code, a usufruct granted for the time that
even in the may elapse
instances where the parties to a contract seem to before a third person reaches a certain age
have dealt with shall subsist for
it separate and apart from the land on which it the number of years specified even if the third
stood in no wise person should
does it change its character as immovable die unless there is an express stipulation in the
property. A building is contract that
an immovable even if not erected by the owner of states otherwise. In the case at bar, there is no
the land. The express
only criterion is union or incorporation with the stipulation that the consideration for the
soil. (Ladera vs. usufruct is the
existence of Petronila's son. Thus, the general
Hodges (CA) 48 rule and not
O.G. 4374) (Reyes and Puno, Outline of
Philippine Civil Law, the exception should apply in this case.
Vol. 2. p.7) ALTERNATIVE ANSWER:
This is a usufruct which is clearly intended for
the benefit of
2. The warehouse built by Pedro on the Manuel until he reaches 30 yrs. of age with
mortgaged property Petronila serving
is real property within the context of Article 415 only as a conduit, holding the property in trust
of the New for his
Civil Code, although it was built by Pedro benefit. The death of Manuel at the age of 26
after the therefore,
foreclosure sale without the knowledge and
consent of the terminated the usufruct.
new owner which makes him a builder in bad
faith, this does
not alter the character of the warehouse as a
real property by
incorporation. It is a structure which cannot be
removed LAND TRANSFER &
without causing injury to the land. So, my
advice to Pedro is
to file the case with the RTC of Bulacan, the
situs of the DEEDS
property,
(Note: If the examinee does not mention that
the structure was built Acquisition of Lands; Citizenship Requirement (2003)
In 1970, the spouses Juan and Juana de la
by a builder in bad faith, it should be given
full credit). Cruz, then
Filipinos, bought the parcel of unregistered
land in the
Sower; Good Faith/ Bad Faith (2000) Philippines on which they built a house which
Felix cultivated a parcel of land and planted it to became their
sugar cane,
residence. In 1986, they migrated to Canada
believing it to be his own. When the crop was and became
eight months Canadian citizens. Thereafter, in 1990, they
old, and harvestable after two more months, a applied, opposed
resurvey of by the Republic, for the registration of the
the land showed that it really belonged to Fred. aforesaid land in
What are the their names. Should the application of the
spouses de la Cruz
options available to Fred? (2%)
be granted over the Republic’s opposition?
SUGGESTED ANSWER: Why? 5%
As to the pending crops planted by Felix in good
faith, Fred
has the option of allowing Felix to continue the SUGGESTED ANSWER:
cultivation Yes, the application should be granted. As a
and to harvest the crops, or to continue the rule, the
cultivation and Constitution prohibits aliens from owning
harvest the crops himself. In the latter option, private lands in the
however, Felix Philippines. This rule, however, does not apply
shall have the right to a part of the expenses of to the spouses
cultivation Juan and Juana de la Cruz because at the time
and to a part of the net harvest, both in they acquired
proportion to the ownership over the land, albeit imperfect, they
were still
time of possession. (Art. 545 NCC),
Filipino citizens. The application for
registration is a mere
ALTERNATIVE ANSWER:
Since sugarcane is not a perennial crop. Felix is confirmation of the imperfect title which the
considered a spouses have
sower in good faith. Being so, Art. 448 applies. already acquired before they became
The options Canadian citizens.
available to Fred are: (a) to appropriate the crop (Republic v. CA, 235 SCRA 567 [1994]).
after paying
Felix the indemnity under Art. 546, or (b) to
require Felix to
Adverse Claims; Notice of Levy (1998)
pay rent. Section 70 of Presidential Decree No. 1529,
concerning
adverse claims on registered land, provides a
Usufruct (1997) 30-day period of
On 1 January 1980, Minerva, the owner of a effectivity of an adverse claim, counted from
building, granted the date of its
Petronila a usufruct over the property until 01 registration. Suppose a notice of adverse claim
June 1998 based upon a
when Manuel, a son of Petronila, would have contract to sell was registered on March 1,
reached his 1997 at the
30th birthday. Manuel, however, died on 1 June instance of the BUYER, but on June 1, 1997, or
1990 when after the
lapse of the 30-day period, a notice of levy on
execution in
he was only 26 years old.
favor of a JUDGMENT CREDITOR was also
registered to
Minerva notified Petronila that the usufruct had enforce a final judgment for money against the
been registered
owner. Then, on June 15, 1997 there having
extinguished by the death of Manuel and
been no formal
demanded that the
cancellation of his notice of adverse claim, the
BUYER pays
CIVIL LAW Answers to the BAR as Arranged by Topics (Year 1990-2006)
to the seller-owner the agreed purchase price in residential, commercial, industrial, or similar
full and productive
registers the corresponding deed of sale. purposes, and only by lease when not needed
Because the by the
annotation of the notice of levy is carried over to
the new title government for public service.
in his name, the BUYER brings an action
against the
JUDGMENT CREDITOR to cancel such (2) If the land is suited or actually used for
annotation, but fishpond or
the latter claims that his lien is superior aquaculture purposes, it comes under the
because it was Jurisdiction of the
annotated after the adverse claim of the BUYER Bureau of Fisheries and Aquatic Resources
had ipso (BFAR) and can
facto ceased to be effective. Will the suit
prosper? [5%] only be acquired by lease. (P.D. 705)
SUGGESTED ANSWER: Regina has been leasing foreshore land from
The suit will prosper. While an adverse claim the Bureau of Fisheries and Aquatic Resources
duly annotated at the back of a title under for the past 15 years. Recently, she learned that
Section 7O of P.D. 1529 is good only for 30 Jorge was able to obtain a free patent from the
days, cancellation thereof is still necessary to Bureau of Agriculture, covering the same land,
render it ineffective, otherwise, the inscription on the basis of a certification by the District
thereof will remain annotated as a lien on the Forester that the same is already "alienable and
property. While the life of adverse claim is 3O disposable". Moreover, Jorge had already
days under P.D. 1529, it continuous to be registered the patent with the Register of
effective until it is canceled by formal petition Deeds of the province, and he was issued an
filed with the Register of Deeds. Original Certificate of Title for the same.
The cancellation of the notice of levy is Regina filed an action for annulment of Jorge's
title on the ground that it was obtained
justified under Section 108 of P.D. 1529
fraudulently. Will the action prosper? (2%)
considering that the levy on execution can not SUGGESTED ANSWER:
be enforced against the buyer whose adverse An action for the annulment of Jorge's Original
claim against the registered owner was Certificate of
recorded ahead of the notice of levy on Title will prosper on the following grounds:
execution. Under Chapter IX of C .A, No. 141, otherwise
known as the Public Land Act, foreshore lands
Annotation of Lis Pendens; When Proper (2001) are disposable for
Mario sold his house and lot to Carmen for P1
million payable in five (5) equal annual
installments. The sale was registered and title
was issued in Carmen's name. Carmen failed
to pay the last three installments and Mario
filed an. action for collection, damages and
attorneys fees against her. Upon filing of the
complaint, he caused a notice of lis pendens to
be annotated on Carmen's title. Is the notice
of lis pendens proper or not? Why? (5%)
SUGGESTED ANSWER:
The notice of lis pendens is not proper for the
reason that the case filed by Mario against
Carmen is only for collection, damages, and
attorney's fees.

Annotation of a lis pendens can only be done


in cases involving recovery of possession of
real property, or to quiet title or to remove
cloud thereon, or for partition or any other
proceeding affecting title to the land or the
use or occupation thereof. The action filed by
Mario does not fall on anyone of these.
Foreshore Lands (2000)
In the case at bar, Rod only forged Cesar's
Free Patent is a mode of concession under signature on the -Deed of Sale. It is very
Section 41, Chapter VII of the Public Land apparent that there was bad faith on the part
Act, which is applicable only for agricultural of Rod from the very beginning. As such, he is
lands. not entitled to the protection of the Land
Registration Act.
The certificate of the district forester that the
Discuss the rights of Don, if any, over the
land is already "alienable and disposable"
property. (2%)
simply means that the land is no longer SUGGESTED ANSWER:
needed for forest purposes, but the Bureau of
Lands could no longer dispose of it by free
patent because it is already covered by a
lease contract between BFAR and Regina.
That contract must be respected.

The free patent of Jorge is highly irregular and


void ab
initio, not only because the Bureau has no
statutory authority to issue a free patent over
a foreshore area, but also because of the false
statements made in his sworn application
that he has occupied and cultivated the land
since July 4, 1945, as required by the free
patent law. Under Section 91 of the Public
Land Act, any patent concession or title
obtained thru false representation is void ab
initio. In cases of this nature, it is the
government that shall institute annulment
proceedings considering that the suit carries
with it a prayer for the reversion of the land
to the state. However, Regina is a party in
interest and the case will prosper because
she has a lease contract for the same land
with the government.

Forgery; Innocent Purchaser; Holder in Bad Faith (2005)


Rod, the owner of an FX taxi, found in his
vehicle an envelope containing TCT No.
65432 over a lot registered in Cesar's name.
Posing as Cesar, Rod forged Cesar's signature
on a Deed of Sale in Rod's favor. Rod
registered the said document with the
Register of Deeds, and obtained a new title in
his name. After a year, he sold the lot to Don,
a buyer in good faith and for value, who also
registered the lot in his name.
Did Rod acquire title to the land? Explain.
(2%)
SUGGESTED ANSWER:
No, Rod did not acquire title to the land. The
inscription in the registry, to be effective,
must be made in good faith. The defense of
indefeasibility of a Torrens Title does not
extend to a transferee who takes the
certificate of title with notice of a flaw. A
holder in bad faith of a certificate of title is
not entitled to the protection of the law, for
the law cannot be used as a shield for frauds.
(Samonte v. Court of Appeals, G.R.
No. 104223, July 12, 2001)
CIVIL LAW Answers to the BAR as Arranged by Topics (Year 1990-2006)
It is a well-known rule in this jurisdiction that persons
dealing with registered land have the legal right The mortgage to Desiderio should be
to rely on the cancelled without
face of the Torrens Certificate of Title and to prejudice to his right to go after Catalino
dispense with and/or the
the need to inquire further, except when the government for compensation from the
party concerned assurance fund.
has actual knowledge of facts and circumstances
that would
impel a reasonably cautious man to make such
inquiry. Fraud; Procurement of Patent; Effect (2000)
(Naawan Community Rural Bank v. Court of In 1979, Nestor applied for and was granted a
Appeals, G.R. No. Free Patent
over a parcel of agricultural land with an area
128573, January 13, 2003) of 30 hectares,
located in General Santos City. He presented
the Free Patent
In the given problem, the property was already to the Register of Deeds, and he was issued a
registered in corresponding
the name of Rod when he bought the same from Original Certificate of Title (OCT) No. 375,
the latter. Subsequently,
Thus, Don could be considered as a buyer in Nestor sold the land to Eddie. The deed of
good faith and sale was
for value. However, since Rod did not actually submitted to the Register of Deeds and on the
sell any basis thereof,
property to him, Don has no right to retain OCT No, 375 was cancelled and Transfer
ownership over Certificate of Title
the property. He has only the right to recover (TCT) No. 4576 was issued in the name of
the purchase Eddie. In 1986,
the Director of Lands filed a complaint for
price plus damages. annulment of
OCT No, 375 and TCT No. 4576 on the ground
that Nestor
obtained the Free Patent through fraud. Eddie
Forgery; Innocent Purchaser; Mirror Principle (1991) filed a motion
Bruce is the registered owner, of a parcel of to dismiss on the ground that he was an
land with a innocent purchaser
building thereon and is in peaceful possession for value and in good faith and as such, he has
thereof. He acquired a title
pays the real estate taxes and collects the
rentals therefrom. to the property which is valid, unassailable and
Later, Catalino, the only brother of Bruce, filed a indefeasible.
petition
where he, misrepresenting to be the attorney-in- Decide the motion. (5%)
fact of Bruce SUGGESTED ANSWER:
and falsely alleging that the certificate of title The motion of Nestor to dismiss the complaint
was lost, for annulment
succeeded in obtaining a second owner's of O.C.T. No. 375 and T.C.T. No. 4576 should be
duplicate copy of denied for
the title and then had the same transferred in the following reasons: 1) Eddie cannot claim
his name protection as an
through a simulated deed of sale in his favor.
Catalino then innocent
mortgaged the property to Desiderio who had purchaser for value nor can he interpose
the mortgage the defense of
annotated on the title. Upon learning of the indefeasibility of his title, because his TCT
fraudulent is rooted on a
transaction, Bruce filed a complaint against void title. Under Section 91 of CA No. 141,
Catalino and as amended,
Desiderio to have the title of Catalino and the otherwise known as the Public Land Act,
mortgage in statements of
favor of Desiderio declared null and void. Will material facts in the applications for public
the complaint land must be
prosper, or will the title of Catalino and the under oath. Section 91 of the same act
mortgage to provides that such
statements shall be considered as essential
Desiderio be sustained? conditions and
parts of the concession, title, or permit
SUGGESTED ANSWER: issued, any false
The complaint for the annulment of Catalino's statement therein, or omission of facts shall
Title will ipso facto
prosper. In the first place, the second owner's produce the cancellation of the concession.
copy of the The patent
title secured by him from the Land Registration issued to Nestor in this case is void ab initio
Court is void not only
ab initio, the owner's copy thereof having never because it was obtained by fraud but
been lost, let also because it
alone the fact that said second owner's copy of covers 30 hectares which is far beyond the
the title was maximum of
fraudulently procured and improvidently issued
by the Court. 24 hectares provided by the free patent law.
In the second place, the Transfer Certificate of
Title procured
by Catalino is equally null and void, it having The government can seek annulment of
been issued on 2) the original
the basis of a simulated or forged Deed of Sale. and transfer certificates of title and the
A forged reversion of the land
deed is an absolute nullity and conveys no title. to the state. Eddie's defense is untenable.
The mortgage The protection
in favor of Desiderio is likewise null and void afforded by the Torrens System to an innocent
because the purchaser for
mortgagor is not the owner of the mortgaged value can be availed of only if the land has
property. While been titled thru
it may be true that under the "Mirror Principle" judicial proceedings where the issue of
of the Torrens fraud becomes
System of Land Registration, a buyer or academic after the lapse of one (1) year from
mortgagee has the the issuance of
right to rely on what appears on the Certificate the decree of registration. In public land
of Title, and grants, the action of
in the absence of anything to excite suspicion, is the government to annul a title fraudulently
under no obtained does
obligation to look beyond the certificate and not prescribe such action and will not be
investigate the barred by the
mortgagor's title, this rule does not find transfer of the title to an innocent purchaser
application in the for value.
case at hand because here. Catalino's title
suffers from two
fatal infirmities, namely: a) The fact that it
emanated from a
forged deed of a Homestead Patents; Void Sale (1999)
In 1950, the Bureau of Lands issued a
Homestead patent to
simulated sale; b) The fact that it was derived A. Three years later, A sold the homestead to B.
from a A died in
1990, and his heirs filed an action to recover
fraudulently the homestead
procured or improvidently issued second from B on the ground that its sale by their
owner's copy, father to the latter
the real owner's copy being still intact and in is void under Section 118 of the Public
the Land Law. B
contends, however, that the heirs of A cannot
possession of the true owner, Bruce. recover the

Page 71 of 119
CIVIL LAW Answers to the BAR as Arranged by Topics (Year 1990-2006)
homestead from him anymore because Cesar bought a residential condominium
their action has unit from High
prescribed and that furthermore, A was in Rise Co. and paid the price in full. He moved
pari delicto. into the unit, but
somehow he was not given the Condominium
Decide. (5%) Certificate of
Title covering the property. Unknown to him,
SUGGESTED ANSWER: High Rise Co.
The sale of the land by A to B 3 years after subsequently mortgaged the entire
issuance of the condominium building to
homestead patent, being in violation of Section Metrobank as security for a loan of P500
118 of the million. High Rise
Public Land Act, is void from its Co. failed to pay the loan and the bank
inception. foreclosed the
mortgage. At the foreclosure sale, the bank
acquired the
The action filed by the heirs of B to declare the building, being the highest bidder. When Cesar
nullity or learned about
inexistence of the contract and to recover the this, he filed an action to annul the foreclosure
land should be sale insofar as
given due his unit was concerned. The bank put up the
course. defense that it
relied on the condominium certificates of title
B's defense of is untenable actio presented by
prescription because an n High Rise Co., which were clean. Hence, it was
which seeks to declare the nullity or inexistence a mortgagee
of A contract and buyer in good faith. Is this defense tenable
does not prescribe. (Article 1410; Banaga vs. or not? Why?
Soler, 2 8CRA (5%.)
765) SUGGESTED ANSWER:
Metrobank's defense is untenable. As a
rule, an innocent
On the other hand, B's defense of pari purchaser for value acquires a good and a
delicto is equally clean title to the
untenable. While as a rule, parties who are in property. However, it is settled that one who
pari delicto have closes his eyes
no recourse against each other on the to facts that should put a reasonable man on
principle that a guard is not an
transgressor cannot profit from his own innocent purchaser for value. In the present
wrongdoing, such problem the
rule does not apply to violations of Section 118 bank is expected, as a matter of standard
of the Public operating procedure,
Land Act because of the underlying public policy to have conducted an ocular inspection, of
in the said the promises
Act "to conserve the land which a homesteader before granting any loan. Apparently,
has acquired by gratuitous Metrobank did not
grant from the government for himself and his follow this procedure. Otherwise, it should
family". In keeping with have discovered
this policy, it has been held that one who that the condominium unit in question was
purchases a occupied by Cesar
homestead within the five-year prohibitory and that fact should have led it to make
period can only further inquiry.
recover the price which he has paid by filing a Under the circumstances, Metrobank cannot be
claim against considered a
the estate of the deceased seller (Labrador vs. mortgagee and buyer in
Delos Santos 66 good faith.
Phil. 579) under the principle that no one shall
enrich himself
at the expense of another. Applying the pari
delicto rule to Mirror Principle (1990)
violation of Section 118 of the Public Land Act, In 1950's, the Government acquired a big
the Court of landed estate in
Appeals has ruled that "the homesteader suffers Central Luzon from the registered owner for
the loss of subdivision into
the fruits realized by the vendee who in small farms and redistribution of bonafide
turn forfeits the occupants, F was
improvement that he has introduced into the a former lessee of a parcel of land, five
land." (Obot vs. hectares in area. After
SandadiUas, 69 OG, April 35, completion of the resurvey and subdivision, F
1966} applied to buy
in
th sai accordanc with the
FIRST ALTERNATIVE ANSWER: e d land e guidelines of the
The action to declare the nullity of the sale did
not prescribe implementing agency. payme pric
(Art. 1410}, such sale being one expressly Upon full nt of the e in
prohibited and 1957, the corresponding deed of absolute sale
declared void by the Public Lands Act [Art. was executed
1409, par. (7)]. in his favor and was registered, and in 1961, a
The prohibition of the law is clearly for the new title was
protection of the issued in his name. In 1963, F sold the said
heirs A such recoveri the woul land to X; and in
of that their ng property d 1965 X sold it to Y, new titles were successively
enhan the polic regardi ownership issued in the
ce public y ng of lands names of the said
acquired by homestead patent (Art. 1416). The purchasers.
defense of
pari delicto is not applicable either, since the In 1977, C filed an action to annul the deeds of
law itself allows sale to F, X
the homesteader to reacquire the land even if it and Y and their titles, on the ground that he (C)
has been sold. had been in
actual physical possession of the land, and that
the sale to F
SECOND ALTERNATIVE ANSWER:
and the subsequent sales should be set aside
Prescription does not arise with respect to
on the ground of
actions to declare a
fraud. Upon motion of defendants, the trial
void contract a nullity (Article 1410). Neither is
court dismissed
the doctrine
th complai upholdi defens o thei
of pari delicto applicable because of public
e nt, ng their es f r being
policy. The law is
innoce purchasers value prescript lache
designed for the protection of the plaintiff so as
nt for , ion and s.
to enhance
Plaintiff appealed.
the public policy of the Public Land Act to give
(a) Is the said appeal meritorious?
land to the Explain your
landle answer(b)Suppose the government agency
ss. concerned joined C in
filing the said action against the
If the heirs are not allowed to recover, it could
defendants, would that
be on the
change the result of the
ground of laches inasmuch as 40 years had litigation? Explain.
elapsed and the SUGGESTED ANSWER:
owner had not brought any action against B (a) The appeal is not meritorious. The trial
especially if the court ruled
latter had improved the land. It would be correctly in granting defendant's motion to
detrimental to B if dismiss for the
the plaintiff is allowed to
recover. following reasons:
1. While there is the possibility that F, a
former lessee of the
Innocent Purchaser for Value (2001) land was aware of the fact that C was the bona
fide
CIVIL LAW Answers to the BAR as Arranged by Topics (Year 1990-2006)
occupant thereof and for this reason his transfer required to explore beyond what the record in
certificate the registry
of title may be vulnerable, the transfer of the indicates on its face in quest for any hidden
same land and defect or
the issuance of new TCTs to X and Y who are inchoate right which may subsequently defeat
innocent his right
purchasers for value render the latter's titles thereto. This is the "mirror principle' of the
indefeasible. A Torrens system
person dealing with registered land may safely which makes it possible for a forged deed to
rely on the be the root of a
correctness of the certificate of title and the law
will not in good title.
any way oblige him to go behind the certificate
to determine
the condition of the property in search for any Besides, it appears that spouses X and Y are
hidden defect guilty of
or inchoate right which may later invalidate or contributory negligence when they delivered
diminish the this OCT to the
right to the land. This is the mirror principle of mortgagee without annotating the mortgage
the Torrens thereon.
Between them and the innocent purchaser for
System of land registration. value, they
should bear the loss.
The action to annul the sale was instituted in 1977 or more ALTERNATIVE ANSWER:
than (10) years from the date of execution If the buyer B, who relied on the teller A's
thereof in 1957, title, was not
aware of the adverse possession of the land by
hence, it has long prescribed. the spouses X
and Y, then the latter cannot recover the
2. Under Sec 45 of Act 496, ―the entry of a certificate of title property from
shall be regarded as an agreement running B. B has in his favor the presumption of good
with the land, and faith which
binding upon the applicant and all his can only be overthrown by adequate proof of
successors in title that bad faith.
the land shall be and always remain However, nobody buys land without seeing
registered land. A title the property,
hence, B could not have been unaware of such
adverse
possession. If after learning of such possession, B simply
under Act 496 is indefeasible and to preserve that character, the

title is cleansed anew with every transfer for value ( De Jesus v


closed his eyes and did nothing about it, then
City of Manila; 29 Phil. 73; Laperal v City of Manila, 62 Phil 313;
the suit for
reconveyance will prosper as the buyer's bad
Penullar v PNB 120 S 111). faith will have
become evident.
loan to the latter, A, however, without the
SUGGESTED ANSWER: knowledge of X and Y, forged a deed of sale of
Even if the government joins C, this will not the aforesaid land in favor of himself, got a
alter the outcome of the case so much TCT in his name, and then sold the land to B,
because of estoppel as an express provision in who bought the land relying on A's title, and
Sec 45 of Act 496 and Sec 31 of PD 1529 that who thereafter also got a TCT in his name. It
a decree of registration and the certificate of was only then that the spouses X and Y
title issued in pursuance thereof ―shall be learned that their land had been titled in B's
conclusive upon and against all persons, name. May said spouses file an action for
including the national government and all reconveyance of the land in question against
branches thereof, whether mentioned by name b? Reason. (5%)
in the application or not.‖ SUGGESTED ANSWER:
The action of X and Y against B for
Mirror Principle; Forgery; Innocent Purchaser (1999) reconveyance of the land will not prosper
The spouses X and Y mortgaged a piece of because B has acquired a clean title to the
registered land to A, delivering as well the property being an innocent purchaser for
OCT to the latter, but they continued to value.
possess and cultivate the land, giving 1/2 of
each harvest to A in partial payment of their
A forged deed is an absolute nullity and
conveys no title. The fact that the forged deed Notice of Lis Pendens (1995)
was registered and a certificate of title was Rommel was issued a certificate of title over a
issued in his name, did not operate to vest parcel of land in Quezon City. One year later
upon an ownership over the property of X and Rachelle, the legitimate owner of the land,
Y. The registration of the forged deed will not discovered the fraudulent registration
cure the infirmity. However, once the title to obtained by Rommel. She filed a complaint
the land is registered in the name of the forger against Rommel for reconveyance and caused
and title to the land thereafter falls into the the annotation of a notice of lis pendens on the
hands of an innocent purchaser for value, the certificate of title issued to Rommel. Rommel
latter acquires a clean title thereto. A buyer of now invokes the indefeasibility of his title
a registered land is not considering that one year has already elapsed
from its issuance. He also seeks the
cancellation of the notice of Lis pendens. May
the court cancel the notice of lis pendens even
before final judgment is rendered? Explain.
SUGGESTED ANSWER:
A Notice of Lis Pendens may be canceled
even before final Judgment upon proper
showing that the notice is for the purpose of
molesting or harassing the adverse party or
that the notice of lis pendens is not necessary
to protect the right of the party who caused it
to be registered. (Section 77, P.D. No. 1529)

In this case, it is given that Rachelle is the


legitimate owner of the land in question. It can
be said, therefore, that when she filed her
notice of lis pendens her purpose was to
protect her interest in the land and not just to
molest Rommel. It is necessary to record the
Lis pendens to protect her interest because if
she did not do it, there is a possibility that the
land will fall into the hands of an innocent
purchaser for value and in that event, the
court loses control over the land making any
favorable judgment thereon moot and
academic. For these reasons, the notice of lis
pendens may not be canceled.

Notice of Lis Pendens; Transferee Pendente Lite (2002)


Sancho and Pacifico are co-owners of a parcel
of land. Sancho sold the property to Bart.
Pacifico sued Sancho and Bart for annulment
of the sale and reconveyance of the
CIVIL LAW Answers to the BAR as Arranged by Topics (Year 1990-2006)
property based on the fact that the sale the obligation. However, the action was
included his one- brought within the
half pro-indiviso share. Pacifico had a notice of ten-year prescriptive period provided by law
lis pendens wherein actions
annotated on the title covering the property and based on written contracts can be instituted. a)
ordered the Will the
cancellation of the notice of lis pendens. The defense prosper? Reason. (3%) b) What are the
notice of lis essential
pendens could not be cancelled immediately
because the title elements of laches? (2%)
over the property was with a bank to which the
property had SUGGESTED ANSWER:
been mortgaged by Bart. Pacifico appealed the No, the defense will not prosper. The problem
case. While did not give
the appeal was pending and with the notice of facts from which laches may be inferred. Mere
lis pendens still delay in filing
uncancelled, Bart sold the property to Carlos, an action, standing alone, does not constitute
who laches (Agra v.
immediately caused the cancellation of the
notice of lis PNB. 309 SCRA 509).
pendens, as well as the issuance of a new title SUGGESTED ANSWER:
in his name. Is b) The four basic elements of laches are; (1)
Carlos (a) a purchaser in good faith, or (b) a conduct on the
transferee part of the defendant or of one under whom he
pendente lite? If your answer is (a), how can the claims, giving
right of rise to the situation of which complainant seeks
Pacifico as co-owner be protected? Explain. a remedy; (2)
(5%) delay in asserting the complainant's rights, the
complainant
having had knowledge or notice of the
SUGGESTED ANSWER: defendant's conduct
A. Carlos is a buyer in bad faith. The notice of and having been afforded an opportunity to
lis pendens institute suit; (3)
was still annotated at the back of the title at the lack of knowledge on the part of the
time he defendant that the
bought the land from Bart. The uncancelled complainant would assert the right on which he
notice of lis bases his suit;
pendens operates as constructive notice of its and (4) injury or prejudice to the defendant in
contents as the event relief
well as interests, legal or equitable, included is accorded to the complainant, or the suit is
therein. All not held to be
persons are charged with the knowledge of
what it contains. barred.
In an earlier case, it was held that a notice of an
adverse claim
remains effective and binding notwithstanding
the lapse of Prescription & Laches; Indefeasibility Rule of Torrens Title
the 30 days from its inscription in the registry.
This ruling is (2002)
even more applicable in a lis pendens. Way back in 1948, Winda’s husband sold in favor of Verde
Sports Center Corp. (Verde) a 10-hectare
property belonging
to their conjugal partnership. The sale was
Carlos is a transferee pendente lite insofar as Sancho’s share made without
in the co-ownership in the land is concerned
because the land Winda’s knowledge, much less consent. In 1950, Winda
was transferred to him during the pendency of learned of the sale, when she discovered
the appeal. the deed of sale
among the documents in her husband’s vault after his demise.
B. Pacifico can protect his right as a co-owner Soon after, she noticed that the construction of
by pursuing the sports
his appeal; asking the Court of Appeals to order complex had started. Upon completion of the
the construction in
re-annotation of the lis pendens on the title of 1952, she tried but failed to get free
Carlos; and by membership privileges in
invoking his right of redemption of Bart’s share under Verde.
Articles 1620 of the New Civil Code. Winda now files a suit against Verde for the
ALTERNATIVE ANSWER: annulment of the
A. Carlos is a purchaser in good faith. A sale on the ground that she did not consent to
possessor in good the sale. In
answer, Verde contends that, in accordance
faith has been defined as ―one who is unaware that there with the Spanish
Civil Code which was then in force, the sale in
exists a flaw which invalidates his acquisition of the thing‖ 1948 of the
property did not need her concurrence. Verde
(Art. 526, NCC). Good faith consists in the possessor’s belief contends that
that the person from whom he received the in any case the action has prescribed or is
thing was the barred by laches.
owner of the same and could convey his title. In Winda rejoins that her Torrens title covering
the case [at the property is
bar], in question, while Carlos bought the
subject property indefeasible, and imprescriptible.
from Bart while a notice of lis pendens was still
annotated A. Define or explain the term ―laches‖. (2%)
thereon, there was also an existing court order B. Decide the case, stating your reasons for
canceling the your decision.
same. Hence, Carlos cannot be considered as being ―aware of (3%)
a flaw which invalidates [their] the acquisition of the thing‖ SUGGESTED ANSWER:
since the alleged flaw, the notice of lis pendens, A. LACHES means failure or neglect, for an
was already unreasonable and
being ordered cancelled at the time of the unexplained length of time, to do what, by
purchase. On this exercising due
ground alone, Carlos can already be considered diligence, could or should have been done
a buyer in earlier. It is
good faith. (Po Lam v. Court of Appeals, 347 negligence or omission to assert a right within
SCRA 86, a reasonable
[2000]). time. (De Vera v. CA, 305 SCRA 624 [1999])
B. To protect his right over the subject property, B. While Article 1413 of the Spanish Civil
Pacifico Code did not
should have timely filed an action for require the consent of the wife for the validity
reconveyance and of the sale, an
alienation by the husband in fraud of the wife is
reinstated the notice of lis pendens. void as held
in Uy Coque v. Navas, 45 Phil. 430 (1923).
Assuming that the
alienation in 1948 was in fraud of Winda
Prescription & Laches; Elements of Laches (2000) and, therefore,
In an action brought to collect a sum of money makes the sale to Verde void, the action to set
based on a aside the sale,
surety agreement, the defense of laches was
raised as the nonetheless, is already barred by
claim was filed more than seven years from the
maturity of
CIVIL LAW Answers to the BAR as Arranged by Topics (Year 1990-2006)
prescription and laches. More than 52 years (a) The mortgage contract executed by O, if at
have already all, is only a
elapsed from her discovery of the sale in voidable contract since it involves a conjugal
1950. partnership
property. The action to annul the same
ALTERNATIVE ANSWER: instituted in 1977, or
B. Winda’s claim that her Torrens Title covering eleven years after the execution of the sheriff's
the property final sale, has
is indefeasible and imprescriptible [does not obviously prescribed because: 1) An action to
hold water] is not annul a
tenable. The rule of indefeasibility of a
Torrens Title contract on the ground of fraud
means that after one year from the date of issue must be brought within four (4) years from
of the decree the date of
of registration or if the land has fallen into the discovery of the fraud. Since this is in
hands of an essence an action
innocent purchaser for value, the title becomes to recover ownership, it must be
incontestable reckoned from the
date of execution of the contract or from
and incontrovertible. the registration
of the alleged fraudulent document with
the assessor's
IMPRESCRIPTIBILITY, on the other hand, office for the purpose of transferring the
means that no tax declaration,
title to the land in derogation of that of the this being unregistered land, (Bael u.
registered owner Intermediate
may be acquired by adverse possession or Appellate Court G. R. L-74423 Jan.30, 1989
acquisitive 169 SCRA 617).
prescription or that the registered owner does
not lose by
extinctive prescription his right to recover 2) If the action is to be treated as an
ownership and action to recover
ownership of land, it would have prescribed
possession of the land. just the same
because more than 10 years have already
elapsed since the
The action in this case is for annulment of the
sale executed by date of the execution of the sale.
the husband over a conjugal partnership
property covered by a SECOND ALTERNATIVE ANSWER:
Torrens Title. Action on contracts are (a) The action to recover has been barred by
subject to acquisitive
prescription in favor of M considering that M
prescription. has possessed
the land under a claim of ownership for ten
(10) years with a
Prescription (1990) just title.
In 1960, an unregistered parcel of land was
mortgaged by
owner O to M, a family friend, as collateral for a (b) If M had secured a Torrens Title to the
loan. O acted land, all the more
through his attorney-in-fact, son S, who was duly S and P could not recover because if at all their
authorized remedies
by way of a special power of attorney, wherein O
declared would be:
that he was the absolute owner of the
land, that the tax 1. A Petition to Review the Decree of
declarations/receipts were all issued in his Registration. This can
name, and that he be availed of within one (1) year from-the entry
has been in open, continuous and adverse thereof, but
possession in the only upon the basis of "actual fraud." There is
no showing
concept of owner.
that M committed actual fraud in securing his
title to the
As O was unable to pay back the loan plus
interest for the land; or
past five [5) years, M had to foreclose the 2. An action in personam against M for the
mortgage. At the reconveyance of
foreclosure sale, M was the highest bidder. the title in their favor. Again, this remedy is
Upon issuance of available within
the sheriff’s final deed of sale and registration four years from the date of the discovery of the
in January, fraud but not
1966, the mortgage property was turned later than ten (10) years from the date of
over to M's registration of the
possession and control M has since then
developed the said title in the name of M.
property. In 1967, O died, survived by sons
S and P.
Prescription; Real Rights (1992)
In 1977, after the tenth (10th) death anniversary A owned a parcel of unregistered land located
of his father on the Tarlac
O. son P filed a suit to annul the mortgage side of the boundary between Tarlac and
deed and Pangasinan. His
subsequent sale of the property, etc., on the brother B owned the adjoining parcel of
ground of fraud. unregistered land on
He asserted that the property in question was
conjugal in the Pangasinan side.
nature actually belonging, at the time of the
mortgage, to O
and his wife, W, whose conjugal share went to A sold the Tarlac parcel to X in a deed of sale
their sons (S executed as a
public instrument by A and X. After X paid in
and P) and to O. full the, price
(a) Is the suit filed by P barred by prescription? of the sale, X took possession of the
Explain your Pangasinan parcel in the
belief that it was the Tarlac parcel covered by
answer. the deed of
(b) After the issuance of the sheriff's final
deed of sale in sale executed by A and X.
1966 in this case, assuming that M applied
for registration
under the Torrens System and was issued a After twelve (12) years, a controversy arose
Torrens Title to between B and X
the said property in question, would that added on the issue of the ownership of the
fact have any Pangasinan parcel, B
significant effect on your conclusion? State claims a vested right of ownership over the
your reason. Pangasinan parcel
because B never sold that parcel to X or to
SUGGESTED ANSWER: anyone else.
Under Art. 173 of the Civil Code, the action
is barred by prescription because the wife
had only ten (10) years from
the transaction and during the marriage to On the other hand, X claims a vested right of
file a suit for the annulment of the mortgage ownership over the Pangasinan parcel by
deed. Alternative Answers to (a) first Alternative acquisitive prescription, because X possessed
Answer: this parcel for over ten (10] years under claim
of ownership.
CIVIL LAW Answers to the BAR as Arranged by Topics (Year 1990-2006)
The right to recover possession of registered
Decide on these claims, giving your reasons. land likewise
does not prescribe because possession is just a
SUGGESTED ANSWER: necessary
At this point in time, X cannot claim the
right of vested incident of ownership.
ownership over the Pangasinan parcel by
acquisitive
prescription. In addition to the requisites
common to ordinary SUGGESTED ANSWER:
and extraordinary acquisitive prescription b) Mikaelo's defense of laches, however,
consisting of appears to be more
uninterrupted, peaceful, public, adverse and sustainable. Renren bought the land and
actual possession had the sale
in the concept of owner, ordinary acquisitive registered way back in 1965. From the facts, it
prescription for appears that it
ten (10) years requires (1) possession in good was only in 1998 or after an inexplicable delay
faith and (2) just of 33 years that
title. "Just title" means that the adverse claimant he took the first step asserting his right to the
came into land. It was not
possession of the property through one of even an action to recover ownership but only
the modes possession of
recognized by law for the acquisition of the land. By ordinary standards, 33 years
ownership but the of neglect or
grantor was not the owner or could not transmit inaction is too long and maybe considered
any right (Art. unreasonable. As
1129. Civil Code). In this case, there is no "just often held by the Supreme Court, the
title" and no principle of
"mode" that can be invoked by X for the imprescriptibility sometimes has to yield to
acquisition of the the equitable
Pangasinan parcel. There was no constructive principle of laches which can convert even a
delivery of the registered land
Pangasinan parcel because it was not the
subject-matter of the owner's claim into a stale demand.
deed of sale. Hence, B retains ownership of the Renren's action to recover possession of the
Pangasinan parcel of land. land will prosper. In 1965, after buying the
land from Robyn, he submitted the Deed of
Sale to the Registry of Deeds for registration
Primary Entry Book; Acquisitive Prescription; Laches (1998)
together with the owner's duplicate copy of
the title, and paid the corresponding
In 1965, Renren bought from Robyn a parcel of
registration fees. Under Section 56 of PD No.
registered land evidenced by a duly executed
1529, the Deed of Sale to Renren is
deed of sale. The owner presented the deed of
considered registered from the time the sale
sale and the owner's certificate of title to the
was entered in the Day Book (now called the
Register of Deeds. The entry was made in the
Primary Entry Book).
daybook and corresponding fees were paid as
evidenced by official receipt. However, no For all legal intents and purposes, Renren is
transfer of certificate of title was issued to considered the registered owner of the land.
Renren because the original certificate of title After all, it was not his fault that the Registry
in Robyn's name was temporarily misplaced of Deeds could not issue the corresponding
after fire partly gutted the Office of the transfer certificate of title.
Register of Deeds. Meanwhile, the land had
been possessed by Robyn's distant cousin, Mikaelo's defense of prescription can not be
Mikaelo, openly, adversely and continuously in sustained. A Torrens title is imprescriptible.
the concept of owner since 1960. It was only in No title to registered land in derogation of the
April 1998 that Renren sued Mikaelo to title of the registered owner shall be acquired
recover possession. Mikaelo invoked a) by prescription or adverse possession.
acquisitive prescription and b) laches, asking (Section 47, P.D. No, 1529)
that he be declared owner of the land. Decide
the case by evaluating these defenses, [5%]
SUGGESTED ANSWER:
Mikaelo's claim of laches, however, is weak
insofar as the element of equity is concerned,
there being no showing in the facts how he
entered into the ownership and possession of
the land.
Reclamation of Foreshore Lands; Limitations (2000)
Republic Act 1899 authorizes municipalities
and chartered cities to reclaim foreshore lands
bordering them and to construct thereon
adequate docking and harbor facilities.
Pursuant thereto, the City of Cavite entered
into an agreement with the Fil -Estate Realty
Company, authorizing the latter to reclaim 300
hectares of land from the sea bordering the
city, with 30% of the land to be reclaimed to be
owned by Fil-Estate as compensation for its
services. The Solicitor General questioned the
validity of the agreement on the ground that it
will mean reclaiming land under the sea which
is beyond the commerce of man. The City
replies that this is authorized by RA. 1899
because it authorizes the construction of docks
and harbors. Who is correct? (3%)
SUGGESTED ANSWER:
The Solicitor General is correct. The authority
of the City of Cavite under RA 1899 to reclaim
land is limited to foreshore lands. The Act did
not authorize it to reclaim land from the sea.
"The reclamation being unauthorized, the City
of Cavite did not acquire ownership over the
reclaimed land. Not being the owner, it could
not have conveyed any portion thereof to the
contractor.
ALTERNATIVE ANSWER:
It depends. If the reclamation of the land from
the sea is necessary in the construction of the
docks and the harbors, the City of Cavite is
correct. Otherwise, it is not. Since RA 1899
authorized the city to construct docks and
harbors, all works that are necessary for such
construction are deemed authorized. Including
the reclamation of land from the sea. The
reclamation being authorized, the city is the
owner of the reclaimed land and it may convey
a portion thereof as payment for the services
of the contractor.
ANOTHER ALTERNATIVE ANSWER:
On the assumption that the reclamation
contract was entered into before RA 1899 was
repealed by PD 3-A, the City of Cavite is
correct. Lands under the sea are "beyond the
commerce of man" in the sense that they are
not susceptible of private appropriation,
ownership or
CIVIL LAW Answers to the BAR as Arranged by Topics (Year 1990-2006)
alienation. The contract in question merely calls answer or show up on the date of initial
for the hearing, does not
reclamation of 300 hectares of land within the
coastal waters guarantee the success of the application. It is still incumbent upon
of the city. Per se, it does not vest, alienate
or transfer the applicant to prove with well nigh incontrovertible evidence that
ownership of land under the sea. The city merely he has acquired a title to the land that is fit for registration. Absent
engaged the such registrable title, it is the clear duty of the Land Registration
services of Fil-Estate to reclaim the land for the
city. Court to dismiss the application and declare the land as public
land.

Registration; Deed of Mortgage (1994)


How do you register now a deed of mortgage of An application for land registration is a
a parcel of proceeding in rem. Its
land originally registered under the Spanish main objective is to establish the status of the
Mortgage Law? res whether it is
still part of our public domain as presumed
SUGGESTED ANSWER: under the
a) After the Spanish Mortgage Law was Regalian doctrine or has acquired the
abrogated by P.D. character of a private
892 on February 16, 1976, all lands covered by property. It is the duty of the applicant to
Spanish titles overcome that
that were not brought under the Torrens system
within six presumption with sufficient evidence.
16] months from the date thereof have been
considered as
"unregistered private lands." Remedies; Judicial Reconstitution of Title (1996)
In 1989, the heirs of Gavino, who died on
August 10, 1987,
Thus, a deed of mortgage affecting land filed a petition for reconstitution of his lost or
originally registered destroyed
under the Spanish Mortgage Law is now Torrens Title to a parcel of land in Ermita,
governed by the Manila. This was
system of registration of transactions or opposed by Marilou who claimed ownership of
instruments affecting the said land
unregistered land under Section 194 of by a series of sales. She claimed that Gavino
the Revised had sold the
Administrative Code as amended by Act No. property to Bernardo way back in 1941 and as
3344. Under this evidence
law, the instrument or transaction affecting thereof, she presented a Tax Declaration in
unregistered land 1948 in the name
is entered in a book provided for the of Bernardo, which cancelled the previous Tax
purpose but the Declaration in
registration thereof is purely voluntary and does the name of Gavino. Then she presented two
not adversely deeds of sale
duly registered with the Register of Deeds, the
affect third persons who have a better right. first one
executed by Bernardo in 1954 selling the same
property to
b) By recording and registering with the Carlos, and the second one executed by Carlos
Register of Deeds of in 1963, selling
the place where the land is located, in the same property to her. She also claimed
accordance with Act that she and her
3344. However, P.D. 892 required holders of predecessors in interest have been in
Spanish title to possession of the
bring the same under the Torrens System within property since 1948. If you were the judge,
6 months how will you
from its effectivity on February 16, 1976. decide the petition? Explain.
Remedies; Judicial Confirmation; Imperfect Title (1993)
On June 30, 1986, A filed in the RTC of Abra an
application SUGGESTED ANSWER:
for registration of title to a parcel of land under If I were the judge, I will give due course to
the petition of
P. D. No. 1529, claiming that since June 12, the heirs of Gavino despite the opposition of
1945, he has been Marilou for the
in open, continuous, exclusive and notorious following reasons: a) Judicial reconstitution of
possession and a certificate of
occupation of said parcel of land of the public
domain which title under RA.
was alienable and disposable, under a bona No. 26 partakes of a land registration
fide claim of proceeding and is
ownership. After issuance of the notice of initial perforce a proceeding in rem. It denotes
hearing and restoration of
publication, as required by law, the petition was an existing instrument which has been lost
heard on July or destroyed
29, 1987. On the day of the hearing nobody but in its original form and condition. The
the applicant purpose of
appeared. Neither was there anyone who reconstitution of title or any document is to
opposed the have the
application. Thereupon, on motion of the same reproduced, after proceedings. In the
applicant, the RTC same form
issued an order of general default and allowed they were when the loss or destruction
the applicant to occurred.
present his evidence. That he did. On September If the Court goes beyond that purpose,
30, 1989, the b) it acts
RTC dismissed A's application for lack of without or in excess of jurisdiction. Thus,
sufficient evidence. where the Torrens
Title sought to be reconstituted is in the name
A appealed to the Court of Appeals. of Gavino, the
court cannot receive evidence proving that
Marilou is the
owner of the land. Marilou's dominical claim to
the land
The appellant urged that the RTC erred in should be ventilated in a separate civil action
dismissing his before the
application for registration and in not ordering Regional Trial Court in its capacity as a court
registration of of general
his title to the parcel of land in question despite
the fact that jurisdiction.
there was no opposition filed by anybody to his REFERENCES: Heirs of Pedro Pinate vs. Dulay.
application. 187 SCRA 12-20
(1990); Bunagan vs. CF1 Cebu Branch VI. 97
Did the RTC commit the error attributed to it? SCRA 72 (1980);
Republic vs. IAC. 157 SCRA 62,66 (1988);
Margolles vs. CA, 230
SUGGESTED ANSWER: SCRA 709; Republic us, Feliciano, 148 SCRA 924.
No, the RTC did not commit the error attributed to it. In an
application for Judicial confirmation of imperfect or incomplete title Remedies; Procedure; Consulta (1994)
What is the procedure of consulta when an
to public agricultural land under Section 48 of the Public Land Act, instrument is
the lack of opposition and the consequent order of default against denied registration?
those who did not SUGGESTED ANSWER:
CIVIL LAW Answers to the BAR as Arranged by Topics (Year 1990-2006)
The Register of Deeds shall notify the An action for reconveyance against Huey is not
interested party in writing, setting forth the the proper remedy, because Huey is an innocent
defects of the instrument or the legal ground purchaser for value. The proper recourse is for
relied upon for denying the registration, and Louie to go after Dewey for damages by reason
advising that if he is not agreeable to such of the fraudulent registration and subsequent
ruling, he may, without withdrawing the sale of the land. If Dewey is insolvent, Louie may
documents from the Registry, elevate the file a claim against the Assurance Fund (Heirs of
matter by Consulta to the Administrator of the Pedro Lopez v. De
Land Registration Authority (LRA). Castro 324 SCRA 591 [2000] citing
Sps. Eduarte v. CA, 323 Phil. 462, 467
[1996]).
Within five {5) days from receipt of
2) notice of
denial, the party-in-interest shall file his (b) Yes, the remedy will prosper because the
Consulta with the action prescribes
Register of Deeds concerned and pay the in ten (10) years, not within one (1) year when
consulta fee. a petition for
After receipt of the Consulta and the reopening of the registration decree may
3) payment of the be filed. The
corresponding fee the Register of Deeds makes action for reconveyance is distinct from the
an annotation petition to
of the pending consulta at the back of the reopen the decree of registration (Grey Alba v.
certificate of title. De la Cruz, 17
Phil. 49 [1910}). There is no need to reopen
the registration
The Register of Deeds then elevates the proceedings, but the property should just be
4) case to the reconveyed to
LRA Administrator with certified records
thereof and a the real owner.
summary of the facts and issues involved.
The LRA Administrator then conducts The action for reconveyance is based on implied
5) hearings or constructive
after due notice or may just require parties to trust, which prescribes in ten (10) years from the
submit their date of issuance
of the original certificate of title. This rule
memoranda. assumes that the
After hearing, the LRA Administrator defendant is in possession of the land. Where it is
6) issues an the plaintiff
order prescribing the step to be taken or the who is in possession of the land, the action for
memorandum to reconveyance
be made. His resolution in consulta shall be would be in the nature of a suit for quieting for
conclusive and the title which
binding upon all Registers of Deeds unless
reversed on appeal action is imprescriptible (David
by the Court of Appeals or by the Supreme
Court. (Section v. Malay, 318 SCRA 711 [1999]).
117, P.D. 1529).
Remedies; Reconveyance; Elements (1995)
Rommel was issued a certificate of title over a
• parcel of land
The procedure of consulta is a mode of appeal from denial
in Quezon City. One year later Rachelle, the legitimate
owner
by the Register of Deeds of the registration of the instrument to the

of the land, discovered the fraudulent


Commissioner of Land Registration. registration obtained by
Rommel. She filed a complaint against Rommel for
• Within five days from receipt of the notice of denial, the
interested party may elevate the matter by reconveyance and caused the annotation of a
consulta to the notice of lis
pendens on the certificate of title issued to
Commissioner of Land Registration who shall Rommel. Rommel
enter an order now invokes the indefeasibility of his title
prescribing the step to be taken or considering that
memorandum to be made. one year has already elapsed from its
issuance. He also seeks
Resolution in consulta shall be binding upon all Registers of Deeds
the cancellation of the notice of Lis pendens.
provided that the party in interest may appeal Will Rachelle's
to the Court of
Appeals within the period prescribed (Sec. 117, suit for reconveyance prosper? Explain.
P.D. 1529).

Remedies; Reconveyance vs. Reopening of a Decree;


Prescriptive Period (2003)
Louie, before leaving the country to train as a chef in a
five-star hotel in New York, U.S.A., entrusted to his
first-degree cousin Dewey an application for registration,
under the Land Registration Act, of a parcel of land located in
Bacolod City. A year later, Louie returned to the Philippines
and discovered that Dewey registered the land and obtained
an Original Certificate of Title over the property in his
Dewey’s name. Compounding the matter, Dewey sold the
land to Huey, an innocent purchaser for value. Louie
promptly filed an action for reconveyance of the parcel of
land against Huey.
Is the action pursued by Louie the proper remedy?
Assuming that reconveyance is the proper
remedy, will the action prosper if the case was
filed beyond one year, but within ten years, from
the entry of the decree of registration?
5%
SUGGESTED ANSWER:
SUGGESTED ANSWER:
Yes, Rachelle's suit will prosper because all
elements for an
action for reconveyance are present, namely: a)
Rachelle is
claiming dominical rights over the same
land. b) Rommel procured his title to the land
by fraud. c)
The action was brought within the statutory
period of

four (4) years from discovery of the fraud


and not later
than ten (10} years from the date of
registration of
Rommel's title. d) Title to the land has not
passed into the
hands of an
innocent purchaser for value.

Rommel can invoke the indefeasibility of his


title if Rachelle
had filed a petition to reopen or review the
decree of
registration. But Rachelle instead filed an
ordinary action in
personam for reconveyance. In the latter
action, indefeasibility
is not a valid defense because, in filing such
action, Rachelle is
not seeking to nullify nor to impugn the
indefeasibility of
Rommel's title. She is only asking the court to
compel Rommel
to reconvey the title to her as the legitimate
owner of the land.
ALTERNATIVE ANSWER:
CIVIL LAW Answers to the BAR as Arranged by Topics (Year 1990-2006)
Yes. The property registered is deemed to be This action does not prescribe. With respect to
held in trust Percival's
for the real owner by the person in whose name action for reconveyance, it would have prescribed,
it is having been filed more
registered. The Torrens system was not than ten (10) years after registration and issuance of an
designed to shield O.C.T. in the
one who had committed fraud or name of Melvin, were it not for the inherent infirmity of
misrepresentation and thus the latter's title.
holds the title in bad faith. (Walstrom v. Mapa Under the facts, the statute of limitations will not apply
Jr., (G .R 38387, to Percival
29 Jan. 1990) as cited in Martinez, D., Summary because Melvin knew that a part of the land covered by
of SC his title actually
belonged to Percival. So, instead of nullifying in toto
Decisions, January to June, 1990, p. 359], the title of Melvin,
the court, in the exercise of equity and jurisdiction, may
grant prayer for
the reconveyance of Lot B to Percival who has actually
Remedies; Reconveyance; Prescriptive Period (1997) possessed the land
On 10 September 1965, Melvin applied for a under a claim of ownership since 1947. After all, if
free patent Melvin's title is
covering two lots - Lot A and Lot B - situated in declared void ab initio and the land is reverted to the
Santiago, public domain,
Isabela. Upon certification by the Public Land Percival would just the same be entitled to preference
Inspector that right to acquire the
Melvin had been in actual, continuous, open, land from the government. Besides, well settled is the
notorious, rule that once
exclusive and adverse possession of the lots public land has been in open, continuous, exclusive and
since 1925, the notorious
Director of Land approved Melvin's application possession under a bonafide claim of acquisition of
on 04 June ownership for the
1967. On 26 December 1967, Original period prescribed by Section 48 of the Public Land Act,
Certificate of Title the same ipso
(OCT) No. P-2277 was issued in the name of jure ceases to be public and in contemplation of law
Melvln. acquired the
character of private land. Thus, reconveyance of the
On 7 September 1971, Percival filed a protest
land from Melvin to
alleging that Percival would be the better procedure, (Vitale vs.
Lot B which he had been occupying and Anore, 90 Phil. 855;
cultivating since
1947 was included in the Free Patent issued in Pena, Land Titles and Deeds, 1982, Page 427)
the name of
Melvin. The Director of Lands ordered the
investigation of
Percival's protest. The Special Investigator who
conducted
the investigation found that Percival had been in
actual
cultivation of Lot B since 1947. ALTERNATIVE ANSWER:
The action of the Solicitor General should
prosper,
On 28 November 1986, the Solicitor General considering that the doctrine of indefeasibility
filed in behalf of title does not
of the Republic of the Philippines a complaint apply to free patent secured through fraud. A
for cancellation certificate of
of the free patent and the OCT issued in the title cannot be used as shield to perpetuate
name of Melvin fraud. The State is
and the reversion of the land to public domain not bound by the period of prescription stated
on the ground in Sec. 38 of
of fraud and misrepresentation in obtaining the Act 496. (Director of Lands vs. Abanilla, 124
free patent. SCRA 358)
On the same date, Percival sued Martin for the
reconveyance
of Lot B. The action for reconveyance filed by Percival may
still prosper
Melvin filed his answers interposing the sole provided that the property has not passed to an
defense in both innocent third party
cases that the Certificate of Title issued in his for value (Dablo us. Court of Appeals. 226 SCRA 618),
name became and provided
incontrovertible and indefeasible upon the lapse that the action is filed within the prescriptive period
of one year of ten years
(Tale vs. Court of Appeals. 208 SCRA 266). Since the
from the issuance of the free patent. action was filed
by Percival 19 years after the issuance of Melvin's
title, it is submitted
Given the circumstances, can the action of the that the same is already barred by prescription.
Solicitor ALTERNATIVE ANSWER
General and the case for reconveyance filed by (to second part of question) The action for
Percival reconveyance filed by
Percival will prosper, because the land has ceased to
possibly prosper? be public land
and has become private land by open, continuous,
SUGGESTED ANSWER: public, exclusive
"If fraud be discovered in the application which possession under a bona fide claim of ownership for
led to the more than thirty
issuance of the patent and Certificate of Title, years, and Percival is still in possession of the
this Title property at present. His
becomes ipso facto null and void. Thus, in a case action for reconveyance can be considered as an
where a action to quiet title,
person who obtained a free patent, knowingly which does not prescribe if the plaintiff is in
made a false possession of the
statement of material and essential facts in his
application for property.
the same, by stating therein that the lot in
question was part of
the public domain not occupied or claimed by
any other
person, his title becomes ipso facto canceled
and consequently (Olviga v. CA. GR 1048013. October 21, 1993)
rendered null and void." "It is to the public
interest that one
who succeeds In fraudulently acquiring title to
public land Remedies; Reopening of a Decree; Elements (1992)
should not be allowed to benefit therefrom and What are the essential requisites or elements
the State, for the allowance
through the Solicitor General, may file the of the reopening or review of a decree of
corresponding registration?
action for annulment of the patent and the
reversion of the
land involved to the public domain" (Dinero us.
Director of SUGGESTED ANSWER:
Lands; Kayaban vs. Republic L-33307,8-20-73; The essential elements are: (1) that the
petitioner has a real or
Director of
dominical right; (2) that he has been deprived
thereof through
Lands us. Hon. Pedro Samson Animas, L-37682, fraud; (3) that the petition is filed within one
3-29-74.) (1) year from the
issuance of the decree; and (4) that the
property has not yet
been transferred to an innocent
CIVIL LAW Answers to the BAR as Arranged by Topics (Year 1990-2006)
purchaser {Rublico vs. Orellana 30 SCRA 511; Administrative Code of 1987 which prohibits
Ubudan vs. Gil officers and
45 SCRA 17). employees of the government from purchasing directly or indirectly
OPTIONAL EXTENDED ANSWER: any property sold by the government for nonpayment of any tax,
Petition for review of the Decree of Registration.
A remedy
fee or other public charge.
expressly provided in Section 32 of P. D. No. (a) Is the sale to Juan valid? If so, what is the
1529 (formerly effect of the
Section 38. Act 496), this remedy has the
following elements: Issuance of the Certificate of Title to Maria?
a) The petition must be filed by a person (b) If the sale is void, may Juan recover the
claiming P10,000.00? If
dominical or other real rights to the land not, why not?
registered in (c) If the sale is void, did it not nevertheless,
operate to divert
Maria of her ownership? If it did, who then is
the name of respondent. the owner of
The registration of the land in the name
b) of the property?
respondent was procured by means of actual,
(not just SUGGESTED ANSWER:
constructive) fraud, which must be Fraud is A. The sale of the land to Juan is not valid,
extrinsic. actual being contrary to
if the registration was made through deceit or law. Therefore, no transfer of ownership of the
any other land was
intentional act of downright dishonesty to enrich effected from the delinquent taxpayer to him.
oneself at the The original
expense of another. It is extrinsic when it is certificates of title obtained by Maria thru a
something that free patent grant
was not raised, litigated and passed upon in the from the Bureau of Lands under Chapter VII,
main CA 141 is
valid but in view of her delinquency, the said
proceedings. title is subject to
The petition must be filed within one (1) the right of the City Government to sell the
c) year from land at public
the date of the issuance of the auction. The issuance of the OCT did not
decree. exempt the land
Title to the land has not passed to an from the tax sales. Section 44 of P.O. No. 1529
d) Innocent provides that
purchaser for value (Libudan vs. Gil, 45_ SCRA every registered owner receiving a Certificate
27, 1972), of Title shall
Rublico vs. Orrelana. 30 SCRA 511, 1969); RP hold the same free from an encumbrances,
vs. CA, 57 G. subject to certain
R No. 40402. March 16, 1987). exemptions.
B. Juan may recover because he was not a
Torrens System vs. Recording of Evidence of Title (1994) party to the
Distinguish the Torrens system of land
registration from the violation of the law.
system of recording of evidence of
title.
SUGGESTED ANSWER: C. No, the sale did not divest Maria of her
The TORRENS SYSTEM OF LAND title precisely because the sale is void. It is as
REGISTRATION is a system for the registration good as if no sale ever took place. In tax
of title to the land. Thus, under this system what sales, the owner is divested of his land
is entered in the Registry of Deeds, is a record of initially upon award and issuance of a
the owner's estate or interest in the land, unlike Certificate of Sale, and finally after the lapse
the system under the Spanish Mortgage Law or of the 1 year period from date of registration,
the system under Section 194 of the Revised to redeem, upon execution by the treasurer
Administrative Code as amended by Act 3344 of an instrument sufficient in form and effects
where only the evidence of such title is recorded. to convey the property. Maria remained
In the latter system, what is recorded is the deed owner of the land until another tax sale is to
of conveyance from hence the owner's title be performed in favor of a qualified buyer.
emanated—and not the title itself.
CONTRACTS
Torrens system of land registration is that
which is prescribed in Act 496 (now PD 1529), Consensual vs. Real Contracts; Kinds of Real
which is either Judicial or quasi-judicial. Contracts (1998)
System or recording of evidence of title is Distinguish consensual from real contracts
merely the registration of evidence of and name at least four (4) kinds of real
acquisitions of land with the Register of contracts under the present law. [3%]
Deeds, who annotates the same on the
existing title, cancels the old one and issues a SUGGESTED ANSWER:
new title based on the document presented CONSENSUAL CONTRACTS are those which
for registration. are perfected by mere consent (Art. 1315. Civil
Code). REAL CONTRACTS are those which are
Unregistered Land (1991) perfected by the delivery of the object of the
Maria Enriquez failed to pay the realty taxes on her unregistered obligation. (Art. 1316, Civil Code) Examples of
agricultural land located in Magdugo, Toledo City. In 1989, to real contracts are deposit, pledge,
satisfy the taxes due, the City sold it at public auction to Juan commodatum and simple loan (mutuum).
Miranda, an employee at the Treasurer's Office of said City,
whose bid at P10,000.00 was the highest. In due time, a final bill Consideration; Validity (2000)
of sale was executed in his favor. Maria refused to turn- over the Lolita was employed in a finance company.
possession of the property to Juan alleging that (1) she had been, Because she could not account for the funds
in the meantime, granted a free patent and on the basis thereof entrusted to her, she was charged with estafa
an Original Certificate of Title was issued to her, and (2) the sale and ordered arrested. In order to secure her
in favor of Juan is void from the beginning in view of the provision release from jail, her parents executed a
in the promissory note to pay the finance company
the amount allegedly misappropriated by their
daughter. The finance company
CIVIL LAW Answers to the BAR as Arranged by Topics (Year 1990-2006)
then executed an affidavit of desistance which led to the
withdrawal of the information against Lolita and My answer will not be the same as to damages.
her release Marvin will be
from jail. The parents failed to comply with their liable for damages for breach of contract of
promissory option. With the
note and the finance company sued them payment of the consideration for the option given,
for specific and with the
performance. Will the action prosper or not? consent of the parties and the object of contract
(3%) being present, a
SUGGESTED ANSWER: perfected contract of option was created.
The action will prosper. The promissory note
executed by (San Miguel, Inc. v. Huang, G.R. No.
Lolita's parents is valid and binding, the 137290, July 31,
consideration being 2000) Under Article 1170 of the Civil Code,
the extinguishment of Lolita's civil liability and those who in the
not the stifling performance of their obligation are guilty of
contravention
of the criminal prosecution. thereof, as in this case, when Marvin did not
give Carlos the
ALTERNATIVE ANSWER:
agreed period of ten days, are liable for
The action will not prosper because the
damages.
consideration for the
promissory note was the non-prosecution of the
criminal case
ALTERNATIVE ANSWER:
for estafa. This cannot be done anymore My answer will not be the same if Carlos
because the paid Marvin
information has already been filed in court and P10,000.00 because an option contract was
to do it is perfected. Thus,
illegal. That the consideration for the if Marvin withdrew the offer prior to the
promissory note is the expiration of the
stifling of the criminal prosecution is evident 10-day period, he breached the option contract.
from the (Article 1324,
execution by the finance company of the
affidavit of Civil Code)
desistance immediately after the execution by
Lolita's parents
of the promissory note. The consideration being c) Supposing that Carlos accepted the
illegal, the offer before
promissory note is invalid and may not be Marvin could communicate his
enforced by court withdrawal thereof?
action. Discuss the legal consequences. (2%)
(Salonga v. Farrales, G.R. No. L-47088, July
Contract of Option; Elements (2005)
10, 1981) Article 1318 of the Civil Code
Marvin offered to construct the house of provides that there can be no contract unless
Carlos for a very reasonable price of
the following requisites concur: (1) consent of
P900,000.00, giving the latter 10 days within
the parties; (2) object certain which is the
which to accept or reject the offer. On the fifth
subject matter of the contract; and (3) cause of
day, before Carlos could make up his mind,
the obligation.
Marvin withdrew his offer.
What is the effect of the withdrawal of Marvin will not be liable to pay Carlos any
Marvin's offer? (2%) damages for withdrawing the offer before the
SUGGESTED ANSWER:
lapse of the period granted. In this case, no
The withdrawal of Marvin's offer will cause the
consideration was given by Carlos for the
offer to cease in law. Hence, even if
option given, thus there is no perfected
subsequently accepted, there could be no
contract of option for lack of cause of
concurrence of the offer and the acceptance. In
obligation. Marvin cannot be held to have
the absence of concurrence of offer and
breached the contract. Thus, he cannot be held
acceptance, there can be no consent. (Laudico
liable for damages.
v. Arias Rodriguez, G.R. No. 16530, March
31, 1922) Without consent, there is no
Will your answer be the same if Carlos paid
perfected contract for the construction of the
Marvin P10,000.00 as consideration for
house of Carlos.
that option? Explain. (2%)
ALTERNATIVE ANSWER: SUGGESTED ANSWER:
My answer will be the same as to the A contract to construct the house of Carlos is
perfection of the contract for the construction perfected. Contracts are perfected by mere
of the house of Carlos. No perfected contract consent manifested by the meeting of the offer
arises because of lack of consent. With the and the acceptance upon the thing and the
withdrawal of the offer, there could be no cause which are to constitute the contract.
concurrence of offer and acceptance. (Gomez v. Court of Appeals, G.R. No.
120747, September 21, 2000)

Under Article 1315 of the Civil Code, Carlos


and Marvin are bound to fulfill what has been
expressly stipulated and all consequences
thereof. Under Article 1167, if Marvin would
refuse to construct the house, Carlos is
entitled to have the construction be done by a
third person at the expense of Marvin. Marvin
in that case will be liable for damages under
Article 1170.
Inexistent Contracts vs. Annullable Contracts (2004)
Distinguish briefly but clearly between
Inexistent contracts and annullable contracts.
SUGGESTED ANSWER:
INEXISTENT CONTRACTS are considered as
not having been entered into and, therefore,
void ob initio. They do not create any
obligation and cannot be ratified or validated,
as there is no agreement to ratify or validate.
On the other hand, ANNULLABLE or
VOIDABLE CONTRACTS are valid until
invalidated by the court but may be ratified. In
inexistent contracts, one or more requisites of
a valid contract are absent. In anullable
contracts, all the elements of a contract are
present except that the consent of one of the
contracting parties was vitiated or one of them
has no capacity to give consent.
Nature of Contracts; Obligatoriness (1991)
Roland, a basketball star, was under contract
for one year to play-for-play exclusively for
Lady Love, Inc. However, even before the
basketball season could open, he was offered a
more attractive pay plus fringes benefits by
Sweet Taste, Inc. Roland accepted the offer
and transferred to Sweet Taste. Lady Love
sues Roland and Sweet Taste for breach of
contract. Defendants claim that the restriction
to play for Lady Love alone is void, hence,
unenforceable, as it
CIVIL LAW Answers to the BAR as Arranged by Topics (Year 1990-2006)
constitutes an undue interference with the right any payment at all. Printado has also a
of Roland standing contract
to enter into contracts and the impairment of
his freedom to with publisher Publico for the printing of 10,000 volumes of school
play and enjoy basketball. textbooks. Suplico was aware of said printing contract. After
Can Roland be bound by the contract he entered printing 1,000 volumes, Printado also fails to perform under its
into with printing contract with Publico. Suplico sues Printado for the value
Lady Love or can he disregard the same? Is he
liable at all? of the unpaid deliveries under their order agreement. At the same
How about Sweet Taste? Is it liable to Lady
Love? time Publico sues Printado for damages for breach of contract
SUGGESTED ANSWER: with respect to their own printing agreement. In the suit filed by
Roland is bound by the contract he entered into
with Lady
Love and he cannot disregard the same, under Suplico, Printado counters that: (a) Suplico cannot demand
the principles
of obligatoriness of contracts. Obligations payment for deliveries made under their order agreement until
arising from
contracts have the force of law between the Suplico has completed performance under said contract; (b)
parties.
Suplico should pay damages for breach of contract; and (c) with
SUGGESTED ANSWER:
Yes, Roland is liable under the contract as far as Publico should be liable for Printado’s breach of his contract with
Lady Love is
concerned. He is liable for damages under Publico because the order agreement between Suplico and
Article 1170 of the
Civil Code since he contravened the tenor of his Printado was for the benefit of Publico. Are the contentions of
obligation.
Not being a contracting party, Sweet Taste is Printado tenable? Explain your answers as to each contention.
not bound by
the contract but it can be held liable under Art.
1314. The (5%)
basis of its liability is not prescribed by
contract but is SUGGESTED ANSWER:
founded on quasi-delict, assuming that Sweet No, the contentions of Printado are untenable.
Taste knew of Printado
the contract. Article 1314 of the Civil Code having failed to pay for the printing paper
provides that any covered by the
third person who induces another to violate his delivery invoices on time, Suplico has the right
contract shall to cease
be liable for damages to the other contracting making further delivery. And the latter did not
party. violate the
order agreement (Integrated Packaging
Corporation v. Court
of Appeals, (333 SCRA 170, G.R. No. 115117,
ALTERNATIVE ANSWER: June 8, [2000]).
It is assumed that Lady Love knew of the contract.
Neither
Roland nor Sweet Taste would be liable, Suplico cannot be held liable for damages, for
because the breach of contract, as
restriction in the contract is violative of Article it was not he who violated the order agreement, but
1306 as being Printado.
contrary to law morals, good customs, public Suplico cannot be held liable for Printado’s breach
order or public of contract with
Publico. He is not a party to the agreement entered
policy. into by and
between Printado and Publico. Theirs is not a
stipulation pour atrui.
Nature of Contracts; Privity of Contract (1996) [Aforesaid] Such contracts do could not affect third
Baldomero leased his house with a telephone to persons like
Jose. The Suplico because of the basic civil law principle of
lease contract provided that Jose shall pay for relativity of
all electricity,
contracts which provides that contracts can only
bind the parties
water and telephone services in the leased who entered into it, and it cannot favor or prejudice
premises during the a third person,
period of the lease. Six months later. Jose even if he is aware of such contract and has acted
surreptitiously with knowledge
vacated the premises. He left behind unpaid
telephone bills thereof. (Integrated Packaging Corporation
for overseas telephone calls amounting to over
P20,000.00.
Baldomero refused to pay the said bills on the
ground that
Jose had already substituted him as the
customer of the v. CA, supra.)
telephone company. The latter maintained
that Baldomero
remained as his customer as far as their service
contract was Rescission of Contracts; Proper Party (1996)
concerned, notwithstanding the lease In December 1985, Salvador and the Star
contract between Semiconductor
Baldomero and Jose. Who is correct, Company (SSC) executed a Deed of
Baldomero or the Conditional Sale wherein
the former agreed to sell his 2,000 square
telephone company? Explain. meter lot in Cainta,
Rizal, to the latter for the price of
SUGGESTED ANSWER: P1,000,000.00, payable
The telephone company is correct because P100,000.00 down, and the balance 60 days
as far as it is after the squatters
concerned, the only person it contracted with in the property have been removed. If the
was Baldomero. squatters are not
The telephone company has no contract with removed within six months, the P100,000.00
Jose. Baldomero down payment
cannot substitute Jose in his stead without the shall be returned by the vendor to the vendee,
consent of the Salvador filed
telephone company (Art. 1293, NCC). ejectment suits against the squatters, but in
Baldomero is, spite of the
decisions in his favor, the squatters still would
therefore, liable under the contract. not leave. In
August, 1986, Salvador offered to return the
P100,000.00
down payment to the vendee, on the ground
Nature of Contracts; Relativity of Contracts (2002) that he is unable
to remove the squatters on the property. SSC
Printado is engaged in the printing business. Suplico supplies refused to
accept the money and demanded that Salvador
printing paper to Printado pursuant to an order agreement under execute a deed
of absolute sale of the property in its favor, at
which Suplico binds himself to deliver the same volume of paper which time it
will pay the balance of the price. Incidentally,
every month for a period of 18 months, with Printado in turn the value of the
agreeing to pay within 60 days after each delivery. Suplico has land had doubled by that time.
been faithfully delivering under the order agreement for 10 months
but thereafter stopped doing so, because Printado has not made
CIVIL LAW Answers to the BAR as Arranged by Topics (Year 1990-2006)
Salvador consigned the P 100,000.00 in court, and filed an property of ZY, his wife may also
sue to recover it under
action for rescission of the deed of conditional sale, plus Article 2016 of the Civil Code if she and
the family needed
damages. Will the action prosper? Explain. the money for support.
SUGGESTED ANSWER: A. 1. The suit by PX to collect the balance of
No, the action will not prosper. The action for what he won from ZY will not prosper. Under
rescission may be brought only by the Article 2014 of the Civil Code, no action can
aggrieved party to the contract. Since it was be maintained by the winner for the collection
Salvador who failed to comply with his of what he has won in a game of chance.
conditional obligation, he is not the aggrieved Although poker may depend in part on ability,
party who may file the action for rescission but it is fundamentally a game of chance.
the Star Semiconductor Company. The
company, however, is not opting to rescind the If the money paid by ZY to PX was conjugal or
contract but has chosen to waive Salvador's community property, the wife of ZY could sue
compliance with the condition which it can do to recover it because Article 117(7) of the
under Art. 1545, NCC. Family Code provides that losses in gambling
or betting are borne exclusively by the loser-
ALTERNATIVE ANSWER:
spouse. Hence, conjugal or community funds
The action for rescission will not prosper. The
may not be used to pay for such losses. If the
buyer has not committed any breach, let alone
money were exclusive
a substantial or serious one, to warrant the
rescission/resolution sought by the vendor. On
the contrary, it is the vendor who appears to
have failed to comply with the condition
imposed by the contract the fulfillment of
which would have rendered the obligation to
pay the balance of the purchase price
demandable. Further, far from being unable to
comply with what is incumbent upon it, ie., pay
the balance of the price the buyer has offered
to pay it even without the vendor having
complied with the suspensive condition
attached to the payment of the price, thus
waiving such condition as well as the 60-day
term in its favor The stipulation that the
P100,000.00 down payment shall be returned
by the vendor to the vendee if the squatters
are not removed within six months, is also a
covenant for the benefit of the vendee, which
the latter has validly waived by implication
when it offered to pay the balance of the
purchase price upon the execution of a deed of
absolute sale by the vendor. (Art. 1545, NCC)

OBLIGATIONS
Aleatory Contracts; Gambling (2004)
Mr. ZY lost P100,000 in a card game called
Russian poker, but he had no more cash to pay
in full the winner at the time the session
ended. He promised to pay PX, the winner,
two weeks thereafter. But he failed to do so
despite the lapse of two months, so PX filed in
court a suit to collect the amount of P50,000
that he won but remained unpaid. Will the
collection suit against ZY prosper? Could Mrs.
ZY file in turn a suit against PX to recover the
P100,000 that her husband lost? Reason. (5%)
SUGGESTED ANSWER:
ALTERNATIVE ANSWER (2): subject to the sole will of the debtor and,
A. (2). Mrs. ZY cannot file a suit to recover therefore the conditional obligation is void.
what her husband lost. Art 2014 of the Civil (Article 1182, NCC).
Code provides that any loser in a game of
chance may recover his loss from the winner, SUGGESTED ANSWER:
with legal interest from the time he paid the The obligation is valid. It is subject to a suspensive
amount lost. This means that only he can file condition, i.e. the future and uncertain event of his
the suit. Mrs. ZY cannot recover as a spouse becoming a lawyer. The performance of this obligation does
who has interest in the absolute community
property or conjugal partnership of gains,
because under Art. 117(7} of the Family Code,
losses are borne exclusively by the loser-
spouse. Therefore, these cannot be charged
against absolute community property or
conjugal partnership of gains. This being so,
Mrs. ZY has no interest in law to prosecute and
recover as she has no legal standing in court to
do so.
Conditional Obligations (2000)
Pedro promised to give his grandson a car if
the latter will pass the bar examinations. When
his grandson passed the said examinations,
Pedro refused to give the car on the ground
that the condition was a purely potestative
one. Is he correct or not? (2%)
SUGGESTED ANSWER:
No, he is not correct. First of all, the condition
is not purely potestative, because it does not
depend on the sole will of one of the parties.
Secondly, even if it were, it would be valid
because it depends on the sole will of the
creditor (the donee) and not of the debtor (the
donor).
Conditional Obligations (2003)
Are the following obligations valid, why, and if
they are valid, when is the obligation
demandable in each case? a) If the debtor
promises to pay as soon as he has the
means to pay; b) If the debtor promises to pay
when he likes; c) If the debtor promises to pay
when he becomes a

lawyer; d) If the debtor promises to pay if


his son, who is sick with cancer, does not
die within one year. 5%
SUGGESTED ANSWER:
The obligation is valid. It is an obligation
subject to an indefinite period because the
debtor binds himself to pay when his means
permit him to do so (Article 1180, NCC). When
the creditor knows that the debtor already has
the means to pay, he must file an action in
court to fix the period, and when the definite
period as set by the court arrives, the
obligation to pay becomes demandable
9Article 1197, NCC).
SUGGESTED ANSWER:
The obligation ―to pay when he likes‖ is a
suspensive condition the fulfillment of which is
CIVIL LAW Answers to the BAR as Arranged by Topics (Year 1990-2006)
not depend solely on the will of the debtor condition of Eva passing the 1998 Bar
but also on Examinations.
other factors outside the debtor’s control. Hence, upon Eva's passing the Bar, the rights of the other buyer
terminated and Eva acquired ownership of the property.
SUGGESTED ANSWER: Assuming that it is Eva who is entitled to buy
The obligation is valid. The death of the son of said house and lot, is she entitled to the rentals
cancer within one year is made a negative collected by Manuel before she passed the
suspensive condition to his making the 1998 bar examinations? Why? (3%)
payment. The obligation is demandable if the
SUGGESTED ANSWER:
son does not die within one year (Article 1185,
NCC). Yes, the sale to the other person is valid as a
sale with a resolutory condition because
what operates as a suspensive condition for
Conditional Obligations; Promise (1997) Eva operates a resolutory condition for the
In two separate documents signed by him, buyer.
FIRST ALTERNATIVE ANS WER:
Juan Valentino "obligated" himself each to Yes, the sale to the other person is valid. However, the buyer
Maria and to Perla, thus - 'To Maria, my true acquired the property subject to a resolutory
love, I obligate myself to give you my one and
only horse when I feel like It." - and -'To Perla,
my true sweetheart, I obligate myself to pay
you the P500.00 I owe you when I feel like it."
Months passed but Juan never bothered to
make good his promises. Maria and Perla
came to consult you on whether or not they
could recover on the basis of the foregoing
settings. What would your legal advice be?
SUGGESTED ANSWER:
I would advise Maria not to bother running
after Juan for the latter to make good his
promise. [This is because a promise is not an
actionable wrong that allows a party to
recover especially when she has not suffered
damages resulting from such promise. A
promise does not create an obligation on the
part of Juan because it is not something which
arises from a contract, law, quasi-contracts or
quasidelicts (Art, 1157)]. Under Art. 1182,
Juan's promise to Maria is void because a
conditional obligation depends upon the sole
will of the obligor.

As regards Perla, the document is an express


acknowledgment of a debt, and the promise to
pay what he owes her when he feels like it is
equivalent to a promise to pay when his means
permits him to do so, and is deemed to be one
with an indefinite period under Art. 1180.
Hence the amount is recoverable after Perla
asks the court to set the period as provided by
Art. 1197, par. 2.

Conditional Obligations; Resolutory Condition (1999)


In 1997, Manuel bound himself to sell Eva a
house and lot which is being rented by
another person, if Eva passes the 1998 bar
examinations. Luckily for Eva, she passed
said examinations.
Suppose Manuel had sold the same house
and lot to another before Eva passed the
1998 bar examinations, is such sale valid?
Why? (2%)
the obligation to deliver their sugar cane to
SECOND ALTERNATIVE ANSWER: the sugar central. As to the obligation to mill
The sale to another person before Eva could the sugar cane, the sugar central is a debtor of
buy it from Manuel is valid, as the contract the farmers. In assigning its rights under the
between Manuel and Eva is a mere promise to contract, the sugar central will also transfer to
sell and Eva has not acquired a real right over the Taiwanese its obligation to mill the sugar
the land assuming that there is a price cane of the farmers. This will amount to a
stipulated in the contract for the contract to novation of the contract by substituting the
be considered a sale and there was delivery or debtor with a third party. Under Article 1293
tradition of the thing sold. of the Civil Code, such substitution cannot take
effect without the consent of the creditor. The
SUGGESTED ANSWER:
formers, who are creditors as far as the
No, she is not entitled to the rentals collected obligation to mill their sugar cane is
by Manuel because at the time they accrued
and were collected, Eva was not yet the owner
of the property.
FIRST ALTERNATIVE ANSWER:
Assuming that Eva is the one entitled to buy
the house and lot, she is not entitled to the
rentals collected by Manuel before she passed
the bar examinations. Whether it is a contract
of sale or a contract to sell, reciprocal
prestations are deemed imposed A for the
seller to deliver the object sold and for the
buyer to pay the price. Before the happening
of the condition, the fruits of the thing and the
interests on the money are deemed to have
been mutually compensated under Article
1187.
SECOND ALTERNATIVE ANSWER:
Under Art. 1164, there is no obligation on the
part of Manuel to deliver the fruits (rentals) of
the thing until the obligation to deliver the
thing arises. As the suspensive condition has
not been fulfilled, the obligation to sell does
not arise.

Extinguishment; Assignment of Rights (2001)


The sugar cane planters of Batangas entered
into a long-term milling contract with the
Central Azucarera de Don Pedro Inc. Ten
years later, the Central assigned its rights to
the said milling contract to a Taiwanese group
which would take over the operations of the
sugar mill. The planters filed an action to
annul the said assignment on the ground that
the Taiwanese group was not registered with
the Board of Investments. Will the action
prosper or not? Explain briefly. (5%)
(Note: The question presupposes
knowledge and requires the application of
the provisions of the Omnibus Investment
Code, which properly belongs to
Commercial law)
SUGGESTED ANSWER:
The action will prosper not on the ground
invoked but on the ground that the farmers
have not given their consent to the
assignment. The milling contract imposes
reciprocal obligations on the parties. The
sugar central has the obligation to mill the
sugar cane of the farmers while the latter have
CIVIL LAW Answers to the BAR as Arranged by Topics (Year 1990-2006)
concerned, may annul such assignment for not Even [if] assuming that there was a perfect
having given right of first
refusal, compensation did not take place
their consent thereto. because the claim is
ALTERNATIVE ANSWER: unliquidated.
The assignment is valid because there is
absolute freedom to
transfer the credit and the creditor need not get
the consent Extinguishment; Compensation vs. Payment (1998)
Define compensation as a mode of
of the debtor. He only needs to notify him. extinguishing an
obligation, and distinguish it from payment.
[2%]
Extinguishment; Cause of Action (2004) SUGGESTED ANSWER:
TX filed a suit for ejectment against BD for non- COMPENSATION is a mode of extinguishing
payment of to the
condominium rentals amounting to P150,000. concurrent amount, the obligations of those
During the persons who in
pendency of the case, BD offered and TX their own right are reciprocally debtors and
accepted the full creditors of each
amount due as rentals from BD, who then filed a other (Tolentino, 1991 ed., p. 365, citing 2
motion to Castan 560 and
dismiss the ejectment suit on the ground that Francia vs. IAC. 162 SCRA 753). It involves the
the action is simultaneous
already extinguished. Is BD’s contention balancing of two obligations in order to
correct? Why or extinguish them to
the extent in which the amount of one is
why not? Reason. (5%) covered by that of
the other. (De Leon, 1992 ed., p. 221, citing 8
SUGGESTED ANSWER: Manresa 401).
BD's contention is not correct. TX can still
maintain the suit
for ejectment. The acceptance by the lessor of
the payment
by the lessee of the rentals in arrears even PAYMENT means not only delivery of
during the money but also
pendency of the ejectment case does not performance of an obligation (Article 1232,
constitute a waiver Civil Code). In
or abandonment of the ejectment case. (Spouses payment, capacity to dispose of the thing paid
Clutario v. and capacity to
receive payment are required for debtor
CA, 216 SCRA 341 [1992]). and creditor,
respectively: in compensation, such capacity is
not necessary,
because the compensation operates by law and
Extinguishment; Compensation (2002) not by the act
Stockton is a stockholder of Core Corp. He of the parties. In payment, the performance
desires to sell his must be complete;
shares in Core Corp. In view of a court suit that while in compensation there may be partial
Core Corp. extinguishment of
has filed against him for damages in the amount
of P 10 an obligation (Tolentino, supra)
million, plus attorney’s fees of P 1 million, as a
result of
statements published by Stockton which are
allegedly Extinguishment; Compensation/Set-Off; Banks (1998)
defamatory because it was calculated to injure X, who has a savings deposit with Y Bank in the
and damage the sum of
corporation’s reputation and goodwill. The P1,000,000.00 incurs a loan obligation with the
articles of said Bank in
incorporation of Core Corp. provide for a right the sum of P800.000.00 which has become due.
of first refusal When X tries
in favor of the corporation. Accordingly, to withdraw his deposit, Y Bank allows only
Stockton gave P200.000.00 to
written notice to the corporation of his offer to be withdrawn, less service charges,
sell his shares claiming that
of P 10 million. The response of Core corp. was compensation has extinguished its obligation
an acceptance under the
of the offer in the exercise of its rights of first savings account to the concurrent amount
refusal, offering of X's debt. X
for the purpose payment in form of contends that compensation is improper
compensation or set-off when one of the
against the amount of damages it is claiming debts, as here, arises from a contract of
against him, deposit. Assuming
exclusive of the claim for attorney’s fees. that the promissory note signed by X to
Stockton rejected the evidence the loan
offer of the corporation, arguing that does not provide for compensation between
compensation between said loan and his
the value of the shares and the amount of
damages demanded savings deposit, who is correct? [3%]
by the corporation cannot legally take effect. Is
Stockton SUGGESTED ANSWER:
Y bank is correct. An. 1287, Civil Code, does
correct? Give reason for your answer. (5%) not apply. All
the requisites of Art. 1279, Civil Code are
present. In the case
of Gullas vs. PNB [62 Phil. 519), the Supreme
SUGGESTED ANSWERS: Court held:
Stockton is correct. There is no right of "The Civil Code contains provisions regarding
compensation compensation
between his price of P10 million and Core (set off) and deposit. These portions of
Corp.’s Philippine law
unliquidated claim for damages. In order that provide that compensation shall take place
compensation when two persons
may be proper, the two debts must be liquidated are reciprocally creditor and debtor of each
and other. In this
demandable. The case for the P 10million connection, it has been held that the relation
damages being still existing between
pending in court, the corporation has as yet no a depositor and a bank is that of creditor and
claim which is debtor, x x x As
a general rule, a bank has a right of set off of
due and demandable against Stockton. the deposits in
its hands for the payment of any indebtedness
ANOTHER MAIN ANSWER: to it on the part
The right of first refusal was not perfected as a
right for the of a depositor." Hence, compensation took
reason that there was a conditional acceptance place between the
equivalent to a
counter-offer consisting in the amount of mutual obligations of X and Y bank.
damages as being
credited on the purchase price. Therefore,
compensation did
not result since there was no valid right of first
refusal (Art. Extinguishment; Condonation (2000)
Arturo borrowed P500,000.00 from his father.
1475 & 1319, NCC) After he had
paid P300,000.00, his father died. When the
ANOTHER MAIN ANSWER: administrator of
his father's estate requested payment of
the balance of
P200,000.00. Arturo replied that the same had
been
CIVIL LAW Answers to the BAR as Arranged by Topics (Year 1990-2006)
condoned by his father as evidenced by a The action will not prosper. The existence of
notation at the inflation or
back of his check payment for the P300,000.00 deflation requires an official declaration by the
reading: "In Bangko
full payment of the loan". Will this be a valid
defense in an Sentral ng Pilipinas.
action for collection? (3%) ALTERNATIVE ANSWER:
The unlawful detainer action will prosper. It is
SUGGESTED ANSWER: a given fact in
It depends. If the notation "in full payment of the the problem, that there was inflation, which
loan" was caused the
written by Arturo's father, there was an implied exchange rate to double. Since the contract
condonation itself authorizes
of the balance that discharges the obligation. In the increase in rental in the event of an
such case, the inflation or
notation is an act of the father from which devaluation of the Philippine peso, the
condonation may doubling of the
be inferred. The condonation being implied, monthly rent is reasonable and is therefore a
it need not valid act under
comply with the formalities of a donation to be the very terms of the contract. Brian's refusal
effective. The to pay is thus a
defense of full payment will, therefore, be valid. ground for ejectment.

Extinguishment; Loss (1994)


When, however, the notation was written by Dino sued Ben for damages because the latter
Arturo himself. had failed to
It merely proves his intention in making that deliver the antique Marcedes Benz car Dino
payment but in had purchased
no way does it bind his father (Yam v. CA, G.R from Ben, which was—by agreement—due for
No. 104726. 11 delivery on
February 1999). In such case, the notation was December 31, 1993. Ben, in his answer to
not the act of Dino's complaint,
his father from which condonation may be said Dino's claim has no basis for the suit,
inferred. There because as the car
being no condonation at all the defense of full was being driven to be delivered to Dino on
payment will January 1, 1994,
a reckless truck driver had rammed into the
not be valid. Mercedes Benz.
The trial court dismissed Dino's complaint,
ALTERNATIVE ANSWER: saying Ben's
If the notation was written by Arturo's father, it obligation had indeed, been extinguished by
amounted to force majeure. Is
an express condonation of the balance which
must comply the trial court correct?
with the formalities of a donation to be valid
under the 2nd
paragraph of Article 1270 of the New Civil Code.
Since the SUGGESTED ANSWER:
amount of the balance is more than 5,000 a) No. Article 1262, New Civil Code provides,
pesos, the "An obligation
acceptance by Arturo of the condonation must which consists in the delivery of a determinate
also be in thing shall be
writing under Article 748. There being no extinguished if it should be lost or destroyed
acceptance in without the fault
writing by Arturo, the condonation is void and of the debtor, and before he has incurred in
the obligation delay. b) The
to pay the balance subsists. The defense of full judgment of the trial court is incorrect. Loss of
payment is, the thing due
therefore, not valid. In case the notation was not by fortuitous events or force majeure is a valid
written by defense for a
Arturo's father, the answer is the same as the debtor only when the debtor has not incurred
answers above. delay.
Extinguishment of liability for fortuitous event
requires that
the debtor has not yet incurred any delay. In
the present case,
the debtor was in delay when the car was
Extinguishment; Extraordinary Inflation or Deflation (2001) destroyed on
On July 1, 1998, Brian leased an office space in a January 1, 1993 since it was due for delivery
building for a on December 31,
period of five years at a rental rate of P1,000.00
a month. The 1993. (Art. 1262 Civil Code)
contract of lease contained the proviso that
"in case of
inflation or devaluation of the Philippine peso,
the monthly
rental will automatically be increased or c) It depends whether or not Ben the seller,
decreased depending was already in
on the devaluation or inflation of the peso default at the time of the accident because a
to the dollar." demand for him
Starting March 1, 2001, the lessor increased to deliver on due date was not complied with
the rental to by him. That
P2,000 a month, on the ground of inflation fact not having been given in the problem, the
proven by the fact trial court
that the exchange rate of the Philippine peso to erred in dismissing Dino's complaint. Reason:
the dollar had There is
increased from P25.00=$1.00 to P50.00=$1.00. default making him responsible for fortuitous
Brian refused events
to pay the increased rate and an action for
unlawful detainer including the assumption of risk or loss.
was filed against him. Will the action prosper?
Why? (5%) If on the other hand Ben was not in default as
no demand
has been sent to him prior to the accident,
SUGGESTED ANSWER: then we must
The unlawful detainer action will not prosper. distinguish whether the price has been paid or
Extraordinary not. If it has
inflation or deflation is defined as the sharp been paid, the suit for damages should
decrease in the prosper but only to
purchasing power of the peso. It does not enable the buyer to recover the price paid. It
necessarily refer to should be noted
the exchange rate of the peso to the dollar. that Ben, the seller, must bear the loss on the
Whether or not principle of res
there exists an extraordinary inflation or perit domino. He cannot be held answerable
deflation is for the for damages as the
courts to decide. There being no showing that loss of the car was not imputable to his fault or
the purchasing fraud. In any
power of the peso had been reduced case, he can recover the value of the car from
tremendously, there the party whose
could be no inflation that would justify the negligence caused the accident. If no price has
increase in the been paid at
amount of rental to be paid. Hence, Brian could all, the trial court acted correctly in dismissing
refuse to pay the complaint.
the increased rate.
ALTERNATIVE ANSWER:
Extinguishment; Loss; Impossible Service (1993)

Page 86 of 119
CIVIL LAW Answers to the BAR as Arranged by Topics (Year 1990-2006)
In 1971, Able Construction, Inc. entered into a has been extinguished by the novation or
contract extinction of the
with Tropical Home Developers, Inc. whereby
the former principal obligation insofar as third parties are concerned.
would build for the latter the houses within its
subdivision.
The cost of each house, labor and materials
included, was Extinguishment; Payment (1995)
P100,000.00. Four hundred units were to be In 1983 PHILCREDIT extended loans to
constructed Rivett-Strom
within five years. In 1973, Able found that it Machineries, Inc. (RIVETTT-STROM),
could no longer consisting of US$10
continue with the job due to the increase in the Million for the cost of machineries imported
price of oil and directly paid
and its derivatives and the concomitant by PHTLCREDIT, and 5 Million in cash
worldwide spiraling payable in
of prices of all commodities, including basic raw installments over a period of ten (10) years on
materials the basis of the
required for the construction of the houses. The value thereof computed at the rate of exchange
cost of of the U.S.
development had risen to unanticipated levels dollar vis-à-vis the Philippine peso at the time
and to such a of payment.
degree that the conditions and factors which
formed the
original basis of the contract had been totally
changed. Able
brought suit against Tropical Homes praying RIVETT-STROM made payments on both loans
that the Court which if
relieve it of its obligation. Is Able Construction based on the rate of exchange in 1983
entitled to the would have fully
relief sought? settled the loans.
SUGGESTED ANSWER: promissory note for P1 million payable two
Yes, the Able Construction. Inc. is entitled to years later or in 1980 became a natural
the relief sought under Article 1267, Civil obligation after the lapse of ten (10) years,
Code. The law provides: "When the service such natural obligation can be a valid
has become so difficult as to be manifestly consideration of a novated promissory note
beyond the contemplation of the parties, the dated in 1991 and payable two years later, or
obligor may also be released therefrom, in in 1993. All the elements of an implied real
whole or in part." novation are present: a) an old valid obligation;
b) a new valid obligation; c) capacity of the
Extinguishment; Novation (1994)
parties; d) animus novandi or intention to
In 1978, Bobby borrowed Pl,000,000.00 from novate; and e) The old and the new obligation
Chito payable in two years. The loan, which should be incompatible with each other on all
was evidenced by a promissory note, was material points (Article 1292). The two
secured by a mortgage on real property. No promissory notes cannot stand together, hence,
action was filed by Chito to collect the loan or the period of prescription of ten (10) years has
to foreclose the mortgage. But in 1991, Bobby, not yet lapsed.
without receiving any amount from Chito,
executed another promissory note which was
worded exactly as the 1978 promissory note,
except for the date thereof, which was the date SUGGESTED ANSWER:
of its execution. 1) Can Chito demand payment No. The mortgage being an accessory contract prescribed with
on the 1991 promissory note in 1994? 2) Can the loan. The novation of the loan, however, did not expressly
include the mortgage, hence, the mortgage is extinguished under
Chito foreclose the real estate mortgage if Article 1296 of the NCC. The contract
Bobby fails to make good his obligation under
the 1991 promissory note?

SUGGESTED ANSWER:
Yes, Chito can demand payment on the 1991
promissory note in 1994. Although the 1978
PHILCREDIT contends that the payments on
both loans should be based on the rate of
exchange existing at the time of payment,
which rate of exchange has been
consistently increasing, and for which
reason there would still be a considerable
balance on each loan. Is the contention of
PHILCREDIT correct? Discuss fully.
SUGGESTED ANSWER:
As regards the loan consisting of dollars, the
contention of PHILCREDIT is correct. It has to
be paid in Philippine currency computed on
the basis of the exchange rate at the TIME OF
PAYMENT of each installment, as held in
Kalalo v. Luz, 34 SCRA 337. As regards the P5
Million loan in Philippine pesos, PHILCREDIT
is wrong. The payment thereof cannot be
measured by the peso-dollar exchange rate.
That will be violative of the Uniform Currency
Act (RA, 529] which prohibits the payment of
an obligation which, although to be paid in
Philippine currency, is measured by a foreign
currency. (Palanca v. CA, 238 SCRA 593).

Liability; Lease; Joint Liability (2001)


Four foreign medical students rented the
apartment of Thelma for a period of one year.
After one semester, three of them returned to
their home country and the fourth transferred
to a boarding house. Thelma discovered that
they left unpaid telephone bills in the total
amount of P80,000.00. The lease contract
provided that the lessees shall pay for the
telephone services in the leased premises.
Thelma demanded that the fourth student pay
the entire amount of the unpaid telephone
bills, but the latter is willing to pay only one
fourth of it. Who is correct? Why? (5%)
SUGGESTED ANSWER:
The fourth student is correct. His liability is
only joint, hence, pro rata. There is solidary
liability only when the obligation expressly so
states or when the law or nature of the
obligation requires solidarity (Art. 1207, CC).
The contract of lease in the problem does not,
in any way, stipulate solidarity.

Liability; Solidary Liability (1998)


Joey, Jovy and Jojo are solidary debtors under a loan
obligation of P300,000.00 which has fallen due. The
creditor has, however, condoned Jojo's entire share in the
debt. Since Jovy has become insolvent, the creditor makes
a demand on Joey to pay the debt.
CIVIL LAW Answers to the BAR as Arranged by Topics (Year 1990-2006)
How much, if any, may Joey be compelled promissory note as a result of the foreclosure
1) to pay? of the chattel
[2%] 2) To what extent, if at all, can Jojo be
compelled by mortgage.
Joey to contribute to such payment? [3%]
(c) The third defense of Y is untenable. Y is a
SUGGESTED ANSWER: surety of X
1. Joey can be compelled to pay only the and the extrajudicial demand against the
remaining balance principal debtor is
of P200.000, in view of the remission of not inconsistent with a judicial demand
Jojo's share by the against the surety. A
creditor. (Art. 1219,
Civil Code) suretyship may co-exist with a mortgage.
(d) The fourth defense of Y is untenable. Y is
liable for the
2. Jojo can be compelled by Joey to contribute entire prestation since Y incurred a solidary
P50.000 Art. obligation with
1217. par. 3, Civil Code provides. "When one of
the solidary X.
debtors cannot, because of his insolvency, (Arts. 1207, 1216. 1252 and 2047 Civil Code; Bicol
reimburse his Savings and Loan
share to the debtor paying the obligation, such
share shall be Associates vs. Guinhawa 188 SCRA 642)
borne by all his co-debtors, in proportion
to the debt of
each." Liability; Solidary Obligation; Mutual Guaranty (2003)
A,B,C,D, and E made themselves solidarity
indebted to X
Since the insolvent debtor's share which for the amount of P50,000.00. When X
Joey paid was demanded payment
P100,000, and there are only two remaining from A, the latter refused to pay on the
debtors - namely following grounds. a)
Joey and Jojo - these two shall share equally the B is only 16 years old. b) C has already been
burden of condoned by X
reimbursement. Jojo may thus be compelled c) D is insolvent. d) E was given by X an
by Joey to extension of 6
contribute
P50.000.00. months without
the consent of the other four co-debtors. State
Liability; Solidary Obligation (1992) the effect of
In June 1988, X obtained a loan from A and each of the above defenses put up by A on his
executed with Y obligation to
as solidary co-maker a promissory note in favor
of A for the pay X, if such defenses are found to be true.
sum of P200,000.00. The loan was payable at
P20,000.00 with
intere monthly firs week of mont
st within the t each h SUGGESTED ANSWERS:
beginning July 1988 until maturity in April 1989. (a) A may avail the minority of B as a defense, but
only for
To secure the B’s share of P 10,000.00. A solidary debtor
payment of the loan. X put up as security a may avail himself
chattel mortgage of any defense which personally belongs to a
on his car, a Toyota Corolla sedan. Because of solidary
failure of X co-debtor, but only as to the share of that
and Y to pay the principal amount of the loan, codebtor.
the car was
extrajudicially foreclosed. A acquired the car at
A's highest bid
of P120,000.00 during the auction sale. (b) A may avail of the condonation by X of C’s
share of P 10,
000.00. A solidary debtor may, in actions filed
by the creditor,
After several fruitless letters of demand against avail himself of all defenses which are derived
X and Y, A from the
nature of the obligation and of those which are
sued Y alone for the recovery of P80.000.00
personal to
constituting the
him or pertain to his own share. With respect
deficiency. Y resisted the suit raising the
to those which
following defenses:
personally belong to others, he may avail
a) That Y should not be liable at all
because X was not himself thereof only
sued together as regards that part of the debt for which the
with Y. latter are
That the obligation has been paid responsible. (Article 1222, NCC).
b) completely by A's
acquisition of the car through "dacion en pago"
(c) A may not interpose the defense of
or payment
insolvency of D as a
by cession.
defense. Applying the principle of mutual
That Y should not be held liable for the
guaranty among
c) deficiency
solidary debtors, A guaranteed the payment of
of P80,000.00 wa no a co-
D’s share and
because he s t mortgagor in the
of all the other co-debtors. Hence, A cannot
chattel mortgage of the car which contract was
avail of the
executed by X
alone as owner and defense of D’s insolvency.
mortgagor.
d) That assuming that Y is liable, he should
only pay the (d) The extension of six (6) months given by X
proportionate sum of P40,000.00. Decide each to E may be
defense with availed of by A as a partial defense but only
reaso for the share of
ns. E, there is no novation of the obligation but
SUGGESTED ANSWER: only an act of
(a) This first defense of Y is untenable. Y is
still liable as liberality granted to E alone.
solidary debtor. The creditor may proceed
against any one of
the solidary debtors. The demand against
one does not Loss of the thing due; Force Majeure (2000)
preclude further demand against the others Kristina brought her diamond ring to a jewelry
so long as the shop for
debt is not fully cleaning. The jewelry shop undertook to
paid. return the ring by
February 1, 1999." When the said date
arrived, the jewelry
(b) The second defense of Y is untenable. Y is shop informed Kristina that the Job was not
still liable. The yet finished.
chattel mortgage is only given as a They asked her to return five days later. On
security and not as February 6, 1999,
payment for the debt in case of failure to pay. Y Kristina went to the shop to claim the ring, but
as a solidary she was
co-maker is not relieved of further liability informed that the same was stolen by a thief
on the who entered the
shop the night before. Kristina filed an action
CIVIL LAW Answers to the BAR as Arranged by Topics (Year 1990-2006)
for damages against the jewelry shop which put up the Bernie 50% of the total payments made.
(Rillo v. Court of
defense of force majeure. Will the action prosper or not? Appeals, G.R. No. 125347, June
19,1997)
(5%)
SUGGESTED ANSWER:
The action will prosper. Since the defendant Period; Suspensive Period (1991)
was already in default not having delivered In a deed of sale of a realty, it was stipulated
the ring when delivery was demanded by that the buyer would construct a commercial
plaintiff at due date, the defendant is liable for building on the lot while the seller would
the loss of the thing and even when the loss construct a private passageway bordering the
was due to force majeure. lot. The building was eventually finished but
the seller failed to complete the passageway
Non-Payment of Amortizations; Subdivision Buyer; When as some of the squatters, who were already
justified (2005) known to be there at the time they entered
Bernie bought on installment a residential into the contract, refused to vacate the
subdivision lot from DEVLAND. After having premises. In fact, prior to its execution, the
faithfully paid the installments for 48 months, seller filed ejectment cases against the
Bernie discovered that DEVLAND had failed to squatters. The buyer now sues the seller for
develop the subdivision in accordance with the specific performance with damages. The
approved plans and specifications within the defense is that the obligation to construct the
time frame in the plan. He thus wrote a letter passageway should be with a period which,
to DEVLAND informing it that he was stopping incidentally, had not been fixed by them,
payment. Consequently, DEVLAND cancelled hence, the need for fixing a judicial period.
the sale and wrote Bernie, informing him that Will the action for specific performance of the
his payments are forfeited in its favor. buyer against the seller prosper?

a) Was the action of DEVLAND proper? SUGGESTED ANSWER:


Explain. (2%) No. the action for specific performance filed
by the buyer is premature under Art. 1197 of
SUGGESTED ANSWER: the Civil Code. If a period has not been fixed
No, the action of DEVLAND is not proper. Under although contemplated by the parties, the
Section 23 of Presidential Decree No. 957, parties themselves should fix that period,
otherwise known as the Subdivision and failing in which, the Court maybe asked to fix
Condominium Buyer's Protection Decree, non- it taking into consideration the probable
payment of amortizations by the buyer is justified contemplation of the parties. Before the
if non-payment is due to the failure of the period is fixed, an action for specific
subdivision owner to develop the subdivision performance is premature.
project according to the approved plans and ALTERNATIVE ANSWER:
within the limit for complying. It has been held in Borromeo vs. CA (47 SCRA
(Eugenio v. Drilon, G.R. No. 109404, January 22, 69), that the Supreme Court allowed the
1996) simultaneous filing of action to fix the
probable contemplated period of the parties
Discuss the rights of Bernie under the where none is fixed in the agreement if this
circums-tances. (2%) would avoid multiplicity of suits. In addition,
SUGGESTED ANSWER: technicalities must be subordinated to
Under P.D. No. 957, a cancellation option is substantial justice.
available to Bernie. If Bernie opts to cancel the ALTERNATIVE ANSWER:
contract, DEVLAND must reimburse Bernie The action for specific performance will not
the total amount paid and the amortizations prosper. The filing of the ejectment suit by the
interest, excluding delinquency interest, plus seller was precisely in compliance with his
interest at legal rate. (Eugenio v. Drilon, G.R. obligations and should not, therefore, be
No. 109404, faulted if no decision has yet been reached by
the Court on the matter.
January 22, 1996) In this case, pursuant to Section 24 of P.D.
No. 957, R.A. No. 6552 otherwise known as
Supposing DEVLAND had fully developed the Realty Installment Buyer Protection Act,
the subdivision but Bernie failed to pay shall govern. Under Section 3 thereof, Bernie
further installments after 4 years due to is entitled: 1) to pay without additional
business reverses. Discuss the rights and interest the unpaid installments due within a
obligations of the parties. (2%) grace period of four (4) months or one month
SUGGESTED ANSWER:
for every year of installment paid; 2) if the
TRUST
contract is cancelled, Bernie is entitled to the
refund of the cash surrender value equal to
50% of the total payments made.
Express Trust; Prescription (1997)
DEVLAND on the other hand has the right to On 01 January 1980, Redentor and Remedies
cancel the contract after 30 days from receipt entered into an agreement by virtue of which
by Bernie of notice of cancellation. DEVLAND the former was to register a parcel of land in
is however obliged to refund to the name of Remedies under the explicit
covenant to reconvey the land to Remigio, son
of Redentor, upon the son's graduation from
college. In 1981, the land was registered in
the name of Remedies.

Redentor died a year later or in 1982. In


March 1983, Remigio graduated from college.
In February 1992, Remigio accidentally found
a copy of the document so constituting
Remedies as the trustee of the land. In May
1994, Remigio filed a case against Remedies
for the reconveyance of the land to him.
Remedies, in her answer, averred that the
action already prescribed. How should the
matter be decided?

SUGGESTED ANSWER:
CIVIL LAW Answers to the BAR as Arranged by Topics (Year 1990-2006)
The matter should be decided in favor of 1. Juana has the right of action to recover (a)
Remigio (trustee) her one-half
because the action has not prescribed. The share in the proceeds of the sale with legal
case at bar interest thereof,
involves an express trust which does not and (b) such damages as she may be able to
prescribe as long as prove as having
they have not been repudiated by the been suffered by her, which may include actual
trustee (Diaz vs. or
compensatory damages as well as moral and
Gorricho. 103 Phil, 261). exemplary
damages due to the breach of trust and bad
faith (Imperial
vs. CA, 259 SCRA 65). Of course, if the buyer
Implied Trust (1998) knew of the
Juan and his sister Juana inherited from their co-ownership over the lot he was buying,
mother two Juana can seek (c)
parcels of farmland with exactly the same reconvenyance of her one-half share instead
areas. For but she must
convenience, the Torrens certificates of title implead the buyer as co-defendant and allege
covering both his bad faith in
lots were placed in Juan's name alone. In 1996, purchasing the entire lot. Finally, consistent
Juan sold to with the ruling in
an innocent purchaser one parcel in its entirety Imperial us. CA. Juana may seek instead (d) a
without the declaration that
knowledge and consent of Juana, and she is now the sole owner of the entire
wrongfully kept for remaining lot on the
theory that Juan has forfeited his one-half
himself the entire price paid. share therein.
What rights of action, if any, does Juana
1. have against
and/or the buyer? |3%] ADDITIONAL ANSWER:
2. Since the two lots have the same area, 1. Juana can file an action for damages against
suppose Juana Juan for having
flies a complaint to have herself declared sole fraudulently sold one of the two parcels which
owner of the he partly held
entire remaining second lot, contending that her in trust for Juana's benefit. Juana may claim
brother had actual or
forfeited his share thereof by wrongfully compensatory damage for the loss of her share
disposing of her in the land;
undivided share in the first lot. Will the suit moral damages for the mental anguish,
prosper? [2%] anxiety, moral shock
and wounded feelings she had suffered;
exemplary damage by
way of example for the common good, and
SUGGESTED ANSWER: attorney's fees.
1. When, for convenience, the Torrens title to Juana has no cause of action against the buyer
the two parcels who acquired
of land were placed in Joan's name alone, there the land for value and in good faith, relying on
was created the transfer
an implied trust (a resulting trust) for the certificate showing that Juan is the registered
benefit of Juana owner of the
with Juan as trustee of one-half undivided or
ideal portion of land.
each of the two lots. Therefore, Juana can file an
action for
damages against Joan for having fraudulently
sold one of the
two parcels which he partly held in trust for
Juana's benefit. SUGGESTED ANSWER:
Juana may claim actual or compensatory 2. Juana's suit to have herself declared as sole
owner of the
damage for the loss
entire remaining area will not prosper because
of while Juan's
her share in the land; moral damages for
the mental act in selling the other lot was wrongful. It did
anguish, anxiety, moral shock and wounded not have the
feelings she had legal effect of forfeiting his share in the
suffered; exemplary damage by way of remaining lot.
example for the However, Juana can file an action against Juan
for partition
common good, and attorney's fees.
or termination of the co-ownership with a
prayer that the lot
Juana has no cause of action against the buyer sold be adjudicated to Juan, and the remaining
who acquired lot be
the land for value and in good faith, relying on
the transfer adjudicated and reconveyed to her.
certificate of title showing that Juan is the ANOTHER ANSWER:
registered owner 2. The suit will prosper, applying the ruling in
Imperial vs.
of the land.
CA cited above. Both law and equity authorize
ANOTHER ANSWER: such a result,
1. Under Article 476 of the Civil Code, Juana can
file an said the Supreme Court.
action for quieting of title as there is a cloud in
the title to the Strictly speaking, Juana's contention that her
subject real property. Second, Juana can also file brother had
an action for forfeited his share in the second lot is
damages against Juan, because the incorrect. Even if the
settled rule is that the
proper recourse of the true owner of the two lots have the same area, it does not follow
property who was that they have
the same value. Since the sale of the first lot
prejudiced and fraudulently dispossessed of the on the Torrens
same is to title in the name of Juan was valid, all that
bring an action for damages against those who Juana may recover
caused or is the value of her undivided interest therein,
employed the same. Third, since Juana had the plus damages.
right to her In addition, she can ask for partition or
share in the property by way of inheritance, she reconveyance of her
can demand undivided interest in the second lot, without
the partition of the thing owned in common, prejudice to any
under Article agreement between them that in lieu of the
494 of the Civil Code, and ask that the title to payment of the
the remaining value of Juana's share in the first lot and
damages, the second
property be declared as exclusively hers.
lot be reconveyed to her.
ALTERNATIVE ANSWER:
However, since the farmland was sold to an
The suit will not prosper, since Juan's
innocent purchaser for value, then Juana has no
wrongful act of pocketing the entire proceeds
cause of action against the buyer consistent with
of the sale of the first lot is not a ground for
the established rule that the rights of an innocent
divesting him of his rights as a co-owner of
purchaser for value must be respected and
the second lot. Indeed, such wrongdoing by
protected notwithstanding the fraud employed by
Juan does not constitute, for the benefit of
the seller in securing his title. (Eduarte vs. CA,
Juana, any of the modes of acquiring
253 SCRA 391)
ownership under Art. 712, Civil Code.
ADDITIONAL ANSWER:
CIVIL LAW Answers to the BAR as Arranged by Topics creditor. Will Peter Co's defense of absence of
Trust; Implied Resulting Trust (1995) agreement to a subrogation of creditor prosper?
In 1960, Maureen purchased two lots in a
SUGGESTED ANSWER:
plush subdivision registering Lot 1 in her name
and Lot 2 in the name of her brother Walter No, Co's defense will not prosper. This is not a
with the latter's consent. The idea was to case of subrogation, but an assignment of
circumvent a subdivision policy against the credit. ASSIGNMENT OF CREDIT is the process
acquisition of more than one lot by one buyer. of transferring the right of the assignor to the
Maureen constructed a house on Lot 1 with an assignee. The assignment may be done either
extension on Lot 2 to serve as a guest house. gratuitously or onerously, in which case, the
In 1987, Walter who had suffered serious assignment has an effect similar to that of a sale
(Nyco Sales Corp.v.BA Finance Corp. G.R
business losses demanded that Maureen
No.71694. Aug.16, 1991 200 SCRA 637). As a
remove the extension house since the lot on
which the extension was built was his property. result of the assignment, the plaintiff acquired
In 1992, Maureen sued for the reconveyance all the rights of the assignor including the right
to her of Lot 2 asserting that a resulting trust to sue in his own name as the legal assignee. In
was created when she had the lot registered in assignment, the debtor's consent is not essential
Walter's name even if she paid the purchase for the validity of the assignment
price. Walter opposed the suit arguing that
assuming the existence of a resulting trust the
action of Maureen has already prescribed
since ten years have already elapsed from the
registration of the title in his name. Decide.
Discuss fully.
SUGGESTED ANSWER:
This is a case of an implied resulting trust. If
Walter claims to have acquired ownership of
the land by prescription or if he anchors his
defense on extinctive prescription, the ten
year period must be reckoned from 1987
when he demanded that Maureen remove the
extension house on Lot No. 2 because such
demand amounts to an express repudiation of
the trust and it was made known to Maureen.
The action for reconveyance filed in 1992 is
not yet barred by prescription.
(Spouses Huang v. Court of Appeals, Sept. 13,
1994).

SALES
Assignment of Credit vs. Subrogation (1993)
Peter Co, a trader from Manila, has dealt
business with Allied Commodities in
Hongkong for five years. All through the
years, Peter Co accumulated an indebtedness
of P500,000.00 with Allied Commodities. Upon
demand by its agent in Manila, Peter Co paid
Allied Commodities by check the amount
owed. Upon deposit in the payee's account in
Manila, the check was dishonored for
insufficiency of funds. For and in consideration
of P1.00, Allied Commodities assigned the
credit to Hadji Butu who brought suit against
Peter Co in the RTC of Manila for recovery of
the amount owed. Peter Co moved to dismiss
the complaint against him on the ground that
Hadji Butu was not a real party in interest
and, therefore, without legal capacity to sue
and that he had not agreed to a subrogation of
(Year 1990-2006) another couple, Bernie and Elena, offered a
(Art. 1624; 1475. CC; Rodriguez v. CA, et al, similar house at a lower price of P 1.5 Million.
G. R No. 84220, March 25. 1992 207 SCRA But Ray insisted on buying the house of Biong
553). and Linda for sentimental reasons. Ray
ALTERNATIVE ANSWER:
prepared a deed of sale to be signed by the
No, the defense of Peter Co will not prosper.
couple and a manager's check for P2 Million.
Hadji Butu validly acquired his right by an
After receiving the P2 Million, Biong signed
assignment of credit under Article 1624 of
the deed of sale. However, Linda was not able
the Civil Code. However, the provisions on
to sign it because she was abroad. On her
the contract of sale (Article 1475 Civil Code)
return, she refused to sign the document
will apply, and the transaction is covered by
saying she changed her mind. Linda filed suit
the Statute of Frauds. (Art. 1403 par. (2)
for nullification of the deed of sale and for
Civil Code)
moral and exemplary damages against Ray.
Conditional Sale vs. Absolute Sale (1997) Will the suit prosper? Explain. (2.5%)
Distinguish between a conditional sale, on ALTERNATIVE ANSWER:
the one hand, and an absolute sale, on the
other hand.
SUGGESTED ANSWER:
A CONDITIONAL SALE is one where the
vendor is granted the right to unilaterally
rescind the contract predicated on the
fulfillment or non-fulfillment, as the case may
be, of the prescribed condition. An
ABSOLUTE SALE is one where the title to the
property is not reserved to the vendor or if
the vendor is not granted the right to rescind
the contract based on the fulfillment or
nonfulfillment, as the case may be, of the
prescribed condition.
Contract of Sale vs. Agency to Sell (1999)
A granted B the exclusive right to sell his
brand of Maong pants in Isabela, the price for
his merchandise payable within 60 days from
delivery, and promising B a commission of
20% on all sales. After the delivery of the
merchandise to B but before he could sell any
of them, B’s store in Isabela was completely
burned without his fault, together with all of
A's pants. Must B pay A for his lost pants?
Why? (5%)
SUGGESTED ANSWER:
The contract between A and B is a sale not
an agency to sell because the price is
payable by B upon 60 days from delivery
even if B is unable to resell it. If B were an
agent, he is not bound to pay the price if he
is unable to resell it.

As a buyer, ownership passed to B upon


delivery and, under Art. 1504 of the Civil
Code, the thing perishes for the owner.
Hence, B must still pay the price.

Contract of Sale; Marital Community Property;


Formalities (2006)
Spouses Biong and Linda wanted to sell their
house. They found a prospective buyer, Ray.
Linda negotiated with Ray for the sale of the
property. They agreed on a fair price of P2
Million. Ray sent Linda a letter confirming
his intention to buy the property. Later,
CIVIL LAW Answers to the BAR as Arranged by Topics (Year 1990-2006)
No, the suit will not prosper. The contract of In a CONTRACT OF SALE, ownership is
sale was transferred to
perfected when Linda and Ray agreed on the the buyer upon delivery of the object to
object of the him while in a
sale and the price [Art. 1475, New Civil Code]. CONTRACT TO SELL, ownership is retained by
The consent the seller
of Linda has already been given, as shown by until the purchase price is fully paid. In a
her agreement contract to sell,
to the price of the sale. There is therefore delivery of the object does not confer
consent on her part ownership upon the
as the consent need not be given in any specific buyer. In a contract of sale, there is only
form. Hence, one contract
her consent may be given by implication, executed between the seller and the buyer,
especially since she while in a contract
was aware of, and participated in the sale of the to sell, there are two contracts, first the
property contract to sell (which
(Pelayo v. CA, G.R. No. 141323, June 8, 2005). is a conditional or preparatory sale) and a
Her action for second, the final
moral and exemplary damages will also not deed of sale or the principal contract which is
prosper because executed after
the case does not fall under any of those
mentioned in Art. full payment of the purchase price.
2219 and 2232 of the Civil Code.
ALTERNATIVE ANSWER: Contract to Sell; Acceptance; Right of First Refusal (1991)
The suit will prosper. Sale of community A is the lessee of an apartment owned by Y. A
property requires allowed his
written consent of both spouses. The failure or married but employed daughter B, whose
refusal of husband works in
Linda to affix her signature on the deed of sale, Kuwait, to occupy it. The relationship
coupled with between Y and A
her express declaration of opposing the sale soured. Since he has no reason at all to
negates any valid eject A, Y, in
consent on her part. The consent of Biong connivance with the City Engineer, secured
by himself is from the latter an
insufficient to effect a valid sale of community order for the demolition of the building. A
property (Art. immediately filed
96, Family Code; Abalos v. Macatangay, G.R. an action in the Regional Trial Court to annul
No. 155043, the order and
to enjoin its enforcement. Y and A were
September 30, 2004). able to forge a
compromise agreement under which A agreed
to a twenty
Does Ray have any cause of action against percent (20%) increase in the monthly rentals.
Biong and They further
Linda? Can he also recover damages from agreed that the lease will expire two (2) years
the spouses? later and that in
the woul sel the property, either
Explain. (2.5%) event that Y dl A or his
Considering that the contract has already been
perfected and daught B th right of first Th
taken out of the operation of the statute of er shall have e refusal. e
frauds, Ray can Compromise Agreement was approved by the
compel Linda and Biong to observe the form court. Six (6)
required by law months before the expiration of the lease, A
in order for the property to be registered in the died. Y sold the
name of Ray property to the Visorro Realty Corp.
without notifying
which can be filed together with the action for B. B then filed an action to rescind the sale in
the recovery favor of the
of house [Art. 1357 New Civil Code]. In the corporation and to compel Y to sell the
alternative, he property to her since
can recover the amount of Two million pesos under the Compromise Agreement, she was
(P2,000,000.00) given the right
that he paid. Otherwise, it would result in of first refusal which, she maintains is a
solutio indebiti or stipulation pour atrui
unjust enrichment. under Article 1311 of the Civil Code. Is she
correct?
Ray can recover moral damages on the ground
that the action SUGGESTED ANSWER:
filed by Linda is clearly an unfounded civil suit B is not correct. Her action cannot prosper.
which falls Article 1311
under malicious prosecution {Ponce v. requires that the third person intended to be
Legaspi, G.R. No. benefited must
communicate his acceptance to the obligor
79184, May 6,1992). before the
revocation. There is no showing that B
manifested her
acceptance to Y at any time before the death of
Contract to Sell (2001) A and before
Arturo gave Richard a receipt which states: the sale. Hence, B cannot enforce any right
under the alleged
Receipt Received from Richard as down stipulation pour
payment for my atrui.
1995 Toyota Corolla with plate No. XYZ-1
23..............
P50.000.00 Double Sales (2001)
On June 15, 1995, Jesus sold a parcel of
registered land to
Jaime. On June 30, 1995, he sold the same land
to Jose. Who
has a better right if: a) the first sale is
Balance payable: 12/30/01........ P50 000.00 registered ahead of the
second sale,
with knowledge of the latter. Why? (3%) b) the
September 15, 2001. second sale is
registered ahead of the first sale,
(Sgd.) Arturo Does this receipt with knowledge of the latter? Why?
evidence a (5%)
contract to sell? Why? (5%) SUGGESTED ANSWER:
(a) The first buyer has the better right if his
SUGGESTED ANSWER: sale was first to
It is a contract of sale because the seller did not be registered, even though the first buyer
reserve knew of the second
ownership until he was fully paid. sale. The fact that he knew of the second sale
at the time of
his registration does not make him as acting in
bad faith
because the sale to him was ahead in time,
Contract to Sell vs. Contract of Sale (1997) hence, has a
State the basic difference (only in their legal priority in right. What creates bad faith in the
effects) Between case of double
a contract to sell, on the one hand, and a sale of land is knowledge of a previous sale.
contract of sale, on
the other.
SUGGESTED ANSWER:
CIVIL LAW Answers to the BAR as Arranged by Topics (Year 1990-2006)
b) The first buyer is still to be preferred, where (2) years, or until 3 June 1973. It is further
the second stated therein
sale is registered ahead of the first sale but that should the Vendor (Juliet) fail to exercise
with knowledge of her right to
the latter. This is because the second buyer, redeem within the said period, the
who at the time conveyance shall be
he registered his sale knew that the property deemed absolute and irrevocable. Romeo did
had already been not take
sold to someone else, acted in bad faith. (Article possession of the property. He did not pay the
1544, C.C.) taxes thereon.

Juliet died in January I973 without having


Double Sales (2004) repurchased the
JV, owner of a parcel of land, sold it to PP. But property. Her only surviving heir, her son X,
the deed of failed to
sale was not registered. One year later, JV sold repurchase the property on or before 3 June
the parcel 1973. In 1975,
again to RR, who succeeded to register the Romeo sold the property to Y for P50,000.00.
deed and to Upon learning
obtain a transfer certificate of title over the of the sale, X filed an action for the
property in his nullification of the sale
own name. Who has a better right over the and for the recovery of the property on the
parcel of land, ground that the
RR or PP? Why? Explain the legal basis for your so-called deed of absolute sale executed by
answer. his mother was
merely an equitable mortgage, taking into
(5%) account the
inadequacy of the price and the failure of
SUGGESTED ANSWER: Romeo to take
It depends on whether or not RR is an innocent possession of the property and to pay the
purchaser taxes thereon.
for value. Under the Torrens System, a deed or Romeo and Y maintain that there was a valid
instrument absolute sale
operated only as a contract between the parties and that the document signed by the former
and as on 3 June 1973
evidence of authority to the Register of Deeds was merely a promise to sell. a) If you were
to make the the Judge, would
registration. It is the registration of the deed or
the you uphold the theory of
instrument that is the operative act that X? b) If you decide in favor of Romeo and Y,
conveys or affects would you
the land. (Sec. 51, P.D. No. 1529).
uphold the validity of the promise to sell?
(1991)On20 December 1970, Juliet, a widow,
In cases of double sale of titled land, it is a borrowed from Romeo P4,000.00 and, as
well-settled rule that the buyer who first security therefore, she executed a deed of
registers the sale in good faith acquires a mortgage over one of her two (2) registered
better right to the land. (Art. 1544, Civil lots which has a market value of P15,000.00.
Code). The document and the certificate of title of
the property were delivered to Romeo.
Persons dealing with property covered by
Torrens title are not required to go beyond On 2 June 1971, Juliet obtained an additional
what appears on its face. sum of P3,000 from Romeo. On this date,
(Orquiola v. CA 386, SCRA 301, [2002]; however, Romeo caused the preparation of a
Domingo v. Races 401 SCRA 197, [2003]). deed of absolute sale of the above property, to
Thus, absent any showing that RR knew which Juliet affixed her signature without first
about, or ought to have known the prior sale reading the document. The consideration
of the land to indicated is P7,000.00. She thought that this
or that he acted in bad faith, and being first to document was similar to the first she signed.
register the sale, RR acquired a good and a When she reached home, her son X, after
clean title to the property as against PP. reading the duplicate copy of the deed,
informed her that what she signed was not a
Equitable Mortgage mortgage but a deed of absolute sale. On the
following day, 3 June 1971, Juliet,
accompanied by X, went back to Romeo and SUGGESTED ANSWER:
demanded the reformation it, Romeo prepared A. I will not uphold the theory of X for the
and signed a document wherein, as vendee in nullification of the sale and for the recovery of
the deed of sale above mentioned, he the property on the ground that the so-called
obligated and bound himself to resell the land sale was only an equitable mortgage. An
to Juliet or her heirs and successors for the equitable mortgage may arise only if, in truth,
same consideration as reflected in the deed of the sale was one with the right of repurchase.
sale (P7,000) within a period of two The facts of the case state that the right to
repurchase was granted after the absolute
deed of sale was executed. Following the rule
in Cruzo vs. Carriaga (174 SCRA 330), a deed
of repurchase executed independently of the
deed of sale where the two stipulations are
found in two instruments instead of one
document, the right of repurchase would
amount only to one option granted by the
buyer to the seller. Since the contract cannot
be upheld as a contract of sale with the right
to repurchase, Art. 1602 of the Civil Code on
equitable mortgage will not apply. The rule
could have been different if both deeds were
executed on the same occasion or date, in
which case, under the ruling in spouses
Claravall v. CA (190 SCRA 439), the contract
may still be sustained as an equitable
mortgage, given the circumstances expressed
in Art. 1602. The reserved right to repurchase
is then deemed an original intention.

B. If I were to decide in favor of Romeo and


Y, I would not uphold the validity of the
promise to sell, so as to enforce it by an
action for specific performance. The promise
to sell would only amount to a mere offer
and, therefore, it is not enforceable unless it
was sought to be exercised before a
withdrawal or denial thereof.

Even assuming the facts given at the end of


the case, there would have been no separate
consideration for such promise to sell. The
contract would at most amount to an option
which again may not be the basis for an
action for specific performance.
Equitable Mortgage vs. Sale (2005)
On July 14, 2004, Pedro executed in favor of
Juan a Deed of
Absolute Sale over a parcel of land covered by
TCT No. Page 93
of 119
CIVIL LAW Answers to the BAR as Arranged by Topics (Year 1990-2006)
6245. It appears in the Deed of Sale that Pedro X sold a parcel of land to Y on 01 January
received 2002, payment
from Juan P120,000.00 as purchase price. and delivery to be made on 01 February 2002.
However, Pedro It was
retained the owner's duplicate of said title. stipulated that if payment were not to be made
Thereafter, Juan, by Y on 01
as lessor, and Pedro, as lessee, executed a February 2002, the sale between the parties
contract of lease would
over the property for a period of one (1) year automatically be rescinded. Y failed to pay on
with a monthly 01 February
rental of Pl,000.00. Pedro, as lessee, was also 2002, but offered to pay three days later,
obligated to pay which payment X
the realty taxes on the property during the period refused to accept, claiming that their contract
of lease. of sale had
already been rescinded. Is X’s contention
correct? Why? 5%
SUGGESTED ANSWER:
Subsequently, Pedro filed a complaint against No, X is not correct. In the sale of immovable
Juan for the property, even
reformation of the Deed of Absolute Sale, though it may have been stipulated, as in this
alleging that the case, that upon
transaction covered by the deed was an equitable failure to pay the price at the time agreed
mortgage. upon the rescission
In his verified answer to the complaint, Juan of the contract shall of right take place, the
alleged that the vendee may pay,
property was sold to him under the Deed of even after the expiration of the period, as long
Absolute Sale, as no demand
and interposed counterclaims to recover for rescission of the contract has been made
possession of the upon him either
property and to compel Pedro to turn over to him judicially or by a notarial act (Article 1592,
the New Civil code).
owner's duplicate of title. Resolve the case with Since no demand for rescission was made on Y,
reasons. (6%) either
judicially or by a notarial act, X cannot refuse
to accept the
payment offered by Y three (3) days after the
SUGGESTED ANSWER: expiration of
The complaint of Pedro against Juan should be
dismissed. the period.
The instances when a contract — regardless
of its ANOTHER SUGGESTED ANSWER:
nomenclature — may be presumed to be an This is a contract to sell and not a contract of
equitable absolute sale,
mortgage are enumerated in Article 1602 of the since as there has been no delivery of the land.
Civil Code: Article 1592 of
"Art. 1602. The contract shall be presumed to be the New Civil code is not applicable. Instead,
an equitable Article 1595 of
the New Civil Code applies. The seller has two
mortgage, in any of the following cases: alternative
When the price of a sale with right to remedies: (1) specific performance, or (2)
1 repurchase is rescission or
resolution under Article 1191 of the New Civil
unusually inadequate: code. In both
When the vendor remains in possession as
2 lessee or remedies, damages are due because of default.
otherwise; ALTERNATIVE ANSWER:
When upon or after the expiration of the Yes, the contract was automatically rescinded
upon Y’s failure
3 right to to pay on 01 February 2002. By the express
terms of the
repurchase another instrument extending the period of redemption
contract, there is no need for X to make a
or granting a new period is executed; demand in order
When the purchaser retains for himself a for rescission to take place. (Article 1191, New
Civil Code, Suria
4 part of the
purchase price; v. IAC 151 SCRA 661 [1987]; U.P. v. de los
When the vendor binds himself to pay the
5 taxes on the Angeles 35 SCRA 102 [1970]).
thing sold;
Maceda Law (2000)
In any other case where it may be fairly Priscilla purchased a condominium unit in
6 inferred that the Makati City from
real intention of the parties is that the the Citiland Corporation for a price of P10
transaction shall secure the Million, payable
payment of a debt or the performance of any P3 Million down and the balance with interest
other obligation. thereon at 14%
per annum payable in sixty (60) equal monthly
"In any of the foregoing cases, any money, installments of P198,333.33. They executed a
Deed of Conditional Sale in which it is
fruits, or other benefit to be received by the
stipulated that should the vendee fail to pay
vendee as rent or otherwise shall be
three
considered as interest which shall be subject
successive installments, the sale shall be
to the usury laws." deemed automatically rescinded without the
Article 1604 states that "the provisions of necessity of judicial action and all payments
made by the vendee shall be forfeited in favor
article 1602 shall also apply to a contract
of the vendor by way of rental for the use and
purporting to be an absolute sale." occupancy of the unit and as liquidated
damages. For 46 months, Priscilla paid the
For Articles 1602 and 1604 to apply, two
monthly installments religiously, but on the
requisites must concur: 1) the parties entered
47th and 48th months, she failed to pay. On
into a contract denominated as a contract of
the 49th month, she tried to pay the
sale; and 2) their intention was to secure an installments due but the vendor refused to
existing debt by way of mortgage. (Heirs of receive the payments tendered by her. The
Balite v. Lim, following month, the vendor sent her a notice
G.R. No. 152168, December 10, 2004)
that it was rescinding the Deed of Conditional
In the given case, although Pedro retained Sale pursuant to the stipulation for automatic
possession of the property as lessee after the rescission, and demanded that she vacate the
execution of the Deed of Sale, there is no premises. She replied that the contract cannot
be rescinded without judicial demand or
showing that the intention of the parties was
notarial act pursuant to
to secure an existing debt by way of mortgage.
Article 1592 of the Civil Code. a) Is Article
Hence, the complaint of Pedro should be
1592 applicable? (3%) b) Can the vendor
dismissed.
rescind the contract? (2%)

Immovable Property; Rescission of Contract (2003) SUGGESTED ANSWER:


Page 94 of 119
CIVIL LAW Answers to the BAR as Arranged by Topics (Year 1990-2006)
a) Article 1592 of the Civil Code does not apply to a
conditional sale. In Valarao v. CA, 304 SCRA
155, the Option Contract (2002)
Supreme Court held that Article 1592 applies
only to a Explain the nature of an option contract. (2%)
contract of sale and not to a Deed of
Conditional Sale where SUGGESTED ANSWER:
the seller has reserved title to the property until An OPTION CONTRACT is one granting a
full payment privilege to buy
of the purchase price. The law applicable is the or sell within an agreed time and at a
Maceda Law. determined price. It
must be supported by a consideration distinct
from the price.
SUGGESTED ANSWER: (Art. 1479 and 1482, NCC)
b) No, the vendor cannot rescind the contract
under the
circumstances. Under the Maceda Law, which is
the law Option Contract; Earnest Money (1993)
applicable, the seller on installment may not LT applied with BPI to purchase a house and
rescind the lot in Quezon
contract till after the lapse of the mandatory City, one of its acquired assets. The
grace period of amount offered was
30 days for every one year of installment Pl,000,000.00 payable, as follows: P200,000.00
payments, and only down payment,
after 30 days from notice of cancellation or the balance of P800,000.00 payable within 90
demand for days from June
rescission by a notarial act. In this case, the 1, 1985. BPI accepted the offer, whereupon LT
refusal of the drew a check
seller to accept payment from the buyer on the for P200,000.00 in favor of BPI which the latter
49th month thereafter
was not justified because the buyer was entitled deposited in its account. On September 5,
to 60 days 1985, LT wrote
grace period and the payment was tendered BPI requesting extension until October 10,
within that 1985 within which
period. Moreover, the notice of rescission to pay the balance, to which BPI agreed. On
served by the seller October 5, 1985,
on the buyer was not effective because the due to the expected delay in the remittance of
notice was not by the needed
a notarial act. Besides, the seller may still pay amount by his financier from the United States,
within 30 days LT wrote BPI
from such notarial notice before rescission may requesting a last extension until October
be effected. 30, 1985, within
All these requirements for a valid rescission which to pay the balance. BPI denied LTs
were not request because
complied with by the seller. Hence, the another had offered to buy the same
rescission is invalid. property for
P1,500,000.00. BPI cancelled its agreement
with LT and
offered to return to him the amount of
P200,000.00 that LT
had paid to it. On October 20, 1985, upon
Maceda Law; Recto Law (1999) receipt of the
What are the so-called "Maceda" and "Recto" amount of P800,000.00 from his US financier,
laws in LT offered to
connection with sales on installments? Give the pay the amount by tendering a cashier's check
most therefor but
which BPI refused to accept. LT then filed a
important features of each law. (5%) complaint against
BPI in the RTC for specific performance and
SUGGESTED ANSWER: deposited in
The MACEDA LAW (R.A. 655) is applicable to sales of court the amount of P800,000.00. Is BPI legally
immovable property on installments. The most correct in
important
features are (Rillo v. CA, 247 SCRA 461): canceling its contract with LT?
(1) After having paid installments for at least
two years, the SUGGESTED ANSWER:
buyer is entitled to a mandatory grace period of BPI is not correct in canceling the contract
one month with LT. In Lina
for every year of installment payments made, to Topacio v Court of Appeals and BPI Investment
pay the (G. R No.
102606, July 3. 1993, 211 SCRA 291) the
unpaid installments without interest. Supreme Court held
that the earnest money is part of the purchase
price and is
If the contract is cancelled, the seller shall proof of the perfection of the contract.
refund to the Secondly, notarial or
buyer the cash surrender value equivalent to judicial rescission under Art. 1592 and 1991 of
fifty percent the Civil Code
(50%) of the total payments made, and after is necessary (Taguba v. de Leon, 132 SCRA
five years of 722.)
installments, an additional five percent (5%)
every year but ALTERNATIVE ANSWER:
not to exceed ninety percent (90%) of the total BPI is correct in canceling its contract with LT
payments but BPI must
do so by way of judicial rescission under Article
made. 1191 Civil
Code. The law requires a judicial action, and
mere notice of
(2) In case the installments paid were less than rescission is insufficient if it is resisted. The
2 years, the law also provides
seller shall give the buyer a grace period of not that slight breach is not a ground for rescission
less than 60 (Song Fo &
days. If the buyer fails to pay the installments Co, vs, Hawaiian Phil Co., 47 Phils. 821),
due at the Delay in the
expiration of the grace period, the seller may fulfillment of the obligation (Art. 1169, Civil
cancel the Code) is a
contract after 30 days from receipt by the buyer ground to rescind, only if time is of the
of the notice essence. Otherwise,
of cancellation or demand for rescission by the court may refuse the rescission if there is a
notarial act. The just cause for
RECTO LAW (Art. 1484} refers to sale of
movables the fixing of a period.
payable in installments and limiting the right of
seller, in case
of default by the buyer, to one of three
remedies: a) exact Perfected Sale; Acceptance of Earnest Money (2002)
fulfillment; b) cancel the sale if two or more Bert offers to buy Simeon’s property under
installments the following
have terms and conditions: P1 million purchase
not price, 10% option
money, the balance payable in cash upon the
clearance of the
property of all illegal occupants. The option
been paid; money is
foreclose the chattel mortgage on the promptly paid and Simeon clears the
c) things sold, property of illegal
also in case of default of two or more occupants in no time at all. However, when
installments, with no Bert tenders
payment of the balance and ask Simeon for the
further action against the purchaser. deed
CIVIL LAW Answers to the BAR as Arranged by Topics (Year 1990-2006)
for absolute sale, Simeon suddenly has a change May Adela still exercise her right of
of heart, redemption? Explain.
claiming that the deal is disadvantageous to him
as he has (5%)
found out that the property can fetch three time
the agreed SUGGESTED ANSWER:
purchase price. Bert seeks specific performance Yes, Adela may still exercise her right of
but Simeon redemption
contends that he has merely given Bert an notwithstanding the lapse of more than 30
option to buy and days from notice
nothing more, and offers to return the option of the sale given to her because Article 1623
money which of the New Civil
Code requires that the notice in writing of the
Bert refuses to accept. sale must come
Will Bert’s action for specific performance prosper? from the prospective vendor or vendor as the case may be. In
this case, the notice of the sale was given by
Explain. (4%) the vendee and
C. May Simeon justify his refusal to proceed the Register of Deeds. The period of 30 days
with the sale by never tolled. She
the fact that the deal is financially
disadvantageous to him? can still avail of that right.
Explain. (4%) ALTERNATIVE ANSWER:
Adela can no longer exercise her right of
SUGGESTED ANSWER: redemption. As
co-owner, she had only 30 days from the time
B. Bert’s action for specific performance will prosper because she received
there was a binding agreement of sale, not just written notice of the sale which in this case
an option took the form of a
contract. The sale was perfected upon copy of the deed of sale being given to her
acceptance by Simeon (Conejero v. CA, 16
of 10% of the agreed price. This amount is in SCRA 775 [1966]). The law does not prescribe
really earnest any particular
form of written notice, nor any distinctive
money which, under Art. 1482, ―shall be considered as part of method for
notifying the redemptioner (Etcuban v. CA, 148
the price and as proof of the perfection of the contract.‖ SCRA 507
(Topacio v. CA, 211 SCRA 291 [1992]; [1987]). So long as the redemptioner was
Villongco Realty v. informed in writing,
he has no cause to complain (Distrito v. CA, 197
SCRA 606, 609
Bormaheco, 65 SCRA 352 [1975]).
[1991]). In fact, in Distrito, a written notice
was held
C. Simeon cannot justify his refusal to proceed unnecessary where the co-owner had actual
with the sale by knowledge of the
the fact that the deal is financially sale, having acted as middleman and being
disadvantageous to him. present when the
Having made a bad bargain is not a legal ground
for pulling vendor signed the deed of sale.
out a biding contract of sale, in the absence of
some actionable
wrong by the other party (Vales Right of First Refusal; Lessee; Effect (1996)
v. Villa, 35 Phil 769 [1916]), and no such wrong Ubaldo is the owner of a building which has
has been been leased by
Remigio for the past 20 years. Ubaldo has
committed by Bert. repeatedly assured
Remigio that if he should decide to sell the
building, he will
give Remigio the right of first refusal. On June
Redemption; Legal; Formalities (2001) 30, 1994,
Betty and Lydia were co-owners of a parcel of Ubaldo informed Remigio that he was willing
land. Last to sell the
January 31, 2001, when she paid her real estate building for P5 Million. The following day,
tax, Betty Remigio sent a
discovered that Lydia had sold her share to letter to Ubaldo offering to buy the building at
Emma on P4.5 Million.
November 10, 2000. The following day, Betty Ubaldo did not reply. One week later, Remigio
offered to received a
redeem her share from Emma, but the latter letter from Santos informing him that the
replied that building has been
Betty's right to redeem has already sold to him by Ubaldo for P5 Million, and that
prescribed. Is Emma he will not
correct or not? Why? (5%) renew Remigio's lease when it expires.
Remigio filed an action
against Ubaldo and Santos for cancellation of
SUGGESTED ANSWER: the sale, and to
Emma, the buyer, is not correct. Betty can still compel Ubaldo to execute a deed of absolute
enforce her sale in his favor,
right of legal redemption as a co-owner. Article based on his right of first refusal. a) Will the
1623 of the action prosper?
Civil Code gives a co-owner 30 days from Explain. b) If Ubaldo had given Remigio an
written notice of option to
the sale by the vendor to exercise his right purchase the
of legal
redemption. In the present problem, the 30-day
period for the
exercise by Betty of her right of redemption had building instead of a right of first refusal,
not even will your
begun to run because no notice in writing of the answer be the same? Explain.
sale appears
to have been given to her by Lydia.
SUGGESTED ANSWER:
Redemption; Legal; Formalities (2002) No, the action to compel Ubaldo to execute
Adela and Beth are co -owners of a parcel of the deed of absolute sale will not prosper.
land. Beth sold her undivided share of the According to Ang Yu v. Court of Appeals (238
property to Xandro, who promptly notified SCRA 602), the right of first refusal is not
Adela of the sale and furnished the latter a based on contract but is predicated on the
copy of the deed of absolute sale. When provisions of human relations and, therefore,
Xandro presented the deed for registration, its violation is predicated on quasi-delict.
the register of deeds also notified Adela of the Secondly, the right of first refusal implies that
sale, enclosing a copy of the deed with the the offer of the person in whose favor that
notice. However, Adela ignored the notices. A right was given must conform with the same
year later, Xandro filed a petition for the terms and conditions as those given to the
partition of the property. Upon receipt of offeree. In this case, however, Remigio was
summons, Adela immediately tendered the offering only P4.5 Million instead of P5
requisite amount for the redemption. Xandro Million.
ALTERNATIVE ANSWER:
contends that Adela lost her right of
No, the action will not prosper. The lessee's
redemption after the expiration of 30 days
right of first refusal does not go so far as to
from her receipt of the notice of the sale given
give him the power to dictate on the lessor
by him.
the price at which the latter should sell
CIVIL LAW Answers to the BAR as Arranged by Topics (Year 1990-2006)
his property. Upon the facts given, the lessor SUGGESTED ANSWER:
had sufficiently complied with his commitment A can exercise his right of repurchase within
to give the lessee a right of first refusal when four (4) years from the date of the contract
he offered to sell the property to the lessee for (Art. 1606, Civil Code).
P5 Million, which was the same price he got in
SUGGESTED ANSWER:
selling it to Santos. He certainly had the right
2} I would advise B to file an action for
to treat the lessee's counter-offer of a lesser
consolidation of title and obtain a judicial
amount as a rejection of his offer to sell at P5
order of consolidation which must be
Million. Thus, he was free to find another
recorded in the Registry of Property (Art.
buyer upon receipt of such unacceptable
1607. Civil Code).
counter-offer (Art. 1319. NCC).
Code). Hence the only action that will prosper
according to the Supreme Court is an "action
SUGGESTED ANSWER:
for damages in a proper forum for the
Yes, the answer will be the same. The action purpose."
will not prosper because an option must be
supported by a consideration separate and Right of Repurchase (1993)
distinct from the purchase price. In this case On January 2, 1980, A and B entered into a
there is no separate consideration. Therefore, contract whereby A sold to B a parcel of land for
the option may be withdrawn by Ubaldo at any and in consideration of
time. (Art. 1324, NCC) P10.000.00. A reserving to himself the right to
repurchase the same. Because they were
friends, no period was agreed upon for the
Right of First Refusal; Lessee; Effect (1998)
repurchase of the property. 1) Until when
In a 20-year lease contract over a building, the must A exercise his right of repurchase? 2) If
lessee is expressly granted a right of first A fails to redeem the property within the
refusal should the lessor decide to sell both allowable period, what would you advise B to
the land and building. However, the lessor do for his better protection?
sold the property to a third person who knew
about the lease and in fact agreed to respect
it. Consequently, the lessee brings an action
against both the lessor-seller and the buyer
to rescind the sale and (b) to compel specific
performance of his right of first refusal in the
sense that the lessor should be ordered to
execute a deed of absolute sale in favor of the
lessee at the same price. The defendants
contend that the plaintiff can neither seek
rescission of the sale nor compel specific
performance of a "mere" right of first refusal.
Decide the case. [5%]
SUGGESTED ANSWER:
The action filed by the lessee, for both
rescission of the offending sale and specific
performance of the right of first refusal which
was violated, should prosper. The ruling in
Equatorial Realty Development, Inc. vs. Mayfair
Theater, Inc. (264 SCRA 483), a case with
similar facts, sustains both rights of action
because the buyer in the subsequent sale knew
the existence of right of first refusal, hence in
bad faith.
ANOTHER ANSWER:
The action to rescind the sale and to compel
the right to first refusal will not prosper. (Ang
Yu Asuncion vs. CA, 238 SCRA 602). The Court
ruled in a unanimous en banc decision that the
right of first refusal is not founded upon
contract but on a quasi-delictual relationship
covered by the principles of human relations
and unjust enrichment (Art. 19, et seq. Civil
Extinguishment; Total Distruction;
Transfer of Ownership; Non-Payment of the Price (1991) Leased Property (1993) A is the owner of a lot
Pablo sold his car to Alfonso who issued a on which he constructed a building in the total cost of
postdated check in full payment therefor. P10,000,000.00. Of that amount B
Before the maturity of the check, Alfonso sold
the car to Gregorio who later sold it to Gabriel.
When presented for payment, the check issued
by Alfonso was dishonored by the drawee bank
for the reason that he, Alfonso, had already
closed his account even before he issued his
check. Pablo sued to recover the car from
Gabriel alleging that he (Pablo) had been
unlawfully deprived of it by reason of Alfonso's
deception. Will the suit prosper?
SUGGESTED ANSWER:
No. The suit will not prosper because Pablo
was not unlawfully deprived of the car
although he was unlawfully deprived of the
price. The perfection of the sale and the
delivery of the car was enough to allow
Alfonso to have a right of ownership over the
car, which can be lawfully transferred to
Gregorio. Art. 559 applies only to a person
who is in possession in good faith of the
property, and not to the owner thereof.
Alfonso, in the problem, was the owner, and,
hence, Gabriel acquired the title to the car.

Non-payment of the price in a contract of sale


does not render ineffective the obligation to
deliver. The obligation to deliver a thing is
different from the obligation to pay its price.
EDCA Publishing Co. v. Santos (1990)

Transfer of Ownership; Risk of Loss (1990)


D sold a second-hand car to E for P150,000.00
The agreement between D and E was that half
of the purchase price, or P75,000.00, shall be
paid upon delivery of the car to E and the
balance of P75,000.00 shall be paid in five
equal monthly installments of P15,000.00 each.
The car was delivered to E, and E paid the
amount of P75.000.00 to D. Less than one
month thereafter, the car was stolen from E's
garage with no fault on E's part and was never
recovered. Is E legally bound to pay the said
unpaid balance of P75.000.00? Explain your
answer.
SUGGESTED ANSWER:
Yes, E is legally bound to pay the balance of
P75,000.00. The ownership of the car sold was
acquired by E from the moment it was
delivered to him. Having acquired ownership,
E bears the risk of the loss of the thing under
the doctrine of res perit domino. [Articles
1496. 1497, Civil Code).

LEASE
CIVIL LAW Answers to the BAR as Arranged by Topics (Year 1990-2006)
contributed P5,000,000.00 provided that phenomenon are still unpredictable despite
the building as a the advances in
whole would be leased to him (B) for a period of
ten years science, the phenomenon is considered unforeseen.
from January 1. 1985 to December 31, 1995 at a
rental of
P100,000.00 a year. To such condition, A
agreed. On Leasee & Lessor; Rights and Obligations (1990)
December 20, 1990, the building was totally A vacant lot several blocks from the center of
burned. Soon the town was
thereafter, A's workers cleared the debris leased by its owner to a young businessman B
and started for a term of
construction of a new building. B then served fifteen (15) years renewal upon agreement of
notice upon A the parties.
that he would occupy the building being After taking possession of the lot, the lessee
constructed upon built thereon a
completion, for the unexpired portion of the building of mixed materials and a store. As the
lease term, years passed,
explaining that he had spent partly for the he expanded his business, earning more
construction of the profits. By the tenth
building that was burned. A rejected B's (10th) year of his possession, he was able to
demand. Did A has a build a three
right in rejecting B's (3)-story building worth at least P300,000.00.
demand? Before the end
of the term of the lease, B negotiated with the
SUGGESTED ANSWER: landowner for
Yes. A was correct in rejecting the demand of B. its renewal, but despite their attempts to do so,
As a result they could not
of the total destruction of the building by agree on the new conditions for the renewal.
fortuitous event, Upon the
the lease was extinguished. (Art. 1655, Civil expiration of the term of the lease, the
Code.) landowner asked B to
vacate the premises and remove his building
and other
improvements. B refused unless he was
Implied New Lease (1999) reimbursed for
Under what circumstances would an implied necessary and useful expenses. B claimed that
new lease or a he was a
tacita reconduccion possessor and builder in good faith, with right
arise? (2%) of retention.
This issue is now before the court for
SUGGESTED ANSWER: resolution in a pending
An implied new lease or tacita reconduccion arises if
at the litigation. a) What are the rights of B? b) What
end of the contract the lessee should continue are the rights
enjoying the
thing leased for 15 days with the acquiescence of the landowner?
of the lessor,
and unless a notice contrary by either
to the partieshas
SUGGESTED ANSWER:
previously been given (Art. 1670). In short, in a) B has the right to remove the building and
order that other
there may be tacita reconduccion there must be improvements unless the landowner decides to
expiration of retain the
the contract; there must be continuation of building at the time of the termination of the
possession for 15 lease and pay
days or more; and there must be no prior the lessee one-half of the value of the
demand to vacate. improvements at that
time. The lessee may remove the building even
though the
Lease of Rural Lands principal thing may suffer damage but B should
(2000) not cause any
more impairment upon the property leased
than is necessary.
In 1995, Mark leased the rice land of Narding in The claim of B that he was a possessor and
Nueva Ecija builder in good
for an annual rental of P1,000.00 per hectare. In faith with the right of retention is not tenable.
1998, due to B is not a
the El Nino phenomenon, the rice harvest fell to builder in good faith because as lessee he does
only 40% not claim
of the average harvest for the previous years.
Mark asked ownership over the property leased.
Narding for a reduction of the rental to P500.00
per hectare SUGGESTED ANSWER:
for that year but the latter refused. Is Mark b) The landowner/lessor may refuse to
legally entitled to reimburse 1/2 of the
value of the improvements and require the
such reduction? (2%) lessee to remove
SUGGESTED ANSWER: the improvements. [Article 1678, Civil Code),
No, Mark is not entitled to a reduction. Under
Article 1680 of
the Civil Code, the lessee of a rural land is
entitled to a Leasee; Death Thereof; Effects (1997)
reduction of the rent only in case of loss of more Stating briefly the thesis to support your
than 1/2 of answer to each of
the fruits through extraordinary and the following cases, will the death - a) of the
unforeseen fortuitous lessee extinguish
events. While the drought brought about by the
"El Nino" the lease agreement?
phenomenon may be classified as
extraordinary, it is not SUGGESTED ANSWER:
considered as No. The death of the lessee will not extinguish
unforeseen. the lease
agreement, since lease is not personal in
ALTERNATIVE ANSWER: character and the
right is transmissible to the heirs. (Heirs of
Yes, Mark is entitled to a reduction of the rent. His loss was more Dimaculangan vs.
than 1/2 of the fruits and the loss was due to an extraordinary and
unforeseen fortuitous event. The "El Nino" phenomenon is IAC, 170 SCRA 393).
extraordinary because it is uncommon; it does not occur with
regularity. And neither could the parties have foreseen its Option to Buy; Expired (2001)
On January 1, 1980, Nestor leased the
occurrence. The event should be foreseeable by the parties so that fishpond of Mario for
a period of three years at a monthly rental of
the lessee can change the time for his planting, or refrain from P1,000.00, with
planting, or take steps to avoid the loss. To be foreseeable, the time an option to purchase the same during the
and the place of the occurrence, as well as the magnitude of the
period of the lease
for the price of P500,000.00. After the
adverse effects of the fortuitous event must be capable of being expiration of the
three-year period, Mario allowed Nestor to
predicted. Since the exact place, the exact time, and the exact remain in the
leased premises at the same rental rate. On
magnitude of the adverse effects of the "El Nino" June 15, 1983,
Nestor tendered the amount of P500,000.00 to
Mario and
demanded that the latter execute a deed of
absolute sale of
the fishpond in his favor. Mario refused, on the
ground that
Nestor no longer had an option to buy the
fishpond.
CIVIL LAW Answers to the BAR as Arranged by Topics (Year 1990-2006)
Nestor filed an action for specific performance. sublessee can invoke no right superior to that
Will the of his
action prosper or not? Why? (5%) sublessor, the moment the sublessor is duly ousted from the
SUGGESTED ANSWER: premises, the sublessee has no leg to stand on. The sublessee's
No, the action will not prosper. The implied
renewal of the right, if any, is to demand reparation for damages from his
lease on a month-to-month basis did not have sublessor, should the latter be at fault.
the effect of
extending the life of the option to purchase
which expired at (Heirs ofSevilla v. Court of Appeals G.R. No.
the end of the original lease period. The lessor 49823, February
is correct in
refusing to sell on the ground that the option 26, 1992).
had expired.

Sublease; Delay in Payment of Rentals (1994)


In January 1993, Four-Gives Corporation
Sublease vs. Assignment of Lease; Rescission of Contract leased the entire
twelve floors of the GQS Towers Complex, for
(2005) a period of
Under a written contract dated December 1, ten years at a monthly rental of P3,000,000.00.
1989, Victor There is a
leased his land to Joel for a period of five (5) provision in the contract that the monthly
years at a rentals should be
monthly rental of Pl,000.00, to be increased to paid within the first five days of the month. For
Pl,200.00 and the month of
Pl,500.00 on the third and fifth year, March, May, June, October and December
respectively. On January 1993, the rentals
1, 1991, Joel subleased the land to Conrad for a were not paid on time with some rentals being
period of delayed up to
ten days. The delay was due to the heavy
two (2) years at a monthly rental of Pl,500.00. paper work
On December 31, 1992, Joel assigned the involved in processing the checks.
lease to his
Four-Gives Corporation also subleased five of
compadre, Ernie, who acted on the belief that
Joel was the the twelve
rightful owner and possessor of the said lot. Joel
floors to wholly-owned subsidiaries. The lease
has been contract
faithfully paying the stipulated rentals to Victor.
expressly prohibits the assignment of the lease
When Victor contract or
learned on May 18, 1992 about the sublease and
any portion thereof. The rental value of the
assignment, building has
he sued Joel, Conrad and Ernie for rescission of
increased by 50% since its lease to Four-Gives
the contract Corporation.
1) Can the building owner eject Four-Gives
of lease and for damages. Corporation on
Will the action prosper? If so, against whom? grounds of the repeated delays in the
payment of the rent? 2}
Can the building owner ask for the cancellation
Explain. (2%) of the
contract for violation of the provision against
SUGGESTED ANSWER: assignment?
Yes, the action of for rescission of the contract of
lease and for SUGGESTED ANSWERS:
damages will prosper. Under Article 1659 of the Civil 1) a) The "repeated delays" in the payment of
Code, "if the rentals would,
lessor or the lessee should not comply with the at best, be a slight or casual breach which does
obligations set forth not furnish a
in Articles 1654 and 1657, the aggrieved party may ground for ejectment especially because the
ask for rescission delays were only
of the contract and indemnification for damages, or due to heavy paper work. Note that there was
only the latter, not even a
allowing the contract to remain in force." Article 1649 demand for payment obviously because the
of the same delay lasted for
Code provides that "the lessee cannot assign the lease only a few days (10 days being the longest), at
without the the end of
consent of the lessor, unless there is a stipulation to which time payments were presumably
the contrary." made and were
Consent is necessary because assignment would accepted. There was, therefore, no default.
cause novation by Note also that
there was no demand made upon the lessee to
the substitution of one of the parties. vacate the
premises for non-payment of the monthly
rent. There is,
(Bangayan v. Court of Appeals, G.R. No. 123581, therefore, no cause of action for ejectment
August 29, arising from the
1997) However, the rule is different in the case
of subleasing. "repeated delays".
When there is no express prohibition in the
Contract of Lease,
the lessee may sublet the thing leased. (Art. b) The building owner cannot eject Four-Gives
1650, Civil Code) Corporation
on the ground of repeated delays in the
payment of rentals.
The delay in the payment of the rentals is
minimal and cannot
In the given case, when Joel assigned the lease be made the basis of an ejectment suit. The
to Ernie, the delay was due to
same was done without the consent of Victor. the heavy paperwork involved in processing
The assignment the checks. It
is void. However, there is no indication that in would be otherwise if the lease contract stated
the written that in the
contract of lease between Victor and Joel, that payment of rentals within the first five days of
subleasing the the month, time
premises is prohibited. Hence, the sublease is of the essence or that the lessee will be in
of Joel with delay if he falls to
Conrad is valid. In view of the foregoing, Victor pay within the agreed period without need of
can file the demand. In this
case of rescission and damages only against Joel case he can judicially eject the tenant on the
and Ernie but ground of lack of
payment of the price stipulated after a
he cannot include Conrad. demand to vacate,
b) In case of rescission, discuss the (Article 1673(2), New Civil Code),
rights and
obligations of the parties. (2%)
c) No. Resolution of a contract will not be
SUGGESTED ANSWER: permitted for a
slight or casual breach, but only for such
Rescission of the lease necessarily requires the return of the thing substantial and
to the lessor. Hence, the judgment granting rescission of the
fundamental breach as would defeat the very
contract should also order the lessee to vacate and return the object of the
parties in making the agreement.(Zepeda v. CA,
leased premises to the lessor. However, since the 216 SCRA
293]. The delay of ten (10)) days is not such a substantial
CIVIL LAW Answers to the BAR as Arranged by Topics (Year 1990-2006)
and fundamental breach to warrant the A, and that he has not been remiss in the
resolution of the payment of rent.
contract of lease specially so when the delay
was due to the Will the action prosper? (3%)
heavy paperwork in processing the checks. SUGGESTED ANSWER:
Yes, the action will prosper. Under Article 1651
of the Civil
Code, the sublessee is bound to the lessor for
SUGGESTED ANSWER: all acts which
2) a) No. Sublease is different from assignment refer to the use and preservation of the thing
of lease. leased in the
Sublease, not being prohibited by the contract manner stipulated between the lessor
of lease is and the lessee.
therefore allowed and cannot be invoked as a
ground to
cancel the lease, Sublease; Validity; Assignment of Sublease (1990)
A leased a parcel of land to B for a period of
two years. The
b) No, the lessor cannot have the lease lease contract did not contain any express
cancelled for alleged prohibition against
violation of the provision against assignment. the assignment of the leasehold or the
The lessee did subleasing of the
not assign the lease, or any portion thereof, to leased premises. During the third year of
the subsidiaries. the lease, B
It merely subleased some floors to its subleased the land to C. In turn, C, without A's
subsidiaries. Since the consent,
problem does not state that the contract of lease assigned the sublease to D. A then filed an
contains a action for the
prohibition against sublease, the sublease is rescission of the contract of lease on the
lawful, the rule ground that B has
being that in the absence of an express violated the terms and conditions of the lease
prohibition a lessee agreement. If
may sublet the thing leased, in whole or in part, you were the judge, how would you decide
without the case,
prejudice to his/its responsibility to the lessor particularly with respect to the validity
for the of:
(a) B's sublease to C?
performance of the contract. and
(b) C's assignment of the sublease to D?
Sublease; Sublessee; Liability (1999) SUGGESTED ANSWER:
May a lessee sublease the property leased (a) B's sublease to C is valid. Although the
without the consent original period
of the lessor, and what are the respective of two years for the lease contract has
liabilities of the expired, the lease
lessee and sub-lessee to the lessor in case of continued with the acquiescence of the lessor
such sublease? during the third
year. Hence, there has been an implied renewal
(3%) of the contract
of lease. Under Art. 1650 of the Civil Code, the
SUGGESTED ANSWER: lessee may
Yes, provided that there is no express sublet the thing leased, in whole or in part,
prohibition against when the contract
subleasing. Under the law, when in the contract of lease does not contain any express [Article
of lease of prohibition. s
things there is no express prohibition, the lessee 1650, 1670 Civil A's action for rescission
may sublet Code). should
the thing leased without prejudice to his not prosper on this
responsibility for the ground.
performance of the contract toward the lessor.
[Art, 1650) In
case there is a sublease of the premises being
leased, the SUGGESTED ANSWER:
sublessee is bound to the lessor for all the acts (b) C's assignment of the sublease to D is not
which refer to valid. Under
the use and preservation of the thing leased in Art. 1649, of the Civil Code, the lessee cannot
the manner assign the lease
stipulated between the lessor and the lessee. without the consent of the lessor, unless there
(Art. 1651} The is a stipulation
sublessee is subsidiarily liable to the lessor for to the contrary. There is no such stipulation in
any rent due the contract.
from the lessee. However, the sublessee shall If the law prohibits assignment of the
not be lease without the
responsible beyond the amount of the rent due consent of the lessor, all the more would the
from him. assignment of a
(Art. 1652) As to the lessee, the latter shall still sublease be prohibited without such
be responsible consent. This is a
to the lessor for the rents; bring to the violation of the contract and is a valid ground
knowledge of the lessor for rescission
every usurpation or untoward act which any
third person may by A.
have committed or may be openly preparing to
carry out upon
the thing leased; advise the owner the need for
all repairs; to
return the thing leased upon the termination of
the lease just COMMON CARRIERS
as he received it, save what has been lost or
impaired by the
lapse of time or by ordinary wear and tear or
from an Extraordinary Diligence (2000)
inevitable cause; responsible for the Despite a warning from the police that an
deterioration or loss of attempt to hijack a
the thing leased, unless he proves that it took PAL plane will be made in the following week,
place without his the airline did
not take extra precautions, such as frisking of
fault. passengers, for
fear of being accused of violating human
rights. Two days
later, an armed hijacker did attempt to hijack a
PAL flight to
Cebu. Although he was subdued by the other
passengers, he
managed to fire a shot which hit and killed a
female passenger.
The victim's parents sued the airline for breach
Sublease; Sublessee; Liability (2000) of contract,
and the airline raised the defense of force
A leased his house to B with a condition that the leased premises majeure. Is the
airline liable or not?
shall be used for residential purposes only. B subleased the (2%)
house to C who used it as a warehouse for fabrics. Upon learning SUGGESTED ANSWER:
The airline is liable. In case of death of a passenger,
this, A demanded that C stop using the house as a warehouse, common
carriers are presumed to have been at fault or
but C ignored the demand, A then filed an action for ejectment to have acted
negligently, unless prove that observ
against C, who raised the defense that there is no privity of they they ed
extraordinary diligence (Article 1756,
contract between him and Civil Code). The
CIVIL LAW Answers to the BAR as Arranged by Topics (Year 1990-2006)
failure of the airline to take extra precautions negate sale because they indicate that
despite a ownership over the
police warning that an attempt to hijack the
plane would be units was never intended to transfer to the distributor.
made, was negligence on the part of the
airline. Being
negligent, it is liable for the death of the
passenger. The Agency; coupled with an interest (2001)
defense of force majeure is not tenable since the Richard sold a large parcel of land in Cebu to
shooting Leo for P100
incident would not have happened had the million payable in annual installments over a
airline taken steps period of ten
that could have prevented the hijacker from years, but title will remain with Richard until
boarding the the purchase
price is fully paid. To enable Leo to pay the
plane. price, Richard
gave him a power-of-attorney authorizing him
ALTERNATIVE ANSWER: to subdivide
Under Article 1763 of the Civil Code, the the land, sell the individual lots, and deliver
common carrier is the proceeds to
not required to observe extraordinary diligence Richard, to be applied to the purchase price.
in preventing Five years later,
injury to its passengers on account of the Richard revoked the power of attorney and
willful acts or took over the
negligence of other passengers or of strangers. sale of the subdivision lots himself. Is the
The common revocation valid or
carrier, in that case, is required to exercise only
the diligence of not? Why? (5%)
a good father of a family; hence, the failure of
the airline to SUGGESTED ANSWER:
take EXTRA precautions in frisking the The revocation is not valid. The power of
passengers and by attorney given to
leaving that matter to the security personnel of the buyer is irrevocable because it is coupled
the airport, with an interest:
does not constitute a breach of that duty so as to the agency is the means of fulfilling the
make the obligation of the
airline liable. Besides, the use of irresistible buyer to pay the price of the land (Article
force by the 1927, CC). In other
hijackers was farce majeure that could not words, a bilateral contract (contract to buy
have been and sell the land)
prevented even by the observance of
extraordinary diligence. is dependent on the agency.
Agency; Guarantee Commission
(2004)
As an agent, AL was given a guarantee
commission, in
AGENCY addition to his regular commission, after he
sold 20 units of
refrigerators to a customer,
Agency (2003) HT Hotel. The customer,
Jo-Ann asked her close friend, Aissa, to buy however, failed to pay for the
some groceries units sold. AL’s principal,
for her in the supermarket. Was there a DRBI, demanded from AL payment for the
nominate contract customer’s
entered into between Jo-Ann and Aissa? In the accountability. AL objected, on the ground
affirmative, that his job was
only to sell and not to collect payment for
what was it? Explain. 5% units bought by
the customer. Is AL’s Can DRBI
SUGGESTED ANSWER: objection valid? collect
Yes, there was a nominate contract. On the from him or not? Reason.
assumption that (5%)
Aissa accepted the request of her close friend
Jo-Ann to but
some groceries for her in the supermarket, what
they entered SUGGESTED ANSWER:
into was a nominate contract of Agency. Article No, AL's objection is not valid and DRBI can
1868 of the collect from
New Civil Code provides that by the contract of AL. Since AL accepted a guarantee
agency a commission, in addition
person binds himself to render some service to his regular commission, he agreed to bear
or to do the risk of
something in representation or on behalf of collection and to pay the principal the
another, with the proceeds of the sale on
the same terms agreed upon with the
consent or authority of the latter. purchaser (Article 1907,
ALTERNATIVE ANSWER: Civil Code)
Yes, they entered into a nominate contract of
lease to service
in the absence of a relation of principal and
agent between Agency; Real Estate Mortgage (2004)
CX executed a special power of attorney
them (Article 1644, New Civil Code). authorizing DY to
secure a loan from any bank and to mortgage
his property
covered by the owner’s In securing a
Agency vs. Sale (2000) certificate of title. loan
A foreign manufacturer of computers and a from MBank, DY did not specify that he was
Philippine acting for CX
distributor entered into a contract whereby in the transaction with said bank. Is CX liable
the distributor for the bank
agreed to order 1,000 units of the loan? Why or why not? Justify your answer.
manufacturer's computers (5%)
every month and to resell them in the
Philippines at the
manufacturer's suggested prices plus 10%. All
unsold units at SUGGESTED ANSWER:
the end of the year shall be bought back by the CX is liable for the bank loan because he
manufacturer authorized the
at the same price they were ordered. The mortgage on his property to secure the loan
manufacturer shall contracted by
hold the distributor free and harmless from any DY. If DY later defaults and fails to pay the
claim for loan, CX is liable
defects in the units. Is the agreement one for to pay. However, his liability is limited to the
sale or agency? extent of the
value of the said property. ALTERNATIVE
(5%) ANSWER: CX
is not personally liable to the bank loan
SUGGESTED ANSWER: because it was
contracted by DY in his personal capacity.
The contract is one of agency, not sale. The notion of sale is Only the property
negated by the following indicia: (1) the price is fixed by the
of CX is liable. Hence, while CX has authorized
manufacturer with the 10% mark-up constituting the commission; (2) the mortgage
on his property to secure the loan of DY, the
the manufacturer reacquires the unsold units at exactly the same bank cannot
sue CX to collect the loan in case DY defaults
price; and (3) warranty for the units was borne by the manufacturer. thereon. The
The foregoing indicia bank can only foreclose the property of CX.
CIVIL LAW Answers to the BAR as Arranged by Topics (Year 1990-2006)
And if the proceeds of the foreclosure are not All those contracts were executed by B while
sufficient to A was
pay the loan in full, the bank cannot run after confined due to illness in the Makati Medical
CX for the Center. Rule on
the validity and binding effect of each of the
deficiency. above contracts
ALTERNATIVE ANSWER: upon A the principal. Explain your answers,
While as a general rule the principal is not
liable for the SUGGESTED ANSWER:
contract entered into by his agent in case the The agency couched in general terms
agent acted in comprised only acts of
his own name without disclosing his principal, administration (Art. 1877, Civil Code). The
such rule does lease contract on
not apply if the contract involves a thing the Manila parcel is not valid, not
belonging to the enforceable and not
principal. In such case, the principal is liable binding upon A. For B to lease the property to
under Article C, for more
1883 of the Civil Code. The contract is deemed than one (1) year, A must provide B with a
made on his special power of
behalf (Sy-juco v. Sy-juco 40 Phil. 634 [1920]). attorney (Art. 1878. Civil Code).
ALTERNATIVE ANSWER: appointed was notoriously incompetent or
CX would not be liable for the bank loan. CX's insolvent.
property would also not be liable on the
mortgage. Since DY did not specify that he General Agency vs. Special Agency (1992)
was acting for CX in the transaction with the A as principal appointed B as his agent
bank, DY in effect acted in his own name. In granting him general and unlimited
the case of Rural Bank of Bombon v. CA, 212 management over A's properties, stating that
SCRA, (1992), the Supreme Court, under the A withholds no power from B and that the
same facts, ruled that "in order to bind the agent may execute such acts as he may
principal by a mortgage on real property consider appropriate.
executed by an agent, it must upon its face
purport to be made, signed and sealed in the Accordingly, B leased A's parcel of land in
name of the principal, otherwise, it will bind Manila to C for four (4) years at P60,000.00
the agent only. It is not enough merely that per year, payable annually in advance.
the agent was in fact authorized to make the
mortgage, if he, has not acted in the name of B leased another parcel of land of A in
the principal. Neither is it ordinarily sufficient Caloocan City to D without a fixed term at
that in the mortgage the agent describes P3,000.00 per month payable monthly.
himself as acting by virtue of a power of
attorney, if in fact the agent has acted in his B sold to E a third parcel of land belonging to
own name and has set his own hand and seal A located in Quezon City for three (3) times
to the mortgage. There is no principle of law the price that was listed in the inventory by A
by which a person can become liable on a real to B.
estate mortgage which she never executed in
person or by attorney in fact".
Appointment of Sub-Agent (1999)
X appoints Y as his agent to sell his products
in Cebu City. Can Y appoint a sub-agent and if
he does, what are the effects of such
appointment? (5%)
SUGGESTED ANSWER:
Yes, the agent may appoint a substitute or
sub-agent if the principal has not prohibited
him from doing so, but he shall be responsible
for the acts of the substitute:
when he was not given the power to appoint
one;
when he was given such power, but without
designating the person, and the person
However, if Jesus was aware of the limitation
The lease of the Caloocan City property to D is of Nestor's power as an agent, and Prime
valid and binding upon A. Since the lease is
Realty Corporation does not
without a fixed term, it is understood to be
from month to month, since the rental is
payable monthly (Art. 1687, Civil Code).

The sale of the Quezon City parcel to E is not


valid and not binding upon A. B needed a
special power of attorney to validly sell the
land (Arts. 1877 and 1878, Civil Code). The
sale of the land at a very good price does not
cure the defect of the contract arising from
lack of authority
Powers of the Agent (1994)
Prime Realty Corporation appointed Nestor
the exclusive agent in the sale of lots of its
newly developed subdivision. Prime Realty
told Nestor that he could not collect or receive
payments from the buyers. Nestor was able to
sell ten lots to Jesus and to collect the down
payments for said lots. He did not turn over
the collections to Prime Realty. Who shall bear
the loss for Nestor's defalcation, Prime Realty
or Jesus?
SUGGESTED ANSWER:
The general rule is that a person dealing with
an agent must inquire into the authority of
that agent. In the present case, if Jesus did
not inquire into that authority, he is liable for
the loss due to Nestor's defalcation unless
Article 1900, Civil Code governs, in which
case the developer corporation bears the loss.

Art. 1900 Civil Code provides: "So far as third


persons are concerned, an act is deemed to
have been performed within the scope of the
agent's authority, if such act is within the
terms of the power of attorney, as written,
even if the agent has in fact exceeded the
limits of his authority according to an
understanding between the principal and the
agent.

However, if Jesus made due inquiry and he was


not informed by the principal Prime Realty of
the limits of Nestor's authority. Prime Realty
shall bear the loss.

Considering that Prime Realty Corporation


only "told" Nestor that he could not receive or
collect payments, it appears that the limitation
does not appear in his written authority or
power of attorney. In this case, insofar as
Jesus, who is a third person is concerned,
Nestor's acts of collecting payments is deemed
to have been performed within the scope
of his authority {Article 1900. Civil Code).
Hence, the principal is liable.
CIVIL LAW Answers to the BAR as Arranged by Topics (Year 1990-2006)
ratify the sale contract, then Jesus shall be allowing the other general partner to bind the
liable (Article corporation
will violate the corporation law principle that
1898. Civil Code). only the board
of directors may bind the corporation.
Termination; Effect of Death of Agent (1997)
Stating briefly the thesis to support your
answer to each of SUGGESTED ANSWER:
the following cases, will the death - (c) of an 3) No, for the same reasons given in the
agent end an Answer to Number
agency? 2 above.
SUGGESTED ANSWER:
Yes. The death of an agent extinguishes the Conveyance of a Partner’s Share Dissolution (1998)
agency, by express provision of par. 3, Art Dielle, Karlo and Una are general partners in a
1919 of the Civil Code. merchandising firm. Having contributed equal
amounts to the capital, they also agree on
equal distribution of whatever net profit is
PARTNERSHIP realized per fiscal period. After two years of
operation, however, Una conveys her whole
Composition of Partnerships; Spouses; Corporations interest in the partnership to Justine, without
(1994) the knowledge and consent of Dielle and Karlo.
1. Is the partnership dissolved?
Can a husband and wife form a limited 12%].What are the rights of Justine, if any,
partnership to engage in real estate business, should she desire to participate in the
with the wife being a limited partner? management of the partnership and in the
Can two corporations organize a general distribution of a net profit of P360.000.00
partnership under the Civil Code of the which was realized after her purchase of
Philippines? 3) Can a corporation and an Una's interest? [3%]
individual form a general partnership? SUGGESTED ANSWER:
No, a conveyance by a partner of his whole
SUGGESTED ANSWER: interest in a partnership does not of itself
a) Yes. The Civil Code prohibits a husband and dissolve the partnership in the absence of an
wife from constituting a universal partnership. agreement. (Art. 1813. Civil Code)
Since a limited partnership is not a universal
partnership, a husband and wife may validly SUGGESTED ANSWER:
form one. b) Yes. While spouses cannot enter Justine cannot interfere or participate in the
into a universal partnership, they can enter management or administration of the
into a limited partnership or be members partnership business or affairs. She may,
thereof (CIR u. Suter, etal. 27 SCRA 152). however, receive the net profits to which Una
would have otherwise been entitled. In this
case, P120.000 (Art. 1813, Civil Code)
SUGGESTED ANSWER: individual provided the following conditions
a) No, A corporation is managed by its are met:
board of The Articles of Incorporation of the
directors. If the corporation were to become a corporation expressly allows the
partner, co-partners would have the power to corporation to enter into partnerships;
make the corporation party to transactions in The Articles of Partnership must provide
an irregular manner since the partners are not that all partners will manage the
agents subject to the control of the Board of partnership, and they shall be jointly and
Directors. But a corporation may enter into a severally liable; and
joint venture with another corporation as long In case of a foreign corporation, it must be
as the nature of the venture is in line with the licensed to do business in the Philippines.
business authorized by its charter. (Tuason &
Co., Inc. v. Bolano, 95 Phil. 106). No. A corporation may not be a general
partner because the principle of mutual
As a general rule a corporation may not form a agency in general partnership
general partnership with another corporation
or an individual because a corporation may not
be bound by persons who are neither directors
nor officers of the corporation.

However, a corporation may form a general


partnership with another corporation or an
Dissolution of Partnership (1995)
Pauline, Patricia and Priscilla formed a
business partnership for the purpose of
engaging in neon advertising for a term of five
(5) years. Pauline subsequently assigned to
Philip her interest in the partnership. When
Patricia and Priscilla learned of the
assignment, they decided to dissolve the
partnership before the expiration of its term
as they had an unproductive business
relationship with Philip in the past. On the
other hand, unaware of the move of Patricia
and Priscilla but sensing their negative
reaction to his acquisition of Pauline's
interest, Philip simultaneously petitioned for
the dissolution of the partnership.
Is the dissolution done by Patricia and
Priscilla without the consent of Pauline or
Philip valid? Explain.
Does Philip have any right to petition for the
dissolution of the partnership before the
expiration of its
specified term? Explain.
SUGGESTED ANSWER:
1, Under Art. 1830 (1) (c) of the NCC, the
dissolution by Patricia and Priscilla is valid
and did not violate the contract of partnership
even though Pauline and Philip did not
consent thereto. The consent of Pauline is not
necessary because she had already assigned
her interest to Philip. The consent of Philip is
not also necessary because the assignment to
him of Pauline's interest did not make him a
partner, under Art, 1813 of the NCC.
ALTERNATIVE ANSWER:
Interpreting Art. 1830 (1) (c) to mean that if
one of the partners had assigned his interest
on the partnership to
CIVIL LAW Answers to the BAR as Arranged by Topics (Year 1990-2006)
another the remaining partners may not dissolve the A should be hired as Secretary. The
decision for the hiring
partnership, the dissolution by Patricia and Priscilla without of A prevails because it is an act
of administration which can
the consent of Pauline or Philip is not valid. be performed by the duly appointed managing
partners, W
and X.
SUGGESTED ANSWER:
No, Philip has no right to petition for dissolution B cannot be hired, because in case of a tie in
because he does not have the standing of a the decision of the managing partners, the
partner (Art. 1813 NCC). deadlock must be decided by the partners
owning the controlling interest. In this case,
the opposition of X and Y prevails because Y
Dissolution of Partnership; Termination (1993)
owns the controlling Interest (Art. 1801, Civil
A, B and C formed a partnership for the
Code).
purpose of contracting with the Government
in the construction of one of its bridges. On Obligations of a Partner; Industrial Partner (2001)
June 30, 1992, after completion of the project, Joe and Rudy formed a partnership to operate
the bridge was turned over by the partners to a car repair shop in Quezon City. Joe provided
the Government. On August 30, 1992, D, a the capital while Rudy contributed his labor
supplier of materials used in the project sued and industry. On one side of their shop, Joe
A for collection of the indebtedness to him. A opened and operated a coffee shop, while on
moved to dismiss the complaint against him the other side, Rudy put up a car accessories
on the ground that it was the ABC partnership store. May they engage in such separate
that is liable for the debt. D replied that ABC businesses? Why? [5%]
partnership was dissolved upon completion of SUGGESTED ANSWER:
the project for which purpose the partnership Joe, the capitalist partner, may engage in the
was formed. Will you dismiss the complaint restaurant business because it is not the
against A If you were the Judge? same kind of business the partnership is
engaged in. On the other hand, Rudy may not
SUGGESTED ANSWER:
engage in any other business unless their
As Judge, I would not dismiss the complaint partnership expressly permits him to do so
against A. because A is still liable as a because as an industrial partner he has to
general partner for his pro rata share of 1/3 devote his full time to the business of the
(Art. 1816, C. C.J. Dissolution of a partnership [Art. 1789, CC).
partnership caused by the termination of the
particular undertaking specified in the
agreement does not extinguish obligations, Commodatum & Mutuum
which must be liquidated during the
"winding up" of the partnership affairs Commodatum (1993)
(Articles 1829 and 1830. par. 1-a, Civil A, upon request, loaned his passenger Jeepney
Code). to B to enable B to bring his sick wife from
Paniqui. Tarlac to the Philippine General
Effect of Death of Partner (1997)
Hospital in Manila for treatment. On the way
Stating briefly the thesis to support your back to Paniqui, after leaving his wife at the
answer to each of the following cases, will the hospital, people stopped the passenger
death - of a partner terminate the Jeepney. B stopped for them and allowed them
partnership? to ride on board, accepting payment from
SUGGESTED ANSWER:
them just as in the case of ordinary passenger
Yes. The death of a partner will terminate the
Jeepneys plying their route. As B was crossing
partnership, by express provision of par. 5,
Bamban, there was an onrush of Lahar from
Art. 1830 of the Civil Code.
Mt Pinatubo, the Jeep that was loaned to him
Obligations of a Partner (1992) was wrecked. 1) What do you call the contract
that was entered into by
W, X, Y and Z organized a general partnership
with W and X as industrial partners and Y and A and B with respect to the passenger
Z as capitalist partners. Y contributed Jeepney that was loaned by A to B to
P50,000.00 and Z contributed P20,000.00 to transport the latter's sick wife to
the common fund. By a unanimous vote of the
Manila? 2) Is B obliged to pay A for the use of
partners, W and X were appointed managing
the passenger
partners, without any specification of their
respective powers and duties.
jeepney? 3) Is B liable to A for the
loss of the Jeepney?
SUGGESTED ANSWER:
A applied for the position of Secretary and B The contract is called "commodatum". [Art.
applied for the position of Accountant of the 1933. Civil Code). COMMODATUM is a
partnership. contract by which one of the parties (bailor)
delivers to another (bailee) something not
The hiring of A was decided upon by W consumable so that the latter may use it for a
and X, but was opposed by Y and Z. certain time and return it.
The hiring of B was decided upon by W No, B is not obliged to pay A for the use of
and Z, but was opposed by X and Y. the passenger Jeepney because commodatum
is essentially gratuitous. (Art. 1933. Civil
Who of the applicants should be hired by
Code]
the partnership? Explain and give your
reasons. Yes, because B devoted the thing to a
SUGGESTED ANSWER: purpose different from that for which it has
been loaned (Art. 1942, par. 2, Civil Code)
CIVIL LAW Answers to the BAR as Arranged by Topics
ALTERNATIVE ANSWER: In the given problem, Pedro left his Adventure
No, because an obligation which consists in van with Tito so that the latter could use it for
the delivery of a determinate thing shall be one year while he was in Riyadh. There was no
extinguished if it should be lost or destroyed mention of a consideration. Thus, the contract
without the fault of the debtor, and before he perfected was commodatum. The amount of
has incurred in delay. (Art. 1262. Civil Code) P15,000.00 was spent by Tito to tune up the
van and to repair its brakes. Such expenses are
Commodatum (2005) extra-ordinary expenses because they are
Before he left for Riyadh to work as a necessary for the preservation of the van Thus,
mechanic, Pedro left his Adventure van with the same should be borne by the bailor, Pedro.
Tito, with the understanding that the latter
could use it for one year for his personal or
family use while Pedro works in Riyadh. He did
not tell Tito that the brakes of the van were
faulty. Tito had the van tuned up and the
brakes repaired. He spent a total amount of
P15,000.00. After using the vehicle for two
weeks, Tito discovered that it consumed too
much fuel. To make up for the expenses, he
leased it to Annabelle.

Two months later, Pedro returned to the


Philippines and asked Tito to return the van.
Unfortunately, while being driven by Tito, the
van was accidentally damaged by a cargo
truck without his fault.
Who shall bear the P15,000.00 spent for
the repair of the van? Explain. (2%)
ALTERNATIVE ANSWER:
Tito must bear the P15,000.00 expenses for the
van. Generally, extraordinary expenses for the
preservation of the thing loaned are paid by
the bailor, he being the owner of the thing
loaned. In this case however, Tito should bear
the expenses because he incurred the expenses
without first informing Pedro about it. Neither
was the repair shown to be urgent. Under
Article 1949 of the Civil Code, bailor generally
bears the extraordinary expenses for the
preservation of the thing and should refund the
said expenses if made by the bailee; Provided,
The bailee brings the same to the attention of
the bailor before incurring them, except only if
the repair is urgent that reply cannot be
awaited.
ALTERNATIVE ANSWER:
The P15,000.00 spent for the repair of the van
should be borne by Pedro. Where the bailor
delivers to the bailee a non-consummable
thing so that the latter may use it for a certain
time and return the identical thing, the
contract perfected is a Contract of
Commodatum. (Art. 1933, Civil Code) The
bailor shall refund the extraordinary expenses
during the contract for the preservation of the
thing loaned provided the bailee brings the
same to the knowledge of the bailor before
incurring the same, except when they are so
urgent that the reply to the notification cannot
be awaited without danger. (Art. 1949 of the
Civil Code)
(Year 1990-2006) bailor to hold bailee liable for the loss of the
Tito must also pay for the ordinary expenses thing loaned.
for the use and preservation of the thing
ALTERNATIVE ANSWER:
loaned. He must pay for the gasoline, oil,
greasing and spraying. He cannot ask for As a rule, Pedro does not have the right to
reimbursement because he has the retrieve the van before the lapse of one year.
obligation to return the identical thing to Article 1946 of the Code provides that "the
the bailor. Under Article 1941 of the Civil bailor cannot demand the return of the thing
Code, the bailee is obliged to pay for the loaned till after the expiration of the period
ordinary expenses for the use and stipulated, or after the accomplishment of the
preservation of the thing loaned. use for which the commodatum has been
constituted. However, if in the meantime, he
Does Pedro have the right to retrieve should have urgent need of the thing, he may
the van even before the lapse of one demand its return or temporary use." In the
year? Explain. (2%) given problem, Pedro allowed Tito to use the
ALTERNATIVE ANSWER: van for one year. Thus, he should be bound by
No, Pedro does not have the right to retrieve the said agreement and he cannot ask for the
the van before the lapse of one year. The return of the car before the expiration of the
parties are mutually bound by the terms of one year period. However, if Pedro has urgent
the contract. Under the Civil Code, there are need of the van, he may demand for its return
only 3 instances when the bailor could or temporary use.
validly ask for the return of the thing loaned
even before the expiration of the period.
These are when: (1) a precarium contract was Who shall bear the expenses for the
entered (Article 1947); (2) if the bailor accidental damage caused by the cargo
urgently needs the thing (Article 1946); and truck, granting that the truck driver and
(3) if the bailee commits acts of ingratitude truck owner are insolvent? Explain. (2%)
(Article 1948). Not one of the situations is
SUGGESTED ANSWER:
present in this case.
Generally, extraordinary expenses arising on
the occasion of the actual use of the thing
The fact that Tito had leased the thing loaned loaned by the bailee, even if incurred without
to Annabelle would not justify the demand for fault of the bailee, shall be shouldered equally
the return of the thing loaned before by the bailor and the bailee. (Art. 1949 of the
expiration of the period. Under Article 1942 Civil Code). However, if Pedro had an urgent
of the Civil Code, leasing of the thing loaned need for the vehicle, Tito would be in delay
to a third person not member of the for failure to immediately return the same,
household of the bailee, will only entitle then Tito would be held liable for the
extraordinary expenses.

b) Who shall bear the costs for the van's


fuel, oil and
other materials while it was with Tito? Commodatum vs. Usufruct
Explain. (2%) (1998)
SUGGESTED ANSWER:
CIVIL LAW Answers to the BAR as Arranged by Topics (Year 1990-2006)
Distinguish usufruct from commodatum and state whether
these may be constituted over consumable
goods. [2%] Mutuum; Interests (2001)
Samuel borrowed P300,000.00 housing loan
SUGGESTED ANSWER: from the bank at
1. USUFRUCT is a right given to a person 18% per annum interest. However, the
(usufructuary) to promissory note
enjoy the property of another with the contained a proviso that the bank "reserves
obligation of the right to
preserving its form and substance. (Art. 562. increase interest within the limits allowed by
Civil Code) law," By virtue
of such proviso, over the objections of Samuel,
the bank
On the other hand, COMMODATUM is a increased the interest rate periodically until it
contract by which reached 48%
one of the parties (bailor) delivers to another per annum. Finally, Samuel filed an action
(bailee) questioning the
something not consumable so that the latter
may use it for a right of the bank to increase the interest rate
up to 48%. The
bank raised the defense that the Central
certain time and return it. Bank of the
Philippines had already suspended the Usury
In usufruct the usufructuary gets the right to Law. Will the
the use and to
the fruits of the same, while in commodatum, action prosper or not? Why? (5%)
the bailee only SUGGESTED ANSWER:
acquires the use of the thing loaned but not its The action will prosper. While it is true
fruits. that the interest
ceilings set by the Usury Law are no longer in
force, it has
Usufruct may be constituted on the whole or a been held that PD No. 1684 and CB Circular
part of the No. 905 merely
fruits of the thing. (Art. 564. Civil Code). It may allow contracting parties to stipulate freely on
even be any adjustment
constituted over consumables like money in the interest rate on a loan or forbearance of
(Alunan v. Veloso, money but do
52 Phil. 545). On the other hand, in not authorize a unilateral increase of the
commodatum, interest rate by one
consumable goods may be subject thereof only
when the party without the other's consent (PNB
purpose of the contract is not the consumption v. CA, 238 SCRA 2O [1994]]). To say otherwise
of the object, will violate the
as when it is merely for exhibition. (Art. 1936, principle of mutuality of contracts under
Civil Code) Article 1308 of the
Civil Code. To be valid, therefore, any change
of interest must
be mutually agreed upon by the parties (Dizon
ANOTHER ANSWER: v, Magsaysay,
1. There are several points of distinction 57 SCRA 25O [1974]). In the present problem,
between usufruct and the debtor not
commodatum. Usufruct is constituted by law, by having given his consent to the increase in
contract, by interest, the
testamentary succession, or by prescription
(Art. 1933. Civil increase is void.
Code). Usufruct creates a real right to the fruits
of another's
property, while commodatum creates only a
purely personal Mutuum; Interests (2002)
right to use another's property, and requires a Carlos sues Dino for (a) collection on a
stipulation to promissory note for a
enable the bailee to "make use" of the fruits loan, with no agreement on interest, on
(Arts. 1939& which Dino
1940, Civil Code). Usufruct maybe onerous defaulted, and (b) damages caused by Dino on
while his (Carlos’)
commodatum is always or essentially gratuitous priceless Michaelangelo painting on which
(Arts. 1933 & Dino is liable on
1935, Civil Code). The contract constituting the promissory note and awards damages to
usufruct is Carlos for the
consensual, while commodatum is a real damaged painting, with interests for both
contract (perfected awards. What rates
only by delivery of the subject matter thereof). of interest may the court impose with respect
However, both to both awards?
involve the enjoyment by a person of the
property of another, Explain. (5%)
differing only as to the extent and scope of such
enjoyment SUGGESTED ANSWER:
[jus fruendi in one and Jus utendi in the other); With respect to the collection of money or
both may have promissory note,
as subject matter either an immovable or a it being a forbearance of money, the legal rate
movable; and, both of interest for
maybe constituted over consumable goods (Arts. having defaulted on the payment of 12% will
574 & 1936, apply. With
Civil Code). A consumable thing may be the respect to the damages to the painting, it is 6%
subject-matter of from the time
an abnormal usufruct but in a normal usufruct, of the final demand up to the time of finality of
the judgment
subject-matter may be used only for exhibition. until judgment credit is fully paid. The court
A considers the
commodatum of a consumable thing may be latter as a forbearance of money. (Eastern
only for the Shipping Lines,
purpose of exhibiting, not consuming it. Inc. v. CA, 234 SCRA 78 [1994]; Art 2210 and
2211, CC)

Mutuum; Interests (2004)


The parties in a contract of loan of money
Mutuum vs. Commodatum (2004) agreed that the
Distinguish briefly but clearly between Mutuum yearly interest rate is 12% and it can be
and increased if there is a
law that would authorize the increase of
commodatum. interest rates.
Suppose OB, the lender, would increase by 5%
SUGGESTED ANSWER: the rate of
In MUTUUM, the object borrowed must be a interest to be paid by TY, the borrower,
consumable without a law
thing the ownership of which is transferred to authorizing such increase, would OB’s
the borrower action be just and
who incurs the obligation to return the same valid? Why? Has TY a remedy against the
consumable to imposition of the
the lender in an equal amount, and of the same
kind and rate increase? Explain. (5%)
quality. In COMMODATUM, the object borrowed
is usually SUGGESTED ANSWER:
a non-consumable thing the ownership of which OB's action is not just and valid. The
is not debtor cannot be
transferred to the borrower who incurs the required to pay the increase in interest there
obligation to being no law
authorizing it, as stipulated in the contract.
return the very thing to the lender. Increasing the
CIVIL LAW Answers to the BAR as Arranged by Topics (Year 1990-2006)
rate in the absence of such law violates the principle of
To whom should a deliver the bag of money?
mutuality of contracts. Decide with
ALTERNATIVE ANSWER: reasons.
Even if there was a law authorizing the
increase in interest SUGGESTED ANSWER:
rate, the stipulation is still void because there B would have no right to claim the money.
is no Article 1990 of
corresponding stipulation to decrease the the Civil Code is not applicable. The law
interest due when refers to another
thing received in substitution of the object
the law reduces the rate of interest. deposited and is
predicated upon something exchanged.
In their hurry, X and Y left in A's bedroom
DEPOSIT one (1) of the bags which they had taken
Compensation; Bank Loan (1997) from the bank.
In order to secure a bank loan, XYZ With X and Y now at large and nowhere to be
Corporation surrendered its deposit found, the bag containing P50.000.00 is now
certificate, with a maturity date of 01 claimed by B, by the Mayor of Manila, and by
September 1997 to the bank. The corporation
the bank.
defaulted on the due repayment of the loan,
prompting the bank to encash the deposit B claims that the depository. A, by force
certificate. XYZ Corporation questioned the majeure had obtained the bag of money in
above action taken by the bank as being a case place of the box of money deposited by B.
of pactum commissorium. The bank disagrees.
What is your opinion? The Mayor of Manila, on the other hand, claims
SUGGESTED ANSWER:
that the bag of money should be deposited with
We submit that there is no pactum the Office of the Mayor as required of the
commissorium here. Deposits of money in finder by the provisions of the Civil Code.
banks and similar institutions are governed by
the provisions on simple loans (Art. 1980. Civil
Code). The relationship between the depositor
and a bank is one of creditor and debtor.
Basically this is a matter of compensation as
all the elements of compensation are present
in this case (BPI vs. CA, 232 SCRA 302).
ADDITIONAL ANSWER:
Where the security for the debt is also money
deposited in a bank, it is not illegal for the
creditor to encash the time deposit
certificates to pay the debtor's overdue
obligation.
(Chu us. CA, et al., G.R 78519, September 26,
1989).

Deposit; Exchange (1992)


X and Y staged a daring bank robbery in
Manila at 10:30 AM in the morning of a
regular business day, and escaped with their
loot of two (2) bags, each bag containing
P50,000,00. During their flight to elude the
police, X and Y entered the nearby locked
house of A, then working in his Quezon City
office. From A's house, X and Y stole a box
containing cash totaling P50,000.00 which
box A had been keeping in deposit for his
friend B.
Yes, he can recover the deficiency. The action
The Mayor of Manila cannot invoke. Article 719 of AB to go after the surety bond cannot be
of the Civil Code which requires the finder to taken to mean a waiver of his right to demand
deposit the thing with the Mayor only when the payment for the whole debt, The amount
previous possessor is unknown. received from the surety is only payment pro
tanto, and an action may be maintained for a
In this case , a must return the bag of money deficiency debt.
to the bank as the previous possessor and
known owner (Arts. 719 and 1990. Civil
Code.) ANTICHRESIS
Antichresis (1995)
SURETY Olivia owns a vast mango plantation which
she can no longer properly manage due to a
Recovery of Deficiency (1997) lingering illness. Since she is indebted to
AB sold to CD a motor vehicle for and in Peter in the amount of P500.000.00 she asks
consideration of P120,000.00 to be paid in Peter to manage the plantation and apply the
twelve monthly equal installments of harvest to the payment of her obligation to
P10,000,00, each installment being due and him, principal and interest, until her
payable on the 15th day of each month indebtedness shall have been fully paid. Peter
starting January 1997. agrees. 1) What kind of contract is entered
into between Olivia
To secure the promissory note, CD (a)
executed a chattel mortgage on the subject and Peter? Explain. 2) What specific
motor vehicle, and (b) furnished a surety bond obligations are imposed by law on Peter
issued by Philam life, CD failed to pay more as a consequence of their contract? 3) Does
than two (2) installments, AB went after the the law require any specific form for the
surety but he was only able to obtain three- validity
fourths (3/4) of the total amount still due and of their contract? Explain 4) May Olivia
owing from CD. AB seeks your advice on how re-acquire the plantation before her
he might, if at all, recover the deficiency. How entire
would you counsel AB? indebtedness shall have been fully paid?
SUGGESTED ANSWER: Explain.
SUGGESTED ANSWER:
The bank resists the claims of B and the Mayor of Manila.
CIVIL LAW Answers to the BAR as Arranged by Topics (Year 1990-2006)
1. A contract of antichresis was entered into payment of the loan. However, the loan was
between not paid on
Olivia and Peter. Under Article 2132 of the New time. A month after 4 years, may the shares
Civil Code, of stock
by a contract of antichresis the creditor pledged be deemed owned by ABC or not?
acquires the right to Reason. (5%)
receive the fruits of an immovable of his debtor,
with the
obligation to apply them to the payment of the
interest, and SUGGESTED ANSWER:
The shares of stock cannot be deemed owned
thereafter to the principal of his credit. by ABC upon
default of MNO. They have to be foreclosed.
Under Article
2088 of the Civil Code, the creditor cannot
SUGGESTED ANSWER: appropriate the
2. Peter must pay taxes and charges upon the things given by way of pledge. And even if the
land and bear parties have
the necessary expenses for preservation and stipulated that ABC becomes the owner of the
repair which he shares in case
MNO defaults on the loan, such stipulation is
may deduct from the fruits. (Art, 2135, NCC) void for being
a pactum commissorium.
SUGGESTED ANSWER: showing that the fall in the value of the
The amount of the principal and interest must pledged property was attributable to the
be specified in writing, otherwise the pledger's fault or fraud. On the contrary, the
antichresis will be void. (Art. 2134, NCC) economic crisis was the culprit. Had the
SUGGESTED ANSWER: pledgee been deceived as to the substance or
No. Art. 2136 specifically provides that the quality of the pledged shares of stock, he
debtor cannot re-acquire the enjoyment of the would have had the right to claim another
immovable without first having totally paid thing in their place or to the immediate
what he owes the creditor. However, it is payment of the obligation. This is not the
potestative on the part of the creditor to do so case here.
in order to exempt him from his obligation
under Art. 2135, NCC, The debtor cannot re- Pledge (2004)
acquire the enjoyment unless Peter compels
ABC loaned to MNO P40,000 for which the
Olivia to enter again the enjoyment of the latter pledged 400 shares of stock in XYZ Inc.
property.
It was agreed that if the pledgor failed to pay
the loan with 10% yearly interest within four
years, the pledgee is authorized to foreclose
PLEDGE on the shares of stock. As required, MNO
delivered possession of the shares to ABC with
Pledge (1994) the understanding that the shares would be
In 1982, Steve borrowed P400.000.00 from returned to MNO upon the
Danny, collateralized by a pledge of shares of
stock of Concepcion Corporation worth
P800,000,00. In 1983, because of the economic
crisis, the value of the shares pledged fell to
only P100,000.00. Can Danny demand that
Steve surrender the other shares worth
P700,000.00?
SUGGESTED ANSWER:
No. Bilateral contracts cannot be changed
unilaterally. A pledge is only a subsidiary
contract, and Steve is still indebted to Danny
for the amount of P400,000.00 despite the fall
in the value of the stocks pledged.

No. Danny's right as pledgee is to sell the


pledged shares at a public sale and keep the
proceeds as collateral for the loan. There is no
Pledge; Mortgage; Antichresis (1996)
In the province, a farmer couple borrowed
money from the local merchant. To guarantee
payment, they left the Torrens Title of their
land with the merchant, for him to hold until
they pay the loan. Is there a - a) contract of
pledge, b) contract of mortgage, c) contract of
antichresis, or d) none of the above? Explain.

SUGGESTED ANSWER:
None of the above. There is no pledge because
only movable property may be pledged (Art.
2094. NCC). If at all, there was a pledge of the
paper or document constituting the Torrens
Title, as a movable by itself, but not of the land
which the title represents.

There is no mortgage because no deed or


contract was executed in the manner required
by law for a mortgage (Arts. 2085 to 2092,
NCC; 2124 to 2131, NCC).

There is no contract of antichresis because


no right to the fruits of the property was
given to the creditor (Art. 2132 NCC).

A contract of simple loan was entered into


with security arrangement agreed upon by the
parties which is not one of those mentioned
above.
ALTERNATIVE ANSWER:
There is a contract of mortgage constituted
over the land. There is no particular form
required for the validity of a mortgage of real
property. It is not covered by the statute of
frauds in Art. 1403, NCC and even assuming
that it is covered, the delivery of the title to the
creditor has taken it out of the coverage
thereof. A contract of mortgage of real property
is consensual and is binding on the parties
despite absence of writing. However, third
parties are not bound because of the absence of
a written instrument evidencing the mortgage
and, therefore the absence of registration. But
this does not affect the validity of the mortgage
between the parties (Art. 2125, NCC), The
creditor may compel the debtor to execute the
mortgage in a public document in order to
allow its registration (Art. 1357.NCC in relation
to Art. 1358. NCC).

QUASI-CONTRACT
CIVIL LAW Answers to the BAR as Arranged by Topics (Year 1990-2006)
Quasi-Contracts; Negotiorium Gestio (1992) house under the principle of negotiorum gestio. He was not liable
In fear of reprisals from lawless elements as the burning of the house is a fortuitous event. Is B liable to A
besieging his barangay, X abandoned his for damages under the foregoing circumstances?
fishpond, fled to Manila and left for Europe.
SUGGESTED ANSWER:
Seeking that the fish in the fishpond were
ready for harvest, Y, who is in the business of No. B is not liable for damages, because he is
managing fishponds on a commission basis, a gestor in negotiorum gestio (Art. 2144,
took possession of the property, harvested the Civil Code) Furthermore, B is not liable to A
fish and sold the entire harvest to Z. because Article 2147 of the Civil Code is not
Thereafter, Y borrowed money from W and applicable.
used the money to buy new supplies of fish fry
B did not undertake risky operations which
and to prepare the fishpond for the next crop.
the owner was not accustomed to embark
a) What is the Juridical relation between X and
upon: a) he has not preferred his own
Y during X's absence? b) Upon the return of X
to the barangay, what are the obligations of Y interest to that of the owner; b) he has not
to X as regards the contract with Z? c) Upon failed to return the property or business after
X's return, what are the obligations of X as demand by the owner; and c) he has not
regards Y's contract with W? d) What legal assumed the management in bad faith.
effects will result if X expressly ratifies Y's
management and what would be the
obligations of X in favor of Y? Explain all your
answers.
they spent for the construction of stores at the ground floor and
the conversion of the second floor into a pension house. While
construction was going on, fire occurred at a nearby house. The
houses at the entire block, including A's were burned. After the
SUGGESTED ANSWER:
EDSA revolution in February 1986, A and his family returned from
The juridical relation is that of the quasi- the United States where they took refuge in 1972. Upon learning
contract of "negotiorum gestio". Y is the of what happened to his house. A sued B for damages, B pleaded
"gestor" or "officious manager" and X is the as a defense that he merely took charge of his
"owner" (Art. 2144, Civil Code).

Y must render an account of his operations


and deliver to X the price he received for the
sale of the harvested fish (Art, 2145, Civil
Code).

X must pay the loan obtained by Y from W


because X must answer for obligations
contracted with third persons in the interest
of the owner (Art. 2150, Civil Code),

Express ratification by X provides the effects


of an express agency and X is liable to pay
the commissions habitually received by the
gestor as manager (Art. 2149, Civil Code).

Quasi-Contracts; Negotiorium Gestio (1993)


In September, 1972, upon declaration of martial rule in the
Philippines. A, together with his wife and children. disappeared
from his residence along A. Mabini Street. Ermita, Manila. B, his
immediate neighbor, noticing that mysterious disappearance of A
and his family, closed the doors and windows of his house to
prevent it from being burglarized. Years passed without B hearing
from A and his family, B continued taking care of A's house, even
causing minor repairs to be done at his house to preserve it. In
1976, when business began to perk up in the area, an
enterprising man. C, approached B and proposed that they build
stores at the ground floor of the house and convert its second
floor into a pension house. B agreed to Cs proposal and together
ALTERNATIVE ANSWER:
He would be liable under Art. 2147 (1) of
the Civil Code, because he used the
property for an operation which the
operator is not accustomed to, and in so
doing, he exposed the house to increased
risk, namely the operation of a pension
house on the second floor and stores on the
first floor

Quasi-Contracts; Negotiorium Gestio (1995)


Armando owns a row of residential
apartments in San Juan, Metro Manila, which
he rents out to tenants. On 1 April 1991 he
left for the United States without appointing
any administrator to manage his apartments
such that uncollected rentals accumulated for
three (3) years. Amparo, a niece of Armando,
concerned with the interest of her uncle, took
it upon herself to administer the property. As a
consequence, she incurred expenses in
collecting the rents and in some instances
even spent for necessary repairs to preserve
the property.

What Juridical relation between Amparo and


Armando, if
any, has resulted from Amparo's unilateral act
of assuming the administration of Armando's
apartments? Explain.
What rights and obligations, if any, does
Amparo have under the circumstances?
Explain.
SUGGESTED ANSWER:
1. Negotiorum gestio existed between Amparo
and Armando, She voluntarily took charge of
the agency or management of the business or
property of her uncle without any power from
her uncle whose property was neglected. She
is called the gestor negotiorum or officious
manager, (Art. 2144, NCC)

It is recommended by the Committee that an


enumeration of any two (2) obligations and
two (2) rights as enumerated in Arts. 2145 to
2152, NCC, would entitle the examinee to full
credit.
Art. 2145. The officious manager shall
perform his duties
with all the diligence of a good father of a family,
and pay the
damages which through his fault or
negligence may be
suffered by the owner of the property or
business under
management.
CIVIL LAW Answers to the BAR as Arranged by Topics (Year 1990-2006)
The courts may, however, increase or moderate the (2) When the contract refers to things
pertaining to the
indemnity according to the circumstances of each case. owner of the business,
imminent and manifest danger to the property
Art. 2146. If the officious manager delegates to or business, the owner is liable as under the
another person all or some of his duties, he first paragraph of the preceding article,
shall be liable for the acts of the delegate, provided:
without prejudice to the direct obligation of The officious manager has acted in good faith,
the latter toward the owner of the business. and
The property or business is intact, ready to be
The responsibility of two or more officious returned to the owner.
managers shall be solidary, unless
management was assumed to save the thing Art. 2152. The officious manager is
or business from imminent danger. personally liable for contracts which he has
entered into with third persons, even though
Art. 2147. The officious manager shall be he acted in the name of the owner, and there
liable for any fortuitous event: shall be no right of action between the owner
If he undertakes risky operations which the and third persons. These provisions shall not
owner was not accustomed to embark apply:
upon; If the owner has expressly or tacitly
If he has preferred his own interest to that of ratified the management, or
the owner;

If he fails to return the property or business


after demand by the owner,
If he assumed the management in bad faith.

Art. 2148. Except when the management


was assumed to save the property or
business from imminent danger, the
officious manager shall be liable for
fortuitous events
If he is manifestly unfit to carry on the
management;
If by his Intervention h e prevented a more
competent person from taking up the
management.

Art. 2149. The ratification of the management


by the owner of the business produces the
effects of an express agency, even if the
business may not have been successful.

Art. 2150, Although the officious management


may not have been expressly ratified, the
owner of the property or business who enjoys
the advantages of the same shall be liable for
obligations incurred in his interest, and shall
reimburse the officious manager for the
necessary and useful expenses and for the
damages which the latter may have suffered in
the performance of his duties.

The same obligation shall be incumbent upon


him when the management had for its purpose
the prevention of an imminent and manifest
loss, although no benefit may have been
derived.

Art. 2151. Even though the owner did not


derive any benefit and there has been no
(NOTE: It is recommended by the Committee responsible for the damages resulting from its
that an enumeration of any two (2) obligations total or partial collapse, if it should be due to
and any two (2) rights as enumerated la Arts. the lack of necessary repairs (Art 2190 Civil
2145 to 2152, NCC would entitle the examinee
Code)
to full credit.)

Quasi-Contracts; Solutio Indebiti (2004) As regards the defense of ―last clear chance,‖ the
DPO went to a store to buy a pack of same is not tenable because according to the
cigarettes worth P225.00 only. He gave the SC in one case (De Roy v CA L-80718, Jan 29,
vendor, RRA, a P500-peso bill. The vendor 1988, 157 S 757) the doctrine of last clear
gave him the pack plus P375.00 change. Was chance is not applicable to instances covered
there a discount, an oversight, or an error in by Art 2190 of the Civil Code.
the amount given? What would be DPO’s duty, if
any, in case of an excess in the amount of
change given by the vendor? How is this
situational relationship between DPO and
RRA denominated? Explain. (5%)
SUGGESTED ANSWER:
There was error in the amount of change given
by RRA. This is a case of solutio indebiti in
that DPO received something that is not due
him. He has the obligation to return the
P100.00; otherwise, he will unjustly enrich
himself at the expense of RRA. (Art. 2154, Civil
Code)
ALTERNATIVE ANSWER:
DPO has the duty to return to RRA the excess
P100 as trustee under Article 1456 of the
Civil Code which provides: If property is
acquired through mistake or fraud, the
person obtaining it is, by force of law,
considered a trustee of an implied trust for
the benefit of the person from whom the
property comes. There is, in this case, an
implied or constructive trust in favor of RRA.

TORTS & DAMAGES


Collapse of Structures; Last Clear Chance (1990)
Mr and Mrs R own a burned-out building, the
firewall of which collapsed and destroyed the
shop occupied by the family of Mr and Mrs S,
which resulted in injuries to said couple and
the death of their daughter. Mr and Mrs S had
been warned by Mr & Mrs R to vacate the
shop in view of its proximity to the weakened
wall but the former failed to do so. Mr & Mrs
S filed against Mr and Mrs R an action for
recovery of damages the former suffered as a
result of the collapse of the firewall. In
defense, Mr and Mrs R rely on the doctrine of
last clear chance alleging that Mr and Mrs S
had the last clear chance to avoid the accident
if only they heeded the former’s warning to vacate
the shop, and therefore Mr and Mrs R’s prior
negligence should be disregarded. If you were the
judge, how would you decide the case? State
your reasons.

SUGGESTED ANSWER:
I would decide in favor of Mr & Mrs S. The
proprietor of a building or structure is
CIVIL LAW Answers to the BAR as Arranged by Topics (Year 1990-2006)
Further, in Phoenix Construction, Inc. v. Availing of that portion of Section 12 of Article
Intermediate II of the
Appellate Court (G.R. L-65295, March 10, 1987.
148 SCRA 353) 1987 Constitution which reads;
the Supreme Court held that the role of the The State x xx shall equally protect the life
common law "last of the mother
clear chance" doctrine in relation to Article and the life of the unborn from conception,
2179 of the Civil "xxx" which
Code is merely to mitigate damages within the he claims confers a civil personality on the
context of unborn from
contributory negligence. the moment of conception.
Boy filed a case for damages against the
Damages (1994) abortionist, praying
On January 5, 1992, Nonoy obtained a loan of therein that the latter be ordered to pay him:
Pl,000,000.00 (a) P30,000.00 as
from his friend Raffy. The promissory note did indemnity for the death of the fetus, (b)
not stipulate P100.000.00 as moral
any payment for Interest. The note was due on damages for the mental anguish and anxiety
January 5, he suffered, (c)
1993 but before this date the two became P50,000.00 as exemplary damages, (d)
political enemies. P20,000.00 as nominal
Nonoy, out of spite, deliberately defaulted in damages, and (e) P25,000.00 as attorney's
paying the note, fees. May actual
thus forcing Raffy to sue him. 1) What actual damages be also recovered? If so, what facts
damages can should be alleged
Raffy recover? 2) Can Raffy ask for moral
damages from and proved?
Nonoy? 3) Can Raffy ask for nominal damages?
4) Can Raffy
ask for temperate damages? 5) Can Raffy ask
for attorney's SUGGESTED ANSWER:
Yes, provided that the pecuniary loss suffered
fees? should be
substantiated and duly proved.
SUGGESTED ANSWER: Raffy may ask for, but would most likely not be
Raffy may recover the amount of the awarded temperate damages, for the reason
promissory note of P1 million, together with that his actual damages may already be
interest at the legal rate from the date of compensated upon proof thereof with the
judicial or extrajudicial demand. In addition, promissory note. TEMPERATE DAMAGES may
however, inasmuch as the debtor is in bad be awarded only when the court finds that
faith, he is liable for all damages which may some pecuniary loss has been suffered but its
be reasonably attributed to the non- amount cannot, from the nature of the case, be
performance of the obligation. (Art. 2201(2). proved with certainty. (Article 2224, Civil
NCC). Code)
Yes, under Article 2220, NCC moral damages
are recoverable in case of breach of contract Yes, under paragraph 2, Article 2208 of the
where the defendant acted fraudulently or in Civil Code, considering that Nonoy's act or
bad faith. omission has compelled Raffy to litigate to
protect his interests. Furthermore. attorneys'
Nominal damages may not be recoverable in fees may be awarded by the court when it is
this case because Raffy may already be just and equitable. (Article 2208(110) Civil
indemnified of his losses with the award of Code).
actual and compensatory damages. NOMINAL
DAMAGES are adjudicated only in order that a Damages arising from Death of Unborn Child (1991)
right of the plaintiff, which has been violated or On her third month of pregnancy,
invaded by the defendant may be vindicated or Rosemarie, married to Boy, for reasons
recognized, and not for the purpose of known only to her, and without informing
indemnifying the plaintiff for any loss suffered Boy, went to the clinic of X, a known
by him. (Article 2231. Civil Code) abortionist, who for a fee, removed and
expelled the fetus from her womb, Boy
learned of the abortion six (6) months later.
Damages arising from Death of Unborn Child (2003)
If a pregnant woman passenger of a bus were
to suffer an abortion following a vehicular
accident due to the gross negligence of the
bus driver, may she and her husband claim
damages from the bus company for the death
of their unborn child? Explain. 5%
SUGGESTED ANSWER:
No, the spouses cannot recover actual damages
in the form of indemnity for the loss of life of the
unborn child. This is because the unborn child is
not yet considered a person and the law allows
indemnity only for loss of life of person. The
mother, however may recover damages for the
bodily injury she suffered from the loss of the
fetus which is considered part of her internal
organ. The parents may also recover damages for
injuries that are inflicted directly upon them, e.g.,
moral damages for mental anguish that attended
the loss of the unborn child. Since there is gross
negligence, exemplary damages can also be
recovered. (Gelus v. CA, 2 SCRA 801
[1961])

Death Indemnity (1994)


Johnny Maton's conviction for homicide was
affirmed by the Court of Appeals and in
addition, although the prosecution had not
appealed at all. The appellate court increased
the indemnity for death from P30,000.00 to
P50,000.00. On his appeal to the Supreme
Court, among the other things Johnny Maton
brought to the high court's attention, was the
increase of indemnity imposed by the Court of
Appeals despite the clear fact that the People
had not appealed from the appellate court's
judgment. Is Johnny Maton correct?
SUGGESTED ANSWER:
In Abejam v. Court of Appeals, the Supreme
Court said that even if the issue of damages
were not raised by the appellant in the Court
of Appeals but the Court of Appeals in its
findings increased the damages, the Supreme
Court will not disturb the findings of the Court
of Appeals.

No, the contention of the accused is not


correct because upon appeal to the Appellate
Court, the court acquired jurisdiction over the
entire case, criminal as well as civil. Since the
conviction of homicide had been appealed,
there
CIVIL LAW Answers to the BAR as Arranged by Topics (Year 1990-2006)
is no finality in the amount of indemnity because A van owned by Orlando and driven by Diego,
the civil while
liability arising from the crime and the judgment negotiating a downhill slope of a city road,
on the crime suddenly gained
speed, obviously beyond the authorized limit in
has not yet become final the area, and
bumped a car in front of it, causing severed
damage to the
c) Yes. Since the civil indemnity is an award in care and serious injuries to its passengers.
the civil action Orlando was not in
arising from the criminal offense, the rule that a the car at the time of the incident. The car
party cannot owner and the
be granted affirmative relief unless he himself injured passengers sued Orlando and Diego for
has appealed damages
should apply. Therefore, it was error for the
Court of Appeals caused by Diego’s negligence. In their defense, Diego claims
to have expanded the indemnity since the that the downhill slope caused the van to gain
judgment on the speed and that,
as he stepped on the brakes to check the
civil liability had become final. acceleration, the
brakes locked, causing the van to go even
faster and
Courts can review matters not eventually to hit the car in front of it. Orlando
d) No. assigned as errors. and Diego
(Hydro Resource vs. CA . 204 SCRA 309). contend that the sudden malfunction of the van’s brake
system is a fortuitous even and that, therefore,
they are
exempt from any liability. Is this contention
Defense; Due Diligence in Selection (2003) tenable? Explain.
As a result of a collision between the taxicab
owned by A (2%)
and another taxicab owned by B, X, a passenger
of the first SUGGESTED ANSWER:
taxicab, was seriously injured. X later filed a No. Mechanical defects of a motor vehicle do
criminal action not constitute
fortuitous event, since the presence of such
against both drivers. defects would
have been readily detected by diligent
maintenance check.
May both taxicab owners raise the defense of The failure to maintain the vehicle in safe
due diligence in running condition
the selection and supervision of their drivers to
be absolved constitutes negligence.
from liability for damages to X? Reason. 5% Liability; Airline Company; Non-Performance of an
SUGGESTED ANSWER: Obligation
It depends. If the civil action is based on a
quasi-delict the (2004)
taxicab owners may raise the defense of DT and MT were prominent members of the
diligence of a good frequent
father of a family in the selection and
supervision of the travelers’ club of FX Airlines. In Hongkong, the couple were
driver; if the action against them is based on assigned seats in Business Class for which they
culpa contractual had bought
or civil liability arising from a crime, they tickets. On checking in, however, they were
cannot raise the told they were
upgraded by computer to First Class for the
defense. flight to Manila
because the Business Section was overbooked.
Filing of Separate Civil Action; Need for Reservation (2003)
As a result of a collision between the taxicab Both refused to transfer despite better seats,
owned by A food, beverage
and another taxicab owned by B, X, a passenger and other services in First Class. They said
of the first they had guests in
taxicab, was seriously injured. X later filed a Business Class they should attend to. They felt
criminal action humiliated,
embarrassed and vexed, however, when the
against both drivers. stewardess
allegedly threatened to offload them if they did
not avail of
Is it necessary for X to reserve his right to the upgrade. Thus they gave in, but during the
institute a civil transfer of
action for damages against both taxicab owners luggage DT suffered pain in his arm and wrist.
before he After arrival in
can file a civil action for damages against them?
Why Manila, they demanded an apology from FX’s management as
well as indemnity payment. When none was
SUGGESTED ANSWER: forthcoming,
It depends. If the separate civil action is to
recover damages they sued the airline for a million pesos in
arising from the criminal act, reservation is damages. Is the
necessary. If the airline liable for actual and moral damages?
civ acti against the taxicab owners is based Why or why not?
il on on culpa
contractual, or on quasi-delict, there is no Explain briefly. (5%)
need for
reservati
on. SUGGESTED ANSWER:
FX Airlines committed breach of contract when
ALTERNATIVE ANSWER: it upgraded
No, such reservation is not necessary. Under DT and MT, over their objections, to First Class
Section 1 of because they
Rule 111 of the 2000 Rules on Criminal had contracted for Business Class passage.
Procedure, what is However,
although there is a breach of contract, DT and
―deemed instituted‖ with the criminal action is only the action MT are
to recover civil liability arising from the crime or entitled to actual damages only for such
ex delicto. pecuniary losses
All the other civil actions under Articles 32, 33, suffered by them as a result of such breach.
34 and 2176 There seems to
be no showing that they incurred such
of the New Civil Code are no longer ―deemed instituted‖, and pecuniary loss. There
may be filed separately and prosecuted is no showing that the pain in DT's arm and
independently even wrist resulted
without any reservation in the criminal action directly from the carrier's acts complained of.
(Section 3, Rule Hence, they
111, Ibid). The failure to make a reservation in are not entitled to actual damages. Moreover,
the criminal DT could have
action is not a waiver of the right to file a avoided the alleged injury by requesting the
separate and airline staff to do
independent civil action based on these articles the luggage transfer as a matter of duty on
of the New their part. There is
Civil Code (Casupanan v. Laroya GR No. 145391, also no basis to award moral damages for such
August 26, breach of
contract because the facts of the problem do
2002). not show bad
faith or fraud on the part of the airline. (Cathay Pacific v.
Fortuitous Event; Mechanical Defects (2002) Vazquez, 399 SCRA 207 [2003]).
However, they
CIVIL LAW Answers to the BAR as Arranged by Topics (Year 1990-2006)
may recover moral damages if the cause of The action may or may not prosper. Moral
action is based damages include
on Article 21 of the Civil Code for the physical suffering, mental anguish, fright,
humiliation and serious anxiety,
embarrassment they felt when the stewardess besmirched reputation, wounded feelings,
threatened to moral shock, social
humiliation, and similar injury. Although
offload them if they did not avail of the upgrade. incapable of
pecuniary computation, moral damages may be
ALTERNATIVE ANSWER: recovered if
If it can be proved that DT's pain in his arm and they are the proximate result of the defendant's
wrist wrongful act
occasioned by the transfer of luggage was or omission. Moral damages predicated
caused by fault or upon a breach of
negligence on the part of the airline's contract of carriage are recoverable only in
stewardess, actual instances where
the carrier is guilty of fraud or bad faith or
damages may be recovered. where the mishap
resulted in the death of a passenger. (Cathay
The airline may be liable for moral damages Pacific Airways,
pursuant to Art. Ltd. v. Court of Appeals, G.R. No. 60501, March
2219 (10) if the cause of action is based on 5, 1993) Where
Article 21 or an there is no showing that the airline acted
act contrary to morals in view of the humiliation fraudulently or in
suffered by bad faith, liability for damages is limited to the
DT and MT when they were separated from natural and
their guests and probable consequences of the breach of the
contract of
were threatened to be offloaded. carriage which the parties had foreseen or
could have
reasona foreseen. In such a case the
bly liability does not
Liability; Airline Company; Non-Performance of an Obligation
include moral and exemplary damages.
(2005)
Dr. and Mrs. Almeda are prominent citizens of In the instant case, if the involuntary
the country upgrading of the
and are frequent travelers abroad. In 1996, they Almedas' seat accommodation was not attended
booked by fraud or
round-trip business class tickets for the Manila- bad faith, the award of moral damages has no
Hong leg to stand on.
Kong-Manila route of the Pinoy Airlines, where
they are
holders of Gold Mabalos Class Frequent Flier
cards. On their
return flight, Pinoy Airlines upgraded their Thus, spouses would not also be entitled to
tickets to first class exemplary
without their consent and, inspite of their damages. It is a requisite in the grant of
protestations to be exemplary damages
allowed to remain in the business class so that that the act of the offender must be
they could be accompanied by bad
with their friends, they were told that the faith or done in wanton, fraudulent or
business class was malevolent manner.
already fully booked, and that they were given (Morris v. Court of Appeals, G.R. No. 127957,
priority in February 21, 2001)
upgrading because they are elite Moreover, to be entitled thereto, the
members/holders of Gold claimant must first
Mabalos Class cards. Since they were establish his right to moral, temperate, or
embarrassed at the compensatory
discussions with the flight attendants, they were damages. (Art. 2234, Civil Code) Since the
forced to take Almedas are not
the flight at the first class section apart from entitled to any of these damages, the award
their friends who for exemplary
were in the business class. Upon their return to damages has no legal basis. Where the awards
Manila, they for moral and
demanded a written apology from Pinoy exemplary damages are eliminated, so must
Airlines. When it the award for
went unheeded, the couple sued Pinoy Airlines attorney's fees be eliminated. (Orosa v. Court of
for breach of Appeals, G.R.
contract claiming moral and exemplary No. 111080, April 5, 2000; Morris v. Court of
damages, as well as Appeals, G.R. No.
attorney's fees. Will the action prosper? Give 127957, February 21, 2001) The most that can
reasons. (5%) be adjudged in
their favor for Pinoy Airlines' breach of contract
is an award
ALTERNATIVE ANSWER:
for nominal damages under Article 2221 of the
Yes, the action will prosper. Article 2201 of the
Civil Code.
Civil Code
(Cathay Pacific Airways v. Sps. Daniel & Maria
entitles the person to recover damages which Luisa Vasquez,
may be
attributed to non-performance of an obligation. G.R. No. 150843, March 14, 2003)
In Alitalia
Airways v. Court of Appeals (G.R. No. 77011, July
24, 1990),
when an airline issues ticket to a passenger However, if spouses Almeda could prove that
confirmed on a there was bad
particular flight, a contract of carriage arises faith on the part of Pinoy Airlines when it
and the passenger breached the
expects that he would fly on that day. When the contract of carriage, it could be liable for
airline moral, exemplary as
deliberately overbooked, it took the risk of
having to deprive well as attorney's fees.
some passengers of their seat in case all of them
would show
up. For the indignity and inconvenience of being
refused the Liability; Employer; Damage caused by Employees (1997)
confirmed seat, said passenger is entitled to a) When would an employer's liability for
moral damages. damage, caused
by an employee in the performance of his
assigned
tasks, be primary and when would it be
subsidiary in
In the given problem, spouses Almeda had a nature? b) Would the defense of due diligence
booked in the
roundtrip business class ticket with Pinoy
Airlines. When selection and
their tickets were upgraded to first class supervision of the employee be available to
without their the
consent, Pinoy Airlines breached the contract. employe
As ruled in r in both instances?
Zulueta v. Pan American (G.R. No. L-28589,
January 8, 1973), SUGGESTED ANSWER::
in case of overbooking, airline is in bad faith. (a) The employer's liability for damage
Therefore, based on culpa
aquiliana under Art, 2176 and 2180 of the
spouses Almeda are entitled to damages. Civil Code is
primary; while that under Art. 103 of the
Revised Penal Code
ALTERNATIVE ANSWER: is subsidiary.
CIVIL LAW Answers to the BAR as Arranged by Topics (Year 1990-2006)
the vehicle at the time of the accident, be held
(b) The defense of diligence in the selection and solidarily
supervision of the employee under Article 2180
of the Civil liable with his driver, John? (5%)
Code is available only to those primarily liable
thereunder, SUGGESTED ANSWER:
but not to those subsidiarily liable under Article Yes. Art may be held solidary liable with John, if
103 of the it was proven
Revised Penal Code (Yumul vs. Juliano, 72 Phil. that the former could have prevented the
94). misfortune with the
use of due diligence. Article 2184 of the Civil
Code states: "In
motor mishaps, the owner is solidary liable
Liability; owner who was in the vehicle (1996) with his driver, if
Marcial, who does not know how to drive, has the former, who was in the vehicle, could have,
always been by the use of
driven by Ben, his driver of ten years whom he due diligence, prevented the misfortune,
had chosen x x x"
carefully and has never figured in a vehicular
mishap. One
day, Marcial was riding at the back seat of his
Mercedes Benz ALTERNATIVE ANSWER:
being driven along EDSA by Ben. Absorbed in 1. It depends. The Supreme Court in Chapman vs,
Underwood
reading a
(27 Phil 374), held: "An owner who sits in his
book, Marcial did not notice that they were
automobile, or
approaching the
other vehicle, and permits his driver to
corner of Quezon Avenue, when the traffic light
continue in a violation
had just
of law by the performance of negligent acts,
turned yellow. Ben suddenly stepped on the gas
after he has had a
to cross the
reasonable opportunity to observe them and to
intersection before the traffic light could turn
direct that the
red. But, too
driver cease therefrom, becomes himself
late. Midway in the intersection, the traffic light
responsible for such
changed, and
acts, x x x On the other hand, if the driver, by a
a Jeepney full of passengers suddenly crossed
sudden act of
the car's path. A
negligen and without owne having a
collision between the two vehicles was
ce, the r reasonable
inevitable. As a result,
opportun to prevent the continuan injure
several jeepney passengers were seriously
ity act or its ce, sa
injured. A suit for
person violates the law, the
damages based on culpa aquiliana was filed
or criminal owner of the
against Marcial
automobile, although present therein at the
and Ben, seeking to hold them jointly and
time the act was
severally liable for
committ is not eith civilly or
such injuries. May Marcial be held liable?
ed responsible, er criminally,
Explain.
therefor. The act complained of must be
continued in the
presence of the owner for such a length of
SUGGESTED ANSWER: time that the
Marcial may not be liable because under Art. owner, by his acquiescence, makes his driver's
2184, NCC, the act his own."
owner who is in the vehicle is not liable with the
driver if by
the exercise of due diligence he could have
prevented the
injury. The law does not require the owner to
supervise the Liability; owner who was in the vehicle (2002)
driver every minute that he was driving. Only Does the presence of the owner inside the
when through vehicle causing
his negligence, the owner has lost an damage to a third party affect his liability for
opportunity to prevent his driver’s
the accident would he be liable (Caedo v. Ytt negligence? Explain
Khe Thai, 26 (2%)
SCRA 410 citing Chapman v. Underwood and
Manlangit v. SUGGESTED ANSWER:
Mauler, 250 SCRA 560). In this case, the fact In motor vehicle mishaps, the owner is made
solidarily liable
that the owner with his driver if he (the owner) was in the
was absorbed in reading a book does not vehicle and could
conclusively show have, by the use of due diligence, misha
that he lost the opportunity to prevent the prevented the p.
accident through (Caedo v. Yu Khe Thai, 26 SCRA 410
his negligence. [1968]).
ALTERNATIVE ANSWER:
Yes, Marcial should be held liable. Art. 2164. Moral Damages & Atty Fees (2002)
NCC makes an owner of a motor vehicle Ortillo contracts Fabricato, Inc. to supply and
solidarily liable with the driver if, being in the install tile materials in a building he is
vehicle at the time of the mishap, he could donating to his province. Ortillo pays 50% of
have prevented it by the exercise of due the contract price as per agreement. It is also
diligence. The traffic conditions along EDSA at agreed that the balance would be payable
any time of day or night are such as to require periodically after every 10% performance until
the observance of utmost care and total completed. After performing about 93% of the
alertness in view of the large number of contract, for which it has been paid an
vehicles running at great speed. Marcial was additional 40% as per agreement, Fabricato,
negligent in that he rendered himself oblivious Inc. did not complete the project due to its
to the traffic hazards by reading a book sudden cessation of operations. Instead,
instead of focusing his attention on the road Fabricato, Inc. demands payment of the last
and supervising the manner in which his car 10% of the contract despite its non-completion
was being driven. Thus he failed to prevent his of the project. Ortillo refuses to pay, invoking
driver from attempting to beat the traffic light the stipulation that payment of the last amount
at the junction of Quezon Avenue and EDSA, 10% shall be upon completion. Fabricato, Inc.
which Marcial, without being a driver himself brings suit for the entire 10%. Plus damages,
could have easily perceived as a reckless Ortillo counters with claims for (a) moral
course of conduct. damages for Fabricato, Inc.’s unfounded suit
which has damaged his reputation as a
Liability; owner who was in the vehicle (1998) philanthropist and respect businessman in his
A Gallant driven by John and owned by Art, and a Corolla driven community, and (b) attorney’s fees.
by its owner, Gina, collided somewhere along Adriatico Street. As A. Does Ortillo have a legal basis for his claim
a result of the accident, Gina had a concussion. Subsequently. for moral damages? (2%)
Gina brought an action for damages against John and Art. There B. How about his claim for attorney’s fees,
is no doubt that the collision is due to John's negligence. Can Art, having hired a lawyer to defend him? (3%)
who was in SUGGESTED ANSWER:
CIVIL LAW Answers to the BAR as Arranged by Topics damages only if X proves reckless negligence of
A. There is no legal basis to Ortillo’s claim the carrier amounting to fraud.
for moral damages. It does not fall under the
coverage of Article 2219 of the New Civil Z can claim moral damages against both
Code. defendants because the rules on damages
arising from death due to a quasi-delict are also
B. Ortillo is entitled to attorney’s fees applicable to death of a passenger caused by
because Fabricato’s complaint is a case of breach of contract by a common carrier (Arts.
malicious prosecution or a clearly 1755. 1756, 1764, 2206 and 2219. Civil Code).
unfounded civil action. (Art. 2208 [4] and
Quasi-Delict (2005)
[11], NCC).
Under the law on quasi-delict, aside from the
Moral Damages; Non-Recovery Thereof (2006) persons who caused injury to persons, who
Under Article 2219 of the Civil Code, moral else are liable under the following
damages may be recovered in the cases circumstances:
specified therein several of which are
enumerated below. Choose the case wherein
you cannot recover moral damages. Explain.
(2.5%) a) A criminal offense resulting in
physical injuries b) Quasi-delicts causing
physical injuries c) Immorality or dishonesty d)
Illegal search e) Malicious prosecution
SUGGESTED ANSWER: Immorality and dishonesty,
per se, are not among those cases enumerated
in Article 2219 which can be the basis of an
action for moral damages. The law specifically
mentions adultery or concubinage, etc. but not
any and every immoral act.

Quasi-Delict (1992)
As the result of a collision between a public
service passenger bus and a cargo truck
owned by D, X sustained physical injuries and
Y died. Both X and Y were passengers of the
bus. Both drivers were at fault, and so X and
Z, the only heir and legitimate child of the
deceased Y, sued the owners of both vehicles.
a) May the owner of the bus raise the defense
of having exercised the diligence of a good
father of a family? b) May D raise the same
defense? c) May X claim moral damages from
both defendants? d) May Z claim moral
damages from both defendants? Give reasons
for all your answers,
SUGGESTED ANSWER:
No. The owner of the bus cannot raise the
defense because the carrier's liability is
based on breach of contract

Yes. D can raise the defense because his


liability is based on a quasi-delict.

Because X suffered physical injuries, X can


claim moral damages against D, but as against
the owner of the bus. X can claim moral
(Year 1990-2006) family to prevent the damage. (Last par., Art.
When a 7-year old boy injures his 2180, Civil Code)
playmate while playing with his
Quasi-Delict; Acts contrary to morals (1996)
father's rifle. Explain. (2%)
SUGGESTED ANSWER: Rosa was leasing an apartment in the city.
The parents of the 7-year old boy who caused Because of the Rent Control Law, her landlord
injury to his playmate are liable under Article could not increase the rental as much as he
219 of the Family Code, in relation to Article wanted to, nor terminate her lease as long as
2180 of the Civil Code since they exercise she was paying her rent. In order to force her
parental authority over the person of the boy. to leave the premises, the landlord stopped
(Tamargo v. making repairs on the apartment, and caused
Court of Appeals, G.R. No. 85044, June 3, the water and electricity services to be
1992; Elcano v. Hill, G.R. No. L-24803, May disconnected. The difficulty of living without
26, 1977) electricity and running water resulted in
Rosa's suffering a nervous breakdown. She
When a domestic helper, while haggling for sued the landlord for actual and moral
a lower price with a fish vendor in the damages. Will the action prosper? Explain.
course of buying foodstuffs for her
employer's family, slaps the fish vendor, SUGGESTED ANSWER:
causing her to fall and sustain injuries.
Explain. (2%)
SUGGESTED ANSWER:
Employer of the domestic helper who
slapped a fish vendor. Under Article 2180,
par. 5 of the Civil Code, "employers shall be
liable for the damages caused by their
employees and household helpers acting
within the scope of their assigned tasks,
even though the former are not engaged in
any business or industry."

A carpenter in a construction
company accidentally hits the right
foot of his co-worker with a hammer.
Explain. (2%)
SUGGESTED ANSWER:
The owner of the construction company.
Article 2180, paragraph 4 states that "the
owners and managers of an establishment
or enterprise are likewise responsible for
damages caused by their employees in the
service of the branches in which the latter
are employed or on the occasion of their
functions."

A 15-year old high school student stabs


his classmate who is his rival for a girl
while they were going out of the
classroom after their last class. Explain.
(2%)
SUGGESTED ANSWER:
The school, teacher and administrator as
they exercise special parental authority.
(Art. 2180, par. 7 in relation to Art. 218 and
Art. 219 of the Family Code)

e) What defense, if any, is available to


them? (2%)
SUGGESTED ANSWER:
The defense that might be available to them
is the observance of a good father of the
CIVIL LAW Answers to the BAR as Arranged by Topics (Year 1990-2006)
Yes, based on quasi-delict under the human for quasi-delict may nonetheless prosper. The
relations Supreme
provisions of the New Civil Code (Articles 19, 20 Court has consistently ruled that the act that
and 21) breaks the
because the act committed by the lessor is contract may also be a tort. There is a
contrary to morals. fiduciary relationship
Moral damages are recoverable under between the bank and the depositor, imposing
Article 2219 utmost
(10) in relation to Article 21. Although the diligence in managing the accounts of the
action is based on depositor. The
quasi-delict and not on contract, actual dishonor of the check adversely affected the
damages may be credit standing
recovered if the lessee is able to prove the of Tony, hence, he is entitled to damages
losses and (Singson v. BPI,
G.R. No. L-24932, June 27, 1968; American
expenses she suffered. Express
International, Inc. v. IAC, G.R. No. 72383,
ALTERNATIVE ANSWERS: November 9, 1988;
a) Yes, based on breach of contract. The Consolidated Bank and Trust v. CA, G.R. No. L-
lessor has the 70766
obligation to undertake repairs to make the
apartment November 9,1998).
habitable and to maintain the lessee in the
peaceful and
adequate enjoyment of the lease for the entire
duration of the Vicarious Liability (1991)
contract (Article 1654. NCC). Since there was Romano was bumped by a minivan owned by
willful breach the Solomon
of contract by the lessor, the lessee is School of Practical Arts (SSPA). The minivan
entitled to moral was driven by
damages under Article 3220, NCC. She is also Peter, a student assistant whose assignment
entitled to was to clean the
actual damages, e. g. loss of income, medical school passageways daily one hour before and
expenses, etc., one hour after
regular classes, in exchange for free tuition.
which she can prove at the trial. Peter was able to
drive the school vehicle after persuading the
regular driver,
b) Yes, based on contract and/or on tort. The Paul, to turn over the wheel to him (Peter).
lessor willfully Romano suffered
breached his obligations under Article 1654. serious physical injuries. The accident
NCC, hence, he happened at night when
is liable for breach of contract. For such breach, only one headlight of the vehicle was
the lessee functioning and Peter
may recover moral damages under Art. 2220 of only had a student driver's permit. As a
the NCC, and consequence, Peter
actual damages that she may have suffered was convicted in the criminal case. Thereafter,
on account Romano sued
thereof. And since the conduct of the lessor was for damages against Peter and SSPA. a) Will
contrary to the action for
morals, he may also be held liable for quasi-
delict. The lessee damages against Peter and SSPA
may recover moral damages under Article
2219 (10) in
relation to Article 21, and all actual damages prosper? b) Will your answer be the same if,
which she may Paul, the regular
have suffered by reason of such conduct under
Articles 9, 20 driver, was impleaded as party defendant
and for allowing
21. Peter to drive the minivan without a
regular driver's
c) Yes, the action should prosper for both actual license. c) Is the exercise of due diligence in
and moral the selection and
damages. In fact, even exemplary damages and
attorney's fees
can be claimed by Rosa, on the authority of supervision of Peter and Paul a material
Magbanua vs. issue to be
IAC (137 SCRA 328), considering that, as given,
the lessor's resolved in this case?
willful and illegal act of disconnecting the water
and electric SUGGESTED ANSWER:
servic resulted Rosa's suffering a nervous A. Yes. It will prosper (Art, 2180) because at the
time he
es in breakdown. drove the vehicle, he was not performing his
Art. NCC Art, 21, NCC authorize assigned tasks as
20 and the award of provided for by Art. 2180. With respect to
SSPA, it is not
damages for such willful and illegal conduct. liable for the acts of Peter because the latter
was not an
employee as held by Supreme Court in Filamer
Quasi-Delict; Mismanagement of Depositor’s Account (2006) Christian
Institute vs. CA. (190 SCRA 485). Peter belongs
to a special
Tony bought a Ford Expedition from a category of students who render service to the
car dealer in school in
Muntinlupa City. As payment, Tony issued a exchange for free tuition fees.
check drawn
against his current account with Premium Bank.
Since he has B. I would maintain the same answer because
a good reputation, the car dealer allowed him to the incident did
immediately not occur while the employee was in the
drive home the vehicle merely on his assurance performance of his
that his check duty as such employee. The incident occurred
is sufficiently funded. When the car at night time,
dealer deposited the and in any case, there was no indication in the
check, it was dishonored on the ground of problem that
"Account Closed."
After an investigation, it was found that an he was performing his duties as a driver.
employee of the
bank misplaced Tony's account ledger. Thus,
the bank C. In the case of Peter, if he were to be
erroneously assumed that his account no longer considered as
exists. Later it employee, the exercise of due diligence in the
turned out that Tony's account has more than selection and
sufficient funds supervision of peter would not be a material
to cover the check. The dealer however, issue since the
immediately filed an conviction of Peter would result in a subsidiary
action for recovery of possession of the vehicle liability where
against Tony the defense would not be available by the
for which he was terribly humiliated and employer.
embarrassed. Does
Tony have a cause of action against Premium In the case of Paul, since the basis of
Bank? Explain. subsidiary liability is the
(5%)
pater familias rule under Art. 2180, the
SUGGESTED ANSWER: defense of selection
and supervision of the employee would be a
Yes, Tony may file an action against Premium Bank for damages valid defense.
under Art. 2176. Even if there exists a contractual relationship
between Tony and Premium Bank, an action ALTERNATIVE ANSWER:
CIVIL LAW Answers to the BAR as Arranged by Topics (Year 1990-2006)
C. In the case of Peter, if he were to be The doctrine of VICARIOUS LIABILITY is that
considered an which
employee, the exercise of due diligence in the renders a person liable for the negligence of
selection and others for whose
supervision of Peter would not be a material acts or omission the law makes him responsible
issue since the on the theory
conviction of Peter would result in a subsidiary
liability where that they are under his control and supervision.
the defense would not be available by the
employer.
Vicarious Liability (2004)
In the case of Paul, since he was in the OJ was employed as professional driver of MM
performance of his Transit bus
work at the time the incident occurred, the owned by Mr. BT. In the course of his work, OJ
school may be hit a
held subsidiarily liable not because of the pedestrian who was seriously injured and later
conviction of Peter, died in the
but because of the negligence of Paul under hospital as a result of the accident. The
Art. 2180. victim’s heirs sued
the driver and the owner of the bus for
damages. Is there a
presumption in this case that Mr. BT, the owner,
had been
neglige If so, is the presumption absolute or
Vicarious Liability (2001) nt? not?
After working overtime up to midnight, Alberto,
an executive Explain. (5%)
of an insurance company drove a company
vehicle to a SUGGESTED ANSWER:
favorite Videoke bar where he had some Yes, there is a presumption of negligence on the
drinks and sang part of the
some songs with friends to "unwind". At 2:00 employer. However, such presumption is
a.m., he drove rebuttable. The
home, but in doing so, he bumped a tricycle, liability of the employer shall cease when they
resulting in the prove that they
death of its driver. May the insurance company observed the diligence of a good father of a
be held liable family to prevent
for the negligent act of Alberto? Why? damage (Article 2180, Civil Code).
When the employee causes damage due to his
SUGGESTED ANSWER: own negligence
The insurance company is not liable because while performing his own duties, there arises
when the the juris tantum
accident occurred, Alberto was not acting within presumption that the employer is negligent,
the assigned rebuttable only by
proof of observance of the diligence of a good
tasks of his employment. father of a
family (Metro Manila Transit v. CA, 223
SCRA 521 [1993];
It is true that under Art. 2180 (par. 5), employers Delsan Transport Lines v, C&tA Construction,
are liable for 412 SCRA 524
damages caused by their employees who were
acting within 2003).
the scope of their assigned tasks. However, the
mere fact that
Alberto was using a service vehicle of the Likewise, if the driver is charged and convicted
employer at the time in a criminal
of the injurious accident does not necessarily case for criminal negligence, BT is subsidiarily
mean that he was liable for the
operating the vehicle within the scope of his
employment. In damages arising from the criminal act.
Castilex Industrial Corp. v. Vasquez Jr (321 Vicarious Liability (2006)
SCRA393 [1999]).
the Supreme Court held that notwithstanding
the fact that the
employee did some overtime work for the Arturo sold his Pajero to Benjamin for P1 Million.
company, the Benjamin
former was, nevertheless, engaged in his took the vehicle but did not register the sale
own affairs or with the Land
carrying out a personal purpose when he went Transportation Office. He allowed his son Carlos,
to a restaurant a minor who
at 2:00 a.m. after coming out from work. The did not have a driver's license, to drive the car to
time of the buy pan de sal
in a bakery. On the way, Carlos driving in a
accident (also reckless manner,
2:00 a. m.) was outside normal working sideswiped Dennis, then riding a bicycle. As a
hours. result, he suffered
serious physical injuries. Dennis filed a criminal
ALTERNATIVE ANSWER: complaint against
The insurance company is liable if Alberto was Carlos for reckless imprudence resulting in
negligent in serious physical
the operation of the car and the car was
assigned to him for injuries.
the benefit of the insurance company, and even 1. Can Dennis file an independent civil action
though he against Carlos
was not within the scope of his assigned and his father Benjamin for damages based on
tasks when the quasi-delict?
accident happened. In decided by the
one case Supreme Explain. (2,5%)
Court, where an executive of a pharmaceutical SUGGESTED ANSWER: Yes, Dennis can file an
independent
company was
civil action against Carlos and his father for
given the use of a company car, and after office
damages based on
hours, the
quasi-delict there being an act or omission
executive made personal use of the car and met
causing damage to
an accident,
another without contractual obligation. Under
the employer was also made liable under Art.
Section 1 of
2180 of the
Rule 111 of the 2000 Rules on Criminal
Civil Code for the injury caused by the negligent
Procedure, what is
operation of
deemed instituted with the criminal action is
the car by the executive, on the ground that the
only the action to
car which
recover civil liability arising from the act or
caused the was to the
omission punished
injury assigned executive by the
by law. An action based on quasi-delict is no
employer for prestige of company. The
longer deemed
the the insurance
company was held liable even though the institut and may be filed separately [Section 3,
employee was not ed Rule 111,
performing within the scope of his assigned Rules of Criminal
tasks when the Procedure].
accident happened [Valenzuela v. CA, 253 SCRA
3O3 (1996)].
Vicarious Liability (2002) Assuming Dennis' action is tenable, can
Explain the concept of vicarious liability in Benjamin raise the defense that he is not liable
quasi-delicts. (1%) because the vehicle is not registered in his
name? Explain. (2.5%)
SUGGESTED ANSWER:
CIVIL LAW Answers to the BAR as Arranged by Topics (Year 1990-2006)
SUGGESTED ANSWER: No, Benjamin cannot
raise the called ―oncomouse‖ in Manila? What will be your advice to
defense that the vehicle is not registered in his
name. His liability, him? (5%)
vicarious in character, is based on Article 2180
because he is the SUGGESTED ANSWER:
father of a minor who caused damage due to (1) The reciprocity principle in private
negligence. While international law may
the suit will prosper against the registered owner, be applied in our jurisdiction. Section 3 of R.A.
it is the actual 8293, the
owner of the private vehicle who is ultimately Intellectual Property Code, provides for
liable (See Duavit v. reciprocity, as follows:
CA, G.R. No. L-29759, May 18, 1989). The "Any person who is a national, or who is
purpose of car domiciled, or has a
registration is to reduce difficulty in identifying the real and effective industrial establishment in a
party liable in country which is
a party to any convention, treaty or agreement
case of accidents relating to
(Villanueva v. Domingo, G.R. No. 144274, intellectual property rights or the repression of
unfair
September 14, 2004).
competition, to which the Philippines is also a
party, or
extends reciprocal rights to nationals of the
Philippines by law,
shall be entitled to benefits to the extent
Vicarious Liability; Public Utility (2000) necessary to give
Silvestre leased a car from Avis-Rent-A-Car effect to any provision of such convention,
Co. at the treaty or reciprocal
Mactan International Airport. No sooner had he law, in addition to the rights to which any
driven the owner of an
car outside the airport when, due to his intellectual property right is otherwise entitled
negligence, he by this Act. (n)"
bumped an FX taxi owned and driven by Victor, To illustrate: the Philippines may refrain from
causing imposing a
damage to the latter in the amount of requirement of local incorporation or
P100,000.00. Victor establishment of a local
filed an action for damages against both domicile for the protection of industrial
Silvestre and Avis, property rights of
based on quasi-delict. Avis filed a motion foreign nationals (citizens of Canada,
to dismiss the Switzerland, U.S.) if the
complaint against it on the ground of failure to countries of said foreign nationals refrain from
state a cause imposing said
of action. Resolve the motion. (3%) requirement on Filipino citizens.
SUGGESTED ANSWER:
The motion to dismiss should be granted, AVIS INTELLECTUAL PROPERTY
is not the employer of Silvestre; hence, there is
no right of action against AVIS under Article Intellectual Creation (2004)
2180 of the Civil Code. Not being the Dr. ALX is a scientist honored for work related
employer, AVIS has no duty to supervise to the human genome project. Among his
Silvestre. Neither has AVIS the duty to observe pioneering efforts concern stem cell research for
due diligence in the selection of its customers. the cure of Alzheimer’s disease. Under corporate
Besides, it was given in the problem that the sponsorship, he helped develop a microbe that
cause of the accident was the negligence of ate and digested oil spills in the sea.
Silvestre.
ALTERNATIVE ANSWER: Now he leads a college team for cancer
The motion should be denied. Under the research in MSS State. The team has
Public Service Law, the registered owner of a experimented on a mouse whose body cells
public utility is liable for the damages suffered replicate and bear cancerous tumor. Called
by third persons through the use of such ―oncomouse‖, it is a life-form useful for medical
public utility. Hence, the cause of action is research and it is a novel creation. Its body
cells do not naturally occur in nature but are
based in law, the Public Service Law.
the product of man’s intellect, industry and ingenuity.
However, there is a doubt whether local
property laws and ethics would allow rights of
exclusive ownership on any life-form. Dr. ALX
ALTERNATIVE ANSWER:
needs your advice: (1) whether the reciprocity
principle in private international law could be Reciprocity principle cannot be applied in
applied in our jurisdiction; and (2) whether our jurisdiction because the Philippines is a
there are legal and ethical reasons that could party to the TRIPS agreement and the WTO.
frustrate his claim of exclusive ownership over The principle involved is the most-favored
the life-form nation clause which is the principle of non-
discrimination.
The protection afforded to intellectual
property protection in the Philippines also
applies to other members of the WTO. Thus, it
is not really reciprocity principle in private
international law that applies, but the most-
favored nation clause under public
international law.

There is no legal reason why "oncomouse"


cannot be protected under the law. Among
those excluded from patent protection are
"plant varieties or animal breeds, or
essentially biological process for the
production of plants and animals" (Section
22.4 Intellectual Property Code, R.A. No.
8293). The "oncomouse" in the problem is not
an essentially biological process for the
production of animals. It is a real invention
because its body cells do not naturally occur
in nature but are the product of man's
ingenuity, intellect and industry.

The breeding of oncomouse has novelty,


inventive step and industrial application.
These are the three requisites of
patentability. (Sec. 29, IPC)

There are no ethical reasons why Dr. ADX and


his college team cannot be given exclusive
ownership over their invention. The use of
such genetically modified mouse, useful for
cancer research, outweighs considerations for
animal rights.

There are no legal and ethical reasons that


would frustrate Dr. ALX's claim of exclusive
ownership over "oncomouse". Animals are
property capable of being appropriated and
owned'. In fact, one can own pet dogs or
cats, or any other animal. If wild animals are
capable of being owned, with more reason
animals technologically enhanced or
corrupted
CIVIL LAW Answers to the BAR as Arranged by Topics (Year 1990-2006)
by man's invention or industry are susceptible to exclusive
ownership by the inventor.
ALTERNATIVE ANSWER:
The oncomouse is a higher life form which
does not fall within the definition of the term
"invention". Neither may it fall within the
ambit of the term "manufacture" which
usually implies a non-living mechanistic
product. The oncomouse is better regarded as
a "discovery" which is the common patrimony
of man.
ALTERNATIVE ANSWER:
The "oncomouse" is a non-patentable invention. Hence,
cannot be owned exclusively by its inventor. It is a method
for the treatment of the human or animal body by surgery or
therapy and diagnostic methods practiced on said bodies are
not patentable under Sec. 22 of the IPC.
Civil Law Q&As (2007-2013) hectorchristopher@yahoo.com dbaratbateladot@gmail.com

A Compilation of the

Questions and Suggested Answers

In the

PHILIPPINE BAR EXAMINATIONS 2007-2013

In

CIVIL LAW
Compiled and Arranged By:

Baratbate-Ladot, Delight

Salise, Hector Christopher “Jay-Arh” Jr. M.

(University of San Jose-Recoletos School of Law)

ANSWERS TO BAR EXAMINATION QUESTIONS by the

UP LAW COMPLEX (2007-2013)

&

PHILIPPINE ASSOCIATION OF LAW SCHOOLS (2008)

“Never Let The Odds Keep You From Pursuing What You Know In Your Heart You Were Meant To Do.”-Leroy Satchel Paige
Page 1 of 180
Civil Law Q&As (2007-2013) hectorchristopher@yahoo.com dbaratbateladot@gmail.com

FOREWORD

This work is a compilation of the ANSWERS TO BAR


EXAMINATION QUESTIONS by the UP LAW COMPLEX ,
Philippine Association of Law Schools from 2007-2010 and
local law students and lawyers’ forum sites from 2011-2013
and not an original creation or formulation of the author.

The authors were inspired by the work of Silliman University’s


College of Law and its students of producing a very good
material to everyone involved in the legal field particularly the
students and the reviewees for free. Hence, this work is a
freeware.

Everyone is free to distribute and mass produce copies of this


work, however, the author accepts no liability for the content of
this reviewer, or for the consequences of the usage, abuse, or
any actions taken by the user on the basis of the information
given.

The answers (views or opinions) presented in this reviewer are


solely those of the authors in the given references and do not
necessarily represent those of the authors of this work.

The Authors.

“Never Let The Odds Keep You From Pursuing What You Know In Your Heart You Were Meant To Do.”-Leroy Satchel Paige
Page 2 of 180
Civil Law Q&As (2007-2013) hectorchristopher@yahoo.com dbaratbateladot@gmail.com

TABLE OF CONTENTS
(Titles are based on Silliman’s Compilation [Arranged by Topic])

Persons
Capacity: Juridical Capacity (2008).......................................................................................................... 12

Capacity; Juridical Capacity of Donee; Requisites for Acceptance (2012)...............................12

Capacity: Legal Capacity; Lex Rei Sitae (2007).....................................................................................13

Correction of Entries; Clerical Error Act (2008)..................................................................................14

Nationality Principle (2009)...........................................................................................................................14

Nationality Principle; Change of Name not Covered (2009)............................................................ 15

Conflict of Laws
Processual Presumption (2009)....................................................................................................................16

Jurisdiction; Courts may Assume Jurisdiction over Conflict of Laws Cases (2010)...........17

Adoption
Adoption; Termination; Death of Adopter (2009).................................................................................17

Adoption; Illegitimate Child (2010)............................................................................................................18

Adoption; Illegitimate Child; Use of Mother’s Surname as Middle Name (2012)..................19

Consent; Consent of the Adopter’s Heirs (2008)..................................................................................19

Qualifications of Adopter (2010).................................................................................................................20

“Never Let The Odds Keep You From Pursuing What You Know In Your Heart You Were Meant To Do.”-Leroy Satchel Paige
Page 3 of 180
Civil Law Q&As (2007-2013) hectorchristopher@yahoo.com dbaratbateladot@gmail.com

Family Code
Marriage; Annulment; Grounds (2009).....................................................................................................20

Marriage; Annulment; Grounds (2007).....................................................................................................21

Marriage; Annulment; Parties (2012)......................................................................................................... 22

Marriage; Annulment; Support Pendente Lite (2010).........................................................................22

Marriage; Divorce Decrees; Filipino Spouse Becoming Alien (2009)..........................................23

Marriage; Divorce Decrees; Foreign Spouse Divorces Filipino Spouse (2012)........................24

Marriage; Divorce Decrees; Foreign Spouse Divorces Filipino Spouse (2010)........................25

Marriage; Legal Separation; Prescription (2012)..................................................................................25

Marriage; Legal Separation; Prescription (2007)..................................................................................26

Marriage; Psychological Incapacity (2013).............................................................................................26

Marriage; Psychological Incapacity (2012).............................................................................................28

Marriage; Requisites (2008)...........................................................................................................................28

Marriage; Subsequent Marriage (2008).....................................................................................................29

Marriage; Void Marriages; By Reason of Public Policy (2008)........................................................ 30

Marriage; Void Marriages; By Reason of Public Policy (2007)........................................................ 30

Marriage; Void Marriages; Property Relations (2009).........................................................................30

Marriage; Void Marriages; Status of Children (2009)..........................................................................31

Parental Authority; Illegitimate Minor Child (2009)..........................................................................32

Parental Authority; In Vitro Fertilization (2010).................................................................................32

“Never Let The Odds Keep You From Pursuing What You Know In Your Heart You Were Meant To Do.”-Leroy Satchel Paige
Page 4 of 180
Civil Law Q&As (2007-2013) hectorchristopher@yahoo.com dbaratbateladot@gmail.com

Paternity & Filiation; Child Born Under a Void Marriage (2010)................................................33

Paternity & Filiation; Impugning Legitimacy (2010)..........................................................................34

Paternity & Filiation; In Vitro Fertilization;

Surrogate Mother’s Remedy to Regain Custody (2010).........................34

Paternity & Filiation; Legitimacy; Presumption (2008)...................................................................35

Paternity & Filiation; Legitimation of a Child from a Previous Valid Marriage (2008)......36

Paternity & Filiation; Legitimation of a Dead Child (2009)............................................................37

Paternity & Filiation; Support: Ascendants & Descendants;

Collateral Blood Relatives (2008).................................................................... 37

Paternity & Filiation; Use of Surname; Illegitimate Child (2009)................................................38

Paternity & Filiation; Who May Impugn Legitimacy (2009)...........................................................39

Property Relations; Adulterous Relationship (2009)..........................................................................39

Property Relations; Accession (2012)........................................................................................................40

Property Relations; Ante-Nuptial Debt (2007)...................................................................................... 40

Property Relations; Unions Without Marriage (2012)........................................................................ 41

Property Relations; Unions Without Marriage (2012)........................................................................ 42

Property Relations; Void Marriages (2010).............................................................................................43

Property Relations; Void Marriages (2010).............................................................................................45

“Never Let The Odds Keep You From Pursuing What You Know In Your Heart You Were Meant To Do.”-Leroy Satchel Paige
Page 5 of 180
Civil Law Q&As (2007-2013) hectorchristopher@yahoo.com dbaratbateladot@gmail.com

Succession
Disposition; Mortis Causa vs. Intervivos; Corpse (2009)..................................................................46

Heirs; Fideicommissary Substitution (2008).........................................................................................46

Heirs; Intestate Succession; Legitime; Computation (2010)..........................................................47

Heirs; Representation; Iron-Curtain Rule (2012).................................................................................49

Heirs; Reserva Troncal (2009).......................................................................................................................49

Intestate Succession (2008)...........................................................................................................................50

Intestate Succession (2008)...........................................................................................................................51

Intestate Succession; Rights of Representation:

Illegitimate, Adopted Child; Iron Curtain Rule (2007).......................... 51

Legitimes; Compulsory Heirs (2012).........................................................................................................53

Legitime; Compulsory Heirs (2008)............................................................................................................53

Preterition; Disinheritance (2008)............................................................................................................. 54

Succession; Proof of Death between persons called to succeed each other (2008).............55

Succession; Rule on Survivorship (2009)...............................................................................................56

Wills; Holographic Wills; Insertions & Cancellations (2012)...........................................................57

Wills; Holographic Wills; Probate (2009).................................................................................................. 57

Wills; Joint Wills (2008)...................................................................................................................................59

Wills; Joint Wills; Probate (2012)................................................................................................................59

“Never Let The Odds Keep You From Pursuing What You Know In Your Heart You Were Meant To Do.”-Leroy Satchel Paige
Page 6 of 180
Civil Law Q&As (2007-2013) hectorchristopher@yahoo.com dbaratbateladot@gmail.com

Wills; Prohibition to Partition of a Co-Owned Property (2010)..................................................... 60

Wills; Notarial Wills; Blind Testator; Requisites (2008).....................................................................61

Wills; Testamentary Disposition; Period to Prohibit Partition (2008)........................................61

Wills; Witnesses to a Will, Presence required; Thumbmark as Signature (2007)..................62

Donation
Donations; Formalities; In Writing (2007).............................................................................................. 63

Donations; Illegal & Impossible Conditions (2007).............................................................................64

Donation; Inter Vivos (2013)......................................................................................................................... 64

Property
Accretion; Alluvium (2008)............................................................................................................................65

Accretion; Rights of the Riparian Owner (2009)..................................................................................67

Builder; Good Faith; Requisites (2013).....................................................................................................68

Easement; Prescription; Acquisitive Prescription (2009)................................................................70

Easement; Right of Way (2013)....................................................................................................................70

Easement; Right of Way (2010)....................................................................................................................72

Hidden Treasure (2008)...................................................................................................................................73

Mortgage; Public or Private Instrument (2013).....................................................................................74

Occupation vs. Possession (2007)...............................................................................................................76

Ownership; Co-Ownership (2009)................................................................................................................76

Ownership; Co-Ownership (2008)................................................................................................................77

“Never Let The Odds Keep You From Pursuing What You Know In Your Heart You Were Meant To Do.”-Leroy Satchel Paige
Page 7 of 180
Civil Law Q&As (2007-2013) hectorchristopher@yahoo.com dbaratbateladot@gmail.com

Property; Movable or Immovable (2007)...................................................................................................78

Land Titles and Deeds


Acquisition of Lands; Sale of Real Property to an Alien (2009)....................................................79

Non-Registrable Properties (2007)...............................................................................................................80

Prescription; Acquisitive Prescription (2008)....................................................................................... 81

Prescription; Judicially Foreclosed Real Property Mortgage (2012)..........................................82

Purchaser in Good Faith; Mortgaged Property (2008)....................................................................... 83

Registration; Governing Law (2007)...........................................................................................................84

Registration; Party Who First took Possession (2013)......................................................................85

Registration; Requisites; Proof (2013)...................................................................................................... 86

Remedies; Fraud; Rights of Innocent Purchaser (2009)...................................................................89

Contracts
Contract to Sell vs. Conditional Contract of Sale (2012)................................................................ 90

Rescission of Contract; Fortuitous Event (2008)................................................................................90

Stipulation; Arbitration Clause (2009).......................................................................................................91

Obligations
Extinguishment; Compensation (2009)....................................................................................................91

Extinguishment; Compensation (2008)....................................................................................................92

Extinguishment; Novation (2008)...............................................................................................................93

Extinguishment; Payment of Check (2013)............................................................................................94

“Never Let The Odds Keep You From Pursuing What You Know In Your Heart You Were Meant To Do.”-Leroy Satchel Paige
Page 8 of 180
Civil Law Q&As (2007-2013) hectorchristopher@yahoo.com dbaratbateladot@gmail.com

Extinguishment; Payment of Check; Legal Tender (2008)..............................................................95

Liability; Solidary Liability (2008).............................................................................................................. 96

Obligations; Without Agreement (2007).................................................................................................. 97

Trust
Trust De Son Tort (2007)................................................................................................................................ 98

Sales
Condominium Act; Partition of a Condominium (2009)...................................................................99

Mortgage; Equitable Mortgage (2012)........................................................................................................99

Option Contract; Liquor & “Pulutan” as Consideration (2013)...................................................100

Right of First Refusal; Lessee; Effect (2008)........................................................................................101

Lease
Builder; Good Faith; Useful Improvements (2013)...........................................................................103

Lease; Caveat Emptor (2009)......................................................................................................................104

Agency
Agency; Sale of a Real Property through an Agent (2010)............................................................104

Partnership
Liability; Liability of a Partner (2010)....................................................................................................105

Oral Partnership (2009)................................................................................................................................106

Share; Demand during the Existence of Partnership (2012)........................................................ 107

“Never Let The Odds Keep You From Pursuing What You Know In Your Heart You Were Meant To Do.”-Leroy Satchel Paige
Page 9 of 180
Civil Law Q&As (2007-2013) hectorchristopher@yahoo.com dbaratbateladot@gmail.com

Commodatum & Mutuum


Mutuum; Interest; Solutio Indebiti (2012)............................................................................................107

Guaranty
Guaranty (2009).................................................................................................................................................108

Surety
Surety (2010)......................................................................................................................................................108

Pledge
Pledge; Pactum Commissorium (2009)..................................................................................................109

Torts and Damages


Damages (2012).................................................................................................................................................109

Damages; Moral & Exemplary (2009)......................................................................................................110

Damages; Public Officers acting in the Performance of their Duties (2012)........................111

Death Indemnity (2009)................................................................................................................................ 112

Doctrine of Discovered Peril (Last Clear Chance) (2007)...............................................................112

Liability; Owner of a Pet; Fortuitous Event (2010)..........................................................................113

Liability; Special Parental Authority (2010).......................................................................................115

Quasi-Delict; Claims; Requisites (2013).................................................................................................115

Quasi Tort (2010)..............................................................................................................................................116

“Never Let The Odds Keep You From Pursuing What You Know In Your Heart You Were Meant To Do.”-Leroy Satchel Paige
Page 10 of 180
Civil Law Q&As (2007-2013) hectorchristopher@yahoo.com dbaratbateladot@gmail.com

MULTIPLE CHOICE QUESTIONS

2013 Civil Law Exam MCQ (October 13, 2013)...….………………………………………..........118

2012 Civil Law Exam MCQ (October 14, 2012).….……………………………………...............130

2011 Civil Law Exam MCQ (November 13, 2011).………………………………….……............149

2010 Civil Law Exam MCQ (September 12, 2010).………………………………….……...........176

2007 Civil Law Exam MCQ (September 09, 2007).………………………………….……...........179

“Never Let The Odds Keep You From Pursuing What You Know In Your Heart You Were Meant To Do.”-Leroy Satchel Paige
Page 11 of 180
Civil Law Q&As (2007-2013) hectorchristopher@yahoo.com dbaratbateladot@gmail.com

Persons mother's womb. However, if the fetus


had an intra-uterine life of less than

Capacity: Juridical Capacity (2008) seven months, it is not deemed born if it


dies within twenty-four (24) hours after
No. II. At age 18, Marian found out that she its complete delivery from the maternal
was pregnant. She insured her own life and womb. The act of naming the unborn
named her unborn child as her sole child as sole beneficiary in the insurance
beneficiary. When she was already due to is favorable to the conceived child and
give birth, she and her boyfriend Pietro, the therefore the fetus acquires presumptive
father of her unboarn child, were or provisional personality. However, said
kidnapped in a resort in Bataan where they presumptive personality only becomes
were vacationing. The military gave chase conclusive if the child is born alive. The
and after one week, they were found in an child need not survive for twenty-four
abandoned hut in Cavite. Marian and Pietro hours as required under Art. 41 of the
were hacked with bolos. Marian and the Code because "Marian was already due to
baby delivered were both found dead, with give birth," indicating that the child was
the baby's umbilical cord already cut. Pietro more than seven months old.
survived.

(A). Can Marian's baby be the beneficiary of


Capacity; Juridical Capacity of Donee;
the insurance taken on the life of the
Requisites for Acceptance (2012)
mother? (2%)

No.I. b) Ricky donated P 1 Million to the


SUGGESTED ANSWER:
unborn child of his pregnant girlfriend,

Yes, the baby can be the beneficiary of which she accepted. After six (6) months of

the life insurance of Marian. Art. 40 NCC pregnancy, the fetus was born and baptized

provides that "birth determines as Angela. However, Angela died 20 hours

personality; but the conceived child after birth. Ricky sought to recover the P 1

shall be considered born for all purposes Million. Is Ricky entitled to recover?

that are favorable to it, provided that it Explain. (5%)

be born later with the conditions


SUGGESTED ANSWER:
specified in Art. 41. Article 41 states
that "for civil purposes, the fetus shall Yes, Ricky is entitled to recover the
be considered born if it is alive at the P1,000,000.00. The NCC considers a
time it is completely delivered from the
“Never Let The Odds Keep You From Pursuing What You Know In Your Heart You Were Meant To Do.”-Leroy Satchel Paige
Page 12 of 180
Civil Law Q&As (2007-2013) hectorchristopher@yahoo.com dbaratbateladot@gmail.com

fetus a person for purposes favorable to acquire a house in Australia because


it provided it is born later in accordance Australian Laws allow aliens to acquire
with the provision of the NCC. While the property from the age of 16.
donation is favorable to the fetus, the
SUGGESTED ANSWER:
donation did not take effect because the
fetus was not born in accordance with
TRUE. Since Australian Law allows alien
the NCC.
to acquire property from the age of 16,

To be considered born, the fetus that Roberta may validly own a house in

had an intrauterine life of less than Australia, following the principle of lex

seven (7) months should live for 24 rei sitae enshrined in Art. 16, NCC,

hours from its complete delivery from which states "Real property as well as

the mother’s womb. Since Angela had an personal property is subject to the law of

intrauterine life of less than seven (7) the country where it is situated."

months but did not live for 24 hours, she Moreover, even assuming that legal

was not considered born and, therefore, capacity of Roberta in entering the

did not become a person. Not being a contract in Australia is governed by

person, she has no juridical capacity to Philippine Law, she will acquire

be a donee, hence, the donation to her ownership over the property bought until

did not take effect. The donation not the contract is annulled.

being effective, the amount donated may


ALTERNATIVE ANSWER:
be recovered. To retain it will be unjust
enrichment. FALSE. Laws relating to family rights
and duties, or to the status, condition or
legal capacity of persons are binding

Capacity: Legal Capacity; Lex Rei Sitae upon the citizens of the Philippines,
even though living abroad (Art. 15, NCC).
(2007)
The age of majority under Philippine law
No.VII. Write "TRUE" if the statement is is 18 years (R.A. No. 6809); hence,
true or "FALSE" if the statement is false. If Roberta, being only 17 years old, has no
the statement is FALSE, state the reason. legal capacity to acquire and own land.
(2% each).

(1). Roberta, a Filipino, 17 years of age,


without the knowledge of his parents, can

“Never Let The Odds Keep You From Pursuing What You Know In Your Heart You Were Meant To Do.”-Leroy Satchel Paige
Page 13 of 180
Civil Law Q&As (2007-2013) hectorchristopher@yahoo.com dbaratbateladot@gmail.com

Correction of Entries; Clerical Error Act the Rules of Court because said changes

(2008) are substantive corrections.

No. IV. Gianna was born to Andy and (B). Instead of a judicial action, can
Aimee, who at the time Gianna's birth were administrative proceedings be brought for
not married to each other. While Andy was the purpose of making the above
single at the time, Aimee was still in the corrections? (2%)
process of securing a judicial declaration of
nullity on her marriage to her ex-husband. SUGGESTED ANSWER:

Gianna's birth certificate, which was signed


No. An administrative proceeding cannot
by both Andy and Aimee, registered the
be brought for the purpose of making the
status of Gianna as "legitimate", her
above corrections. R.A. 9048, otherwise
surname carrying that of Andy's and that
known as the Clerical Error Act, which
her parents were married to each other.
authorizes the city or municipal civil

(A). Can a judicial action for correction of registrar or the consul general to correct

entries in Gianna's birth certificate be a clerical or typographical error in an

successfully maintained to: entry and/or change the first name or


nickname in the civil register without
a). Change her status from "legitimate" to need of a judicial order. Errors that
"illegitimate" (1%); involve the change of nationality, age,
status, surname or sex of petitioner are
and
not included from the coverage of the
said Act (Silverio v. Republic, G.R. No.
b). Change her surname from that of Andy's
174689, 22 Oct., 2007).
to Aimee's maiden surname? (1%)

SUGGESTED ANSWER:
Nationality Principle (2009)

Yes, a judicial action for correction of


No.XII. Emmanuel and Margarita, American
entries in Gianna's birth certificate can
citizens and employees of the U.S. State
be successfully maintained to change (a)
Department, got married in the African
her status from "legitimate" to
state of Kenya where sterility is a ground
"illegitimate," and (b) her surname from
for annulment of marriage. Thereafter, the
that of Andy's to Aimee's maiden
spouses were assigned to the U.S. Embassy
surname in accordance with Rule 108 of
in Manila. On the first year of the spouses’

“Never Let The Odds Keep You From Pursuing What You Know In Your Heart You Were Meant To Do.”-Leroy Satchel Paige
Page 14 of 180
Civil Law Q&As (2007-2013) hectorchristopher@yahoo.com dbaratbateladot@gmail.com

tour of duty in the Philippines, Margarita


filed an annulment case against Emmanuel ALTERNATIVE ANSWER:
before a Philippine court on the ground of The forum has jurisdiction over an
her husband’s sterility at the time of the action for the annulment of marriage
celebration of the marriage. solemnized elsewhere but only when the
party bringing the actions is domiciled
(A). Will the suit prosper? Explain your in the forum. In this case, none of the
answer. (3%) parties to the marriage is domiciled in
the Philippines. They are here as
SUGGESTED ANSWER:
officials of the US Embassy whose stay in
No, the suits will not prosper. As applied
the country is merely temporary, lasting
to foreign nationals with the respect to
only during their fixed tour of duty.
family relations and status of persons,
Hence, the Philippine courts have no
the nationality principle set forth in
jurisdiction over the action.
Article 15 of the Civil Code will govern
the relations of Emmanuel and
Margarita. Since they are American
Nationality Principle; Change of Name
citizens, the governing law as to the
not Covered (2009)
ground for annulment is not Kenyan Law
which Magarita invokes in support of No.XX. (A). If Ligaya, a Filipino citizen
sterility as such ground; but should be residing in the United States, files a petition
U.S. Law, which is the national Law of for change of name before the District Court
both Emmanuel and Margarita as of New York, what law shall apply? Explain.
recognized under Philippine Law. Hence, (2%)
the Philippine court will not give due
course to the case based on Kenyan Law. SUGGESTED ANSWER:
The nationality principle as expressed in New York law shall apply. The petition of
the application of national law of foreign change of name file din New York does
nationals by Philippine courts is not concern the legal capacity or status
established by precedents (Pilapil v. Ibay- of the petitioner. Moreover, it does nto
Somera, 174 SCRA 653[1989], Garcia v. affect the registry of any other country
Recio, 366 SCRA 437 [2001], Llorente v. including the country of birth of the
Court of Appeals 345 SCRA 92 [2000], petitioner. Whatever judgment is
and Bayot v. Court of Appeals 570 SCRA rendered in that petition will have effect
472 [2008]). only in New York. The New York court

“Never Let The Odds Keep You From Pursuing What You Know In Your Heart You Were Meant To Do.”-Leroy Satchel Paige
Page 15 of 180
Civil Law Q&As (2007-2013) hectorchristopher@yahoo.com dbaratbateladot@gmail.com

cannot, for instance, order the Civil petitioner and his transactions in the
Registrar in the Philippines to change its Philippines. The Philippine court can
records. The judgment of the New York never acquire jurisdiction over the
court allowing a change in the name of custodian in the US of the records of the
the petitioner will be limited to the petitioner. Moreover, change of name
records of the petitioner in New York has nothing to do with the legal capacity
and the use of her new name in all or status of the alien. Since Philippine
transactions in New York. Since the records and transactions are the only
records and processes in New York are ones affected, the Philippine court may
the only ones affected, the New York effect the change only in accordance
court will apply New YorK law in with the laws governing those records
resolving the petition. and transactions that law cannot be but
ALTERNATIVE ANSWER: Philippine law.
Philippine law shall apply (Art 15, NCC). ALTERNATIVE ANSWER:
Status, conditions, family rights and U.S. law shall apply as it is his national
duties are governed by Philippine laws as law. This is pursuant to the application
to Filipinos even though sojourning of lex patriae or the nationality
abroad. principle, by which his legal status is
ALTENATIVE ANSWER: governed by national law, the matter of
If Ligaya, a Filipino, files a petition for change of name being included in the
change of name with the District Court legal status. The Supreme Court has
of New YoRk, the laws of New York will reiterate in several cases, that the lex
govern since change of name is not one patriae as provided in Article 15 of the
of those covered by the principles of Civil Code is applicable to foreign
nationality. nationals in determining their legal
status (supra).
(B). If Henry, an American citizen residing
in the Philippines, files a petition for change Conflict of Laws
of name before a Philippine court, what law
shall apply? Explain. (2%) Processual Presumption (2009)

SUGGESTED ANSWER:
No.I. TRUE or FALSE. Answer TRUE if the
Philippine law will apply. The petition
statement is true, or FALSE if the
for change of name in the Philippines
statement is false. Explain your answer in
will affect only the records of the
not more than two (2) sentences.

“Never Let The Odds Keep You From Pursuing What You Know In Your Heart You Were Meant To Do.”-Leroy Satchel Paige
Page 16 of 180
Civil Law Q&As (2007-2013) hectorchristopher@yahoo.com dbaratbateladot@gmail.com

(A). The doctrine of "processual Public Order. To maintain peace and


presumption" allows the court of the forum order, disputes that disturb the peace of
to presume that the foreign law applicable the forum should be settled by the court
to the case is the same as the local or of the forum even though the application
domestic law. (1%) of the foreign law is necessary for the
purpose.
SUGGESTED ANSWER:
TRUE. If the foreign law necessary to the Humanitarian Principle. An aggrieved

resolve an issue is not proven as a fact, party should not be left without remedy

the court of the forum may presume that in a forum even though the application

the foreign law is the same as the law of of the foreign law by the courts of the

the forum. forum is unavoidable in order to extend


relief.

Adoption
Jurisdiction; Courts may Assume
Jurisdiction over Conflict of Laws Cases
Adoption; Termination; Death of Adopter
(2010)
(2009)

No.III. Define, Enumerate or Explain. (2%


No.XIII. Rafael, a wealthy bachelor, filed a
each)
petition for the adoption of Dolly, a one-year
old foundling who had a severe heart ailment.
Give at least two reasons why a court may
During the pendency of the adoption
assume jurisdiction over a conflict of laws
proceedings, Rafael died of natural causes.
case.
The Office of the Solicitor General files a
SUGGESTED ANSWER: motion to dismiss the petition on the ground
that the case can no longer proceed because
Statute theory. There is a domestic law
of the petitioner’s death.
authorizing the local court to assume
jurisdiction. (A). Should the case be dismissed? Explain.
(2%)
Comity theory. The local court assumes
jurisdiction based on the principle of SUGGESTED ANSWER:
comity or courtesy. It depends on the stage of the

ALTERNATIVE ANSWER: proceedings when Rafael died. If he died


after all the requirements under the law

“Never Let The Odds Keep You From Pursuing What You Know In Your Heart You Were Meant To Do.”-Leroy Satchel Paige
Page 17 of 180
Civil Law Q&As (2007-2013) hectorchristopher@yahoo.com dbaratbateladot@gmail.com

have been complied with and the case is Adoption; Illegitimate Child (2010)
already submitted for resolution, the
court may grant the petition and issue a No.VIII. Spouses Rex and Lea bore two

decree of adoption despite the death of children now aged 14 and 8. During the

the adopter (Section 13, RA 8552). subsistence of their marriage, Rex begot a

Otherwise, the death of the petitioner child by another woman. He is now 10

shall have the effect terminating the years of age.

proceedings.
On Lea’s discovery of Rex’s fathering a child

(B). Will your answer be the same if it was by another woman, she filed a petition for

Dolly who died during the pendency of the legal separation which was granted.

adoption proceedings? Explain. (2%)


Rex now wants to adopt his illegitimate
SUGGESTED ANSWER: child.
No, if it was Dolly who died, the case
Whose consent is needed for Rex’s adoption
should be dismissed. Her death
of his illegitimate child? (2.5%)
terminates the proceedings (Art. 13,
Domestic Adoption Law).
SUGGESTED ANSWER:

ALTERNATIVE ANSWER: The consent of the 14-year-old


It depends. If all the requirements under legitimate child, of the 10- year -old
the law have already been complied with illegitimate child and of the biological
and the case is already submitted for mother of the illegitimate child are
resolution, the death of the adoptee needed for the adoption (Section 7 and
should not abate the proceedings. The 9, RA 8552). The consent of Lea is no
court should issue the decree of longer required because there was
adoption if will be for the best interest of already a final decree of legal separation.
the adoptee. While RA8552 provides only
for the case where it is the petitioner If there was no legal separation, can Rex

who dies before the decree is issued, it is still adopt his illegitimate child? Explain.

with more compelling reason that the (2.5%)

decree should be allowed in case it is the


SUGGESTED ANSWER:
adoptee who dies because adoption is
primarily for his benefit. Yes, he can still adopt his illegitimate
child but with the consent of his spouse,

“Never Let The Odds Keep You From Pursuing What You Know In Your Heart You Were Meant To Do.”-Leroy Satchel Paige
Page 18 of 180
Civil Law Q&As (2007-2013) hectorchristopher@yahoo.com dbaratbateladot@gmail.com

of his 14-year-old legitimate child, of the Adoption of Stephanie Nathy Astorga


illegitimate child, and of the biological Garcia, G.R. No. 148311, March 31,
mother of the illegitimate child (Section 2005, the Supreme Court ruled that the
7 and 9, RA 8552). adopted child may use the surname of
the natural mother as his middle name
because there is no prohibition in the
law against it. Moreover, it will also be
Adoption; Illegitimate Child; Use of
for the benefit of the adopted child who
Mother’s Surname as Middle Name
shall preserve his lineage on his
(2012)
mother’s side and reinforce his right to
inherit from his mother and her family.
No.IV.b) Honorato filed a petition to adopt
Lastly, it will make the adopted child
his minor illegitimate child Stephanie,
conform with the time-honored Filipino
alleging that Stephanie’s mother is Gemma
tradition of carrying the mother’s
Astorga Garcia; that Stephanie has been
surname as the person’s middle name.
using her mother’s middle name and
surname; and that he is now a widower and
qualified to be her adopting parent. He
prayed that Stephanie’s middle name be Consent; Consent of the Adopter’s Heirs
changed from "Astorga" to "Garcia," which (2008)
is her mother’s surname and that her
surname "Garcia" be changed to "Catindig," No.V. Despite several relationships with
which is his surname. This the trial court different women, Andrew remained
denied. Was the trial court correct in unmarried. His first relationship with
denying Hororato’s request for Stephanie’s Brenda produced a daughter, Amy, now 30
use of her mother’s surname as her middle years old. His second, with Carla, produced
name? Explain. (5%) two sons: Jon and Ryan. His third, with
Donna, bore him no children although
SUGGESTED ANSWER: Elena has a daughter Jane, from a previous
relationship. His last, with Fe, produced no
No, the trial court was not correct. There is
biological children but they informally
no law prohibiting an illegitimate child
adopted without court proceedings, Sandy's
adopted by his natural father to use as
now 13 years old, whom they consider as
middle name his mother’s surname. The
their own. Sandy was orphaned as a baby
law is silent as to what middle name an
and was entrusted to them by the midwife
adoptee may use. In case of In re:
who attended to Sandy's birth. All the

“Never Let The Odds Keep You From Pursuing What You Know In Your Heart You Were Meant To Do.”-Leroy Satchel Paige
Page 19 of 180
Civil Law Q&As (2007-2013) hectorchristopher@yahoo.com dbaratbateladot@gmail.com

children, including Amy, now live with Qualifications of Adopter (2010)


andrew in his house.
No.IX. Eighteen-year old Filipina Patrice
(A). Is there any legal obstacle to the legal had a daughter out of wedlock whom she
adoption of Amy by Andrew? To the legal named Laurie. At 26, Patrice married
adoption of Sandy by Andrew and Elena? American citizen John who brought her to
(2%) live with him in the United States of
America. John at once signified his
SUGGESTED ANSWER:
willingness to adopt Laurie.

Yes, there is a legal obstacle to the legal Can John file the petition for adoption? If
adoption of Amy by Andrew. Under Sec. yes, what are the requirements? If no, why?
9(d) of RA 8552, the New Domestic
(5%)
Adoption Act of 1998, the written
consent of the illegitimate SUGGESTED ANSWER:
sons/daughters, ten (10) years of age or
No, John cannot file the petition to
over, of the adopter, if living with said
adopt alone. Philippine law requires
adopter and the latter's spouse, if any, is
husband and wife to adopt jointly except
necessary to the adoption. All the
on certain situations enumerated in the
children of Andrew are living with him.
law. The case of John does not fall in
Andrew needs to get the written consent
any of the exceptions (R.A. 8552).
of Jon, Ryan, Vina and Wilma, who are
all ten (10) years old or more. Sandy's Family Code
consent to Amy's adoption is not
necessary because she was not legally Marriage; Annulment; Grounds (2009)
adopted by Andrew. Jane's consent is
likewise not necessary because she is No.XII. Emmanuel and Margarita, American

not a child of Andrew. Sandy, an orphan citizens and employees of the U.S. State

since birth, is eligible for adoption under Department, got married in the African

Sec. 8(f) of RA 8552, provided that state of Kenya where sterility is a ground

Andrew obtains the written consent of for annulment of marriage. Thereafter, the

the other children mentioned above, spouses were assigned to the U.S. Embassy

including Amy and Elena obtains the in Manila. On the first year of the spouses’

written consent of Jane, if she is over tour of duty in the Philippines, Margarita

ten years old (Sec. 9(d), RA 8552). filed an annulment case against Emmanuel
before a Philippine court on the ground of

“Never Let The Odds Keep You From Pursuing What You Know In Your Heart You Were Meant To Do.”-Leroy Satchel Paige
Page 20 of 180
Civil Law Q&As (2007-2013) hectorchristopher@yahoo.com dbaratbateladot@gmail.com

her husband’s sterility at the time of the shall be determined by applying Kenyan
celebration of the marriage. law and not Philippine law.

(B). Assume Emmanuel and Margarita are However, while Kenyan law governs the
both Filipinos. After their wedding in formal validity of the marriage, the legal
Kenya, they come back and take up capacity of the Filipino parties to the
residence in the Philippines. Can their marriage is governed not by Kenyan law
marriage be annulled on the ground of but by Philippine law (Article 15, NCC).
Emmanuel’s sterility? Explain. (3%) Sterility of a party as a ground for the
annulment of the marriage is not a
SUGGESTED ANSWER:
matter of form but a matter of legal
No, the marriage cannot be annulled
capacity. Hence, the Philippine court
under the Philippine law. Sterility is not
must apply Phillippine law in
a ground for annulment of marriage
determining the status of the marriage
under Article 45 of the Family Code.
on the ground of absence or defect in the
legal capacity of the Filipino parties.
ALTERNATIVE ANSWER:
Since sterility does not constitute
No, the marriage cannot be annulled in
absence or defect in the legal capacity of
the Philippines.
the parties under Philippine law, there is
no ground to avoid or annul the
The Philippine court shall have
marriage. Hence, the Philippine court
jurisdiction over the action to annul the
has to deny the petition.
marriage not only because the parties
are residents of the Philippines but
because they are Filipino citizens. The
Philippine court, however, shall apply Marriage; Annulment; Grounds (2007)
the law of the place where the marriage
was celebrated in determining its formal No. VII. Write "TRUE" if the statement is
validity (Article 26, FC; Article 17, NCC). true or "FALSE" if the statement is false. If
the statement is FALSE, state the reason.
Since the marriage was celebrated in (2% each).
Kenya in accordance with Kenyan law,
the formal validity of such marriage is (4). The day after John and Marsha got
governed by Kenyan law and any issue as married, John told her that he was
to the formal validity of that marriage impotent. Marsha continued to live with

“Never Let The Odds Keep You From Pursuing What You Know In Your Heart You Were Meant To Do.”-Leroy Satchel Paige
Page 21 of 180
Civil Law Q&As (2007-2013) hectorchristopher@yahoo.com dbaratbateladot@gmail.com

John for 2 years. Marsha is now estopped No, I do not agree. There are others who
from filing an annulment case against may file a petition for declaration of
John. nullity such as the other spouse in
bigamous marriages.
SUGGESTED ANSWER:

FALSE. Marsha is not estopped from


filing an annulment case against John on Marriage; Annulment; Support Pendente
the ground of his impotence, because Lite (2010)
she learned of his impotence after the
celebration of the marriage and not No.V. G filed on July 8, 2000 a petition for

before. Physical incapacity to declaration of nullity of her marriage to B.

consummate is a valid ground for the During the pendency of the case, the couple

annulment of marriage if such incapacity entered into a compromise agreement to

was existing at the time of the marriage, dissolve their absolute community of

continues and appears to be incurable. property. B ceded his right to their house

The marriage may be annulled on this and lot and all his shares in two business

ground within five years from its firms to G and their two children, aged 18

celebration. and 19.

B also opened a bank account in the


amount of P3 million in the name of the two

Marriage; Annulment; Parties (2012) children to answer for their educational


expenses until they finish their college
No.IX.b) A petition for declaration of nullity degrees.
of a void marriage can only be filed by
either the husband or the wife? Do you For her part, G undertook to shoulder the

agree? Explain your answer. (5%) day-to-day living expenses and upkeep of
the children. The Court approved the
SUGGESTED ANSWER: spouses’ agreement on September 8, 2000.

Yes, I agree. Under the rules Suppose the business firms suffered
promulgated by the Supreme Court, a reverses, rendering G unable to support
direct action for declaration of nullity herself and the children. Can G still ask for
may only be filed by any of the spouses. support pendente lite from B? Explain. (3%)

ALTERNATIVE SUGGESTED ANSWER:

“Never Let The Odds Keep You From Pursuing What You Know In Your Heart You Were Meant To Do.”-Leroy Satchel Paige
Page 22 of 180
Civil Law Q&As (2007-2013) hectorchristopher@yahoo.com dbaratbateladot@gmail.com

SUGGESTED ANSWER: Marriage; Divorce Decrees; Filipino

Spouse Becoming Alien (2009)


If B acquiesces and does not file the
action to impugn the legitimacy of the No.IV. Harry married Wilma, a very wealthy
child within the prescriptive period for woman. Barely five (5) years into the
doing so in Article 170 of the Family marriage, Wilma fell in love with Joseph.
Code, G's daughter by another man shall Thus, Wilma went to a small country in
be conclusively presumed as the Europe, became a naturalized citizen of
legitimate daughter of B by G. that country, divorced Harry, and married
Joseph. A year thereafter, Wilma and
Suppose in late 2004 the two children had
Joseph returned and established
squandered the P3 million fund for their
permanent residence in the Philippines.
education before they could obtain their
college degrees, can they ask for more (A). Is the divorce obtained by Wilma from
support from B? Explain. (3%) Harry recognized in the Philippines?
Explain your answer. (3%)
SUGGESTED ANSWER:

SUGGESTED ANSRWER :
Yes, the two children can still ask for
support for schooling or training for
As to Wilma, the divorced obtained by
some professions, trade or vocation,
her is recognized as valid in the
even beyond the age of majority until
Philippines because she is now a
they shall have finished or completed
foreigner. Philippine personal laws do
their education (Article 194, Paragraph
not apply to a foreigner. However,
2, Family Code; Javier v. Lucero, 94
recognition of the divorce as regards
Phil. 634 {1954}].Their having
Harry will depend on the applicability to
squandered the money given to them for
his case of the second paragraph of
their education will not deprive them of
Article 26 of the Family Code. If it is
their right to complete an education, or
applicable, divorce is recognized as to
to extinguish the obligation of the
him and, therefore, he can remarry.
parents to ensure the future of their
However, if it is not applicable, divorce
children.
is not recognized as to him and,
consequently, he cannot remarry.

ALTERNATIVE ANSWER:

“Never Let The Odds Keep You From Pursuing What You Know In Your Heart You Were Meant To Do.”-Leroy Satchel Paige
Page 23 of 180
Civil Law Q&As (2007-2013) hectorchristopher@yahoo.com dbaratbateladot@gmail.com

Yes , the divorce obtained by Wilma is SUGGESTED ANSWER :


recognized as valid in the Philippines. At
the time she got the divorce, she was Yes, he can validly marry Elizabeth,
already a foreign national having been applying the doctrine laid down by the
naturalized as a citizen of that “small Supreme Court in Republic v. Obrecido
country in Europe.” Based on precedents (427 SCRA 114 [2005]). Under the second
established by the Supreme Court paragraph of Article 26 of the Family
( Bayot v. CA, 570 SCRA 472 [2008]), Code, for the Filipino spouse to have
divorce obtained by a foreigner is capacity to remarry, the law expressly
recognized in the Philippines if validly requires the spouse who obtained the
obtained in accordance with his or her divorce to be a foreigner at the time of
national law . the marriage. Applying this requirement
to the case of Harry it would seem that
(B). If Harry hires you as his lawyer, what he is not given the capacity to remarry.
legal recourse would you advise him to This is because Wilma was a Filipino at
take? Why? (2%) the time of her marriage to Harry.

SUGGESTED ANSWER:
In Republic v. Obrecido, however, the
Supreme Court ruled that a Filipino
I will advice Harry to:
spouse is given the capacity to remarry
even though the spouse who obtained
Dissolve and liquidate his property
the divorce was a Filipino at the time of
relations with Wilma ; and
the marriage, if the latter was already a
foreigner when the divorce was already
If he will remarry, file a petition for the
obtained abroad. According to the court,
recognition and enforcement of the
to rule otherwise will violate the equal
foreign judgment of divorced (Rule
protection clause of the Constitution.
39,Rules of Court ).

(C). Harry tells you that he has fallen in


Marriage; Divorce Decrees; Foreign
love with another woman, Elizabeth, and
Spouse Divorces Filipino Spouse (2012)
wants to marry her because, after all,
Wilma is already married to Joseph. Can Cipriano and Lady Miros married each
Harry legally marry Elizabeth? Explain. other. Lady Miros then left for the US and
(2%) there, she obtained American citizenship.

“Never Let The Odds Keep You From Pursuing What You Know In Your Heart You Were Meant To Do.”-Leroy Satchel Paige
Page 24 of 180
Civil Law Q&As (2007-2013) hectorchristopher@yahoo.com dbaratbateladot@gmail.com

Cipriano later learned all about this proving only that the foreign spouse has
including the fact that Lady Miros has obtained a divorce against her or him
divorced him in America and that she had abroad. (1%)
remarried there. He then filed a petition for
authority to remarry, invoking Par. 2, Art. SUGGESTED ANSWER :

26 of the Family Code. Is Cipriano


FALSE, In Garcia v. Recio , 366 SCRA
capacitated to re-marry by virtue of the
437 (2001) , the SC held that for a
divorce decree obtained by his Filipino
Filipino spouse to have capacity to
spouse who was later naturalized as an
contract a subsequent marriage, it must
American citizen? Explain. (5%)
also be proven that the foreign divorced
obtained abroad by the foreigner spouse
SUGGESTED ANSWER:
give such foreigner spouse capacity to
Yes, he is capacitated to remarry. While remarry.
the second paragraph of Art 26 of the
ALTERNATIVE ANSWER:
Family Code is applicable only to a
Filipino who married a foreigner at the TRUE, Art 26 (2) (FC), clearly provides
time of marriage, the Supreme Court that the decree of divorce obtained
ruled in the case of Republic v. Orbecido, abroad by the foreigner spouse is
G.R. No. 154380, 5 Oct, 2005, that the sufficient to capacitate the Filipino
said provision equally applies to a spouse to remarry.
Filipino who married another Filipino at
the time of the marriage, but who was
already a foreigner when the divorce was
obtained. Marriage; Legal Separation; Prescription

(2012)

No.IV.a) After they got married, Nikki


Marriage; Divorce Decrees; Foreign discovered that Christian was having an
Spouse Divorces Filipino Spouse (2010) affair with another woman. But Nikki
decided to give it a try and lived with him
No.I. True or False.
for two (2) years. After two (2) years, Nikki
filed an action for legal separation on the
(A). Under Article 26 of the Family Code,
ground of Christian’s sexual infidelity. Will
when a foreign spouse divorces his/her
the action prosper? Explain. (5%)
Filipino spouse, the latter may re-marry by

“Never Let The Odds Keep You From Pursuing What You Know In Your Heart You Were Meant To Do.”-Leroy Satchel Paige
Page 25 of 180
Civil Law Q&As (2007-2013) hectorchristopher@yahoo.com dbaratbateladot@gmail.com

SUGGESTED ANSWER: the sexual infidelity committed in 2003,


the prescriptive period runs from 2003
Although the action for legal separation
and so on. The action for legal
has not yet prescribed, the prescriptive
separation for the last act of sexual
period being 5 years, if Obecido’s affair
infidelity in 2005 will prescribe in 2010.
with another woman was ended when
Nikki decided to live with him again,
Nikki’s action will not prosper on
account of condonation. However, if Marriage; Psychological Incapacity
such affair is still continuing, Nikki’s (2013)
action would prosper because the action
will surely be within five (5) years from No.I. You are a Family Court judge and

the commission of the latest act of before you is a Petition for the Declaration

sexual infidelity. Every act of sexual of Nullity of Marriage (under Article 36 of

liaison is a ground for legal separation. the Family Code)filed by Maria against Neil.
Maria claims that Neil is psychologically
incapacitated to comply with the essential
obligations of marriage because Neil is a
Marriage; Legal Separation; Prescription
drunkard, a womanizer, a gambler, and a
(2007)
mama's boy- traits that she never knew or

No.VII. Write "TRUE" if the statement is saw when Neil was courting her. Although
summoned, Neil did not answer Maria's
true or "FALSE" if the statement is false. If
petition and never appeared in court.
the statement is FALSE, state the reason.
(2% each).
To support her petition, Maria presented
three witnesses- herself, Dr. Elsie Chan,
(2). If a man commits several acts of sexual
and Ambrosia. Dr. Chan testified on the
infidelity, particularly in 2002, 2003, 2004,
psychological report on Neil that she
2005, the prescriptive period to file for legal
prepared. Since Neil never acknowledged
separation runs from 2002.
n9r responded to her invitation for
SUGGESTED ANSWER: interviews, her report is solely based on her
interviews with Maria and the spouses'
FALSE. The five-year prescriptive period minor children. Dr. Chan concluded that
for filing legal separation runs from the Neil is suffering from Narcissistic
occurrence of sexual infidelity Personality Disorder, an ailment that she
committed in 2002 runs from 2002, for found to be already present since Neil's

“Never Let The Odds Keep You From Pursuing What You Know In Your Heart You Were Meant To Do.”-Leroy Satchel Paige
Page 26 of 180
Civil Law Q&As (2007-2013) hectorchristopher@yahoo.com dbaratbateladot@gmail.com

early adulthood and one that is grave and mere conclusions. Being a drunkard, a
incurable. Maria testified on the specific womanizer, a gambler and a mama’s boy,
instances when she found Neil drunk, with merely shows Neil’s failure to perform
another woman, or squandering the his marital obligations. In a number of
family's resources in a casino. Ambrosia, cases, the Supreme Court did not find
the spouses' current household help, the existence of psychological incapacity
corroborated Maria's testimony. in cases where the respondent showed
habitual drunkenness (Republic v.
On the basis of the evidence presented, will Melgar, G.R. No. 139676, 2006), blatant
you grant the petition? (8%) display of infidelity and irresponsibility
(Dedel v. CA, 2004) or being hooked to
SUGGESTED ANSWER:
gambling and drugs (Republic v. Tanyag-
San Jose, G.R. No. 168328, 2007).
No. The petition should be denied.

ALTERNATIVE ANSWER:
The psychological incapacity under Art.
36 of the Family Code must be
Yes. The petition should be granted.
characterized by (a) gravity, (b) juridical
antecedence, and (c) incurability. It is The personal medical or psychological
not enough to prove that the parties examination of respondent is not a
failed to meet their responsibilities and requirement for declaration of
duties as married persons; it is essential psychological incapacity. It is the
that they must be shown to be incapable totality of the evidence presented which
of doing so, due to some physiological shall determine the existence of
(not physical) illness (Republic v. CA and psychological incapacity (Marcos v.
Molina, G.R. No. 108763, Feb 13, 1997). Marcos, G.R. No. 136490, Oct 19, 2000).
Dr. Chan’s report corroborated by
In this case, the pieces of evidence
Maria’s and Ambrosia’s testimonies,
presented are not sufficient to conclude
therefore, sufficiently prove Neil’s
that indeed Neil is suffering from
psychological incapacity to assume his
psychological incapacity [Narcissistic
marital obligations.
Personality Disorder] existing already
before the marriage, incurable and
serious enough to prevent Neil from
performing his essential marital
obligations. Dr. Chan’s report contains

“Never Let The Odds Keep You From Pursuing What You Know In Your Heart You Were Meant To Do.”-Leroy Satchel Paige
Page 27 of 180
Civil Law Q&As (2007-2013) hectorchristopher@yahoo.com dbaratbateladot@gmail.com

Marriage; Psychological Incapacity the said report is the only evidence of

(2012) respondent’s psychological incapacity.

No.II.b) The petitioner filed a petition for


declaration of nullity of marriage based Marriage; Requisites (2008)
allegedly on the psychological incapacity of
the respondent, but the psychologist was No. III. Roderick and Faye were high school

not able to personally examine the sweethearts. When Roderick was 18 and

respondent and the psychological report Faye, 16 years old, they started to live

was based only on the narration of together as husband and wife without the

petitioner. Should the annulment be benefit of marriage. When Faye reached 18

granted? Explain. (5%) years of age, her parents forcibly took her
back and arranged for her marriage to Brad.
SUGGESTED ANSWER: Although Faye lived with Brad after the
marriage, Roderick continued to regularly
The annulment cannot be guaranteed
visit Faye while Brad was away at work.
solely on the basis of the psychological
During their marriage, Faye gave birth to a
report. For the report to prove the
baby girl, Laica. When Faye was 25 years old,
psychological incapacity of the
Brad discovered her continued liason with
respondent, it is required that the
Roderick and in one of their heated
psychologist should personally examine
arguments, Faye shot Brad to death. She lost
the respondent and the psychological
no time in marrying her true love Roderick,
report should be based on the
without a marriage license, claiming that they
psychologist’s independent assessment
have been continuously cohabiting for more
of the facts as to whether or not the
than 5 years.
respondent is psychologically

incapacitated. (A). Was the marriage of Roderick and Faye


valid? (2%)
Since, the psychologist did not
personally examine the respondent, and SUGGESTED ANSWER:
his report is based solely on the story of
the petitioner who has an interest in the No. The marriage of Roderick and Faye is
outcome of the petition, the marriage not valid. Art. 4, FC provides that the
cannot be annulled on the ground of absence of any of the essential or formal
respondent’s psychological incapacity if requisites renders the marriage void ab
initio. However, no license shall be

“Never Let The Odds Keep You From Pursuing What You Know In Your Heart You Were Meant To Do.”-Leroy Satchel Paige
Page 28 of 180
Civil Law Q&As (2007-2013) hectorchristopher@yahoo.com dbaratbateladot@gmail.com

necessary for the marriage of a man and reqiured to submit the required certificate
a woman who have lived together as of capacity to marry from the German
husband and wife for at least 5 years and Embassy in Manila, Adolf stated in the
without any legal impediment to marry application for marriage license that he was
each other. In Republic v. Dayot, G.R. a Filipino citizen. With the marriage license
No. 175581, 28 March 2008, reiterating stating that Adolf was a Filipino, the couple
the doctrine in Niñal v. Bayadog, G.R. got married in a ceremony officiated by the
No. 133778, 14 March 2000, this five- Parish Priest of Calamba, Laguna in a
year period is characterized by beach in Nasugbu, Batangas, as the local
exclusivity and continuity. In the parish priest refused to solemnize
present case, the marriage of Roderick marriages except in his church. Is the
and Faye cannot be considered as a marriage valid? Explain fully. (5%)
marriage of exceptional character,
because there were 2 legal impediments SUGGESTED ANSWER:

during their cohabitation: minority on


No. The marriage is not valid. Art. 41 FC
the part of Faye, during the first two
allows the present spouse to contract a
years of cohabitation; and, lack of legal
subsequent marriage during the
capacity, since Faye married Brad at the
subsistence of his previous marriage
age of 18. The absence of a marriage
provided that: (a) his prior spouse in the
license made the marriage of Faye and
first marriage had been absent for four
Roderick void ab initio.
consecutive years; (b) that the spouse
present has a well-founded belief that

Marriage; Subsequent Marriage (2008) the absent spouse was already dead, and
present spouse instituted a summary
No. I. Ana Rivera had a husband, a Filipino proceeding for the declaration of the
citizen like her, who was among the presumptive death of absent spouse.
passengers on board a commercial jet plane Otherwise, the second marriage shall be
which crashed in the Atlantic Ocean ten null and void. In the instant case, the
years earlier and had never been heard of husband of Ana was among the
ever since. Believing that her husband had passengers on board a commercial jet
died, Ana married Adolf Cruz Staedtler, a plane which crashed in the Atlantic
divorced German national born of a Ocean. The body of the deceased
German father and a Filipino mother husband was not recovered to confirm
residing in Stuttgart. To avoid being his death. Thus, following Art. 41, Ana

“Never Let The Odds Keep You From Pursuing What You Know In Your Heart You Were Meant To Do.”-Leroy Satchel Paige
Page 29 of 180
Civil Law Q&As (2007-2013) hectorchristopher@yahoo.com dbaratbateladot@gmail.com

should have first secured a judicial while Jane is a child of Elena from a
declaration of his presumptive death previous relationship. Thus, their
before she married Adolf. The absence of marriage is not one of the prohibited
the said judicial declaration marriages enumerated under Art. 38 of
incapacitated Ana from contracting her the FC.
second marriage, making it void ab
initio.

Marriage; Void Marriages; By Reason of

Marriage; Void Marriages; By Reason of Public Policy (2007)

Public Policy (2008) No. VII. Write "TRUE" if the statement is


true or "FALSE" if the statement is false. If
No.V. Despite several relationships with the statement is FALSE, state the reason.
different women, Andrew remained (2% each).
unmarried. His first relationship with
Brenda produced a daughter, Amy, now 30 (5). Amor gave birth to Thelma when she

years old. His second, with Carla, produced was 15 years old. Thereafter, Amor met

two sons: Jon and Ryan. His third, with David and they got married when she was

Donna, bore him no children although 20 years old. David had a son, Julian, with

Elena has a daughter Jane, from a previous his ex-girlfriend Sandra. Julian and Thelma

relationship. His last, with Fe, produced no can get married.

biological children but they informally


SUGGESTED ANSWER:
adopted without court proceedings, Sandy's
now 13 years old, whom they consider as
TRUE. Julian and Thelma can get
their own. Sandy was orphaned as a baby
married. Marriage between stepbrothers
and was entrusted to them by the midwife
and stepsisters are not among the
who attended to Sandy's birth. All the
marriages prohibited under the Family
children, including Amy, now live with
Code.
andrew in his house.

(D). Can Jon and Jane legally marry? (1%)


Marriage; Void Marriages; Property

SUGGESTED ANSWER: Relations (2009)

Yes. Jon and Jane can marry each other; No. III. In December 2000, Michael and

Jon is an illegitimate child of Andrew Anna, after obtaining a valid marriage

“Never Let The Odds Keep You From Pursuing What You Know In Your Heart You Were Meant To Do.”-Leroy Satchel Paige
Page 30 of 180
Civil Law Q&As (2007-2013) hectorchristopher@yahoo.com dbaratbateladot@gmail.com

license, went to the Office of the Mayor of impediment for them to validity marry
Urbano, Bulacan, to get married. The Mayor each other.
was not there, but the Mayor’s secretary
asked Michael and Anna and their witnesses
to fill up and sign the required marriage Marriage; Void Marriages; Status of
contract forms. The secretary then told them Children (2009)
to wait, and went out to look for the Mayor
who was attending a wedding in a neighboring No. III. In December 2000, Michael and Anna,

municipality. after obtaining a valid marriage license, went


to the Office of the Mayor of Urbano, Bulacan,
When the secretary caught up with the to get married. The Mayor was not there, but
Mayor at the wedding reception, she the Mayor’s secretary asked Michael and
showed him the marriage contract forms Anna and their witnesses to fill up and sign
and told him that the couple and their the required marriage contract forms. The
witnesses were waiting in his office. The secretary then told them to wait, and went out
Mayor forthwith signed all the copies of the to look for the Mayor who was attending a
marriage contract, gave them to the wedding in a neighboring municipality.
secretary who returned to the Mayor’s
office. She then gave copies of the marriage
contract to the parties, and told Michael When the secretary caught up with the

and Anna that they were already married. Mayor at the wedding reception, she

Thereafter, the couple lived together as showed him the marriage contract forms

husband and wife, and had three sons. and told him that the couple and their
witnesses were waiting in his office. The
(C). What property regime governs the Mayor forthwith signed all the copies of the
properties acquired by the couple? Explain. marriage contract, gave them to the
(2%) secretary who returned to the Mayor’s
office. She then gave copies of the marriage
SUGGESTED ANSWER:
contract to the parties, and told Michael
and Anna that they were already married.
The marriage being void, the property
Thereafter, the couple lived together as
relationship that governed their union is
husband and wife, and had three sons.
special co-ownership under Article 147
of the Family Code. This is on the (A). Is the marriage of Michael and Anna
assumption that there was no valid, voidable, or void? Explain your
answer. (3%)

“Never Let The Odds Keep You From Pursuing What You Know In Your Heart You Were Meant To Do.”-Leroy Satchel Paige
Page 31 of 180
Civil Law Q&As (2007-2013) hectorchristopher@yahoo.com dbaratbateladot@gmail.com

SUGGESTED ANSWER : (C). When Rona reaches seven (7) years old,
she tells Rodolfo that she prefers to live
The marriage is void because the formal with him, because he is better off
requisite of marriage ceremony was financially than Nanette. If Rodolfo files an
absent ( Art.3, F.C. 209, Family Code). action for the custody of Rona, alleging that
he is Rona’s choice as custodial parent, will
ALTERNATIVE ANSWER: the court grant Rodolfo’s petition? Why or
The marriage is void because an why not? (2%)
essential requisite was absent: consent
SUGGESTED ANSWER:
of the parties freely given in the
No, because Rodolfo has no parental
presence of the solemnizing officer (Art .
authority over Rona. He who has the
2, FC).
parental authority has the right to
(B). What is the status of the three children custody. Under the Family Code, the
of Michael and Anna? Explain your answer. mother alone has parental authority over
(2%) the illegitimate child. This is true even if
illegitimate father recognized the child
SUGGESTED ANSWER: and even though he is giving support for
The children are illegitimate, having the child. To acquire custody over Rona,
been born outside a valid marriage. Rodolfo should first deprive Nanette of
parental authority if there is ground
under the law, and in a proper court
Parental Authority; Illegitimate Minor proceedings. In the same action, the
Child (2009) court may award custody of Rona to
Rodolfo if it is for her best interest.
No.XIV. Rodolfo, married to Sharon, had an
illicit affair with his secretary, Nanette, a
19-year old girl, and begot a baby girl,
Parental Authority; In Vitro Fertilization
Rona. Nanette sued Rodolfo for damages:
(2010)
actual, for hospital and other medical
expenses in delivering the child by No.VI. Gigolo entered into an agreement
caesarean section; moral, claiming that with Majorette for her to carry in her womb
Rodolfo promised to marry her, his baby via in vitro fertilization. Gigolo
representing that he was single when, in undertook to underwrite Majorette’s pre-
fact, he was not; and exemplary, to teach a natal expenses as well as those attendant
lesson to like-minded Lotharios.

“Never Let The Odds Keep You From Pursuing What You Know In Your Heart You Were Meant To Do.”-Leroy Satchel Paige
Page 32 of 180
Civil Law Q&As (2007-2013) hectorchristopher@yahoo.com dbaratbateladot@gmail.com

to her delivery. Gigolo would thereafter pay in Parañaque. After four (4) years or in
Majorette P2 million and, in return, she 2001, G having completed her 4-year
would give custody of the baby to him. college degree as a fulltime student, she
and B contracted marriage without a
After Majorette gives birth and delivers the license.
baby to Gigolo following her receipt of P2
million, she engages your services as her The marriage of B and G was, two years
lawyer to regain custody of the baby. later, declared null and void due to the
absence of a marriage license.
Who of the two can exercise parental
authority over the child? Explain. (2.5%) (B). Is Venus legitimate, illegitimate, or
legitimated? Explain briefly. (3%)
SUGGESTED ANSWER:
SUGGESTED ANSWER:
Majorette, the mother, can exercise
parental authority. Since the child was Venus is illegitimate. She was conceived
born out of wedlock, the child is and born outside a valid marriage. Thus,
illegitimate and the mother has the she is considered illegitimate (Art 165,
exclusive parental authority and custody Family Code). While Venus was
over the child. legitimated by the subsequent marriage
of her parents, such legitimation was
ALTERNATIVE ANSWER:
rendered ineffective when the said
Gigolo can exercise parental authority marriage was later on declared null and
over the child. Majorette has no blood void due to absence of a marriage
relation to the child. She is just a license.
“carrier” of the child.
Under Article 178 of the Family Code,
“legitimation shall take place by a
subsequent valid marriage between
Paternity & Filiation; Child Born Under a parents. The annulment of a voidable

Void Marriage (2010) marriage shall not affect the


legitimation.” The inclusion of the
No.X. In 1997, B and G started living underscored portion in the Article
together without the benefit of marriage. necessarily implies that the Article's
The relationship produced one offspring, application is limited to voidable
Venus. The couple acquired a residential lot marriages. It follows that when the

“Never Let The Odds Keep You From Pursuing What You Know In Your Heart You Were Meant To Do.”-Leroy Satchel Paige
Page 33 of 180
Civil Law Q&As (2007-2013) hectorchristopher@yahoo.com dbaratbateladot@gmail.com

subsequent marriage is null or void, the action to impugn, B can pray for the
legitimation must also be null and void. correction of the status of the said
In the present problem, the marriage daughter in her record of birth.
between B and G was not voidable but
void. Hence, Venus has remained an (B). If B acquiesces to the use of his

illegitimate child. surname by G’s daughter by another man,


what is/are the consequence/s? Explain.
(5%)

Paternity & Filiation; Impugning SUGGESTED ANSWER:


Legitimacy (2010)
If B acquiesces and does not file the

No.IV. Spouses B and G begot two action to impugn the legitimacy of the

offsprings. Albeit they had serious child within the prescriptive period for

personality differences, the spouses doing so in Article 170 of the Family

continued to live under one roof. B begot a Code, G's daughter by another man shall

son by another woman. G also begot a be conclusively presumed as the

daughter by another man. legitimate daughter of B by G.

(A). If G gives the surname of B to her


daughter by another man, what can B do to
Paternity & Filiation; In Vitro
protect their legitimate children's interests?
Fertilization; Surrogate Mother’s
Explain. (5%)
Remedy to Regain Custody (2010)
SUGGESTED ANSWER:
No.VI. Gigolo entered into an agreement
B can impugn the status of G's daughter with Majorette for her to carry in her womb
by another man as his legitimate his baby via in vitro fertilization. Gigolo
daughter on the ground that for undertook to underwrite Majorette’s pre-
biological reason he could not have been natal expenses as well as those attendant
the father of the child, a fact that may to her delivery. Gigolo would thereafter pay
be proven by the DNA test. Having been Majorette P2 million and, in return, she
born during the marriage between B and would give custody of the baby to him.
G, G's daughter by another man is
After Majorette gives birth and delivers the
presumed as the child of B under Article
164 of the Family Code. In the same baby to Gigolo following her receipt of P2

“Never Let The Odds Keep You From Pursuing What You Know In Your Heart You Were Meant To Do.”-Leroy Satchel Paige
Page 34 of 180
Civil Law Q&As (2007-2013) hectorchristopher@yahoo.com dbaratbateladot@gmail.com

million, she engages your services as her No, he cannot. Both he and Majorette are
lawyer to regain custody of the baby. guilty of violating the provision of the
Anti-Child Abuse Law (RA7610) on child
What legal action can you file on behalf of trafficking. Being in pari delicto, the
Majorette? Explain. (2.5%) partners shall be left where they are and
Gigolo cannot demand the return of what
SUGGESTED ANSWER:
he paid.
As her lawyer, I can file a petition for
ALTERNATIVE ANSWER:
habeas corpus on behalf Majorette to
recover custody of her child. Since she is Yes. The agreement between Gigolo and
the mother of the child that was born Majorette is a valid agreement.
out of wedlock, she has exclusive
parental authority and custody over the Is the child entitled to support and

child. Gigolo, therefore, has no right to inheritance from Gigolo? Explain. (2.5%)

have custody of the child and his refusal


SUGGESTED ANSWER:
to give up custody will constitute illegal
detention for which habeas corpus is the If Gigolo voluntarily recognized the child
proper remedy. as his illegitimate child in accordance
with Article 175 in relation to Article
ALTERNATIVE ANSWER:
172 of the Family Code, the child is
The action to regain custody will not entitled to support and inheritance from
prosper. In the first place Majorette Gigolo.
cannot regain custody of the baby. As
ALTERNATIVE ANSWER:
surrogate mother she merely carries the
child in her womb for its development. Yes, because Gigolo is the natural and
The child is the child of the natural biological parent of the baby.
parents- Gigolo and his partner. The
agreement between Gigolo and Majorette
is a valid agreement.
Paternity & Filiation; Legitimacy;

Can Gigolo demand from Majorette the Presumption (2008)

return of the P2 million if he returns the No. III. Roderick and Faye were high school

baby? Explain. (2.5%) sweethearts. When Roderick was 18 and


Faye, 16 years old, they started to live
SUGGESTED ANSWER: together as husband and wife without the

“Never Let The Odds Keep You From Pursuing What You Know In Your Heart You Were Meant To Do.”-Leroy Satchel Paige
Page 35 of 180
Civil Law Q&As (2007-2013) hectorchristopher@yahoo.com dbaratbateladot@gmail.com

benefit of marriage. When Faye reached 18 March 2002, the Supreme Court ruled
years of age, her parents forcibly took her that impugning the legitimacy of the
back and arranged for her marriage to Brad. child is a strictly personal right of
Although Faye lived with Brad after the husband, except: (a) when the husband
marriage, Roderick continued to regularly died before the expiration of the period
visit Faye while Brad was away at work. fixed for bringing the action; (b) if he
During their marriage, Faye gave birth to a should die after the filing of the
baby girl, Laica. When Faye was 25 years old, complaint, without having desisted
Brad discovered her continued liason with therefrom, or (c) if the child was born
Roderick and in one of their heated after the death of the husband. Laica's
arguments, Faye shot Brad to death. She lost case does not fall under any of the
no time in marrying her true love Roderick, exceptions.
without a marriage license, claiming that they
have been continuosly cohabiting for more (D). Can Laica be legitimated by the

than 5 years. marriage of her biological parents? (1%)

(B). What is the filiation status of Laica? SUGGESTED ANSWER:

(2%)
No. Laica cannot be legitimated by the

SUGGESTED ANSWER: marriage of her biological parents


because only children conceived and
Laica is legitimate because children born outside of wedlock of parents who
conceived or born during the marriage of at the time of the conception of the
the parents are presumed to be former were not disqualified by any
legitimate (Art. 164, FC). impediment to marry each other may be
legitimated (Art. 177, FC).
(C).Can Laica bring an action to impugn her
own status on the ground that based on
DNA results, Roderick is her biological
father? (2%) Paternity & Filiation; Legitimation of a
Child from a Previous Valid Marriage
SUGGESTED ANSWER: (2008)
No. IV. Gianna was born to Andy and
No. Laica cannot bring an action to
Aimee, who at the time Gianna's birth were
impugn her own status. In Liyao Jr. v.
not married to each other. While Andy was
Tanhoti-Liyao, G.R. No. 138961, 07
single at the time, Aimee was still in the

“Never Let The Odds Keep You From Pursuing What You Know In Your Heart You Were Meant To Do.”-Leroy Satchel Paige
Page 36 of 180
Civil Law Q&As (2007-2013) hectorchristopher@yahoo.com dbaratbateladot@gmail.com

process of securing a judicial declaration of statement is false. Explain your answer in


nullity on her marriage to her ex-husband. not more than two (2) sentences.
Gianna's birth certificate, which was signed
by both Andy and Aimee, registered the (E). A dead child can be legitimated. (1%)

status of Gianna as "legitimate", her


SUGGESTED ANSWER:
surname carrying that of Andy's and that
her parents were married to each other.
TRUE To be legitimated, the law does
not require a child to be alive at the
(C). Assuming that Aimee is successful in
same time of the marriage of his / her
declaring her former marriage void, and
parents ( Article 177, FC ). Furthermore,
Andy and Aimee subsequently married each
Art. 181 of the Family Code which states
other, would Gianna be legitimated? (1%)
that “[Th]e llegitimation of children who

SUGGESTED ANSWER: died before the celebration of marriage


will benefit their descendants,” does not
Gianna cannot be legitimated by the preclude instances where such
subsequent marriage of Andy and Aimee. legitimation will benefit no one but the
Art. 177 of the FC provides that "only child's ascendants ,or other relatives .
children conceived and born outside of
wedlock of parents who, at the time of
the conception of the former, were not Paternity & Filiation; Support:

disqualified by any impediment to marry Ascendants & Descendants; Collateral

each other may be legitimated." In the Blood Relatives (2008)


present case, a legal impediment was
No.V. Despite several relationships with
existing at the time of the conception of
different women, Andrew remained
Gianna. Her mother, Aimee, was still
unmarried. His first relationship with
alive in the process of securing judicial
Brenda produced a daughter, Amy, now 30
declaration of nullity on her marriage to
years old. His second, with Carla, produced
her ex-husband.
two sons: Jon and Ryan. His third, with
Donna, bore him no children although
Paternity & Filiation; Legitimation of a Elena has a daughter Jane, from a previous

Dead Child (2009) relationship. His last, with Fe, produced no


biological children but they informally
No. I. TRUE or FALSE. Answer TRUE if the adopted without court proceedings, Sandy's
statement is true, or FALSE if the now 13 years old, whom they consider as

“Never Let The Odds Keep You From Pursuing What You Know In Your Heart You Were Meant To Do.”-Leroy Satchel Paige
Page 37 of 180
Civil Law Q&As (2007-2013) hectorchristopher@yahoo.com dbaratbateladot@gmail.com

their own. Sandy was orphaned as a baby Paternity & Filiation; Use of Surname;
and was entrusted to them by the midwife Illegitimate Child (2009)
who attended to Sandy's birth. All the
children, including Amy, now live with No.XIV. Rodolfo, married to Sharon, had an

andrew in his house. illicit affair with his secretary, Nanette, a


19-year old girl, and begot a baby girl,
(B). In his old age, can Andrew be legally Rona. Nanette sued Rodolfo for damages:
entitled to claim support from Amy, Jon, actual, for hospital and other medical
Ryan, Vina, Wilma, and Sandy assuming expenses in delivering the child by
that all of them have the means to support caesarean section; moral, claiming that
him? (1%) Rodolfo promised to marry her,
representing that he was single when, in
SUGGESTED ANSWER: fact, he was not; and exemplary, to teach a
lesson to like-minded Lotharios.
Andrew, in his old age, cannot be legally
entitled to claim support because Art. (B). Suppose Rodolfo later on acknowledges
195, par 2 of the FC limits the giving of Rona and gives her regular support, can he
support to "legitimate ascendants and compel her to use his surname? Why or
descendants." why not? (2%)

(C). Can Amy, Jon, Ryan, Vina, Wilma, and SUGGESTED ANSWER:
Sandy legally claim support from each No. he has no right to compel Rona to
other? (2%) use his surname. The law does not give
him the right simply because he gave her
SUGGESTED ANSWER:
support (RA 9255).

Amy, Jon, Ryan, Vina, Wilma and Sandy


Under the Family Code, an illegitimate
cannot legally claim support from each
child was required to use only the
other because Art. 195, par 5 limits the
surname of the mother. Under RA 9255,
giving of support to "legitimate brothers
otherwise known as the Revilla law,
and sisters, whether full or half blood."
however, the illegitimate child is given
the option to use the surname of the
illegitimate father when the latter has
recognized the former in accordance
with law. Since the choice belongs to the
illegitimate child, Rodolfo cannot

“Never Let The Odds Keep You From Pursuing What You Know In Your Heart You Were Meant To Do.”-Leroy Satchel Paige
Page 38 of 180
Civil Law Q&As (2007-2013) hectorchristopher@yahoo.com dbaratbateladot@gmail.com

compel Rona, if already of age, to use They are not related at all to Edilberto.
the surname against her will. If Rona is They were born during the marriage of
still a minor, to use the surname of Conrado and Clarita, hence, are
Rodolfo will require the consent of considered legitimate children of the
Rona's mother who has sole parental said spouses. This status is conferred on
authority over her. them at birth by law.

Under Philippine law, a person cannot


Paternity & Filiation; Who May Impugn have more than one natural filiation.
Legitimacy (2009) The legitimate filiation of a person can
be changed only if the legitimate father
No.V. Four children, namely: Alberto,
will successfully impugn such status.
Baldomero, Caridad, and Dioscoro, were
born to the spouses Conrado and Clarita de In the problem, therefore, the filiation of
la Costa. The children’s birth certificates Alberto and Baldomero as legitimate
were duly signed by Conrado, showing children of Condrado cannot be changed
them to be the couple’s legitimate children. by their recognition by Edilberto as his
illegitimate children. Before they can be
Later, one Edilberto de la Cruz executed a
conferred the status of Edilberto’s
notarial document acknowledging Alberto
illegitimate children, Condrado must
and Baldomero as his illegitimate children
first impugn their legitimacy. Since
>with Clarita. Edilberto died leaving
Condrado has not initiated any action to
substantial properties. In the settlement of
impugn their legitimacy, they continue
his estate, Alberto and Baldomero
to be the legitimate of Condrado. They
intervened claiming shares as the
cannot be the illegitimate children of
deceased’s illegitimate children. The
Edilberto at the same time. Not being
legitimate family of Edilberto opposed the
the illegitimate children of Edilberto,
claim.
they have no right to inherit from him.

Are Alberto and Baldomero entitled to share


in the estate of Edilberto? Explain. (4%)
Property Relations; Adulterous
SUGGESTED ANSWER: Relationship (2009)

No. XI. TRUE or FALSE. Answer TRUE if


No, Alberto and Baldomero are not
the statement is true, or FALSE if the
entitled to share in Edilberto’s estate.

“Never Let The Odds Keep You From Pursuing What You Know In Your Heart You Were Meant To Do.”-Leroy Satchel Paige
Page 39 of 180
Civil Law Q&As (2007-2013) hectorchristopher@yahoo.com dbaratbateladot@gmail.com

statement is false. Explain your answer in SUGGESTED ANSWER:


not more than two (2) sentences.
It depends. If the value of the building is

(B). If there is no marriage settlement, the more than the value of the land, the

salary of a "spouse" in an adulterous building is conjugal and the land


becomes conjugal property under Art.
marriage belongs to the conjugal
120 of the Family Code. This is a case of
partnership of gains. (1%)
reverse accession, where the building is
SUGGESTED ANSWER: considered as the principal and the land,
False. In adulterous relationship, the the accessory. If, on the other hand, the
salary of a married partner belongs to value of the land is more than the value
the absolute community, or conjugal of the building, then the ordinary rule of
partnership, of such married partner accession applies where the land is the
with his or her lawful spouse. Under principal and the building, the
Articles 148 of the Family Code, the accessory. In such case, the land
property relations between married remains paraphernal property and the
partner and his/her paramour is building becomes paraphernal propery.
governed by ordinary co-ownership
Note: The rule on reverse accession is
where the partners become co-owners
applicable only to the regime of conjugal
only when they contributed to the
partnership of gains in both the Family
acquisition of the property. The
Code and the New Civil Code. The foregoing
paramour is deemed to have not
answer assumes that CPG is the regime of
contributed in the earning of the salary
the property relations of the spouses.
of the married partner.

Property Relations; Ante-Nuptial Debt


Property Relations; Accession (2012)
(2007)

No.III.(a) Maria, wife of Pedro, withdrew P 5


No. VII. Write "TRUE" if the statement is
Million from their conjugal funds. With this
true or "FALSE" if the statement is false. If
money, she constructed a building on a lot
the statement is FALSE, state the reason.
which she inherited from her father. Is the
(2% each).
building conjugal or paraphernal? Reasons.
(5%) (3). An individual, While single, purchases a
house and lot in 1990 and borrows money

“Never Let The Odds Keep You From Pursuing What You Know In Your Heart You Were Meant To Do.”-Leroy Satchel Paige
Page 40 of 180
Civil Law Q&As (2007-2013) hectorchristopher@yahoo.com dbaratbateladot@gmail.com

in 1992 to repair it. In 1995, such Property Relations; Unions Without


individual gets married while the debt is Marriage (2012)
still being paid. After the marriage, the debt
is still the responsibility of such individual. No.V. a) Spouses Primo and Monina Lim,
childless, were entrusted with the custody
SUGGESTED ANSWER: of two (2) minor children, the parents of
whom were unknown. Eager of having
FALSE. The absolute Community of children of their own, the spouses made it
property is liable for the ante-nuptial appear that they were the children’s
debts of either spouse in so far as the parents by naming them Michelle P. Lim
same redounded to the benefit of the and Michael Jude Lim. Subsequently,
family (Art. 94 par.7, FC). Monina married Angel Olario after Primo’s
death.
ALTERNATIVE ANSWER:

She decided to adopt the children by


FALSE. The debt is already the
availing the amnesty given under R.A. 8552
responsibility of the community
to those individuals who simulated the
property, because the property already
birth of a child. She filed separate petitions
constitutes absolute community
for the adoption of Michelle, then 25 years
property under Art. 91 of FC which took
old and Michael, 18. Both Michelle and
effect in 1988 while the house and lot
Michael gave consent to the adoption.
here involved was purchased in 1990.
There is no indication that the spouse The trial court dismissed the petition and
who bought the property had legitimate ruled that Monina should have filed the
descendants by a former marriage, which petition jointly with her new husband.
would exclude the house and lot from Monina, in a Motion for Reconsideration
the community property, Art. 92 par 3, argues that mere consent of her husband
FC). If the spouses established a conjugal would suffice and that joint adoption is not
partnership, the property belongs to the needed, for the adoptees are already
individual spouse if full ownership was emancipated.
vested before marriage (Art. 118, FC).
Is the trial court correct in dismissing the
petitions for adoption? Explain. (5%)

SUGGESTED ANSWER:

“Never Let The Odds Keep You From Pursuing What You Know In Your Heart You Were Meant To Do.”-Leroy Satchel Paige
Page 41 of 180
Civil Law Q&As (2007-2013) hectorchristopher@yahoo.com dbaratbateladot@gmail.com

Yes, the trial court was correct. At the Borromeo discovered that titles to the three
time the petitions for adoptions were lots have been transfereed in the name of
filed, petitioner had already remarried. Descallar. Who is the rightful owner of the
Under the law, husband and wife shall properties? Explain. (5%)
adopt jointly, except in the cases
enumerated in the law. The adoption SUGGESTED ANSWER:

cases of Michelle and James do not fall


It depends. On the assumption that the
in any of the exceptions provided in the
Family Code is the applicable law, the
law where a spouse is permitted to adopt
ownership of the properties depends on
alone. Hence, Monina should adopt
whether or not, Jambrich and Descallar
jointly with her husband Angel (Adoption
are capacitated to marry each other
of Michelle P. Lim, G.R. Nos. 168992-93,
during their cohabitation, and whether
May 21, 2009).
or not both have contributed funds for
the acquisition of the properties.

If both of them are capacitated to marry


Property Relations; Unions Without
each other, Art 147- co-ownership will
Marriage (2012)
apply to their property relations and the

No.V. b) Jambrich, an Austrian, fell in-love properties in question are owned by

and lived together with Descallar and them in equal shares even though all the

bought their houses and lots at Agro-Macro funds used in acquiring the properties

Subdivision. In the Contracts to Sell, came only from the salaries or wages, or

Jambrich and Descallar were referred to as the income of Jambrich from his

the buyers. When the Deed of Absolute Sale business or profession. In such case,

was presented for registration before the while Jambrich is disqualified to own

Register of Deeds, it was refused because any part of the properties, his

Jambrich was an alien and could not subsequent transfer of all his interest

acquire alienable lands of the public therein to Borromeo, a Filipino, was

domain. After Jambrich and Descallar valid as it removed the disqualification.

separated, Jambrich purchased an engine In such case, the properties are owned

and some accessories for his boat from by Borromeo and Descallar in equal

Borromeo. To pay for his debt, he sold his shares.

rights and interests in the Agro-Macro If, on the other hand, Jambrich and
properties to Borromeo.
Descallar were not capacitated to marry

“Never Let The Odds Keep You From Pursuing What You Know In Your Heart You Were Meant To Do.”-Leroy Satchel Paige
Page 42 of 180
Civil Law Q&As (2007-2013) hectorchristopher@yahoo.com dbaratbateladot@gmail.com

each other Art. 148-co-ownership marriage, the couple possessed the


governs their property relations. Under following properties:
this regime, Jambrich and Descallar are
co-owners of the properties but only if a house and lot acquired by B on
both of them contributed in their August 3, 1988, one third (1/3) of
acquisition. If all the funds used in the purchase price (representing
acquiring the properties in question downpayment) of which he paid; one
came from Jambrich, the entire property third (1/3) was paid by G on
is his even though he is disqualified from February 14, 1990 out of a cash gift
owning it. His subsequent transfer to given to her by her parents on her
Borromeo, however, is valid as it graduation on April 6, 1989; and the
removed the disqualification. In such balance was paid out of the spouses’
case, all the properties are owned by joint income; and
Borromeo. If, on the other hand an apartment unit donated to B by an
Descallar contributed to their uncle on June 19, 1987.
acquisition, the properties are co-owned
Who owns the foregoing properties?
by Descallar and Borromeo in proportion
Explain. (5%)
to the respective contributions of the
Descallar and Jambrich.
SUGGESTED ANSWER:
Note: The facts of the problem are not
Since the marriage was declared void ab
exactly the same as in the case of
initio in 2001, no Absolute Community
Borromeo v. Descallar, G.R. NO. 159310,
or Conjugal Partnership was ever
Feb 24, 2009, hence, the difference in
established between B and G. Their
the resulting answer.
property relation is governed by a
“special co-ownership” under Article 147
Property Relations; Void Marriages of the Family Code because they were
(2010) capacitated to marry each other.

Under that Article 147, wages and


No.VII. G and B were married on July 3,
salaries of the “former spouses” earned
1989. On March 4, 2001, the marriage,
during their cohabitation shall be owned
which bore no offspring, was declared void
by them in equal shares while properties
ab initio under Article 36 of the Family
acquired thru their work for industry
Code. At the time of the dissolution of the
shall be owned by them in proportion to

“Never Let The Odds Keep You From Pursuing What You Know In Your Heart You Were Meant To Do.”-Leroy Satchel Paige
Page 43 of 180
Civil Law Q&As (2007-2013) hectorchristopher@yahoo.com dbaratbateladot@gmail.com

their respective contributions. Care and She is an undivided co-owner to the


maintenance of the family is recognized extent for her contribution in its
as a valuable contribution. In the acquisition when she paid 1/3 of the
absence of proof as to the value of their purchase price using the gift from her
respective contributions, they shall parents. Although the gift was acquired
share equally. by G during her cohabitation with B, it is
her exclusive property. It did not consist
If ownership of the house and lot was
of wage or salary or fruit of her work or
acquired by B on August 3, 1988 at the
industry.
time he bought it on installment before
he got married, he shall remain owner of
1/3 of the house is co-owned by B and G
the house and lot but he must reimburse
because the payment came from their
G for all the amounts she advanced to
co-owned funds, i.e., their joint income
pay the purchase price and for one-half
during their cohabitation which is
share in the last payment from their
shared by them equally in the absence of
joint income. In such case, the house
any proof to the contrary.
and lot were not acquired during their
cohabitation, hence, are not co-owned by
After summing up their prospective
B and G.
shares, B and G are undivided co-owners
But if the ownership of the house and lot of the house and lot in equal shares.
was acquired during the cohabitation,
As to the apartment, it is owned
the house and lot will be owned as
exclusive by B because he acquired it
follows:
before their cohabitation. Even if he
1/3 of the house and lot is owned by B. acquired it during their cohabitation, it
He is an undivided co-owner to that will still be his exclusive property
extent for his contributions in its because it did not come from his wage or
acquisition in the form of the down salary, or from his work or industry. It
payment he made before the celebration was acquired gratuitously from his uncle.
of the marriage. The money he used to
If G and B had married on July 3, 1987 and
pay the down payment was not earned
their marriage was dissolved in 2007, who
during the cohabitation, hence, it is his
owns the properties? Explain. (5%)
exclusive property.

SUGGESTED ANSWER:
1/3 of the house and lot is owned by

“Never Let The Odds Keep You From Pursuing What You Know In Your Heart You Were Meant To Do.”-Leroy Satchel Paige
Page 44 of 180
Civil Law Q&As (2007-2013) hectorchristopher@yahoo.com dbaratbateladot@gmail.com

The answer is the same as in letter A. in Parañaque. After four (4) years or in
Since the parties to the marriage which 2001, G having completed her 4-year
was later declared void ab initio were college degree as a fulltime student, she
capacitated to marry each other, the and B contracted marriage without a
applicable law under the New Civil Code license.
was Article 144.This Article is
substantially the same as Article 147 of The marriage of B and G was, two years

the Family Code. later, declared null and void due to the
absence of a marriage license.
Hence, the determination of ownership
will remain the same as in question A. (A). If you were the judge who declared the
And even assuming that the two nullity of the marriage, to whom would you
provisions are not the same, Article 147 award the lot? Explain briefly. (3%)
of the Family Code is still the law that
will govern the property relations of B SUGGESTED ANSWER:

and G because under Article 256, the


Since the marriage was null and void, no
Family Code has retroactive effect
Absolute Community or Conjugal
insofar as it does not prejudice or impair
Partnership was established between B
vested or acquired rights under the new
and G. Their properties are governed by
Civil Code or other laws. Applying Article
the “special co-ownership” provision of
147 retroactively to the case of G and B
Article 147 of the Family Code because
will not impair any vested right. Until
both B and G were capacitated to marry
the declaration of nullity of the marriage
each other. The said Article provides
under the Family Code, B and G have not
that when a man and a woman who are
as yet acquired any vested right over the
capacitated to marry each other, live
properties acquired during their
exclusively with each other as husband
cohabitation.
and wife without the benefit of marriage,
or under a void marriage: (1) their wages
and salaries shall be owned by them in
Property Relations; Void Marriages
equal shares; and (2) property acquired
(2010)
by both of them through their work or
industry shall be governed by the rules
No.X. In 1997, B and G started living
on co-ownership. In co-ownership, the
together without the benefit of marriage.
parties are co-owners if they contributed
The relationship produced one offspring,
something of value in the acquisition of
Venus. The couple acquired a residential lot

“Never Let The Odds Keep You From Pursuing What You Know In Your Heart You Were Meant To Do.”-Leroy Satchel Paige
Page 45 of 180
Civil Law Q&As (2007-2013) hectorchristopher@yahoo.com dbaratbateladot@gmail.com

the property. Their share is in Succession


proportion to their respective
contributions. In an ordinary co- Disposition; Mortis Causa vs. Intervivos;
ownership the care and maintenance of
Corpse (2009)
the family is not recognized as a
valuable contribution for the acquisition No. XI. TRUE or FALSE. Answer TRUE if
of a property. In the Article 147 “special the statement is true, or FALSE if the
co-ownership” however, care and statement is false. Explain your answer in
maintenance is recognized as a valuable not more than two (2) sentences.
contribution which will entitle the
(E). A person can dispose of his corpse
contributor to half of the property
acquired. through an act intervivos. (1%)

Having been acquired during their SUGGESTED ANSWER:


cohabitation, the residential lot is False. A persons cannot dispose of his
presumed acquired through their joint corpse through an act inter vivos, i.e., an
work and industr under Article 147, act to take effect during his lifetime.
hence, B and G are co-owners of the said Before his death there is no corpse to
property in equal shares. dispose. But he is allowed to do so
through an act mortis causa, i.e., an act
Article 147 also provides that when a
to take effect upon his death.
party to the void marriage was in bad
faith, he forfeits his share in the co-
ownership in favor of the common
children or descendants, the default of Heirs; Fideicommissary Substitution
children or descendants, the forfeited (2008)
share shall belong to the innocent party.
In the foregoing problem, there is no No. XIII. Raymond, single, named his sister

showing that one party was in bad faith. Ruffa in his will as a devisee of a parcel of

Hence, both shall be presumed in good land which he owned. The will imposed

faith and no forfeiture shall take place. upon Ruffa the obligation of preseving the
land and transferring it, upon her death, to
her illegitimate daughter Scarlet who was
then only one year old. Raymond later died,
leaving behind his widowed mother, Ruffa
and Scarlet.

“Never Let The Odds Keep You From Pursuing What You Know In Your Heart You Were Meant To Do.”-Leroy Satchel Paige
Page 46 of 180
Civil Law Q&As (2007-2013) hectorchristopher@yahoo.com dbaratbateladot@gmail.com

(A). Is the condition imposed upon Ruffa, to Ruffa (Art. 992, Civil Code). Moreover,
preserve the property and to transmit it Scarlet is not a compulsory heir of
upon her death to Scarlet, valid? (1%) Raymond, hence she can inherit only by
testamentary succession. Since
SUGGESTED ANSWER: Raymond executed a will in the case at
bar, Scarlet may inherit from Raymond.
Yes, the condition imposed upon Ruffa
to preserve the property and to transmit
it upon her death to Scarlet is valid
because it is tantamount to Heirs; Intestate Succession; Legitime;
fideicommissary substitution under Art. Computation (2010)
863 of the Civil Code.
No.XI. The spouses Peter and Paula had
(B). If Scarlet predeceases Ruffa, who three (3) children. Paula later obtained a
inherits the property? (2%) judgment of nullity of marriage. Their
absolute community of property having
SUGGESTED ANSWER: been dissolved, they delivered P1 million to
each of their 3 children as their
Ruffa will inherit the property as
presumptive legitimes.
Scarlet's heir. Scarlet acquires a right to
the succession from the time of Peter later re-married and had two (2)
Raymond's death, even though she children by his second wife Marie. Peter
should predecease Ruffa (Art. 866, Civil and Marie, having successfully engaged in
Code). business, acquired real properties. Peter
later died intestate.
(C). If Ruffa predeceases Raymond, can
Scarlet inherit the property directly from (A). Who are Peter’s legal heirs and how will
Raymond? (2%) his estate be divided among them? (5%)

SUGGESTED ANSWER: SUGGESTED ANSWER:

If Ruffa predeceases Raymond, The legal heirs of Peter are his children
Raymond's widowed mother will be by the first and second marriages and his
entitled to the inheritance. Scarlet, an surviving second wife.
illegitimate child, cannot inherit the
Their shares in the estate of Peter will
property by intestate succession from
depend, however, on the cause of the
Raymond who is a legitimate relative of

“Never Let The Odds Keep You From Pursuing What You Know In Your Heart You Were Meant To Do.”-Leroy Satchel Paige
Page 47 of 180
Civil Law Q&As (2007-2013) hectorchristopher@yahoo.com dbaratbateladot@gmail.com

nullity of the first marriage. If the If the ground of nullity is not


nullity of the first marriage was psychological capacity:
psychological incapacity of one or both 2 legitimate ¼ of the estate for
spouses, the three children of that void children each of second
marriage are legitimate and all of the marriage
legal heirs shall share the estate of Peter
Surviving ¼ of the estate
in equal shares. If the judgment of
second spouse
nullity was for other causes, the three
children are illegitimate and the estate 3 illegitimate 1/12 of estate for

shall be distributed such that an children each of first marriage

illegitimate child of the first marriage


shall receive half of the share of a
Note: The legitime of an illegitimate
legitimate child of the second marriage,
child is supposed to be ½ the legitime of
and the second wife will inherit a share
a legitimate child or 1/8 of the estate.
equal to that of a legitimate child. In no
But the estate will not be sufficient to
case may the two legitimate children of
pay the said legitime of the 3
the second marriage receive a share less
illegitimate children, because only ¼ of
than one-half of the estate which is their
the estate is left after paying the
legitime. When the estate is not
legitime of the surviving spouse which is
sufficient to pay all the legitimes of the
preferred.
compulsory heirs, the legitime of the
spouse is preferred and the illegitimate Hence, the remaining ¼ of the estate
children suffer the reduction. shall be divided among the 3 illegitimate
children.
Computation:

If the ground of nullity is psychological (B). What is the effect of the receipt by

incapacity: Peter’s 3 children by his first marriage of


their presumptive legitimes on their right to
3 children by first 1/6 of the estate
inherit following Peter’s death? (5%)
marriage for each

2 children by second 1/6 of the estate SUGGESTED ANSWER:


marriage for each
In the distribution of Peter’s estate, ½ of
Surviving second 1/6 of the estate
the presumptive received by the 3
spouse
children of the first marriage shall be
collated to Peter’s estate and shall be

“Never Let The Odds Keep You From Pursuing What You Know In Your Heart You Were Meant To Do.”-Leroy Satchel Paige
Page 48 of 180
Civil Law Q&As (2007-2013) hectorchristopher@yahoo.com dbaratbateladot@gmail.com

imputed as an advance of their Art 992 of the NCC, an illegitimate child


respective inheritance from Peter. Only has no right to inherit ab intestato from
half of the presumptive legitime is the legitimate children and relatives of
collated to the estate of Peter because his father or mother. Arnel is
the other half shall be collated to the disqualified to inherit from Ricky
estate of his first wife. because Arnel is an illegitimate child of
Franco and Ricky is a legitimate relative
of Franco.

Heirs; Representation; Iron-Curtain Rule

(2012)
Heirs; Reserva Troncal (2009)
No.VIII.a) Ricky and Arlene are married.
They begot Franco during their marriage. No. I. TRUE or FALSE. Answer TRUE if the
Franco had an illicit relationship with statement is true, or FALSE if the
Audrey and out of which, they begot Arnel. statement is false. Explain your answer in
Frnaco predeceased Ricky, Arlene and not more than two (2) sentences.
Arnel. Before Ricky died, he executed a will
which when submitted to probate was (B).In reservatroncal, all reservatarios (reser

opposed by Arnel on the ground that he vees) inherit as a class and in equal shares

should be given the share of his father, regardless of their proximity in degree to

Franco. Is the opposition of Arnel correct? the prepositus. (1%)

Why? (5%)
SUGGESTED ANSWER:

SUGGESTED ANSWER: FALSE. Not all the relatives within the


third degree will inherit as reservatario ,
No, his opposition is not correct. Arnel and not all those who are entitled to
cannot inherit from Ricky in the inherit will inherit in the equal shares .
representation of his father Franco. In The applicable laws of intestate
representation, the representative must succession will determine who among
not only be a legal heir of the person he the relatives will inherit as reservatarios
is representing, he must also be a legal and what shares they will tak, i.e., the
heir of the decedent he seeks to inherit direct line excludes the collateral, the
from. descending direct line excludes the

While Arnel is a legal heir of Franco, he ascending ,the nearer excludes the more
remote, the nephews and nieces exclude
is not a legal heir of Ricky because under

“Never Let The Odds Keep You From Pursuing What You Know In Your Heart You Were Meant To Do.”-Leroy Satchel Paige
Page 49 of 180
Civil Law Q&As (2007-2013) hectorchristopher@yahoo.com dbaratbateladot@gmail.com

the uncles and the aunts, and half blood (1). The wife of Ramon will, therefore,
relatives inherit half the share of full- receive one half (½) of the estate or the

blooded relatives. amount of P5,000,000.00.


(2). The three (3) full-blood brothers, will,
therefore, receive P1,000,000.00 each.
(3). The nephew will receive
Intestate Succession (2008)
P1,000,000.00 by right of
representation.
No. VII. Ramon Mayaman died intestate,
(4). The two (2) half-brothers will receive
leaving a net estate of P10,000,000.00.
P500,000.00 each.
Determine how much each heir will receive
from the estate:
(B). If Ramon is survived by his wife, a half-
sister, and three nephews (sons of a
(A). If Ramon is survived by his wife, three
deceased full-blood brother)? Explain. (3%)
full-blood brothers, two half-brothers, and
one nephew (the son of a deceased full-
SUGGESTED ANSWER:
blood brother)? Explain. (3%)
The wife will receive one half (1/2) of the
estate or P5,000,000.00. The other half
SUGGESTED ANSWER:
shall be inherited by (1) the full-blood
Having died intestate, the estate of
brother, represented by his three
Ramon shall be inherited by his wife and
children, and (2) the half-sister. They
his full and half blood siblings or their
will divide the other half between them
respective representatives. In intestacy,
such that the share of the half-sister is
if the wife concurs with no one but the
just half the share of the full-blood
siblings of the husband, all of them are
brother. The share of the full-blood
the intestate heirs of the deceased
brother shall in turn be inherited by the
husband. The wife will receive half of the
three nephews in equal shares by right of
intestate estate, while the siblings or
presentation.
their respective representatives, will
inherit the other half to be divided
Therefore, the three (3) nephews will
among them equally. If some siblings are
receive P1,111,111.10 each the half-
of the full-blood and the other of the half
sister will receive the sum of
blood, a half blood sibling will receive
P1,666,666.60.
half the share of a full-blood sibling.

“Never Let The Odds Keep You From Pursuing What You Know In Your Heart You Were Meant To Do.”-Leroy Satchel Paige
Page 50 of 180
Civil Law Q&As (2007-2013) hectorchristopher@yahoo.com dbaratbateladot@gmail.com

Intestate Succession (2008) (D). How should the house and lot, and the
cash be distributed? (1%)
No.X. Arthur executed a will which contained
only: (i) a provision disinheriting his daughter SUGGESTED ANSWER:
Bernica for running off with a married man,
and (ii) a provision disposing of his share in Since the probate of the will cannot be

the family house and lot in favor of his other allowed, the rules on intestate

children Connie and Dora. He did not make succession apply. Under Art. 996 of the

any provisions in favor of his wife Erica, Civil Code, if a widow or widower and

because as the will stated, she would anyway legitimate children or descendants are

get ½ of the house and lot as her conjugal left, the surviving spouse has the same

share. The will was very brief and share as of the children. Thus, ownership

straightforward and both the above provisions over the house and lot will be created

were contained in page 1, which Arthur and among wife Erica and her children

his instrumental witness, signed at the Bernice, Connie and Dora. Similarly, the

bottom. Page 2 contained the attestation amount of P 1 million will be equally

clause and the signatures, at the bottom divided among them.

thereof, of the 3 instrumental witnesses which


included Lambert, the driver of Arthur; Yoly,
the family cook, and Attorney Zorba, the
Intestate Succession; Rights of
lawyer who prepared the will. There was a 3rd
Representation: Illegitimate, Adopted
page, but this only contained the notarial
Child; Iron Curtain Rule (2007)
acknowledgement. The attestation clause
stated the will was signed on the same No. X. For purpose of this question, assume
occasion by Arthur and his instrumental all formalities and procedural requirements
witnesses who all signed in the presence of have been complied with.
each other, and the notary public who
notarized the will. There are no marginal In 1970, Ramon and Dessa got married.
signatures or pagination appearing on any of Prior to their marriage, Ramon had a child,
the 3 pages. Upon his death, it was discovered Anna. In 1971 and 1972, Ramon and Dessa
that apart from the house and lot, he had a P legally adopted Cherry and Michelle
1 million account deposited with ABC bank. respectively. In 1973, Dessa died while
giving birth to Larry Anna had a child, Lia.
Anna never married. Cherry, on the other
hand, legally adopted Shelly. Larry had

“Never Let The Odds Keep You From Pursuing What You Know In Your Heart You Were Meant To Do.”-Leroy Satchel Paige
Page 51 of 180
Civil Law Q&As (2007-2013) hectorchristopher@yahoo.com dbaratbateladot@gmail.com

twins, Hans and Gretel, with his girlfriend, also of the person from whom the person
Fiona. In 2005, Anna, Larry and Cherry being represented was supposed to
died in a car accident. In 2007, Ramon inherit. While Shelly is a legal heir of
died. Who may inherit from Ramon and Cherry, Shelly is not a legal heir of
who may not? Give your reason briefly. Ramon. Adoption created a purely
(10%) personal legal relation only between
Cherry and Shelly.
SUGGESTED ANSWER:
(2). Hans and Gretel are barred from
The following may inherit from Ramon: inheriting from Ramon under Art. 992,
NCC. Being illegitimate children, they
(1). Michelle, as an adopted child of
cannot inherit ab intestao from Ramon.
Ramon, will inherit as a legitimate child
of Ramon. As an adopted child, Michelle ALTERNATIVE ANSWER:
has all the rights of a legitimate child
(Sec 18, Domestic Adoption Law). The problem expressly mentioned the
dates of the adoption of Cherry and
(2). Lia will inherit in representation of Michelle as 1971 and 1972. During that
Anna. Although Lia is an illegitimate time, adoption was governed by the New
child, she is not barred by Articles 992, Civil Code. Under the New Civil Code,
because her mother Anna is an husband and wife were allowed to adopt
illegitimate herself. She will represent separately or not jointly with the other
Anna as regards Anna's legitime under spouse. And since the problem does not
Art. 902, NCC and as regards Anna's specifically and categorically state, it is
intestate share under Art. 990, NCC. possible to construe the use of the word
"respectively" in the problem as
The following may not inherit from
indicative of the situation that Cherry
Ramon:
was adopted by Ramon alone and
Michelle was adopted by Dessa alone. In
(1). Shelly, being an adopted child, she
such case of separate adoption the
cannot represent Cherry. This is because
alternative answer to the problem will be
adoption creates a personal legal relation
as follows: Only Lia will inherit from
only between the adopter and the
Ramon in representation of Ramon's
adopted. The law on representation
illegitimate daughter Anna. Although Lia
requires the representative to be a legal
is an illegitimate child, she is not barred
heir of the person he is representing and
from inheriting from Ramon because her

“Never Let The Odds Keep You From Pursuing What You Know In Your Heart You Were Meant To Do.”-Leroy Satchel Paige
Page 52 of 180
Civil Law Q&As (2007-2013) hectorchristopher@yahoo.com dbaratbateladot@gmail.com

mother is herself illegitimate. Shelly SUGGESTED ANSWER:


cannot inherit in representation of
A testator may dispose of by will the free
Cherry because Shelly is just an adopted
portion of his estate. Since the legitime
child of Cherry. In representation, the
of JCP is 1/8 of the estate, SGO is ¼ of
representative must not only be a legal
the estate and that of HBR and RVC is ½
heir of the person he is representing but
of the hereditary estate under Art 889 of
also of the decedent from whom the
the NCC, the remaining 1/8 of the estate
represented person is supposed to
is the free portion which the testator
inherit. In the case of Shelly, while she
may dispose of by will.
is a legal heir of Cherry by virtue of
adoption, she is not a legal heir of
Ramon. Adoption creates a personal
legal relation only between the adopting Legitime; Compulsory Heirs (2008)
parent and the adopted child (Teotico v.
Del Val, 13 SCRA 406, 1965. Michelle No. XII. Ernesto, an overseas Filipino
cannot inherit from Ramon, because she worker, was coming home to the Philippines
was adopted not by Ramon but by Dessa. after working for so many years in the
In the eyes of the law, she is not related Middle East. He had saved P100.000 in his
to Ramon at all. Hence, she is not a legal saving account in Manila which intended to
heir of Ramon. Hans and Gretel are not use to start a business in his home
entitled to inherit from Ramon, because country. On his flight home, Ernesto had a
they are barred by Art. 992 NCC. Being fatal heart attack. He left behind his
illegitimate children of Larry, they widowed mother, his common-law wife and
cannot inherit from the legitimate their twins sons. He left no will, no debts,
relatives of their father Larry. Ramon is no other relatives and no other properties
a legitimate relative of Larry who is the except the money in his saving account.
legitimate father. Who are the heirs entitled to inherint from
him and how much should each receive?
(3%)

Legitimes; Compulsory Heirs (2012) SUGGESTED ANSWER:

No.VIII.b) How can RJP distribute his estate The mother and twin sons are entitled to
by will, if his heirs are JCP, his wife; HBR inherit from Ernesto. Art. 991 of the
and RVC, his parents; and an illegitimate Civil Code, provides that if legitimate
child, SGO?

“Never Let The Odds Keep You From Pursuing What You Know In Your Heart You Were Meant To Do.”-Leroy Satchel Paige
Page 53 of 180
Civil Law Q&As (2007-2013) hectorchristopher@yahoo.com dbaratbateladot@gmail.com

ascendants are left, the twin sons shall The attestation clause stated the will was
divide the inheritance with them taking signed on the same occasion by Arthur and
one-half of the estate. Thus, the widowed his instrumental witnesses who all signed
mother gets P50,000.00 while the twin in the presence of each other, and the
sons shall receive P25,000.00 each. The notary public who notarized the will. There
common-law wife cannot inherit from are no marginal signatures or pagination
him because when the law speaks "widow appearing on any of the 3 pages. Upon his
or widower" as a compulsory heir, the death, it was discovered that apart from the
law refers to a legitimate spouse (Art. house and lot, he had a P 1 million account
887, par 3, Civil Code). deposited with ABC bank.

(A). Was Erica preterited? (1%)

Preterition; Disinheritance (2008) SUGGESTED ANSWER:

No.X. Arthur executed a will which contained Erica cannot be preterited. Art. 854 of
only: (i) a provision disinheriting his daughter the Civil Code provides that only
Bernica for running off with a married man, compulsory heirs in the direct line can
and (ii) a provision disposing of his share in be preterited.
the family house and lot in favor of his other
children Connie and Dora. He did not make (B). What other defects of the will, if any,

any provisions in favor of his wife Erica, can cause denial of probate? (2%)

because as the will stated, she would anyway


SUGGESTED ANSWER:
get ½ of the house and lot as her conjugal
share. The will was very brief and
The other defects of the will that can
straightforward and both the above provisions
cause its denial are as follows: (a) Atty.
were contained in page 1, which Arthur and
Zorba, the one who prepared the will was
his instrumental witness, signed at the
one of the three witnesses, violating the
bottom. Page 2 contained the attestation
three-witnesses rule; (b) no marginal
clause and the signatures, at the bottom
signature at the last page; (c ) the
thereof, of the 3 instrumental witnesses which
attestation did not state the number of
included Lambert, the driver of Arthur; Yoly,
pages upon which the will is written;
the family cook, and Attorney Zorba, the
and, (d) no pagination appearing
lawyer who prepared the will. There was a 3rd
correlatively in letters on the upper part
page, but this only contained the notarial
of the three pages (Azuela v. C.A., G.R.
acknowledgement.

“Never Let The Odds Keep You From Pursuing What You Know In Your Heart You Were Meant To Do.”-Leroy Satchel Paige
Page 54 of 180
Civil Law Q&As (2007-2013) hectorchristopher@yahoo.com dbaratbateladot@gmail.com

No. 122880, 12 Apr 2006 and cited cases (B). Between Marian and the baby, who is
therein, Art 805 and 806, Civil Code). presumed to have died ahead? (1%)

(C). Was the disinheritance valid? (1%) SUGGESTED ANSWER:

SUGGESTED ANSWER: Marian is presumed to have died ahead


of the baby. Art. 43 applies to persons
Yes, the disinheritance was valid. Art. who are called to succeed each other.
919, par 7, Civil Code provides that The proof of death must be established
"when a child or descendant leads a by positive or circumstantial evidence
dishonorable or disgraceful life, like derived from facts. It can never be
running off with a married man, there is established from mere inference. In the
sufficient cause for disinheritance." present case, it is very clear that only
Marian and Pietro were hacked with
bolos. There was no showing that the
baby was also hacked to death. The
Succession; Proof of Death between
baby's death could have been due to lack
persons called to succeed each other
of nutrition.
(2008)

ALTERNATIVE ANSWER:
No. II. At age 18, Marian found out that she
was pregnant. She insured her own life and
The baby is presumed to have died ahead
named her unborn child as her sole
of Marian. Under Par. 5, rule 131, Sec. 5
beneficiary. When she was already due to
of the Rules of Court, if one is under 15
give birth, she and her boyfriend Pietro, the
or above 60 and the age of the other is in
father of her unboarn child, were
between 15 and 60, the latter is
kidnapped in a resort in Bataan where they
presumed to have survived. In the
were vacationing. The military gave chase
instant case, Marian was already 18
and after one week, they were found in an
when she found out that she was
abandoned hut in Cavite. Marian and Pietro
pregnant. She could be of the same age
were hacked with bolos. Marian and the
or maybe 19 years of age when she gave
baby delivered were both found dead, with
birth.
the baby's umbilical cord already cut. Pietro
survived. (C). Will Pietro, as surviving biological
father of the baby, be entitled to claim the

“Never Let The Odds Keep You From Pursuing What You Know In Your Heart You Were Meant To Do.”-Leroy Satchel Paige
Page 55 of 180
Civil Law Q&As (2007-2013) hectorchristopher@yahoo.com dbaratbateladot@gmail.com

proceeds of the life insurance on the life of


Marian? (2%) Marilyn is not entitled to a share in the
estate of Dr. Lopez. For purpose of
SUGGESTED ANSWER: succession, Dr. Lopez and his son
Roberto are presumed to have died at
Pietro, as the biological father of the
the same time, there being no evidence
baby, shall be entitled to claim the
to prove otherwise, and there shall be no
proceeds of life insurance of the Marian
transmission of rights from one to the
because he is a compulsory heir of his
other (Article 43, NCC). Hence, Roberto,
child.
inherited nothing from his father that
Marilyn would in turn inherit from
Roberto .The children of Roberto,

Succession; Rule on Survivorship (2009) however, will succeed their grandfather,


Dr. Lopez ,in representation of their
No. II. Dr. Lopez, a 70-year old widower, father Roberto and together Roberto will
and his son Roberto both died in a fire that receive 1/3 of the estate of Dr. Lopez
gutted their home while they were sleeping since their father Roberto was one of the
in their air-conditioned rooms. Roberto’s three children of Dr. Lopez . Marilyn
wife, Marilyn, and their two children were cannot represent her husband Roberto
spared because they were in the province at because the right is not given by the law
the time. Dr. Lopez left an estate worth to a surviving spouse.
P20M and a life insurance policy in the
amount of P1M with his three children --- As to the proceeds of the insurance on
one of whom is Roberto --- as beneficiaries. the life of Dr. Lopez:

Marilyn is now claiming for herself and her Since succession is not involved as
children her husband’s share in the estate regards the insurance contract, the
left by Dr. Lopez, and her husband’s share provisions of the Rules of Court (Rule
in the proceeds of Dr. Lopez’s life insurance 131, Sec. 3 , [jj] [5] ) on survivorship
policy. Rule on the validity of Marilyn’s shall apply. Under the Rules, Dr. Lopez,
claims with reasons. (4%) who was 70 years old, is presumed to
have died ahead of Roberto who is
SUGGESTED ANSWER :
presumably between the ages 15 and 60.
Having survived the insured, Roberto's
As to the Estate of Dr. Lopez:
right as a beneficiary became vested

“Never Let The Odds Keep You From Pursuing What You Know In Your Heart You Were Meant To Do.”-Leroy Satchel Paige
Page 56 of 180
Civil Law Q&As (2007-2013) hectorchristopher@yahoo.com dbaratbateladot@gmail.com

upon the death of Dr. Lopez. When should be given effect must be denied.
Roberto died after Dr. Lopez, his right to The said cancellation has revoked the
receive the insurance became part of his entire will as nothing remains of the will
hereditary estate, which in turn was after the name of Rosa was cancelled.
inherited in equal shares by his legal Such cancellation is valid revocation of
heirs, namely, his spouse and children. the will and does not require
Therefore, Roberto's children and his authentication by the full signature of
spouse are entitled to Roberto's one- the testator to be effective.
third share in the insurance proceeds.
However, if the cancellation of Rosa’s
name was not done by the testator
himself, such cancellation shall not be
effective and the will in its original tenor
Wills; Holographic Wills; Insertions & shall remain valid. The effectively of the
Cancellations (2012) holographic will cannot be left to the
mercy of unscrupulous third parties.
No.VII.a) Natividad’s holographic will, which
The writing of Gregorio’s name as sole
had only one (1) substantial provision, as
heir was ineffective, even though written
first written, named Rosa as her sole heir.
by the testator himself, because such is
However, when Gregorio presented it for
an alteration that requires
probate, it already contained an alteration,
authentication by the full signature of
naming Gregorio, instead of Rosa, as sole
heir, but without authentication by the testator to be valid and effective. Not

Natividad’s signature. Rosa opposes the having an authenticated, the designation

probate alleging such lack of proper of Gregorio as an heir was ineffective,

authentication. She claims that the (Kalaw v. Relova, G.R. No. L-40207, Sept

unaltered form of the will should be given 28, 1984).

effect. Whose claim should be granted?


Explain. (5%)

Wills; Holographic Wills; Probate (2009)


SUGGESTED ANSWER:

It depends. If the cancellation of Rosa’s No.VI. On December 1, 2000, Dr. Juanito

name in the will was done by the Fuentes executed a holographic will,

testator himself, Rosa’s claimed that the wherein he gave nothing to his recognized

holographic will in its original tenor illegitimate son, Jay. Dr. Fuentes left for the

“Never Let The Odds Keep You From Pursuing What You Know In Your Heart You Were Meant To Do.”-Leroy Satchel Paige
Page 57 of 180
Civil Law Q&As (2007-2013) hectorchristopher@yahoo.com dbaratbateladot@gmail.com

United States, passed the New York medical court shall apply the New Civil Code in
licensure examinations, resided therein, determining the formal validity of the
and became a naturalized American citizen. holographic will. The subsequent change
He died in New York in 2007. The laws of in the citizenship of Dr. Fuentes did not
New York do not recognize holographic wills affect the law governing the validity of
or compulsory heirs. his will. Under the new Civil Code, which
was the law used by Dr. Fuentes, the law
(A). Can the holographic will of Dr. Fuentes
enforced at the time of execution of the
be admitted to probate in the Philippines? will shall govern the formal validity of
Why or why not? (3%) the will (Art. 795, NCC).

SUGGESTED ANSWER:
(B). Assuming that the will is probated in
Yes, the holographic will of Dr. Fuentes
the Philippines, can Jay validly insist that
may be admitted to probate in the
he be given his legitime? Why or why not?
Philippines because there is no public
(3%)
policy violated by such probate. The only
issue at probate is the due execution of SUGGESTED ANSWER:
the will which includes the formal No, Jay cannot insist because under New
validity of the will. As regards formal York law he is not a compulsory heir
validity, the only issue the court will entitled to a legitime.
resolve at probate is whether or not the
will was executed in accordance with the The national law of the testator

form prescribed by the law observed by determines who his heirs are, the order

the testator in the execution of his will. that they succeed, how much their

For purposes of probate in the successional rights are, and whether or

Philippines, an alien testator may not a testamentary disposition in his will

observe the law of the place where the is valid (Art 16, NCC). Since, Dr. Fuentes

will was executed (Art 17, NCC), or the was a US citizen, the laws of the New

formalities of the law of the place where York determines who his heirs are. And

he resides, or according to the since the New York law does not

formalities of the law of his own country, recognize the concept of compulsory
or in accordance with the Philippine heirs, Jay is not a compulsory heir of Dr.
Civil Code (Art. 816, NCC). Since Dr. Fuentes entitled to a legitime.
Fuentes executed his will in accordance
with the Philippine law, the Philippine

“Never Let The Odds Keep You From Pursuing What You Know In Your Heart You Were Meant To Do.”-Leroy Satchel Paige
Page 58 of 180
Civil Law Q&As (2007-2013) hectorchristopher@yahoo.com dbaratbateladot@gmail.com

Wills; Joint Wills (2008) SUGGESTED ANSWER:

No. XI. John and Paula, British citizens at No. The testamentary dispositions are
birth, acquired Philippine citizenship by not valid because (a) omission of Mary, a
naturalization after their marriage. During legitimate child, is tantamount to
their marriage the couple acquired preterition which shall annul the
substanial landholdings in London and in institution of Peter and Paul as heirs
Makati. Paula bore John three children, (Art. 854, Civil Code); and, (b) the
Peter, Paul and Mary. In one of their trips disposition that Peter and Paul could not
to London, the couple executed a joint will dispose of nor divide the London estate
appointing each other as their heirs and for more than 20 years is void (Art. 870,
providing that upon the death of the Civil Code).
survivor between them the entire estate
would go to Peter and Paul only but the two
could not dispose of nor divide the London
Wills; Joint Wills; Probate (2012)
estate as long as they live. John and Paul
died tragically in the London Subway
No.VII.b) John Sagun and Maria Carla
terrorist attack in 2005. Peter and Paul
Camua, British citizens at birth, acquired
filed a petition for probate of their parent's
Philippine citizenship by naturalization
will before a Makati Regional Trial Court.
after their marriage. During their marriage,
the couple acquired substantial
(A). Should the will be admitted to probate?
landholdings in London and in Makati.
(2%)
Maria begot three (3) children, Jorge,
SUGGESTED ANSWER: Luisito, and Joshur. In one of their trips to
London, the couple executed a joint will
No. The will cannot be admitted to appointing each other as their heirs and
probate because a joint will is expressly providing that upon the death of the
prohibited under Art. 818 of the Civil survivor between them, the entire estate
Code. This provision applies John and would go to Jorge and Luisito only but the
Paula became Filipino citizens after their two (2) could not dispose of nor divide the
marriage. London estate as long as they live. John
and Maria died tragically in the London
(B). Are the testamentary dispositions subway terrorist attack in 2005. Jorge and
valid? (2%) Luisito filed a petition for probate of their
parents’ will before a Makati Regional Trial

“Never Let The Odds Keep You From Pursuing What You Know In Your Heart You Were Meant To Do.”-Leroy Satchel Paige
Page 59 of 180
Civil Law Q&As (2007-2013) hectorchristopher@yahoo.com dbaratbateladot@gmail.com

Court. Joshur vehemently objected because SUGGESTED ANSWER:


he was preterited.
Assuming the will of John and Maria was

Should the will be admitted to probate? valid, the testamentary prohibition on the
division of the London estate shall be valid
Explain. (2%)
but only for 20 years. Under Arts 1083 and
SUGGESTED ANSWER: 494 of the NCC, a testamentary disposition
of the testator cannot forbid the partition of
No, the will should not be admitted to
all or part of the estate for a period longer
probate. Since the couples are both
than twenty (20) years.
Filipino citizens, Art 818 and 819 of the
NCC shall apply. Said articles prohibits
the execution of joint wills and make
them void, even though authorized of Wills; Prohibition to Partition of a Co-

the country where they were executed. Owned Property (2010)

Are the testamentary dispositions valid? No.I. True or False.


Explain. (2%)
X, a widower, died leaving a will stating
SUGGESTED ANSWER: that the house and lot where he lived
cannot be partitioned for as long as the
Since the joint will is void, all the
youngest of his four children desires to stay
testamentary disposition written therein
there. As coheirs and co-owners, the other
are also void. However, if the will is
three may demand partition anytime. (1%)
valid, the institutions of the heirs shall
be annulled because Joshur was SUGGESTED ANSWER:
preterited. He was preterited because he
will receive nothing from the will, will FALSE, The other three co – heirs may

receive nothing in testacy, and the facts not anytime demand the partition of the

do not show that he received anything as house and lot since it was expressly

an advance on his inheritance. He was provided by the decedent in his will that

totally excluded from the inheritance of the same cannot be partitioned while his

his parents. youngest child desires to stay there.


Article 1083 of the New Civil Code allows
Is the testamentary prohibition against the a decedent to prohibit, by will, the
division of the London estate valid? partition of a property and his estate for
Explain. (1%) a period not longer than 20 years no

“Never Let The Odds Keep You From Pursuing What You Know In Your Heart You Were Meant To Do.”-Leroy Satchel Paige
Page 60 of 180
Civil Law Q&As (2007-2013) hectorchristopher@yahoo.com dbaratbateladot@gmail.com

matter what his reason maybe. Hence, (B). Act as a witness to a will? (1%)
the three co-heir cannot demand its
partition at anytime but only after 20 SUGGESTED ANSWER:

years from the death of their father.


Stevie cannot be a witness to a will. Art.
Even if the deceased parent did not leave
820 of the Civil Code provides that "any
a will, if the house and lot constituted
person of sound mind and of the age of
their family home, Article 159 of the
eighteen years or more, and not blind,
Family Code prohibits its partition for a
deaf or dumb, and able to read and write,
period of ten (10) years, or for as long as
may be a witness to the execution of a
there is a minor beneficiary living in the
will.
family home.

(C). In either of the above instances, must


the will be read to him? (1%)
Wills; Notarial Wills; Blind Testator;
SUGGESTED ANSWER:
Requisites (2008)

If Stevie makes a will, the will must be


No. XIV. Stevie was born blind. He went to
read to him twice, once by one of the
school for the blind, and learned to read in
subscribing witnesses, and again, by the
Baille Language. He Speaks English
notary public before whom the will is
fluently. Can he:
acknowledged (Art. 808, Civil Code).
(A). Make a will? (1%)

SUGGESTED ANSWER:
Wills; Testamentary Disposition; Period

Assuming that he is of legal age (Art. to Prohibit Partition (2008)


797, Civil Code) and of sound mind at
No. XI. John and Paula, British citizens at
the time of execution of the will (Art.
birth, acquired Philippine citizenship by
798, Civil Code), Stevie, a blind person,
naturalization after their marriage. During
can make a notarial will, subject to
their marriage the couple acquired
compliance with the "two-reading rule"
substanial landholdings in London and in
(Art. 808, Civil Code) and the provisions
Makati. Paula bore John three children,
of Arts. 804, 805 and 806 of the Civil
Peter, Paul and Mary. In one of their trips
Code.
to London, the couple executed a joint will
appointing each other as their heirs and

“Never Let The Odds Keep You From Pursuing What You Know In Your Heart You Were Meant To Do.”-Leroy Satchel Paige
Page 61 of 180
Civil Law Q&As (2007-2013) hectorchristopher@yahoo.com dbaratbateladot@gmail.com

providing that upon the death of the that she can sign her full name later. While
survivor between them the entire estate the will was being signed, Roberta
would go to Peter and Paul only but the two experienced a stomach ache and kept going
could not dispose of nor divide the London to the restroom for long periods of time.
estate as long as they live. John and Paul Hannah, while waiting for her turn to sign
died tragically in the London Subway the will, was reading the 7th Harry Potter
terrorist attack in 2005. Peter and Paul book on the couch, beside the table on
filed a petition for probate of their parent's which everyone was signing. Benjamin,
will before a Makati Regional Trial Court. aside from witnessing the will, also offered
to notarize it. A week after, Clara was run
(C). Is the testamentary prohibition against over by a drunk driver while crossing the
the division of the London estate valid? (2%) street in Greenbelt.

SUGGESTED ANSWER: May the will of Clara be admitted to


probate? Give your reasons briefly. (10%)
No. the testamentary prohibition against
the division of the London estate is void SUGGESTED ANSWER:
(Art. 870, Civil Code). A testator,
however, may prohibit partition for a Probate should be denied. The
period which shall not exceed twenty requirement that the testator and at
years (Art. 870 in relation to Art. 494, least three (3) witnesses must sign all in

par 3, Civil Code). the "presence" of one another was not


complied with. Benjamin who notarized
the will is disqualified as a witness,
hence he cannot be counted as one of
Wills; Witnesses to a Will, Presence
the three witnesses (Cruz v. Villasor, 54
required; Thumbmark as Signature
SCRA 31, 1973). The testatrix and the
(2007) other witnesses signed the will not in
the presence of Roberta because she was
No.VI. Clara, thinking of her mortality,
in the restroom for extended periods of
drafted a will and asked Roberta, Hannah,
time. Inside the restroom, Roberta could
Luisa and Benjamin to be witnesses.
not have possibly seen the testatrix and
During the day of signing of her will, Clara
the other witnesses sign the will by
fell down the stairs and broke her arms.
merely casting her eyes in the proper
Coming from the hospital, Clara insisted on
direction (Jaboneta v. Gustilo, 5 Phil
signing her will by thumb mark and said
541, 1906; Nera v. Rimando, 18 Phil

“Never Let The Odds Keep You From Pursuing What You Know In Your Heart You Were Meant To Do.”-Leroy Satchel Paige
Page 62 of 180
Civil Law Q&As (2007-2013) hectorchristopher@yahoo.com dbaratbateladot@gmail.com

451, 1914). Therefore, the testatrix Because the Picasso painting reminded
signed the will in the presence of only Angie of him, Brad in his will bequeathed
two witnesses, and only two witnesses the painting to Angie. Brad died in 1995.
signed the will in the presence of the Saddened by Brad's death, Jennifer asked
testatrix and of one another. for the Picasso painting as a remembrance
of him. Angie refused and claimed that
It is to be noted, however, that the Brad, in his will, bequeathed the painting to
thumb mark intended by the testator to her. Is Angie correct? Why or why not?
be his signature in executing his last will (10%)
and testament is valid (Payad v.
Tolentino, 62 Phil 848, 1936; Matias v. SUGGESTED ANSWER:
Salud, L-104 Phil 1046, 23 June, 1958).
The problem, however, states that Clara NO. Angie is not correct. The Picasso

"said that she can sign her full name painting is not given or donated by

later;" Hence, she did not consider her Jennifer to Brad. She merely "placed it

thumb mark as her "complete" signature, in his bedroom." Hence, she is still the

and intended further action on her part. owner of the painting. Not being the

The testatrix and the other witness owner of the Picasso painting, Brad

signed the will in the presence of cannot validly bequeath the same to

Hannah, because she was aware of her Angie (Art. 930, NCC). Even assuming

function and role as witness and was in a that the painting was impliedly given or

position to see the testatrix and the donated by Jennifer to Brad, the

other witnesses sign by merely casting donation is nevertheless void for not

her eyes in the proper direction. being in writing. The Picasso painting
must be worth more than 5,000 pesos.
Donation
Under Art. 748, NCC, the donation and
acceptance of a movable worth more
Donations; Formalities; In Writing (2007)
than 5,000 pesos must be in writing,

No. VIII. In 1986, Jennifer and Brad were otherwise the donation is void. The

madly in love. In 1989, because a certain donation being void, Jennifer remained

Picasso painting reminded Brad of her, the owner of the Picasso painting and

Jennifer acquired it and placed it in his Brad could not have validly disposed of

bedroom. In 1990, Brad and Jennifer broke said painting in favor of Angie in his will.

up. While Brad was mending his broken


ALTERNATIVE ANSWER:
heart, he met Angie and fell in love.

“Never Let The Odds Keep You From Pursuing What You Know In Your Heart You Were Meant To Do.”-Leroy Satchel Paige
Page 63 of 180
Civil Law Q&As (2007-2013) hectorchristopher@yahoo.com dbaratbateladot@gmail.com

YES. Angie is correct. Even assuming illegal and impossible donations imposed
that there was void donation because the in an onerous donation shall annul the
same was not in writing, Brad was in donation (Art. 1183, NCC). This is so,
uninterrupted possession of the Picasso because onerous donations are governed
painting from 1989 to 1995, lasting for by the law on contracts (Art. 733, NCC).
six (6) years prior to his death. Brad has
already acquired ownership of the
painting through acquisitive
Donation; Inter Vivos (2013)
prescription. Under Art. 1132, NCC,
ownership of movables prescribes
No.V. Josefa executed a deed of donation
through continuous possession for four
covering a one-hectare rice land in favor of
years in good faith and for eight (8) years
her daughter, Jennifer. The deed
without need of other conditions. A void
specifically provides that:
donation may be the basis of possession
in the concept of owner and of just title "For and in consideration of her love
for purposes of acquisitive prescription. and service Jennifer has shown and
given to me, I hereby freely,
voluntarily and irrevocably donate to
her my one-hectare rice land covered
Donations; Illegal & Impossible
by TCT No. 11550, located in San
Conditions (2007)
Fernando, Pampanga. This donation

No.I. Distinguish the following concepts: shall take effect upon my death."

(B). Illegal and impossible conditions in a The deed also contained Jennifer's signed

simple donation v. illegal and impossible acceptance, and an attached notarized

conditions in an onerous donation. (5%) declaration by Josefa and Jennifer that the
land will remain in Josefa's possession and
SUGGESTED ANSWER: cannot be alienated, encumbered, sold or
disposed of while Josefa is still alive.
Illegal and impossible conditions in a
simple donation are considered as not Advise Jennifer on whether the deed is a
written. Such conditions, shall therefore, donation inter vivos or mortis causa and
be disregarded but the donation remains explain the reasons supporting your advice.
valid (Art. 727, NCC). On the other hand, (8%)

“Never Let The Odds Keep You From Pursuing What You Know In Your Heart You Were Meant To Do.”-Leroy Satchel Paige
Page 64 of 180
Civil Law Q&As (2007-2013) hectorchristopher@yahoo.com dbaratbateladot@gmail.com

SUGGESTED ANSWER: same should be harmonized with its


express irrevocability (Austria-Magat v.
The donation is a donation inter vivos.
CA, G.R. No. 106755, Feb 1, 2002).

When the donor intends that the ALTERNATIVE ANSWER:


donation shall take effect during the
lifetime of the donor, though the The donation is donation mortis causa.
property shall not be delivered till after
the donor’s death, this shall be a The deed clearly states that the donation
shall take effect upon the death of the
donation inter vivos (Art. 729, Civil
donor, Josefa. The donor, moreover,
Code).
retained ownership of the subject
The Civil Code prefers inter vivos property as it was declared that the
transmissions. Moreover, mortis causa property cannot be alienated,
donations should follow the formalities of a encumbered, sold or disposed of while
will (Art. 728, Civil Code). Here there is no the donor is still alive.
showing that such formalities were
As the donation is in the nature of a
followed. Thus, it is favorable to Jennifer
mortis causa disposition, the formalities
that the deed is a donation inter vivos.
of a will should have been complied with
Furthermore, what is most significant in under Art. 728 of the Civil Code,
determining the type of donation is the otherwise, the donation is void and
absence of stipulation that the donor would produce no effect (The National
could revoke the donation; on the Treasure of the Philippines v. Vda. de
contrary, the deeds expressly declare Meimban, G.R. No. L-61023, Aug 22,
them to be “irrevocable,” a quality 1984).
absolutely incompatible with the idea of
conveyances mortis causa where Property
revocability is the essence of the act, to
the extent that a testator cannot Accretion; Alluvium (2008)
lawfully waive or restrict his right of
revocation. The provisions of the deed of No. IX. The properties of Jessica and Jenny,

donation which state that the same will who are neighbors, lie along the banks of

only take effect upon the death of the the Marikina River. At certain times of the

donor and that there is a prohibition to year, the river would swell and as the water

alienate, encumber, dispose, or sell the recedes, soil, rocks and other materials are

“Never Let The Odds Keep You From Pursuing What You Know In Your Heart You Were Meant To Do.”-Leroy Satchel Paige
Page 65 of 180
Civil Law Q&As (2007-2013) hectorchristopher@yahoo.com dbaratbateladot@gmail.com

deposited on Jessica's and Jenny's land but is also the consequences of the
properties. This pattern of the river direct and deliberate intervention of
swelling, receding and depositing soil and man, it is man-made accretion and a
other materials being deposited on the part of the public domain (Tiongco v.
neighbors' properties have gone on for Director of Lands, 16 C.A. Rep 211, cited
many years. Knowing his pattern, Jessica in Nazareno v. C.A., G.R. No. 98045, 26
constructed a concrete barrier about 2 June 1996). Thus, Jessica cannot legally
meters from her property line and claim ownership of the additional 2
extending towards the river, so that when meters of land along her property
the water recedes, soil and other materials because she constructed a concrete
are trapped within this barrier. After several barrier about 2 meters from her property
years, the area between Jessica's property causing deposits of soil and other
line to the concrete barrier was completely materials when the water recedes. In
filled with soil, effectively increasing other words, the increase in her property
Jessica's property by 2 meters. Jenny's was not caused by nature but was man-
property, where no barrier was constructed, made.
also increased by one meter along the side
of the river. (B). If Jessica's and Jenny's properties are
registered, will the benefit of such
(A). Can Jessica and Jenny legally claim registration extend to the increased area of
ownership over the additional 2 meters and their properties? (2%)
one meter, respectively, of land deposited
SUGGESTED ANSWER:
along their properties?(2%)

SUGGESTED ANSWER: If the properties of Jessica and Jenny


are registered, the benefit of such
Only Jenny can claim ownership over registration does not extend to the
the additional one meter of land increased area of their properties.
deposited along her property. Art. 457 of Accretion does not automatically
the Civil Code provides that "to the become registered land because there is
owners of lands adjoining the banks of a specific technical description of the lot
river belong the accretion which they in its Torrens title. There must be a
gradually receive from the effects of the separate application for registration of
current of the water." Where the land is the alluvial deposits under the Torrens
not formed solely by the natural effect of System (Grande v. CA, G.R. No. L-17652,
the water current of the river bordering 30 June, 1962).

“Never Let The Odds Keep You From Pursuing What You Know In Your Heart You Were Meant To Do.”-Leroy Satchel Paige
Page 66 of 180
Civil Law Q&As (2007-2013) hectorchristopher@yahoo.com dbaratbateladot@gmail.com

(C). Assume the two properties are on a cliff Ulpiano built three huts on this additional
adjoining the shore of Laguna Lake. Jessica area, where he and his two married
and Jenny had a hotel built on the children live. On this same area, Ulpiano
properties. They had the erath and rocks and his family planted peanuts, monggo
excavated from the properties dumped on beans and vegetables. Ulpiano also
the adjoining shore, giving rise to a new regularly paid taxes on the land, as shown
patch of dry land. Can they validly lay claim by tax declarations, for over thirty years.
to the patch of land? (2%)
When Marciano learned of the increase in
SUGGESTED ANSWER: the size of the land, he ordered Ulpiano to
demolish the huts, and demanded that he
No. Jessica and Jenny cannot validly lay be paid his share in the proceeds of the
claim to the patch of land because in harvest. Marciano claims that under the
order to acquire land by accretion, there Civil Code, the alluvium belongs to him as a
should be a natural and actual continuity registered riparian owner to whose land the
of the accretion to the land of the accretion attaches, and that his right is
riparian owner caused by natural ebb and enforceable against the whole world.
flow of the current of the river (Delgado
v. Samonte, CA-G.R. No. 34979-R, 10 (A). Is Marciano correct? Explain. (3%)
Aug 1966).
SUGGESTED ANSWER:
Marciano’s contention is correct. Since
that accretion was deposited on his land
Accretion; Rights of the Riparian Owner by the action of the waters of the river
(2009) and he did not construct any structure
to increase the deposition of soil and
No.XVI. Marciano is the owner of a parcel of
silt, Marciano automatically owns the
land through which a river runs out into
accretion. His real right of ownership is
the sea. The land had been brought under
enforceable against the whole world
the Torrens System, and is cultivated by
including Ulpiano and his two married
Ulpiano and his family as farmworkers
children. Although Marciano’s land is
therein. Over the years, the river has
registered, the three (3) hectares land
brought silt and sediment from its sources
deposited through accretion was not
up in the mountains and forests so that
automatically registered. As an
gradually the land owned by Marciano
unregistered land, it is subject to
increased in area by three hectares.
acquisitive prescription by third persons.

“Never Let The Odds Keep You From Pursuing What You Know In Your Heart You Were Meant To Do.”-Leroy Satchel Paige
Page 67 of 180
Civil Law Q&As (2007-2013) hectorchristopher@yahoo.com dbaratbateladot@gmail.com

production, gathering and preservation


Although Ulpiano and his children live in of the fruits (Art 443, NCC).
the three (3) hectare unregistered land
owned by Marciano, they are farm He may also ask for reimbursement of
workers; therefore, they are possessors the taxes he has paid, as these are
not in the concept of owners but in the charges on the land owned by Marciano.
concept of mere holders. Even if they This obligation is based on a quasi-
possess the land for more than 30 years, contract (Art 2175, NCC).
they cannot become the owners thereof
through extraordinary acquisitive
prescription, because the law requires
Builder; Good Faith; Requisites (2013)
possession in the concept of the owner.
Payment of taxes and tax declaration are
No.VIII. Ciriaco Realty Corporation (CRC)
not enough to make their possession one
sold to the spouses Del a Cruz a500-square
in the concept of owner. They must
meter land (Lot A) in Paranaque. The land
repudiate the possession in the concept
now has a fair market value of Pl,200,000.
of holder by executing unequivocal acts
CRC likewise sold to the spouses Rodriguez,
of repudiation amounting to ouster of
a 700-square meter land (Lot B) which is
Marciano, known to Marciano and must
adjacent to Lot A. Lot B has a present fair
be proven by clear and convincing
market value of P1,500,000.
evidence. Only then would his
possession become adverse. The spouses Dela Cruz constructed a house
on Lot B, relying on their presentation of
(B). What rights, if any, does Ulpiano have
the CRC sales agent that it is the property
against Marciano? Explain. (3%)
they purchased. Only upon the completion
of their house did the spouses Dela Cruz
SUGGESTED ANSWER:
discovered that they had built on Lot B
Although Ulpiano is a possessor in bad
owned by the spouses Rodriguez, not on Lot
faith, because he knew he does not own
A that they purchased. They spent P 1
the land, he will lose the three huts he
000,000 for the house.
built in bad faith and make an
accounting of the fruits he has gathered,
As their lawyer, advise the spouses Dela
he has the right to deduct from the value
Cruz on their rights and obligations under
of the fruits the expenses for
the given circumstances, and the recourses

“Never Let The Odds Keep You From Pursuing What You Know In Your Heart You Were Meant To Do.”-Leroy Satchel Paige
Page 68 of 180
Civil Law Q&As (2007-2013) hectorchristopher@yahoo.com dbaratbateladot@gmail.com

and options open to them to protect their However, the builder cannot be obliged
interests. (8%) to buy the land if its value is
considerable more than that of the
SUGGESTED ANSWER: building.. In such case, he shall pay
reasonable rent of the owner of the land
Based on the fact as stated, the spouses
does not choose to appropriate the
Dela Cruz as builders and the spouses
building or trees after proper indemnity
Rodriguez as land owners, are both in
(Art 448, Civil Code).
good faith. The spouses Dela Cruz are
builder in good faith because before The house constructed by the spouses
constructing the house they exercised Dela Cruz is considered as a useful
due diligence by asking the Agent of CRC expense, since it increased the value of
the location of the lot A, and they relied the lot. As such, should the spouses
on the information given by the agent Rodriguez decides to appropriate the
who is presumed to know the identity of house, the spouses Dela Cruz are
the lot purchased by the Dela Cruz entitled to the right of retention pending
spouses (Pleasantville v. CA, 253 SCRA reimbursement of the expenses they
10, 1996). On the other hand, there is no incurred or the increase in value which
showing that the land owners, spouse the thing may have acquired by reason
Rodriguez acted in bad faith. The facts of the improvement (Art 546, Civil
do not show that the building was done Code). Thus, the spouses Dela Cruz may
with their knowledge and without demand P1,000,000.00 as payment of
opposition on their part (Art 453, Civil the expenses in building the house or
Code). The good faith is always presumed increase in value of the land because of
(Art. 527, Civil Code). the house as a useful improvement, as
may be determined by the court form
The owner of the land on which anything
the evidence presented during the trial
has been built, sown, or planted in good
(Depra v. Dumlao, 136 SCRA 475, 1985;
faith shall have the right:
Technogas Phils v. CA, 268 SCRA 5,
1997).
to appropriate as his own the works after
payment of the indemnity provided for
in Art 546 and 548, or

to oblige the one who built to pay the

price of the land.

“Never Let The Odds Keep You From Pursuing What You Know In Your Heart You Were Meant To Do.”-Leroy Satchel Paige
Page 69 of 180
Civil Law Q&As (2007-2013) hectorchristopher@yahoo.com dbaratbateladot@gmail.com

Easement; Prescription; Acquisitive In 2006, Brand0 fenced off his property,


Prescription (2009) thereby blocking Andres' access to the
national highway. Andres demanded that
No. XI. TRUE or FALSE. Answer TRUE if part of the fence be removed to maintain
the statement is true, or FALSE if the his old access route to the highway
statement is false. Explain your answer in (pathway A), but Brando refused, claiming
not more than two (2) sentences. that there was another available pathway
(pathway B) for ingress and egress to the
(C). Acquisitive prescription of a negative
highway. Andres countered that pathway B
easement runs from the time the owner of
has defects, is circuitous, and is extremely
the dominant estate forbids, in a notarized
inconvenient to use.
document, the owner of the servient estate
from executing an act which would be To settle their dispute, Andres and Brando
lawful without the easement. (1%) hired Damian, a geodetic and civil engineer,
to survey and examine the two pathways
SUGGESTED ANSWER:
and the surrounding areas, and to
True. In negative easements, acquisitive
determine the shortest and the least
prescription runs from the moment the
prejudicial way through the servient
owner of the dominant estate forbade, by
estates. After the survey, the engineer
an instrument acknowledged before
concluded that pathway B is the longer
notary public, the owner of the servient
route and will need improvements and
estate from executing an act which
repairs, but will not significantly affect the
would be lawful without the easement
use of Brando's property. On the other
(Art. 621, NCC).
hand, pathway A that had long been in
place, is the shorter route but would
significantly affect the use of Brando's
Easement; Right of Way (2013) property.

No.VII.In 2005, Andres built a residential In light of the engineer's findings and the

house on a lot whose only access to the circumstances of the case, resolve the

national highway was a pathway crossing parties' right of way dispute. (6%)

Brando's property. Andres and others have


SUGGESTED ANSWER:
been using this pathway (pathway A) since
1980.

“Never Let The Odds Keep You From Pursuing What You Know In Your Heart You Were Meant To Do.”-Leroy Satchel Paige
Page 70 of 180
Civil Law Q&As (2007-2013) hectorchristopher@yahoo.com dbaratbateladot@gmail.com

Andres is not entitled to the easement of (Pathway B). Second, the right of way
right of way for Pathway A. Pathway B obtained (Pathway A) is not the least
must be used. prejudicial to Brando’s property, as
evidence by the reports of the geodetic
The owner of a dominant estate may
and civil engineer.
validly obtain a compulsory right of way
only after he has established the When there is already an existing
existence of four requisites, to wit: adequate outlet from the dominant
estate to the public highway, even if the
The (dominant) estate is surrounded by said outlet, for one reason or another, be
other immovables and is without inconvenient, the need to open up
adequate outlet to a public highway; another servitude is entirely unjustified
(Costabella Corporation v. CA, G.R. No.
After payment of the proper indemnity;
80511, Jan 25, 1991). The rule that the

The isolation was not due to the easement of right of way shall be
established at the point least prejudicial
proprietor’s own acts; and
to the servient estate is controlling
The right of way claimed is at a point (Quimen v. Quimen and CA, G.R. No.
least prejudicial to the servient estate, 112331, May 29, 1996).
and insofar as consistent with this rule,
(Note: It is not clear from the problem if there
where the distance from the dominant
exists an easement in favor of the lot
estate to the public highway maybe the
belonging to Andres and if Brando’s lot is
shortest (Art 650, civil Code).
burdened as a servient estate by a right of
However, the Supreme Court has way as a servient estate. If there is such an
consistently ruled that in case both easement burdening Brando’s lot, was it
criteria cannot be complied with, the created as legal easement or as a voluntary
right of way shall be established at the easement. If the used pathway was only a
point least prejudicial to the servient tolerance, then Brando may close it. Andres
estate. must ask for the constitution of a legal
easement through Brando’s lot by proving
The first and fourth requisites are not the four requisites required by Art 649 and
complied with. First, there is another 65, Civil Code).
available outlet to the national highway

“Never Let The Odds Keep You From Pursuing What You Know In Your Heart You Were Meant To Do.”-Leroy Satchel Paige
Page 71 of 180
Civil Law Q&As (2007-2013) hectorchristopher@yahoo.com dbaratbateladot@gmail.com

Easement; Right of Way (2010) easement or servitude, even if the deed

of sale is silent on the matter.


No.XIII. Franz was the owner of Lot E which
was surrounded by four (4) lots one of The vendee of the property in which a
which – Lot C – he also owned. He promised servitude or easement exists cannot
Ava that if she bought Lot E, he would give close or put obstructions thereon to
her a right of way in Lot C. prevent the dominant estate from using
it.
Convinced, Ava bought Lot E and, as
promised, Franz gave her a right of way in
Ava’s working abroad for more than ten
Lot C.
(10) years should not be construed as
non-user, because it cannot be implied
Ava cultivated Lot E and used the right of
from the fact that she or those she left
way granted by Franz.
behind to cultivate the lot no longer use

Ava later found gainful employment abroad. the right of way.


On her return after more than 10 years, the
right of way was no longer available to her Note: Since a right of way is a

because Franz had in the meantime sold discontinuous easement, the period of

Lot C to Julia who had it fenced. ten years of non-user, shall be computed
from the day it ceased to be used under
(A). Does Ava have a right to demand from Act 6341 (2) CC.
Julia the activation of her right of way?
Renunciation or waiver of an easement
Explain. (2.5%)
must be specific, clear, express and

SUGGESTED ANSWER: made in a public instrument in


accordance of Art 1358 of the New Civil
Yes. Ava has the right to demand from Code.
Julia the activation of the right of way, ALTERNATIVE ANSWER:
for the following reasons:
Yes. Ava has the right to demand from
The easement of the right of way is a Julia the activation of the right of way.
real right which attaches to, and is A voluntary easement of right of way,
inseperable from, the estate to which it like any other contract, could be
belongs. extinguished only by mutual agreement
or by renunciation of the owner of the
The sale of the property includes the
dominant estate. Also, like any other

“Never Let The Odds Keep You From Pursuing What You Know In Your Heart You Were Meant To Do.”-Leroy Satchel Paige
Page 72 of 180
Civil Law Q&As (2007-2013) hectorchristopher@yahoo.com dbaratbateladot@gmail.com

contract, an easement is generally her in lot C if Ava purchase lot E. The


effective between parties, their heirs and promise was not reduced to writing (Obra
assignees, except in case where the v. Baldria, 529 SCRA 621 [2007]). Hence,
rights and obligations arising from the it was not or could not have been
contract are not transmissible by their registered as to warn buyers of lot C
nature, or by stipulations or by provision about the existence of the easement on
of law (Unisource Commercial v. Chung, the property. Not having been annotated
593 SCRA 530 [2009]). on the TCT to lot C, the buyer acquired
lot C free from such right of way granted
(B). Assuming Ava opts to demand a right of
to Ava.
way from any of the owners of Lots A, B,
and D, can she do that? Explain. (2.5%)

SUGGESTED ANSWER: Hidden Treasure (2008)

Yes. Ava has the option to demand a


No. VIII. Adam, a building contractor, was
right of way on any of the remaining lots
engaged by Blas to construct a house on a
of Franz more so after Franz sold lot C to
lot which he (Blas) owns. While digging on
Julia. The essential elements of a legal the lot in order to lay down the foudation of
right of way under Art 649 and 650 of the house, Adam hit a very hard object. It
the New Civil Code are complied with. turned out to be the vault of the old Banco

ALTERNATIVE ANSWER: de las Islas Filipinas. Using a detonation


device, Adam was able to open the vault
Yes. Ava has the option to demand a containing old notes and coins which were
right of way from the other lots. The law in circulation during the Spanish era. While
provides that whenever a piece of land the notes and coins are no longer legal
acquired by sale, exchange or partition is tender, they were valued at P100 million
surrounded by other estates of the because of their historical value and the
vendor, exchanger, or co-owner, he shall coins silver nickel content. The following
be obliged to grant a right of way filed legal claims over the notes and coins:
without indemnity (Art 652, NCC).
(i). Adam, as finder;
ALTERNATIVE ANSWER:

(ii). Blas, as owner of the property where


No. There was merely a promise to Ava
they were found;
that a right of way shall be granted to

“Never Let The Odds Keep You From Pursuing What You Know In Your Heart You Were Meant To Do.”-Leroy Satchel Paige
Page 73 of 180
Civil Law Q&As (2007-2013) hectorchristopher@yahoo.com dbaratbateladot@gmail.com

(iii). Bank of the Philippine Islands, as present case, Adam, as finder, and Blas,
successor-in-interest of the owner of the as owner of the land, are entitled to
vault; and share 50-50 in the treasure. The
government can only claim if it can
(iv). The Philippine Government because of establish that the notes and coins are of
their historical value. interest to science or the arts, then it
must pay just price of the things found,
(A). Who owns the notes and coins? (4%)
to be divided equally between Adam and

SUGGESTED ANSWER: Blas (Art. 438, Civil Code).

(B). Assuming that either or both Adam and


The notes and coins are no longer owned
Blas are adjudged as owners, will the notes
by the Bank of the Philippine Islands,
and coins be deemed part of their absolute
which has either lost or abandoned the
community or conjugal partnership of gains
vault and its contents, and it has not
with their respective spouses? (2%)
taken any effort to search, locate or
recover the vault. In any case, since the
SUGGESTED ANSWER:
vault is in actual possession of Adam,
BPI may attempt, in a judicial action to If either or both Adam and Blas are
recover, to rebut the presumption of adjudged as owners, the notes and coins
ownership in favor of Adam and Blas shall be deemed part of their absolute
(Art. 433, Civil Code). Hidden treasure is community or conjugal partnership of
any hidden and unknown deposit of
gains with their respective spouses (Art.
money, jewelry, or other precious
117, par 4, FC).
objects, the lawful ownership of which
does not appear. Given the age and
importance of the items found, it would
be safe to consider the vault, notes and Mortgage; Public or Private Instrument

coins abandoned by BPI and its (2013)


predecessor (Art. 439, Civil Code). It
No.VI. Lito obtained a loan of P1,000,000
belongs to the owner of the land on
from Ferdie, payable within one year. To
which it is found. When the discovery is
secure payment, Lito executed a chattel
made on the property of another, or of
mortgage on a Toyota Avanza and a real
the State and by chance, one-half of it
estate mortgage on a 200-square meter
shall belong to the finder who is not a
piece of property.
trespasser (Art. 438, Civil Code). In the

“Never Let The Odds Keep You From Pursuing What You Know In Your Heart You Were Meant To Do.”-Leroy Satchel Paige
Page 74 of 180
Civil Law Q&As (2007-2013) hectorchristopher@yahoo.com dbaratbateladot@gmail.com

Would it be legally significant - from the exceeds Five Hundred pesos (P500.00)
point of view of validity and enforceability - must appear in writing, even in private
if the loan and the mortgages were in public one. However, the requirement is not for
or private instruments? (6%) validity of the contract, but only for its
greater efficacy.
SUGGESTED ANSWER:
With regard to the chattel mortgage, Art.
From the point of view of validity and 1508, the Chattel Mortgage Law,
enforceability, there would be legal requires an affidavit of good faith stating
significance if the mortgage was in a that the chattel mortgage is supposed to
public or private instrument. As for the stand as security of the loan; thus, for
loan, there is no legal significance the validity of the chattel mortgage, it
except of interest were charged on the must be in a public document and
loan, in which case, the charging of recorded in the Chattel Mortgage
interest must be in writing. Register in the Register of Deeds. A real
estate mortgage, under the provisions of
A contract of loan is a real contract and
Art. 2125 of the Civil Code, requires that
is perfected upon delivery of the object
in order that a mortgage may be validly
of the obligation (Art 1934, Civil Code).
constituted the document in which it
Thus, a contract of loan is valid and
appears be recorded. If the instrument is
enforceable even if it is neither in a
not recorded, the mortgage is
private nor in a public document.
nevertheless valid and binding between
the parties. Hence, for validity of both
As a rule, contracts shall be obligatory in
chattel and real estate mortgages, they
whatever form they may have been
must appear in a public instrument. But
entered into provided all the essential
the purpose of enforceability, it is
requisites for their validity are present.
submitted that the form of the contract,
With regards to its enforceability, a
whether in a public or private document,
contact of loan is not among those
would be immaterial (Mobil Oil v.
enumerated under Art. 1403 (2) of the
Diocaresa, 29 SCRA 656, 1969).
Civil Code, which are covered by the
Statute of Frauds. Also, under Art 1358, acts and contracts
which have for their object the creation
It is important to note that under Art.
or transmission of real rights over
1358 of the Civil Code, all the other
immovable property must be in a public
contracts where the amount involved

“Never Let The Odds Keep You From Pursuing What You Know In Your Heart You Were Meant To Do.”-Leroy Satchel Paige
Page 75 of 180
Civil Law Q&As (2007-2013) hectorchristopher@yahoo.com dbaratbateladot@gmail.com

document for greater efficacy and a real right. Possession may be the real right of
estate mortgage is a real right over possession or jus possessiones or it can

immovable property. be merely the right to possess or jus


possedendi, which are among the basic
rights of ownership. If the real right of
possession is possession in the concept
Occupation vs. Possession (2007)
of owner, but subject to certain
limitations, it may ripen into full
No.I. Distinguish the following concepts:
ownership of the thing or property right

(A). Occupation v. possession. (5%) through acquisitive prescription


depending on whether it is a case of
SUGGESTED ANSWER: ordinary or extraordinary prescription
and whether the property is movable or
Occupation is an original mode of
immovable.
acquiring ownership (Art. 712, NCC).
Things appropriable by nature which are
without an owner, such as animals that
Ownership; Co-Ownership (2009)
are the object of hunting and fishing,
hidden treasure and abandoned No. XI. TRUE or FALSE. Answer TRUE if
movables, are acquired by occupation the statement is true, or FALSE if the
(Art. 713, NCC). However, ownership of a statement is false. Explain your answer in
piece of land cannot be acquired by not more than two (2) sentences.
occupation (Art. 714, NCC).
(D). The renunciation by a co-owner of his
ALTERNATIVE ANSWER: undivided share in the co-owned property
in lieu of the performance of his obligation
Occupation is a mode of acquiring
to contribute to taxes and expenses for the
dominion by the seizure of corporeal
preservation of the property constitutes
things which have no owner, with the
dacion en pago. (1%)
intention of acquiring the ownership
thereof. It is an original mode of SUGGESTED ANSWER:
acquiring ownership upon seizure of a True, Under the Civil Code, a co-owner
res nullius by the occupant who has the may renounce his share in the co-owned
intention to become the owner thereof. property in lieu of paying for his share in
Possession, on the other hand, is the the taxes and expenses for the
holding of the thing or an enjoyment of a preservation of the co-owned property.

“Never Let The Odds Keep You From Pursuing What You Know In Your Heart You Were Meant To Do.”-Leroy Satchel Paige
Page 76 of 180
Civil Law Q&As (2007-2013) hectorchristopher@yahoo.com dbaratbateladot@gmail.com

In effect, there is dacion en pago SUGGESTED ANSWER:


because the co-owner is discharging his
monetary obligation by paying it with his Yes, Cathy can lawfully ask for the

non-monetary interest in the co-owned demolition of Bobby's house. Where

property. The fact that he is giving up there are two or more heirs, the whole

his entire interest simply means that he estate of the decedent, is, before
partition, owned in common by such
is accepting the value of his interest as
heirs, subject to the payment of debts of
equivalent to his share in the taxes and
the deceased (Art. 1078, Civil Code),
expenses of preservation.
Under the rules on co-ownership, "none
of the co-owners shall, without the

Ownership; Co-Ownership (2008) consent of the others make alterations


in the thing owned in common, even
No. VI. Alex died without a will, leaving only though benefits for all would results
an undeveloped and untitled lot in Tagiug therefrom." In Cruz v. Catapang, G.R. No.
City. He is survived by his wife and 4 164110, 12 Feb., 2008, the Court held
children. His wife told the children that she that "alterations include any act of strict
is waiving her share in the property, and dominion or ownership such as
allowed Bobby, the eldest son who was construction of a house." In the present
about to get married, to construct his case, of Alex is the real owner of the
house on ¼ of the lot, without however undeveloped and untitled lot in Taguig,
obtaining the consent of his siblings. After co-ownership is created among his wife
settlement of Alex's estate and partition and four children over said property
among the heirs, it was discovered that upon his death. Since the construction
Bobby's house was constructed on the of the house by Bobby was done without
portion allocated to his sister, Cathy asked obtaining the consent of his siblings, the
Bobby to demolish his house and vacate alteration effected is illegal. Bobby is
the portion alloted to her. In leiu of considered to be in bad faith and as a
demolition, Bobby offered to purchase from sanction for his conduct, he can be
Cathy the lot portion on which his house compelled by Cathy to demolish or
was constructed. At that time, the house remove the structure at his own
constructed was valued at P350.000. expense.

(A). Can Cathy lawfully ask for demolition of (B). Can Bobby legally insist on purchasing
Bobby's house? (3%) the land? (2%)

“Never Let The Odds Keep You From Pursuing What You Know In Your Heart You Were Meant To Do.”-Leroy Satchel Paige
Page 77 of 180
Civil Law Q&As (2007-2013) hectorchristopher@yahoo.com dbaratbateladot@gmail.com

SUGGESTED ANSWER: nature and object to remain at a fixed


place on a river, lake or coast." Since the
No. Bobby cannot legally insist on floating platform is a petroleum
purchasing the land. Being in bad faith, operation facility, it is intended to
he has no option to pay for the price of remain permanently where it is situated,
the lot (Art. 450, Civil Code). even if it is tethered to a ship which is
anchored to the seabed.

ALTERNATIVE ANSWER:
Property; Movable or Immovable (2007)
The platform is a movable property
No.II. Manila Petroleum Co. owned and because it is attached to a movable
operated a petroleum operation facility off property, i.e. the vessel which was
the coast of Manila. The facility was located merely anchored to the seabed. The fact
on a floating platform made of wood and that the vessel is merely anchored to the
metal, upon which was permanently sea bed only shows that it is not
attached the heavy equipment for the intended to remain at a fixed place;
petroleum operations and living quarters of hence, it remains a movable property. If
the crew. The floating platform likewise the intention was to make the platform
contained a garden area, where trees, stay permanent where it was moored, it
plants and flowers were planted. The would not have been simply tethered to
platform was tethered to a ship, the MV a vessel but itself anchored to the
101, which was anchored to the seabed. seabed.

Please briefly give the reason for your (B). Are the equipment and living quarters
answers. (10%) movable or immovable property?

(A).Is the platform movable or immovable SUGGESTED ANSWER:


property?
The thing and living quarters of the crew
SUGGESTED ANSWER: are immovable property under Art. 415
NCC, classifies as an immovable
The platform is an immovable property
"everything attached to an immovable in
under Art. 415 (9) NCC, which provides
a fixed manner, in such a way that it
that "docks and structures which,
cannot be separated therefrom without
though floating, are intended by their
breaking the material or deterioration of

“Never Let The Odds Keep You From Pursuing What You Know In Your Heart You Were Meant To Do.”-Leroy Satchel Paige
Page 78 of 180
Civil Law Q&As (2007-2013) hectorchristopher@yahoo.com dbaratbateladot@gmail.com

the object." Both the equipment and the The trees, plants and flowers planted in
living quarters are permanently attached the garden area of the platform are
to the platform which is also an immovable property under Art. 415 (2)
immovable. The equipment can also be NCC which classifies as an immovable
classified as an immovable property property "trees, plants and growing
under Art. 415 (5) NCC because such fruits, while they are attached to the
equipment are "machinery, receptacles, land or form an integral part of an
instruments or implements intended by immovable, the petroleum operation
the owner of the tenement for an facility.
industry or works which may be carried
ALTERNATIVE ANSWER:
on in a building or on a piece of land and
which tend directly to meet the needs of
The trees, plants and flowers planted in
the industry or works." It is logically
the garden area of the platform are
assumed that the petroleum industry
movable property because they are not
may be carried on in a building or on a
permanently attached t the land and do
piece of land and the platform is
not form an integral part of an
analogous to a building.
immovable. The platform is not an
ALTERNATIVE ANSWER: immovable property for the same reason
already given in the Alternative Answer
The equipment and living quarters of the to Item (a) above.
crew are movable properties since they
are attached to a platform which is also Land Titles and Deeds
a movable property, because it is simply
attached to a vessel is likewise a Acquisition of Lands; Sale of Real
movable property since it was merely Property to an Alien (2009)
anchored on the seabed only shows that
it is not intended to remain at a fixed No.XIX. In 1972, Luciano de la Cruz sold to

place; hence, it remains a movable Chua Chung Chun, a Chinese citizen, a


parcel of land in Binondo. Chua died in
property.
1990, leaving behind his wife and three
(C). Are the trees, plants and flowers children, one of whom, Julian, is a
immovable or movable property? naturalized Filipino citizen. Six years after
Chua’s death, the heirs executed an
SUGGESTED ANSWER: extrajudicial settlement of estate, and the
parcel of land was allocated to Julian. In

“Never Let The Odds Keep You From Pursuing What You Know In Your Heart You Were Meant To Do.”-Leroy Satchel Paige
Page 79 of 180
Civil Law Q&As (2007-2013) hectorchristopher@yahoo.com dbaratbateladot@gmail.com

2007, Luciano filed suit to recover the land Non-Registrable Properties (2007)
he sold to Chua, alleging that the sale was
void because it contravened the No.IV. (B). What properties are not

Constitution which prohibits the sale of registrable? (5%)

private lands to aliens. Julian moved to


Supply this information.
dismiss the suit on grounds of pari delicto,
laches and acquisitive prescription. Decide
SUGGESTED ANSWER:
the case with reasons. (4%)
The following properties are not
SUGGESTED ANSWER:
registrable:

The case must be dismissed. Julian, who


(1.) Properties of the Public dominion;
is a naturialized Filipino citizen and to
whom the property was allocated in a n (2.) Properties for public use or public
extra-judicial partition of the estate, is
service;
now the owner of the property. The
defect in ownership of the property of (3.) Inalienable lands of the public
Julian’s alien father has already been domain;
cured by its transfer to Julian. It has
been validated by the transfer of the (4.) Military installations, civil and quasi-

property to a Filipino citizen. Hence, public lands; and


there is no more violation of the
(5.) All lands not classified as alienable
Constitution because the subject real
and disposable.
property is now owned by a Filipino
citizen (Halili v. CA, 287 SCRA 465,
ALTERNATIVE ANSWER:
[1998]). Further, after the lapse of 35
year, laches has set in and the motion to (1). Properties of public dominium
dismiss may be granted, for the failure of intended for public use, like roads, canals,
Luciano to question the ownership of rivers, torrents, ports and bridges
Chua before its transfer of ownership to constructed by the State, banks, shores,
Julian. roadsteads, and the like, are incapable of
private appropriation, much less
registration (Art. 420 NCC). This includes
public markets, public plazas, municipal
streets and public buildings

“Never Let The Odds Keep You From Pursuing What You Know In Your Heart You Were Meant To Do.”-Leroy Satchel Paige
Page 80 of 180
Civil Law Q&As (2007-2013) hectorchristopher@yahoo.com dbaratbateladot@gmail.com

(Municipality of Antipolo v. Zapanta, 133 (7.) Lands reclaimed by the government


SCRA 820, 1986; Martinez v. CA, 56 from the sea, lakes, or other bodies of
SCRA 647, 1974; Navera v. Quicho, 5 water are disposed or acquisible only by
SCRA 454, 1962). lease and not otherwise, under the
Public Land Act.
(2.) Lands proclaimed or classified as
forest, timberlands, mineral lands and
national parks. Under Sec 2, Art XII,
Constitution of the Philippines, these Prescription; Acquisitive Prescription

lands are inalienable. (2008)

(3.) Lands that are reserved by law or No. VII. Anthony bought a piece of untitled
Presidential proclamation for military, agricultural land from Bert. Bert, in turn,

civic or quasi-public purpose, Under Sec acquired the property by forging carlo's

88, Chapter XII of the Public Land Act, signature in a deed of sale over the

such lands shall be inalienable and shall property. Carlo had been in possession of

not be subject to occupation, entry, sale, the property for 8 years, declared it for tax

lease or other disposition. purposes, and religiously paid all taxes due
on the property. Anthony is not aware of
(4.) In general, all lands of the public the defect in Bert's title, but has been in
domain that has not been classified as actual physical possession of the property
alienable and disposable under the Public from the time he bought it from Bert, who
Land Act. had never been in possession of the
property for one year.
(5.) Lands that form part of the seabed,
riverbed or lakebed. These lands are not (A). Can Anthony acquire ownership of the
susceptible to private appropriation. property by acquisitive prescription? How
many more years does he have possess it to
(6.) Foreshore lands is that strip of land
acquire ownership? (2%)
that lies between the high and low water
marks and alternately wet and dry SUGGESTED ANSWER:
according to the flow of the tide belong
to the public domain, and can only be Yes, Anthony can acquire ownership of

acquired by lease if not needed by the the property through acquisitive

government for public or quasi-public prescription. In the present case,

purposes. Anthony is a buyer/possessor in good

“Never Let The Odds Keep You From Pursuing What You Know In Your Heart You Were Meant To Do.”-Leroy Satchel Paige
Page 81 of 180
Civil Law Q&As (2007-2013) hectorchristopher@yahoo.com dbaratbateladot@gmail.com

faith because he was not aware of the shall have a right to a part of the
defect in Bert's title (Art. 526, Civil expenses of cultivation, and to a part of
Code). As such, Anthony can acquire the net harvest of the standing crops,
ownership and other real rights over both in proportion to the time of the
immovable property through open, possession (Art 545, Civil Code).
continuous possession of 10 years (Art.
1134, Civil Code). Anthony needs nine
more years of possession, in addition to
Prescription; Judicially Foreclosed Real
his one (1) year of possession in good
Property Mortgage (2012)
faith.

No.IX.a) Does the right to request for the


(B).If Carlo is able to legally recover his
issuance of a writ of possession over a
property, can he require Anthony to
foreclosed real property prescribe in five (5)
account for all the fruits he has harvested
years? (5%)
from the property while in possession? (2%)

SUGGESTED ANSWER:
SUGGESTED ANSWER:

Yes, it prescribes in five (5) years. If the


If Carlo is able to legally recover his
real property mortgaged is judicially
property, he cannot require Anthony to
foreclosed, the action for judicial
account for all the fruits harvested from
foreclosure should be filed within a
the property. Anthony is entitled to the
period of ten (10) years. The request for
fruits harvested in good faith before his
issuance of a writ of possession should
possession was legally interrupted (Art.
be filed upon motion of the winning
544, Civil Code).
bidder within five (5) years after the
judgment of foreclosure. The writ of
(C).If there are standing crops on the
possession is an order commanding the
property when Carlo recovers possession,
sheriff to place a person named therein
can Carlo appropriate them? (2%)
in possession of real property (BPI v.
SUGGESTED ANSWER: Icot. G.R. No. 168081, Oct 12, 2009).

Yes, Carlos can appropriate only a


portion of the standing crops on the
property once he recovers possession.
Anthony being a possessor in good faith,

“Never Let The Odds Keep You From Pursuing What You Know In Your Heart You Were Meant To Do.”-Leroy Satchel Paige
Page 82 of 180
Civil Law Q&As (2007-2013) hectorchristopher@yahoo.com dbaratbateladot@gmail.com

Purchaser in Good Faith; Mortgaged of Deeds under Act. 3344 and obtained a
Property (2008) tax declaration in its name.

No. XIX. Juliet offered to sell her house and (A). Was Dehlma a purchaser in good faith?
lot, together with all the furniture and (2%)
appliances therein to Dehlma. Before
agreeing to purchase the property, Dehlma SUGGESTED ANSWER:

went to the Register of Deeds to verify


Yes, Dehlma is a purchaser in good faith.
Juliet's title. She discovered that while the
In the present case, before Dehlma
property was registered in Juliet's name
bought the property, she went to the
under the Land Registration Act, as
Register of Deeds to verify Juliet's title.
amended by the Property Registration
When she discovered that the property
Decree, it property, Dehlma told Juliet to
was mortgaged to Elaine, she gave an
redeem the property from Elaine, and gave
advance payment so that Juliet could
her an advance payment to be used for
release the mortgage. It was only after
purposes of realesing the mortgage on the
the mortgage was released and free from
property. When the mortgage was released,
the claims of other persons that Dehlma
Juliet executed a Deed of Absolute Sale
bought the property. Thus, Dehlma is a
over the property which was duly registered
purchaser in good faith (Mathay v. CA,
with the Registry of Deeds, and a new TCT
G.R. No. 115788, 17 Sept, 1998).
was issued in Dehlma's name. Dehlma
immediately took possession over the house
(B). Who as between Dehlma and XYZ Bank
and lot and the movables therein.
has a better right to the house and lot? (2%)
Thereafter, Dehlma went to theAssessor's
Office to get a new tax declaration under SUGGESTED ANSWER:
her name. She was surprised to find out
that the property was already declared for Between Dehlma and XYZ Bank, Dehlma
tax purposes in the name of XYZ Bank has a better right to the house and lot.
which had foreclosed the mortgage on the After the release of the mortgage, the
property before it was sold to her. XYZ Deed of Absolute Sale was registered and
Bank was also the purchaser in the a new title was issued in Dehlma's name.
foreclosure sale of the property. At that Act 3344 is applicable exclusively to
time, the property was still unregistered but instruments resulting from agreement of
XYZ Bank registered the Sheriff's Deed of parties thereto and does not apply to
Conveyance in the day book of the Register deeds of a sheriff conveying to a

“Never Let The Odds Keep You From Pursuing What You Know In Your Heart You Were Meant To Do.”-Leroy Satchel Paige
Page 83 of 180
Civil Law Q&As (2007-2013) hectorchristopher@yahoo.com dbaratbateladot@gmail.com

purchaser unregistered lands sold to him land registration and acquisition of title to
under execution (Williams v. Suñer, 49 land. The manual should include the
Phil. ,534). following items:

(C). Who owns the movables inside the (A). What is the governing law? (5%)
house? (2%)
SUGGESTED ANSWER:
SUGGESTED ANSWER:
The governing law is the Land
Dehlma owns the movables because Registration Act as amended by Property
when she acquired the house and lot Registration Decree (Act 496 as amended
from Juliet, all the furniture and by PD 1529).
appliances therein were included in the
[Note: It is respectfully recommended
sale. As owner of the real property,
that full credit be given to examinees
Dehlma also owns the movables found
who did not give the exact title or
therein (Art. 542, Civil Code).
number of the law but merely stated a
description of the law.]

Registration; Governing Law (2007) ALTERNATIVE ANSWER:

No.IV. Bedrock Land & Property In general, the governing law relating to
Development Corp. is a development registration and acquisition of title to
company engaged in developing and selling land is Act 496 of 1902 as amended by
subdivisions, condominium units and PD 1529, otherwise known as Property
industrial estates. In order to replenish its Registration Decree of June 11, 1978.
inventories, it embarked on an aggressive
land banking program. It employed "scouts" (1.) Chapter III-I governs original

who roam all over the Philippines to look for registration of land title under the

and conduct investigations on prospective Torrens System by voluntary ordinary

sites for acquisition and development, judicial proceedings.

whether developed, semi-developed or raw


(2.) Chapter II-II governs compulsory
land. The management of Bedrock asks you
registration of lands through cadastral
as the company counsel to prepare a
proceedings.
manual containing a summary of the
pertinent laws and regulations relating to

“Never Let The Odds Keep You From Pursuing What You Know In Your Heart You Were Meant To Do.”-Leroy Satchel Paige
Page 84 of 180
Civil Law Q&As (2007-2013) hectorchristopher@yahoo.com dbaratbateladot@gmail.com

(3.) Section 103 governs registration of Replacement of lost or destroyed owner's


homestead, sales, free patent under CA duplicate certificate of title.
No. 141, as amended, otherwise known
(9.) R.A. No. 26 governs judicial
as the Public Land Act.
reconstitution of lost or destroyed
(4.) Section 104 governs registration of originals of the certificate of title.
certificates of land transfers,
emancipation patents and Certificates of (10.) R.A. No. 6732 governs

Land Ownership Award (CLOA) under administrative reconstitution of lost or

Comprehensive Land Reform Law. destroyed original certificates of title.

(5.) Chapter V governs the registration of (11.) Section 113 governs the

land dealings on registered land like registration of instruments affecting

conveyances, transfers, mortgages, unregistered private lands.

leases, powers of attorney, trusts and


(12.) Section 117 governs "consultas,"
similar contracts inter vivos.
where the Register of Deeds refuses to

(6.) Chapter V-II governs the registration register a deed or when he is in doubt as
of involuntary dealings on registered to what action to take on an instrument
land like attachments, adverse claims, presented for registration.
enforcement of liens on registered land,
notices of lis pendens. (7.) Chapter VI
governs the registration of judgments,
Registration; Party Who First took
orders and partitions, condemnation in
Possession (2013)
eminent domain proceedings, judicial
and extra-judicial settlement of estates. No.IX.Rica petitioned for the annulment of
her ten-year old marriage to Richard.
(8.) Sections 107, 108 and 109 govern
Richard hired Atty. Cruz to represent him
petitions and actions after original
in the proceedings. In payment for Atty.
registration like: (a).Compulsory
Cruz's acceptance and legal fees, Richard
surrender of withheld owner's duplicate
conveyed to Atty. Cruz a parcel of land in
certificate of title;
Taguig that he recently purchased with his
lotto winnings. The transfer documents
Amendment and alteration of certificate
were duly signed and Atty. Cruz
of title;

“Never Let The Odds Keep You From Pursuing What You Know In Your Heart You Were Meant To Do.”-Leroy Satchel Paige
Page 85 of 180
Civil Law Q&As (2007-2013) hectorchristopher@yahoo.com dbaratbateladot@gmail.com

immediately took possession by fencing off the liquidation of the absolute


the property's entire perimeter. community or conjugal partnership of
the spouses as the case may be (Art. 50
Desperately needing money to pay for his in relation to Art 43 of the Family Code).
mounting legal fees and his other needs Richard purchased the land with his
and despite the transfer to Atty. Cruz, lotto winnings during the pendency of
Richard offered the same parcel of land for the suit for annulment and on the
sale to the spouses Garcia. After inspection assumption that the parties are governed
of the land, the spouses considered it a by the regime of absolute community or
good investment and purchased it from conjugal partnership, winnings from
Richard. Immediately after the sale, the gambling or betting will form part
spouses Garcia commenced the thereof. Also, since the land is part of
construction of a three-story building over the absolute community or conjugal
the land, but they were prevented from partnership of Richard and Rica, it may
doing this by Atty. Cruz who claimed he not be sold or alienated without the
has a better right in light of the prior consent of the latter and any disposition
conveyance in his favor. or encumbrance of the property of the
community or the conjugal property
Is Atty. Cruz's claim correct? (8%)
without the consent of the other spouse
is void (Art 96 and Art 124, Family
SUGGESTED ANSWER:
Code).

No. Atty. Cruz is not correct. At first


glance, it may appear that Atty. Cruz is
the one who has the better right because
Registration; Requisites; Proof (2013)
he first took possession of the property.
However, a lawyer is prohibited under No.X. Manuel was born on 12 March 1940
Art 1491 of the Civil Code from in a 1 000-square meter property where he
acquiring the property and rights which grew up helping his father, Michael,
may be the object of any litigation in cultivate the land. Michael has lived on the
which they may take part by virtue of property since the land was opened for
their profession. While the suit is for settlement at about the time of the
annulment of marriage and it may be Commonwealth government in 1935, but
urged that the land itself is not the for some reason never secured any title to
object of the litigation, the annulment of the property other than a tax declaration in
marriage, if granted, will carry with it his name. He has held the property through

“Never Let The Odds Keep You From Pursuing What You Know In Your Heart You Were Meant To Do.”-Leroy Satchel Paige
Page 86 of 180
Civil Law Q&As (2007-2013) hectorchristopher@yahoo.com dbaratbateladot@gmail.com

the years in the concept of an owner and ownership since June 12, 1945, or
his stay was uncontested by others. He has earlier. However, it is only necessary
also conscientiously and continuously paid that the land is already declared A & D
the realty taxes on the land. land “at the time for application for
registration is filed” (Malabanan v.
Michael died in 2000 and Manuel - as Republic, G.R. No. 180067, June 30,
Michael’s only son and heir -now wants to
2009).
secure and register title to the land in his
own name. He consults you for legal advice Manuel could also invoke Sec 14 (2) of
as he wants to perfect his title to the land the same Decree, which allows
and secure its registration in his name. registration through ordinary acquisitive
prescription for thirty years, provided,
What are the laws that you need to consider however, that the land is “patrimonial”
in advising Manuel on how he can perfect in character, i.e. already declared by the
his title and register the land in his name? government (a) as A & D, and (b) no
Explain the relevance of these laws to your longer needed for public use or public
projected course of action. (4%) service (Malabanan, supra).

SUGGESTED ANSWER: Manuel could also file an application for


“confirmation of imperfect or
(Note: With all due respect, it is
incomplete title’ through “judicial
recommended that the examiner accept and
legalization” under Sec. 48 (b) of CA no.
give full credit to any of the answers given in
141, or the Public Land Act (PLA). But,
each of the following paragraphs.)
as held in Malabanan, there is no
substantial difference between this
I would advice Manuel to file an
provision and Sec 14 (1) of the PRD.
application for registration under Sec 14
Both refer to agricultural lands already
of Pres. Decree No. 1529, or the Property
classified as alienable and disposable at
Registration Decree (PRD), specifically
the time the application is filed, and
Sec14 (1) which requires (a) that the land
require possession and occupation since
applied for forms part of the alienable
June 12, 1945. The only difference is
and disposable (A & D) portion of the
that under the PRD, there already exists
public domain, and (b) that the applicant
a title which is to be confirmed, whereas
has been in open, continuous and
under the PLA, the presumption is that
notorious possession and occupation
land is still public land (Republic v.
thereof under bona fide claim of

“Never Let The Odds Keep You From Pursuing What You Know In Your Heart You Were Meant To Do.”-Leroy Satchel Paige
Page 87 of 180
Civil Law Q&As (2007-2013) hectorchristopher@yahoo.com dbaratbateladot@gmail.com

Aquino, G.R. No. L-33983, January 27, Manuel has a the burden to overcome

1983). the presumption of State ownership by


“well-nigh incontrovertible” evidence
Manuel may also invoke “vested rights’ (Ong v. Republic, G.R. No. 175746,
acquired under Rep. Act. No. 1942, dated March 12, 2008). Accordingly, he must
June 2, 1957, which amended Sec. 48 (b) show that ht eland is already classified
of the PLA by providing for a prescriptive as A & D “at the time the application for
period of thirty years for judicial registration is filed” and that he has
confirmation of imperfect title. It must been in “possession and occupation
only be demonstrated that possession thereof” in the manner required by law
and occupation commenced on January since June 12, 1945, or earlier.
24, 1947 and the 30-year period was
completed prior to the effectivity of PD Manuel may tack his possession to that
No. 1073 on January 25, 1977. PD No. of his predecessor-in-interest (Michael)
1073 now requires possession and by the testimony of disinterested and
occupation since June 12, 1945 knowledgeable eyewitnesses. Overt acts
(Republic v. Espinosa, G.R. No. 171514, of possession may consist in introducing
July 18, 2012). valuable improvements like fencing the
land, constructing a residential house
Another alternative is for Manuel to thereon, cultivating the land and
secure title through administrative planting fruit bearing trees, declaring
proceedings under the homestead or free the land for taxation purposes and
patent provisions of the PLA. The title paying realty taxes, all of which are
issued has the same efficacy and validity corroborative proof of possession.
as a title issued through judicial
proceedings, but with the limitations To identify the land, he must submit the
that the land cannot be sold or disposed tracing cloth plan or a duly-certified
of within five years from the issuance of blueprint or whiteprint copy thereof
patent (Sec. 118, CA No. 141, as (Director of Lands v. Reyes, G.R. No. L-
amended). 27594, November 28, 1975; Director of
Lands v. CA and Iglesia ni Cristo, G.R.
What do you have to prove to secure No. L-56613, March 14, 1988).
Manuel's objectives and what
documentation are necessary? (4%) To show the classification of the land as
A & D, the application must be
SUGGESTED ANSWER:
accompanied by (1) a CENRO or PENRO

“Never Let The Odds Keep You From Pursuing What You Know In Your Heart You Were Meant To Do.”-Leroy Satchel Paige
Page 88 of 180
Civil Law Q&As (2007-2013) hectorchristopher@yahoo.com dbaratbateladot@gmail.com

certification; and (2) a certified true


copy of the original classification I will institute the following actions
approved by the DENR Secretary against Atty. Tan:
(Republic v. Bantigue, G.R.No. 162322,
March 14, 2012). A presidential or (a). A civil action for damage for the
legislative act may also be considered. fraudulent transfer of the title in his
name and to recover the value of the
property;

Remedies; Fraud; Rights of Innocent


(b). An action against the National
Purchaser (2009)
Treasurer for compensation from the
State Assurance Fund which is set aside
No.IX. Before migrating to Canada in 1992,
by law to pay those who lose their land
the spouses Teodoro and Anita entrusted
suffer damages as a consequence of the
all their legal papers and documents to
operation of the Torrens system;
their nephew, Atty. Tan. Taking advantage
of the situation, Atty. Tan forged a deed of
(c). A criminal action for forgery or
sale, making it appear that he had bought
falsification of public document;
the couple’s property in Quezon City. In
2000, he succeeded in obtaining a TCT over
(d). A complaint with the Supreme
the property in his name. Subsequently,
Court/Integrated Bar of the Philippines
Atty. Tan sold the same property to Luis,
to disbar or suspend him or other
who built an auto repair shop on the
disciplinary action for violation or the
property. In 2004, Luis registered the deed
Code of Professional Ethics.
of conveyance, and title over the property
was transferred in his name.
Any action against Luis will not prosper

In 2006, the spouses Teodoro and Anita because he is an innocent purchaser for

came to the Philippines for a visit and value. The Title to the land he bought

discovered what had happened to their was already in the name of the person

property. They immediately hire you as who sold the property to him, and there

lawyer. What action or actions will you is nothing on the title which will make

institute in order to vindicate their rights? him suspect about the fraud committed

Explain fully. (4%) by Atty. Tan.

SUGGESTED ANSWER:

“Never Let The Odds Keep You From Pursuing What You Know In Your Heart You Were Meant To Do.”-Leroy Satchel Paige
Page 89 of 180
Civil Law Q&As (2007-2013) hectorchristopher@yahoo.com dbaratbateladot@gmail.com

Contracts Rescission of Contract; Fortuitous Event

(2008)
Contract to Sell vs. Conditional Contract
No.XVIII. AB Corp. entered into a contract
of Sale (2012)
with XY Corp. whereby the former agreed to

No.X.a) A contract to sell is the same as a construct the research and laboratory

conditional contract of sale. Do you agree? facilities of the latter. Under the terms of

Explain your answer. (5%) the contract, AB Corp. agreed to complete


the facility in 18 months, at the total
SUGGESTED ANSWER: contract price of P10 million. XY Corp. paid
50% of the total contract price, the balance
No. A contract to sell is a species of
to be paid upon completion of the work. The
conditional sale. The contract to sell
work stated immediately, but AB Corp. later
does not sell a thing or property; it sells
experienced work slippage because of labor
the right to buy property. A conditional
unrest in his company. AB Corp.'s
sale is a sale subject to the happening or
employees claimed that they are not being
performance of a condition, such as
paid on time; hence, the work slowdown. As
payment of the full purchase price, or
of the 17th month, work was only 45%
the performance of other prestation to
completed. AB Corp. asked for extension of
give, to do or not to do. Compliance with
time, claiming that its labor problems is a
the condition automatically gives the
case of fortuitous event, but this was
right to the vendee to demand the
denied by XY Corp. When it became certain
delivery of the object of the sale. In a
that the contruction could not be finished
contract to sell, however, the
on time, XY Corp. sent written notice
compliance with the condition does not
cancelling the contract, and requiring AB
automatically sell the property to the
Corp. to immediately vacate the premises.
vendee. It merely gives the vendee the
right to compel the vendor to execute (A). Can the labor unrest be considered a
the deed of absolute sale. fortuitous event? (1%)

SUGGESTED ANSWER:

No. The labor unrest cannot be


considered a fortuitous event under Art.
1174 of the Civil Code. A fortuitous
event should occur independent of the

“Never Let The Odds Keep You From Pursuing What You Know In Your Heart You Were Meant To Do.”-Leroy Satchel Paige
Page 90 of 180
Civil Law Q&As (2007-2013) hectorchristopher@yahoo.com dbaratbateladot@gmail.com

will of the debtor or without his statement is false. Explain your answer in
participation or aggravation (Paras, Civil not more than two (2) sentences.
Code Annotated, vol. IV, 2000 ed., p
159). As mentioned in the facts, labor (A). A clause in an arbitration contract

unrest of the employees was caused by granting one of the parties the power to

AB Corp.'s failure to pay its employees choose more arbitrators than the other

on time. renders the arbitration contract void. (1%)

(B). Can XY Corp. unilaterrally and SUGGESTED ANSWER:


True. The Civil Code provides that “Any
immediately cancel the contract? (2%)
clause giving one of the parties power to
SUGGESTED ANSWER: choose more arbitrators than the other
is void and of no effect” (Art 2045, NCC).
No, XY Corp. cannot unilaterally and
Obligations
immediately cancel the contract. In the
absence of any stipulation for automatic
Extinguishment; Compensation (2009)
rescission, rescission must be judicial
(Art. 1191, Civil Code). No.XV. Sarah had a deposit in a savings
account with Filipino Universal Bank in the
(C). Must AB Corp. return the 50%
amount of five million pesos
downpayment? (2%)
(P5,000,000.00). To buy a new car, she
obtained a loan from the same bank in the
SUGGESTED ANSWER:
amount of P1,200,000.00, payable in twelve

AB Corp. need not return the 50% down monthly installments. Sarah issued in favor

payment because 45% of the work was of the bank post-dated checks, each in the

already completed, otherwise, XY Corp. amount of P100,000.00, to cover the twelve

would be unjustly enriching itself at the monthly installment payments. On the


third, fourth and fifth months, the
expense of AB Corp.
corresponding checks bounced.

The bank then declared the whole


Stipulation; Arbitration Clause (2009) obligation due, and proceeded to deduct the
amount of one million pesos
No. XI. TRUE or FALSE. Answer TRUE if (P1,000,000.00) from Sarah’s deposit after
the statement is true, or FALSE if the notice to her that this is a form of

“Never Let The Odds Keep You From Pursuing What You Know In Your Heart You Were Meant To Do.”-Leroy Satchel Paige
Page 91 of 180
Civil Law Q&As (2007-2013) hectorchristopher@yahoo.com dbaratbateladot@gmail.com

compensation allowed by law. Is the bank Extinguishment; Compensation (2008)


correct? Explain. (4%)
No. XV. Eduardo was granted a loan by XYZ
SUGGESTED ANSWER: Bank for the purpose of improving a
No, the bank is not correct. While the building which XYZ leased from him.
Bank is correct about the applicability of Eduardo, executed the promissory note
compensation, it was not correct as to ("PN") in favor of the bank, with his friend
the amount compensated. Recardo as co-signatory. In the PN, they
both acknowledged that they are
A bank deposit is a contract of loan, "individually and collectively" liable and
where the depositor is the creditor and waived the need for prior demand. To
the bank the debtor. Since Sarah is also secure the PN, Recardo executed a real
the debtor of the bank with respect to estate mortgage on his own property. When
the loan, both are mutually principal Eduardo defaulted on the PN, XYZ stopped
debtors and creditors of each other. Both payment of rentals on the building on the
obligation are due, demandable and ground that legal compensation had set in.
liquidated but only up to the extent of Since there was still a balance due on the
P300,000.00 (covering the unpaid third, PN after applying the rentals, XYZ
fourth and fifth monthly installments). foreclosed the real estate mortgage over
The entire one million was not yet due Recardo's property. Recardo opposed the
because the loan has no acceleration foreclosure on the ground that he is only a
clause in case of default. And since there co-signatory; that no demand was made
is no retention or controversy upon him for payment, and assuming he is
commenced by third person and liable, his liability should not go beyond
communicated in due time to the debtor, half the balance of the loan. Further,
then all the requisites of legal Recardo said that when the bank invoked
compensation are present but only up to compensation between the reantals and the
the amount of P300,000.00. The bank, amount of the loan, it amounted to a new
therefore, may deduct P300,000.00 from contract or novation, and had the effect of
Sarah’s bank deposit by way of extinguishing the security since he did not
compensation. give his consent (as owner of the property
under the real estate mortgage) thereto.

(A). Can XYZ Bank validly assert legal


compensation? (2%)

“Never Let The Odds Keep You From Pursuing What You Know In Your Heart You Were Meant To Do.”-Leroy Satchel Paige
Page 92 of 180
Civil Law Q&As (2007-2013) hectorchristopher@yahoo.com dbaratbateladot@gmail.com

SUGGESTED ANSWER: ground that legal compensation had set in.


Since there was still a balance due on the
Yes, XYZ Bank can validly assert legal PN after applying the rentals, XYZ
compensation. In the present case, all of foreclosed the real estate mortgage over
the elements of legal compensation are Recardo's property. Recardo opposed the
present: (1) XYZ Bank is the creditor of foreclosure on the ground that he is only a
Eduardo while Eduardo is the lessor of co-signatory; that no demand was made
XYZ Bank; (2) both debts consist in a upon him for payment, and assuming he is
sum of money, or if the things due are liable, his liability should not go beyond
consumable, they be of the same kind, half the balance of the loan. Further,
and also of the same quality if the latter Recardo said that when the bank invoked
has been stated; (3) the two debts be compensation between the reantals and the
due; (4) they be liquidated and amount of the loan, it amounted to a new
demandable, and (5) over neither of them contract or novation, and had the effect of
there be any retention or controversy, extinguishing the security since he did not
commenced by third persons and give his consent (as owner of the property
communicated in due time to the debtor under the real estate mortgage) thereto.
(Art. 1279, Civil Code).
(C). Does Recardo have basis under the
Civil Code for claiming that the original
contract was novated? (2%)
Extinguishment; Novation (2008)

SUGGESTED ANSWER:
No. XV. Eduardo was granted a loan by XYZ
Bank for the purpose of improving a No. Recardo has no basis for claiming
building which XYZ leased from him. novation of the original contract when
Eduardo, executed the promissory note the bank invoked compensation because
("PN") in favor of the bank, with his friend there was simply partial compensation
Recardo as co-signatory. In the PN, they (Art. 1290, Civil Code) and this would
both acknowledged that they are not bar the bank from recovering the
"individually and collectively" liable and remaining balance of the obligation.
waived the need for prior demand. To
secure the PN, Recardo executed a real ALTERNATIVE ANSWER:
estate mortgage on his own property. When
Eduardo defaulted on the PN, XYZ stopped No. In order that an obligation may be

payment of rentals on the building on the extinguished by another, it is imperative

“Never Let The Odds Keep You From Pursuing What You Know In Your Heart You Were Meant To Do.”-Leroy Satchel Paige
Page 93 of 180
Civil Law Q&As (2007-2013) hectorchristopher@yahoo.com dbaratbateladot@gmail.com

that it be so declared in unequivocal or creditors (Philippine Airlines v. CA


terms, or that the old and new and Amelia Tan, G.R. No. L-49188,
obligations be on every point compatible 1990). Mere delivery of checks does not
with each other. Novation is never discharge the obligation under a
presumed (Art. 1292, Civil Code). judgment. A check shall produce the
effect of payment only when they have
been cashed or where through the fault
of the creditor they have been impaired
Extinguishment; Payment of Check
(Art 1249, Civil Code).
(2013)
However, it is not necessary that the
No.VI. Lito obtained a loan of P1,000,000
right of redemption be exercised by
from Ferdie, payable within one year. To
delivery of legal tender. A check may be
secure payment, Lito executed a chattel
used for the exercise of right of
mortgage on a Toyota Avanza and a real
redemption, the same being a right and
estate mortgage on a 200-square meter
not an obligation. The tender of a check
piece of property.
is sufficient to compel redemption but is
not in itself a payment that relieves the
Lito's failure to pay led to the extra-judicial
redemptioner from his liability to pay
foreclosure of the mortgaged real property.
the redemption price (Biana v. Gimenez,
Within a year from foreclosure, Lito
G.R. No. 132768, Sept 9, 2005, citing
tendered a manager's check to Ferdie to
Fortunado v. CA).
redeem the property. Ferdie refused to
accept payment on the ground that he
Redemption within the period allowed by
wanted payment in cash: the check does
law is not a matter of intent but a
not qualify as legal tender and does not
question of payment or valid tender of
include the interest payment. Is Ferdie's
full redemption prices within the said
refusal justified? (4%)
period. Whether redemption is being
made under Art. 3135 or under the
SUGGESTED ANSWER:
General Banking Law, the mortgagor or

A check, whether a manager’s check or his assignee is required to tender

an ordinary check is not legal tender, payment to make said redemption valid

and an offer of a check in payment of a (Heirs of Quisumbing v. PNB and SLDC,

debt is not a valid tender of payment and G.R. No. 178242, Jan 20, 2009).

may be refused receipt by the oblige

“Never Let The Odds Keep You From Pursuing What You Know In Your Heart You Were Meant To Do.”-Leroy Satchel Paige
Page 94 of 180
Civil Law Q&As (2007-2013) hectorchristopher@yahoo.com dbaratbateladot@gmail.com

Moreover, Ferdie’s refusal was justified there has been extraordinary deflation since
on the ground that the amount tendered 1998, and therefore, Felipe should pay him
does not include interest. In order to the value of the debt at the time it was
effect the redemption of the foreclosed incurred. Felipe refused to pay him again,
property, the payment to the purchaser claiming that Gustavo is estopped from
must include the following sums: (a) the raising the issue of legal tender, having
bid price; (b) the interest on the bid accepted the check in March, and that it
price, computed at one per centum (1%) was Gustavo's negligence in not depositing
per month; and (c) the assessments and the check immediately that caused the
taxes, if any, paid by the purchaser with check to become stale.
the same rate of interest (Sec 28, 1997
Rules of Civil Procedure). Unless there is (A). Can Gustavo now raised the issue that

an express stipulation to that effect, the the cashier's check is not legal tender? (2%)

creditor cannot be compelled to receive


SUGGESTED ANSWER:
partial payment of the prestation (Art.
1248, Civil Code). No. Gustavo previously accepted a check
as payment. It was his fault why the
check became stale. He is now estopped

Extinguishment; Payment of Check; from raising the issue that a cashier's

Legal Tender (2008) check is not legal tender.

No. XVII. Felipe borrowed $100 from (B). Can Felipe validly refuse to pay

Gustavo in 1998, when the Phil P - US$ Gustavo again? (2%)

exchange rate was P56 - US$1. On March


SUGGESTED ANSWER:
1, 2008, Felipe tendered to Gustavo a
cashier's check in the amount of P4,135 in
Yes, Felipe can refuse to pay Gustavo,
payment of his US$ 100 debt, based on the
who allowed the check to become stale.
Phil P - US$ exchange rat at that time.
Although a check is not legal tender
Gustavo accepted the check, but forgot to
(Belisario v. Natividad. 60 Phil 156),
deposit it until Sept. 12, 2008. His bank
there are instances when a check
refused to accepted the check because it
produces the effects of payment, for
had become stale. Gustavo now wants
example: (a) when the creditor is in
Felipe to pay him in cash the amount of
estoppel or he had previously promised
P5,600. Claiming that the previous
he would accept a check (Paras, Civil
payment was not in legal tender, and that

“Never Let The Odds Keep You From Pursuing What You Know In Your Heart You Were Meant To Do.”-Leroy Satchel Paige
Page 95 of 180
Civil Law Q&As (2007-2013) hectorchristopher@yahoo.com dbaratbateladot@gmail.com

Code Annotated, Vol IV, 2000 ed., p. secure the PN, Recardo executed a real
394); (b) when the check has lost its estate mortgage on his own property. When
value because of the fault of the creditor Eduardo defaulted on the PN, XYZ stopped
(Art. 1249, 2nd par.),as when he was payment of rentals on the building on the
unreasonably delayed in presenting the ground that legal compensation had set in.
check for payment (PNB v. Seeto, G.R. Since there was still a balance due on the
No, L-4388, 13 August 1952). PN after applying the rentals, XYZ
foreclosed the real estate mortgage over
(C). Can Felipe compel Gustavo to receive Recardo's property. Recardo opposed the
US$100 instead? (1%) foreclosure on the ground that he is only a
co-signatory; that no demand was made
SUGGESTED ANSWER:
upon him for payment, and assuming he is
liable, his liability should not go beyond
Felipe cannot compel Gustavo to receive
half the balance of the loan. Further,
US$100 because under RA 529, payment
Recardo said that when the bank invoked
of loans should be at Philippine currency
compensation between the reantals and the
at the rate of exchange prevailing at the
amount of the loan, it amounted to a new
time of the stipulated date of payment.
contract or novation, and had the effect of
Felipe could only compel Gustavo to
extinguishing the security since he did not
receive US$ 100 if they stipulated that
give his consent (as owner of the property
obligation be paid in foreign currency
under the real estate mortgage) thereto.
(R.A. 4100).

(B). Can Recardo's property be foreclosed to


pay the full balance of the loan? (2%)
Liability; Solidary Liability (2008)
SUGGESTED ANSWER:

No. XV. Eduardo was granted a loan by XYZ


Yes, Recardo's property can be
Bank for the purpose of improving a
foreclosed to pay the full balance of the
building which XYZ leased from him.
loan because when he signed as co-
Eduardo, executed the promissory note
signatory in the promissory note, he
("PN") in favor of the bank, with his friend
acknowledged he is solidarily liable with
Recardo as co-signatory. In the PN, they
Eduardo. In solidary obligations, a
both acknowledged that they are
creditor has the right to demand full
"individually and collectively" liable and
payment of the obligation from any of
waived the need for prior demand. To

“Never Let The Odds Keep You From Pursuing What You Know In Your Heart You Were Meant To Do.”-Leroy Satchel Paige
Page 96 of 180
Civil Law Q&As (2007-2013) hectorchristopher@yahoo.com dbaratbateladot@gmail.com

the solidary debtors (Art. 1207, Civil latter, is obliged to continue the same

Code). until the termination of the affair and its


incidents, or to require the person
concerned to substitute him, if the
owner is in a position to do so (Art.
Obligations; Without Agreement (2007)
2144, NCC).

No.V. What are obligations without an


Second example, a case of solutio
agreement"? Give five examples of
indebiti may also give rise to an
situations giving rise to this type of
obligation without an agreement. This
obligations? (10%)
refers to the obligation to return which
arises when something is received when
SUGGESTED ANSWER:
there is no right to demand it, and it was
"Obligations without an agreement" are unduly delivered through mistake (Art.
obligations that do not arise from 2154, NCC).
contract such as those arising from: 1.
Third example, is when without the
delicts; 2. quasi-delicts; 3. solutio
knowledge of the person obliged to give
indebiti; 4. negotiorum gestio; and 5. all
support, it is given by a stranger, the
other obligations arising from law.
latter shall have a right to claim the

ALTERNATIVE ANSWER: same from the former, unless it appears


that he gave it out of piety and without
"Obligations without an agreement" refer intention of being repaid (Art. 2164,
to the juridical relation of quasi-contract NCC).
which arise from certain lawful,
Fourth example, is when through
voluntary and unilateral acts to the end
accident or other causes a person is
that no one shall be unjustly enriched or
injured or becomes seriously ill, and he
benefited at the expense of another. (Art.
is treated or helped while he is not in a
2142, NCC)
condition to give consent to a contract,
First Example of an obligation without he shall be liable to pay for the services
an agreement is a case of negotiorum of the physician or other person aiding
gestio, whereby one who voluntarily him, unless the service has been
takes charge of the agency or rendered out of pure generosity (Art.
management of the business or property 2167, NCC).
of another without any power from the
“Never Let The Odds Keep You From Pursuing What You Know In Your Heart You Were Meant To Do.”-Leroy Satchel Paige
Page 97 of 180
Civil Law Q&As (2007-2013) hectorchristopher@yahoo.com dbaratbateladot@gmail.com

Fifth instance of an obligation without ought not, in equity and good


an agreement is when the person obliged conscience, to hold (Heirs of Lorenzo
to support an orphan or an insane or Yap v. CA, 371 Phil 523, 1991). The
other indigent person unjustly refuses to following are examples of constructive
give support to the latter, any third trust: 1. Art. 1456 NCC which provides:
person may furnish support to the needy "If property is acquired through mistake
individual, with right of reimbursement or fraud, the person obtaining it is, by
from the person obliged to give support. force of law considered a trustee of an
The provisions of this article apply when implied trust for the benefit of the
the father or mother of a child under person for whom the property comes." 2.
eighteen years of age unjustly refuses to Art 1451 NCC which provides: "When
support him (Art. 2166, NCC). land passes by succession through any
person and he causes the legal title to be
Trust put in the name of another, a trust is
established by implication of law for the
Trust De Son Tort (2007) benefit of the true owner." 3. Art 1454
NCC which provides: "If an absolute
No.III. Explain the following concepts and conveyance of property is made in order
doctrines and give an example of each: to secure the performance of an
obligation of the grantor toward the
(A). concept of trust de son
grantee, a trust by virtue of law is
tort (constructive trust) (5%)
established. If the fulfillment of the
obligation is offered by the grantor when
SUGGESTED ANSWER:
it becomes due, he may demand the
A constructive trust is a trust NOT reconveyance of the property to him." 4.
created by any word or phrase, either Art 1455 NCC which provides: "When any

expressly or impliedly, evincing a direct trustee, guardian or any person holding a

intention to create a trust, but is one fiduciary relationship uses trust funds

that arises in order to satisfy the for the purchase of property and causes

demands of justice. It does not come conveyance to be made to him or to

about by agreement or intention but third person, a trust is established by

mainly operation of law and construed as operation of law in favor of the person to

a trust against one who, by fraud, duress whom the funds belong."

or abuse of confidence, obtains or holds


the legal right to property which he

“Never Let The Odds Keep You From Pursuing What You Know In Your Heart You Were Meant To Do.”-Leroy Satchel Paige
Page 98 of 180
Civil Law Q&As (2007-2013) hectorchristopher@yahoo.com dbaratbateladot@gmail.com

Sales advanced for her employees (biyaheros).


She required them to surrender TCT of
Condominium Act; Partition of a
their properties and to execute the
Condominium (2009) corresponding Deeds of Sale in her favor.
Domeng Bandong was not required to post
No.XVIII. The Ifugao Arms is a
any security but when Eulalia discovered
condominium project in Baguio City. A
that he incurred shortage in cattle
strong earthquake occurred which left huge
procurement operation, he was required to
cracks in the outer walls of the building. As
execute a Deed of Sale over a parcel of land
a result, a number of condominium units
in favor of Eulalia. She sold the property to
were rendered unfit for use. May Edwin,
her grandneice Jocelyn who thereafter
owner of one of the condominium units
instituted an action for ejectment against
affected, legally sue for partition by sale of
the Spouses Bandong.
the whole project? Explain. (4%)

To assert their right, Spouses Bandong filed


SUGGESTED ANSWER:
an action for annulment of sale against
Yes, Edwin may legally sue for partition
Eulalia and Jocelyn alleging that there was
by sale of the whole condominium
no sale intended but only equitable
project under the following conditions:
mortgage for the purpose of securing the
the damage or destruction caused by the
shortage incurred by Domeng in the
earthquake has rendered one-half (1/2)
amount of P 70, 000.00 while employed as
or more of the units therein
"biyahero" by Eulalia. Was the Deed of Sale
untenantable, and (b) that the
between Domeng and Eulalia a contract of
condominium owners holding an
sale or an equitable mortgage? Explain.
aggregate of more than thirty percent
(5%)
(30%) interests of the common areas are
opposed to the restoration of the SUGGESTED ANSWER:
condominium project (Sec 8 [b], Republic
Act No. 472 “Condominium Act”). The contract between Domeng Bandong
and Eulalia was an equitable mortgage
rather than a contract of sale. The
Mortgage; Equitable Mortgage (2012) purported deed of sale was actually
intended to merely secure the payment
No.VI. (b) Eulalia was engaged in the of the shortage incurred by Domeng in
business of buying and selling large cattle. the conduct of the cattle-buying
In order to secure the financial capital, she operations. Under Art 1602, Civil Code,

“Never Let The Odds Keep You From Pursuing What You Know In Your Heart You Were Meant To Do.”-Leroy Satchel Paige
Page 99 of 180
Civil Law Q&As (2007-2013) hectorchristopher@yahoo.com dbaratbateladot@gmail.com

the contract shall be presumed to be an offering P800,000 in ready cash for the
equitable mortgage when it may be fairly land. When Roberto confirmed that he
inferred that the real intention of the could pay in cash as soon as Sergio could
parties is simply to secure the payment get the documentation ready, Sergio
of a debt or the performance of any decided to withdraw his offer to Marcelo,
other obligation. The present transaction hoping to just explain matters to his friend.
was clearly intended to just secure the Marcelo, however, objected when the
shortage incurred by Eulalia because withdrawal was communicated to him,
Bandung remained in possession of the taking the position that they have a firm
property inspite of the execution of the and binding agreement that Sergio cannot
sale. simply walk away from because he has an
option to buy that is duly supported by a
duly accepted valuable consideration.

Option Contract; Liquor & “Pulutan” as


Does Marcelo have a cause of action
Consideration (2013)
against Sergio? (5%)

No.III.Sergio is the registered owner of a


SUGGESTED ANSWER:
500-square meter land. His friend, Marcelo,
who has long been interested in the Yes. Marcelo has a cause of action
property, succeeded in persuading Sergio to against Sergio.
sell it to him. On June 2, 2012, they agreed
on the purchase price of P600,000 and that Under Art. 1324, when the offerer has
Sergio would give Marcelo up to June30, allowed the offeree a certain period to
2012 within which to raise the amount. accept, the offer may be withdrawn at
Marcelo, in a light tone usual between any time before acceptance by
them, said that they should seal their communicating such withdrawal, except
agreement through a case of Jack Daniels when the option is founded upon
Black and P5,000 "pulutan" money which consideration, as something paid or
he immediately handed to Sergio and which promised.
the latter accepted. The friends then sat
An accepted unilateral promise to buy or
down and drank the first bottle from the
sell a determinate thing for a price
case of bourbon.
certain is binding upon him if the
On June 15, 2013, Sergio learned of promise is supported by a consideration
another buyer, Roberto, who was distinct from the price (Art. 1479).

“Never Let The Odds Keep You From Pursuing What You Know In Your Heart You Were Meant To Do.”-Leroy Satchel Paige
Page 100 of 180
Civil Law Q&As (2007-2013) hectorchristopher@yahoo.com dbaratbateladot@gmail.com

Consideration in an option contract may The Statute of Frauds covers an


be anything of value,, unlike in sale agreement for the sale of real property or
where it must be the price certain in of an interest therein. Such agreement is
money or its equivalent (San Miguel unenforceable by action, unless the
Properties Inc. v. Spouses Huang, G.R. same, or some note or memorandum,
No. 137290, July 31, 2000). thereof, be in writing, (Art. 1403 (e),
Civil Code). Here, Marcelo and Sergio
Here, the case of Jack Daniels Black and merely entered into an Option Contract,
the P5,000.00 “pulutan” money was a which refers to a unilateral promise to
consideration to “seal their agreement,” buy or sell, which need not be in writing
an agreement that Marcelo is given until to be enforceable (Sanchez v. Rigos, G.R.
June 30, 2012 to buy the parcel of land. No. L-25494, June 14, 1972, citing
There is also no showing that such Atkins, Kroll and Co. Inc. v. Cua Hian
consideration will be considered part of Tek and Southwestern Sugar & Molasses
the purchase price. Thus, Sergio’s Co. v. Atlantic Gulf & Pacific Co.).
unilateral withdrawal of the offer
violated the Option Contract between ALTERNATIVE ANSWER:
him and Marcelo.
No. Sergio’s claim has no legal basis.
Can Sergio claim that whatever they might
have agreed upon cannot be enforced The contract of sale has already been

because any agreement relating to the sale partially executed which takes it outside

of real property must be supported by the ambit of the Statute of Frauds is

evidence in writing and they never reduced applicable only to executory contracts,

their agreement to writing? (3%) not to contracts that are totally or


partially performed (Carbonnel v. Poncio,
SUGGESTED ANSWER: G.R. No. L-11231, May 12, 1958).

No. Sergio’s claim has no legal basis.

The contract at issue in the present case Right of First Refusal; Lessee; Effect
is the option contract, not the contract (2008)
of sale for the real property. Therefore,
Art. 1403 does not apply. No.XVI. Dux leased his house to Iris for a
period of 2 years, at the rate of P25,000.00

“Never Let The Odds Keep You From Pursuing What You Know In Your Heart You Were Meant To Do.”-Leroy Satchel Paige
Page 101 of 180
Civil Law Q&As (2007-2013) hectorchristopher@yahoo.com dbaratbateladot@gmail.com

monthly, payable annually in advance. The refusal. This makes the mother a buyer
contract stipulated that it may be renewed in bad faith, hence giving more ground
for another 2-year period upon mutual for rescission of the sale to her
agreement of the parties. The contract also (Equatorial Realty, et al. v. Mayfair
granted Iris the right of first refusal to Theater, G.R. No. 106063, 21 Nov. 1996).
purchase the property at any time during
the lease, if Dux decides to sell the property ALTERNATIVE ANSWER:

at the same price that the property is


No, Iris cannot seek rescission of the
offered for sale to a third party. Twenty-
sale of the property to Dux’s mother
three months after execution of the lease
because the sale is not one of those
contract, Dux sold breach of her right of
rescissible contracts under Art. 1381 of
first refusal. Dux said there was no breach
the Civil Code.
because the property was sold to his
mother who is not a third party. Iris filed an
(B). Will the alternative prayer for extension
action to rescind the sale and to compel
of the lease prosper? (2%)
Dux to sell the property to her at the same
price. Alternatively, she asked the court to SUGGESTED ANSWER:
extend the lease for another 2 years on the
same terms. No. The contract stipulated that it may
be renewed for another 2-year period
(A). Can Iris seek rescission of the sale of upon mutual agreement of the parties.
the property to Dux's mother? (3%) Contracts are binding between the
parties; validity or compliance cannot be
SUGGESTED ANSWER:
left to the will of one of the parties (Art.

Yes, because the right of first refusal is 1308, Civil Code).

included in the contract signed by the


ALTERNATIVE ANSWER:
parties. Only if the lessee failed to
exercise the right of first refusal could It depends. The alternative prayer for
the lessor lawfully sell the subject the extension of the lease may prosper if
property to others, under no less than there is a stipulation in the contract of
the same terms and conditions sale; (b) Dux's mother is aware of the
previously offered to the lessee. Granting existing contract of lease; or (c) the lease
that the mother is not a third party, this
is recorded in the Registry of Property
would make her privy to the agreement
(Art. 1676, Civil Code).
of Dux and Iris, aware of the right of first
“Never Let The Odds Keep You From Pursuing What You Know In Your Heart You Were Meant To Do.”-Leroy Satchel Paige
Page 102 of 180
Civil Law Q&As (2007-2013) hectorchristopher@yahoo.com dbaratbateladot@gmail.com

Lease reimbursed the value of the improvements


he introduced. (4%)
Builder; Good Faith; Useful
SUGGESTED ANSWER:
Improvements (2013)

Boboy’s claim that he is a builder in


No.IV.Anselmo is the registered owner of a
good faith has no basis. A builder in good
land and a house that his friend Boboy
faith is someone who occupies the
occupied for a nominal rental and on the
property in concept of an owner. The
condition that Boboy would vacate the
provisions on builder-planter-sower
property on demand. With Anselmo's
under the Civil Code cover cases in
knowledge, Boboy introduced renovations
which the builder, planter and sower
consisting of an additional bedroom, a
believe themselves to be owners of the
covered veranda, and a concrete block
land, or at least, to have a claim of title
fence, at his own expense.
thereto.
Subsequently, Anselmo needed the property
As Boboy is a lessee of the property,
as his residence and thus asked Boboy to
even if he was paying nominal rental,
vacate and turn it over to him. Boboy,
Art. 1678, Civil Code, is applicable.
despite an extension, failed to vacate the
Under this provision, if the lessee
property, forcing Anselmo to send him a
makes, in good faith, useful
written demand to vacate.
improvements which are suitable to the

In his own written reply, Boboy signified use for which the lease is intended,

that he was ready to leave but Anselmo without altering the form or substance of

must first reimburse him the value of the the property leased, the lessor upon the

improvements he introduced on the termination of the lease, shall pay the

property as he is a builder in good faith. lessee one-half of the value of

Anselmo refused, insisting that Boboy improvements at that time. Should the

cannot ask for reimbursement as he is a lessor refuse to reimburse said amount,

mere lessee. Boboy responded by removing the lessee may remove the

the improvements and leaving the building improvements, even though the
in its original state. principal thing may suffer damage
thereby.
(IVa) Resolve Boboy's claim that as a
builder in good faith, he should be

“Never Let The Odds Keep You From Pursuing What You Know In Your Heart You Were Meant To Do.”-Leroy Satchel Paige
Page 103 of 180
Civil Law Q&As (2007-2013) hectorchristopher@yahoo.com dbaratbateladot@gmail.com

(IVb) Can Boboy be held liable for damages lease contracts between Jude and his
for removing the improvements over tenants? Explain your answer. (3%)
Anselmo's objection? (4%)
SUGGESTED ANSWER:
SUGGESTED ANSWER: Yes, Ildefonso must respect the lease
contracts between Jude and his tenants.
No. Boboy cannot be held liable for While it is true that the said lease
damages. contracts were not registered and
annotated on the title to the property,
The lessor, Anselmo, refused to
Ildefonso is still not an innocent
reimburse one-half of the value of the
purchaser for value. He ought to know
improvements, so the lessee, Boboy, may
the existence of the lease because the
remove the same, even though the
building was already occupied by the
principal thing may suffer damage
tenants at the time he bought it.
thereby. If in removing the useful
Applying the principle of caveat emptor,
improvements Boboy caused more
he should have checked and known the
impairment in the property leased than
status of the occupants of their right to
is necessary he will be liable for damages
occupy the building before buying it.
(Art. 1678, Civil Code).

Agency

Lease; Caveat Emptor (2009)


Agency; Sale of a Real Property through
an Agent (2010)
No.VIII. Jude owned a building which he
had leased to several tenants. Without
No.XVI. X was the owner of an unregistered
informing his tenants, Jude sold the
parcel of land in Cabanatuan City. As she
building to Ildefonso. Thereafter, the latter
was abroad, she advised her sister Y via
notified all the tenants that he is the new
overseas call to sell the land and sign a
owner of the building. Ildefonso ordered the
contract of sale on her behalf.
tenants to vacate the premises within thirty
days from notice because he had other Y thus sold the land to B1 on March 31,
plans for the building. The tenants refused 2001 and executed a deed of absolute sale
to vacate, insisting that they will only do so on behalf of X. B1 fully paid the purchase
when the term of their lease shall have price.
expired. Is Ildefonso bound to respect the

“Never Let The Odds Keep You From Pursuing What You Know In Your Heart You Were Meant To Do.”-Leroy Satchel Paige
Page 104 of 180
Civil Law Q&As (2007-2013) hectorchristopher@yahoo.com dbaratbateladot@gmail.com

B2, unaware of the sale of the land to B1, double sales of an immovable property,
signified to Y his interest to buy it but the ownership shall pertain to the
asked Y for her authority from X. Without person who is in good faith was first in
informing X that she had sold the land to possession and in the absence thereof to
B1, Y sought X for a written authority to the person who presents the oldest title,
sell. provide there is good faith.

X e-mailed Y an authority to sell the land. Y In a case, the Supreme Court has held

thereafter sold the land on May 1, 2001 to that in a sale of real estate the execution

B2 on monthly installment basis for two of a notarial document of sale is


tantamount to delivery of the possession
years, the first installment to be paid at the
of the property sold. The ownership of
end of May 2001.
the land therefore pertains to the first
Who between B1 and B2 has a better right buyer. It may also be mentioned that
over the land? Explain. (5%) under Art 3344 no instruments or deed
establishing, transmitting,
SUGGESTED ANSWER: acknowledging, modifying, or
extinguishing right to real property not
B-2 has a better title. This is not a case
registered under Act 496 shall be valid
of double sale. Since the first sale was
except as between the parties. Thus, the
void. The law provides that when a sale
Deed of Sale of B-2 has no binding effect
of a piece of land or any interest therein
on B-1.
is through an agent, the authority of the
latter shall be in writing; otherwise, the
Partnership
sale shall be void (Art 1874, NCC). The
property was sold by Y to B1 wihtout any
Liability; Liability of a Partner (2010)
written authority from the owner X.
Hence, the sale to B1 was void. No.XV. A, B, and C entered into a
partnership to operate a restaurant
ALTERNATIVE ANSWER:
business. When the restaurant had gone
Under the facts, B-1 has a better right to past break-even stage and started to garner
the land. Given the fact that the Deed of considerable profits, C died. A and B
Sale in favor of B-1 and B-2 are not continued the business without dissolving
inscribed in the Registry of Deeds, the the partnership. They in fact opened a
case is governed by Art 1544 of the New branch of the restaurant, incurring
Civil Code which provides that in case of obligations in the process. Creditors started

“Never Let The Odds Keep You From Pursuing What You Know In Your Heart You Were Meant To Do.”-Leroy Satchel Paige
Page 105 of 180
Civil Law Q&As (2007-2013) hectorchristopher@yahoo.com dbaratbateladot@gmail.com

demanding for the payment of their individual properties shall be subject


obligations. first to the payment of his separate
debts (Art 1835. NCC).
(A). Who are liable for the settlement of the
partnership’s obligations? Explain? (3%)

SUGGESTED ANSWER: Oral Partnership (2009)

The two remaining partners, A and B, are No.I. TRUE or FALSE. Answer TRUE if the
liable. When any partner dies and the statement is true, or FALSE if the
business is continued without any statement is false. Explain your answer in
settlement of accounts as between him
not more than two (2) sentences.
or his estate, the surviving partners are
held liable for continuing the business (C). An oral partnership is valid. (1%)
despite the death of C (Art 1841, 1785,
SUGGESTED ANSWER:
par 2, and Art 1833 of NCC).
TRUE. Partnership is a consensual
(B).What are the creditors’ recourse/s? contract, hence, it is valid even though
Explain. (3%) not in writing.

SUGGESTED ANSWER: ALTERNATIVE ANSWER:


TRUE. An oral is a consensual of the
Creditors can file the appropriate
partnership is valid even though not in
actions, for instance, an action for
writing. However, If it involves
collection of sum of money against the
contribution of an immovable property
“partnership at will” and if there are no
or a real right, an oral contract of
sufficient funds, the creditors may go
partnership is void. In such a case, the
after the private properties of A and B
contract of partnership to be valid, must
(Art 816, NCC). Creditors may also sue
be in a public instrument ( Art. 1771
the estate of C. The estate is not
,NCC ), and the inventory of said
excused from the liabilities of the
property signed by the parties must be
partnership even if C is dead already but
attached to said public instrument (Art.
only up to the time that he remained a
1773, NCC).
partner (Art 1829, 1835, par 2, NCC;
Testate Estate of Mota v. Serra, 47 Phil
ALTERNATIVE ANSWER:
464 [1925]). However, the liability of C’s

“Never Let The Odds Keep You From Pursuing What You Know In Your Heart You Were Meant To Do.”-Leroy Satchel Paige
Page 106 of 180
Civil Law Q&As (2007-2013) hectorchristopher@yahoo.com dbaratbateladot@gmail.com

TRUE. Partnership is a consensual Yes, he is not entitled to the return of


contract, hence, it is valid even though his contribution to the capital of the
not in writing. The oral contract of partnership, but only to the net profits
partnership is also valid even if an from the partnership business during the
immovable property or real right is life of the partnership period. If he is a
contributed thereto. While the law, in limited partner, however, he may ask for
such a case, requires the partnership to the return of his contributions as
be in a public document, the law does provided in Art 1856 and 1857, Civil
not expressly declare the contract void if Code.
not executed in the required form
(Article 1409 (7 ,NCC ). And there being Commodatum & Mutuum
nothing in the law from which it can be Mutuum; Interest; Solutio Indebiti
inferred that the said requirement is
(2012)
prohibitory or mandatory (Article 5,
NCC), the said oral contract of No.VI.a) Siga-an granted a loan to
partnership must also be valid. The Villanueva in the amount of P 540, 000.00.
interested party may simply require the Such agreement was not reduced to writing.
contract to be made into a public Siga-an demanded interest which was paid
document in order to comply with the by Villanueva in cash and checks. The total
required form (Article 1357, NCC). The amount Villanueva paid accumulated to P
purpose of the law in requiring a public 1, 200, 000.00. Upon advice of her lawyer,
document is simply to notify the public Villanueva demanded for the return of the
about the contribution. excess amount of P 660, 000.00 which was
ignored by Siga-an.

Share; Demand during the Existence of Is the payment of interest valid? Explain.
Partnership (2012) (3%)

No.X.b) A partner cannot demand the SUGGESTED ANSWER:


return of his share (contribution) during the
existence of a partnership. Do you agree? No, Art. 1956, Civil Code, provides that

Explain your answer. (5%) “no interest shall be due unless it has
been expressly stipulated in writing.”
SUGGESTED ANSWER:
Is solution indebiti applicable? Explain.
(2%)

“Never Let The Odds Keep You From Pursuing What You Know In Your Heart You Were Meant To Do.”-Leroy Satchel Paige
Page 107 of 180
Civil Law Q&As (2007-2013) hectorchristopher@yahoo.com dbaratbateladot@gmail.com

SUGGESTED ANSWER: enforceable ( Article 1403 [2] b, NCC).The


validity of the contract should be
Yes, Solutio Indebiti is applicable distinguished from its enforceability .
because Villanueva Overpaid by
Surety
P600,000.00 representing interest
payment which is not due. He can,
Surety (2010)
therefore, demand its return.
No.III. Define, Enumerate or Explain. (2%
Guaranty each)

Guaranty (2009) (A). What is the difference between


"guaranty" and "suretyship"?
No.I. TRUE or FALSE. Answer TRUE if the
statement is true, or FALSE if the SUGGESTED ANSWER:
statement is false. Explain your answer in
Guaranty and Suretyship distinguished
not more than two (2) sentences.
(1)The obligation in guaranty is
(D). An oral promise of guaranty is valid
secondary; whereas, in suretyship, it is
and binding. (1%)
primary.

SUGGESTED ANSWER : In guranty, the undertaking is to pay if


the principal debtor cannot pay;
FALSE. An oral contract of guaranty, whereas, in suretyship, the undertaking
being a special promise to answer for the is to pay if the principal debtor does not
debt of
pay .
another, is unenforceable unless in
In guranty, the guarantor is entitled to
writing (Article 1403 [2] b, NCC ).
the benefit of excussion; whereas, in

ALTERNATIVE ANSWER: suretyship the surety is not entitled.

TRUE. An oral promise of guaranty is


Liability in guaranty depends upon an
valid and binding. While the contract is
independent agreement to pay the
valid, however ,it is unenforceable
obligations of the principal if he fails to
because it is not writing . Being a special
do so; whereas, in suretyship, the surety
promise answer for the debt, or
assumes liability as a regular party.
miscarriage of another, the Statute of
Frauds requires it to be in writing to be

“Never Let The Odds Keep You From Pursuing What You Know In Your Heart You Were Meant To Do.”-Leroy Satchel Paige
Page 108 of 180
Civil Law Q&As (2007-2013) hectorchristopher@yahoo.com dbaratbateladot@gmail.com

(5)The Guarantor insures the solvency (B). Will your answer to [a] be the same if
of the principal debtor; whereas, the the contract stipulates that upon failure of
surety insures the debt. Rosario to redeem the ring on due date,
Jennifer may immediately sell the ring and
(6)In a guaranty, the guarantor is
appropriate the entire proceeds thereof for
subsidiarlty liable; whereas, in a
herself as full payment of the loan?
suretyship, the surety binds himself
Reasons. (3%)
solidarity with the principal debtor (Art
2047, Civil Code). SUGGESTED ANSWER:
No, my answer will be different. While
Pledge
the contract of pledge is valid, the
stipulation authorizing the pledgee to
Pledge; Pactum Commissorium (2009)
immediately sell the thing pledged is
void under Art 2088 of the New Civil
No.XVII. Rosario obtained a loan of
Code, which provides that “the creditor
P100,000.00 from Jennifer, and pledged
cannot appropriate the things given by
her diamond ring. The contract signed by
way of pledge or mortgage, or dispose of
the parties stipulated that if Rosario is
them xxx.” Jennifer cannot immediately
unable to redeem the ring on due date, she
sell by herself the thing pledged. It must
will execute a document in favor of Jennifer
be foreclosed by selling it at a public
providing that the ring shall automatically
auction in accordance with the
be considered full payment of the loan.
procedure under Art 2112 of the New

(A). Is the contract valid? Explain. (3%) Civil Code.

Torts and Damages


SUGGESTED ANSWER:
The contract is valid because Rosario Damages (2012)
has to execute a document in favor of
Jennifer to transfer the ownership of the No.I. a) Roberto was in Nikko Hotel when he
pledged ring to the latter. The contract bumped into a friend who was then on her
does not amount to pactum way to a wedding reception being held in
commissorium because it does not said hotel. Roberto alleged that he was then
provide for the automatic appropriation invited by his friend to join her at the
by the pledgee of the thing pledged in wedding reception and carried the basket
case of default by the pledgor. full of fruits which she was bringing to the
affair. At the reception, the wedding

“Never Let The Odds Keep You From Pursuing What You Know In Your Heart You Were Meant To Do.”-Leroy Satchel Paige
Page 109 of 180
Civil Law Q&As (2007-2013) hectorchristopher@yahoo.com dbaratbateladot@gmail.com

coordinator of the hotel noticed him and prosper. Otherwise, Roberto’s action will
asked him, allegedly in a loud voice, to not prosper.
leave as he was not in the guest list. He
The hotel is liable for the wrongful acts
retorted that he had been invited to the
of its employees.
affair by his friend, who however denied
doing so. Deeply embarrassed by the COMMENT:
incident, Roberto then sued the hotel for
The facts of the problem are almost
damages under Articles 19 and 21 of the
similar to the facts of Nikko Hotel
Civil Code. Will Roberto’s action prosper?
Manila Garden v. Reyes, G.R. No.
Explain. (5%)
154259, Feb 28, 2005. In the said case,
SUGGESTED ANSWER: however, there is a categorical finding
that the hotel employee did not, exposed
No. Roberto’s action will not prosper.
the complainant to the ridicule, shame
From the facts given in the problem, the
or embarrassment; hence, did not
wedding coordinator did not abuse her
commit any abuse of right. The present
right when she asked him to leave the
problem makes no statement of that
wedding reception because he was not in
finding. In the contrary, the problem
the guest list. Hotel Nikko could not be
states that it is a mere allegation.
held liable for damages as its liable
spring from the liability of its employee
(Nikko Hotel Manila Garden v. Reyes,
G.R. No. 154259, Feb 28, 2005). Damages; Moral & Exemplary (2009)

ALTERNATIVE ANSWER: No.XIV. Rodolfo, married to Sharon, had an


illicit affair with his secretary, Nanette, a
It depends. While the hotel has the right
19-year old girl, and begot a baby girl,
to exclude an uninvited guest from the
Rona. Nanette sued Rodolfo for damages:
wedding reception, that does not give
actual, for hospital and other medical
the hotel the license to humiliate
expenses in delivering the child by
Roberto. If the wedding coordinator of
caesarean section; moral, claiming that
the hotel acted wrongfully e.g. with the
Rodolfo promised to marry her,
abuse of right, unfairly, or in a matter
representing that he was single when, in
that exposed Roberto to unnecessary
fact, he was not; and exemplary, to teach a
ridicule or shame, his action will
lesson to like-minded Lotharios.

“Never Let The Odds Keep You From Pursuing What You Know In Your Heart You Were Meant To Do.”-Leroy Satchel Paige
Page 110 of 180
Civil Law Q&As (2007-2013) hectorchristopher@yahoo.com dbaratbateladot@gmail.com

(A). If you were the judge, would you award Vinzons-Chato filed a Motion to Dismiss
all the claims of Nanette? Explain. (3%) arguing that she cannot be held liable for
damages for acts she performed while in
SUGGESTED ANSWER:
the discharge of her duties as BIR
If Rodolfo's marriage could not have
Commissioner. Is she correct? Explain. (5%)
been possibly known to Nanette or there
is no gross negligence on the part of SUGGESTED ANSWER:
Nanette, Rodolfo could be held liable for
Yes. As a general rule, a public officer is
moral damages.
not liable for acts performed in the
discharge of his duties. The exceptions
If there is gross negligence in a suit for
are when he acted with malice, bad faith,
quasi-delict, exemplary could be
or gross negligence in the performance
awarded.
of his duty, or when his act is in
violation of a Constitutional guaranteed
right and liberties of a person under
Damages; Public Officers acting in the Art32 of the NCC.
Performance of their Duties (2012)
The public officer is not automatically
considered to have violated the rights or
No.II.a) Liwayway Vinzons-Chato was then
liberties of a person simply because the
the Commissioner of Internal Revenue
rule the public officer issued was
while Fortune Tobacco Corporation is an
entity engaged in the manufacture of declared invalid by the court. The

different brands of cigarettes, among which complainant must still allege and prove

are "Champion," "Hope," and "More" the particular injury or prejudice he has

cigarettes. suffered from the violation of his


constitutional right by the issuance of
Fortune filed a complaint against Vinzons- the invalidated rule.
Chato to recover damages for the alleged
The problem does not state any fact
violation of its constitutional rights arising
from which any malice, bad faith or
from Vinzons-Chato’s issuance of Revenue
gross negligence on the part of Vinzons-
Memorandum Circular No. 37-934 (which
Chato may be inferred, or the particular
re-classified Fortune cigarettes as locally
injury or prejudice the complainant may
manufactured with foreign brands and
have suffered as a result of the violation
thereby imposed higher taxes), which the
of his constitutional right. Hence, she
Supreme Court later declared invalid.

“Never Let The Odds Keep You From Pursuing What You Know In Your Heart You Were Meant To Do.”-Leroy Satchel Paige
Page 111 of 180
Civil Law Q&As (2007-2013) hectorchristopher@yahoo.com dbaratbateladot@gmail.com

cannot be held liable. The facts the driver and the defense of diligence is
presented are similar to facts of the case not available.
of Vinzons-Chato v. Fortune, G.R. No.
(B).Would your answer be the same if
141309, Dec 23, 2008.
Rommel was in the car at the time of the
accident? Explain. (2%)

Death Indemnity (2009) SUGGESTED ANSWER:


Yes, my answer would be the same.
No. X. Rommel’s private car, while being Rommel, who was in the car, shall be
driven by the regular family driver, Amado, liable for damages if he could have
hits a pedestrian causing the latter’s death. prevented the misfortune by the use of
Rommel is not in the car when the incident due diligence in supervising his driver
happened. but failed to exercise it (Art. 2184, NCC).
In such case, his liability is solidary with
(A). Is Rommel liable for damages to the
his driver.
heirs of the deceased? Explain. (2%)
ALTERNATIVE ANSWER:
Yes, my answer will be the same except
SUGGESTED ANSWER:
that in such case the liability of the
Yes, Rommel may be held liable for
owner is not presumed. When the owner
damages if he fails to prove that he
is inside the vehicle, he becomes liable
exercised the diligence of a good father
only when it is shown that he could have
of a family (Art. 2180, par 5, NCC) in
prevented the misfortune by the use of
selecting and supervising his family
due diligence (Art. 2184, NCC). For the
driver. The owner is presumed liable
owner to be held liable, the burden of
unless he proves the defense of
proving that he could have prevented
diligence. If the driver was performing
the misfortune rests on the shoulder of
his assigned task when the accident
the victim.
happened, Rommel shall be solidarily
liable with the driver.

In case the driver is convicted of Doctrine of Discovered Peril (Last Clear


reckless imprudence and cannot pay the
Chance) (2007)
civil liability, Rommel is subsidiarily
liable for the damage awarded against No.III. Explain the following concepts and
doctrines and give an example of each:

“Never Let The Odds Keep You From Pursuing What You Know In Your Heart You Were Meant To Do.”-Leroy Satchel Paige
Page 112 of 180
Civil Law Q&As (2007-2013) hectorchristopher@yahoo.com dbaratbateladot@gmail.com

(B). doctrine of discovered peril (last clear typhoon knocked down the fence of the
chance) (5%) pond and the iguana crawled out of the
gate of Primo’s residence. N, a neighbor
SUGGESTED ANSWER: who was passing by, started throwing
stones at the iguana, drawing the iguana to
The doctrine of last clear chance states
move toward him. N panicked and ran but
that where the plaintiff was guilty of
tripped on something and suffered a broken
prior or antecedent negligence, but the
leg.
defendant, who had the ultimate
opportunity to avoid the impending Is anyone liable for N’s injuries? Explain.
harm failed to do so, it is the defendant (4%)
who is liable for all the consequences of
the accident notwithstanding the prior SUGGESTED ANSWER:
negligence of the plaintiff. An example is
No one is liable. The possessor of an
where a person was riding a pony on a
animal or whoever may make use of the
bridge and improperly pulled the pony to
same is responsible for the damage it
the wrong side when he saw a car
may cause, although it may escape or be
coming. The driver of the car did not
lost. This responsibility shall cease only
stop or change direction, and nearly hit
in case the damage should come from
the horse, and, the frightened animal
force majeure or from the fault of the
jumped to its death. The driver of the
person who has suffered damage (Art
car is guilty of negligence because he
2183, NCC).
had a fair opportunity to avoid the
accident and failed to avail himself of
that opportunity. He is liable under the
doctrine of last clear chance (Picart v.
Liability; Special Parental Authority
Smith, 37 Phil. 809, 1918).
(2010)

No.XII. On May 5, 1989, 16-year old


Liability; Owner of a Pet; Fortuitous Rozanno, who was issued a student permit,
Event (2010) drove to school a car, a gift from his
parents. On even date, as his class was
No.XIV. Primo owns a pet iguana which he scheduled to go on a field trip, his teacher
keeps in a man-made pond enclosed by a requested him to accommodate in his car,
fence situated in his residential lot. A as he did, four (4) of his classmates

“Never Let The Odds Keep You From Pursuing What You Know In Your Heart You Were Meant To Do.”-Leroy Satchel Paige
Page 113 of 180
Civil Law Q&As (2007-2013) hectorchristopher@yahoo.com dbaratbateladot@gmail.com

because the van rented by the school was (B). How about the damage to the jeepney?
too crowded. On the way to a museum Explain. (2%)
which the students were scheduled to visit,
Rozanno made a wrong maneuver, causing SUGGESTED ANSWER:

a collision with a jeepney. One of his


With respect to the damages caused to
classmates died. He and the three (3) others
the jeepney, only Rozanno should be
were badly injured.
held liable because his negligence or
tortuous act was the sole, proximate and
(A). Who is liable for the death of Rozanno’s
immediate cause thereof.
classmate and the injuries suffered by
Rozanno and his 3 other classmates?
(C). Under the same facts, except the date
Explain. (2%)
of occurrence of the incident, this time in
mid-1994, what would be your answer?
SUGGESTED ANSWER:
Explain. (2%)
At the time the incident occurred in May
1989, Rozanno was still a minor. Being a SUGGESTED ANSWER:

minor, Art 218 of the Family Code


Since Rozanno was 16 years old in 1989,
applies. Pursuant to Art 218, the school,
if the incident happened sometime in
its administrators and teachers shall be
the middle of 1994, Rozanno have been
liable for the acts of minor Rozanno
21 years old at the time. Hence, he was
because of the special parental authority
already of legal age. The law reducing
and responsibility that they exercise
the age of majority to 18 years took
over him. The authority applies to all
effect in December 1989.
authorized activities, whether inside or
outside the premises of the school, Being of legal age, articles 218, 219, and

entity or institution. The field trip on 221 of the Family Code are no longer

which occasion Rozanno drove the car, applicable. In such case, only Rozanno

was an authorized activity, and , thus, will be personally responsible for all the

covered by the provision. Furthermore, consequences of his act unless his

the parents of Rozanno are subsidiarily school or his parents were themselves

liable pursuant to Art 219 (FC), and also negligent and such negligence

principally liable under Art 221 (FC), if contributed to the happening of the

they are negligent. incident. In that event, the school or his


parents are not liable under Art 218, 218
or 221 of the Family Code, but will be

“Never Let The Odds Keep You From Pursuing What You Know In Your Heart You Were Meant To Do.”-Leroy Satchel Paige
Page 114 of 180
Civil Law Q&As (2007-2013) hectorchristopher@yahoo.com dbaratbateladot@gmail.com

liable under general provision on the court to justify the damages that your client
Civil Code on quasi-delict. claims? (8%)

SUGGESTED ANSWER:

Quasi-Delict; Claims; Requisites (2013)


I will the base the claim of my client on
quasi-delict under Art 2176 of the Civil
No.II. A collision occurred at an intersection
Code of the Philippines. The requisites
involving a bicycle and a taxicab. Both the
for a claim under quasi-delict to prosper
bicycle rider (a businessman then doing his
are as follows:
morning exercise) and the taxi driver
claimed that the other was at fault. Based
Act or omission, there being fault or
on the police report, the bicycle crossed the
negligence;
intersection first but the taxicab, crossing
at a fast clip from the bicycle's left, could Damage or injury; and
not brake in time and hit the bicycle's rear
wheel, toppling it and throwing the bicycle Causal connection between the damage
rider into the sidewalk 5 meters away. and the act or omission.

The bicycle rider suffered a fractured right The case clearly involves quasi-delict
knee, sustained when he fell on his right where my client, the bicycle rider,
side on the concrete side walk. He was suffered injury as a result of the
hospitalized and was subsequently negligence of the over-speeding taxi
operated on, rendering him immobile for 3 driver, without fault on my client’s part.
weeks and requiring physical rehabilitation
for another 3 months. In his complaint for To prove actual damages aside from the

damages, the rider prayed for the award testimony of client, I will present his

ofP1,000,000 actual damages,P200,000 hospital and medical bills. Receipts paid

moral damages, P200,000 exemplary on the rehabilitation will also be

damages, P1 00,000 nominal damages and presented. [The sentence in red should

P50,000 attorney's fees. be replaced with the following sentence


because he is a businessman and not an
Assuming the police report to be correct employee. Furthermore, I will present
and as the lawyer for the bicycle rider, what income tax returns, contracts and other
evidence (documentary and testimonial) documents to prove unrealized profits as
and legal arguments will you present in a result of this temporary injury.] I will

“Never Let The Odds Keep You From Pursuing What You Know In Your Heart You Were Meant To Do.”-Leroy Satchel Paige
Page 115 of 180
Civil Law Q&As (2007-2013) hectorchristopher@yahoo.com dbaratbateladot@gmail.com

also call the attending physician to fraudulent, reckless, oppressive, or


testify as to the extent of the injuries malevolent manner. While the amount of
suffered by my client, and to corroborate exemplary damages may not be proved,
the contents of the medical documents. the plaintiff must show that he is
entitled to moral or compensatory
Based on Art. 2202, in quasi-delicts, the damages. In support of this, I will
defendant shall be liable for all damages present the police report showing the
which are the natural and probable circumstance under which the accident
consequences of the act or omission took place, taking into account the
complained of. It is not necessary that actions of the parties. I will ask the
the damages have been foreseen or could officials who responded to the accident
have been foreseen by the defendant. to testify as to the conduct of the
parties at the time of the accident in
Unlike actual damages, no proof of
order to determine whether defendant
pecuniary loss is necessary in order that
was guilty of gross negligence.
moral, nominal, temperate liquidated or
exemplary damages may be adjudicated. Finally, attorney’s fees may be recovered
The assessment is left to the discretion when exemplary damages are awarded
of the Court (Art. 2216, Civil Code). (Art 2208, Civil Code).
There must be proof pecuniary
estimation, however.

Moral damages can be recovered by my Quasi Tort (2010)


client under Articles 2219 and 2200.
Moral damages may be recovered in case No.III. Define, Enumerate or Explain. (2%

of a quasi-delict causing physical each)


injuries. Additionally, it must be proved
(B). Define quasi tort. Who are the persons
that such damages were the proximate
liable under quasi torts and what are the
result of the act complained of. Medical
defenses available to them?
certificates will be presented, along with
the testimony from my client and other
Note: It is recommended that the examiner
eyewitness accounts, in order to support
exercise leniency and liberality in grading the
the award for moral damages.
answers given to this question. The term quasi-

Exemplary damages may be granted if tort is not part of legal developments in civil
law. In Philippine legal tradition, quasi-
the defendant acted in wanton,

“Never Let The Odds Keep You From Pursuing What You Know In Your Heart You Were Meant To Do.”-Leroy Satchel Paige
Page 116 of 180
Civil Law Q&As (2007-2013) hectorchristopher@yahoo.com dbaratbateladot@gmail.com

delict has been treated as the closest civil who can be held liable and their defenses
law equivalent of the common law tort. In would also apply.
fact, in a number of Supreme Court
Those liable for quasi-delict include:
decisions, the two terms have been
considered synonymous. In reality, however, Those tortfeasor or the person causing
the common law tort is much broader in damage to another through fault or
scope than the civil law quasi-delict. In negligence ( Article 2176 NCC ); and
recent developments in common law, the Persons vicariously liable under Article
concept of “quasi-torts” can be considered as 2180 (NCC ).
the closest common law equivalent of the
civil law concept of quasi-delict. This is
The defenses available include:
because it is argued that the growing
recognition of quasi-torts as a source of That the defendant was not negligent or
obligation is hinged on the acceptance at
that he exercised due diligence ( Article
common law of the civil law principles of
2176 NCC );
quasi-delict.
That although the defendant is negligent
SUGGESTED ANSWER:
his negligence is not the proximate

Quasi -tort is a legal concept upholding cause of the injury ( Article 2179 NCC );

the doctrine that some legal duty exists That the plaintiff's own negligence was
that cannot be classified strictly as a
the immediate and proximate cause of
personal duty (thus resulting in a tort),
his injury ( Article 2179 NCC );
nor as a contractual duty but rather
some other kind of duty recognizable by (d ) That the person vicariously liable

the law. ” Tort “ or ” Quasi-tort” is an has observed all the diligence of a good

Anglo American or Common Law father of a family to prevent damage

concept, while “Delict” or “Quasi-Delict“ ( Article 2180 NCC ); and

is a Civil Law concept (Wikipedia That the cause of action has


encyclopedia). prescribed after the lapse s (Article 2179

ALTERNATIVE ANSWER: NCC ).

Quasi -tort is considered as the The fact that the plaintiff had

equivalent of quasi-delict. Hence the committed contributory negligence is a

rules of the latter pertaining to persons partial defense (Art 2179, NCC).

“Never Let The Odds Keep You From Pursuing What You Know In Your Heart You Were Meant To Do.”-Leroy Satchel Paige
Page 117 of 180
Civil Law Q&As (2007-2013) hectorchristopher@yahoo.com dbaratbateladot@gmail.com

MULTIPLE CHOICE per capita, and not by right of

representation (Art 975, Civil Code)


QUESTIONS
I. (2) How much is Dante's share in the net
2013 Taxation Law Exam
estate? (1%)
MCQ (October 13, 2013) P150,000.
P200,000.
Armand died intestate. His full-blood P300,000.
brothers, Bobby and Conrad, and half- P400,000.
blood brothers, Danny, Edward and Floro, None of the above.
all predeceased him. The following are the
surviving relatives: SUGGESTED ANSWER:
1. Benny and Bonnie, legitimate children of E. None of the above.
Bobby; There is no showing that Danny is an
2. Cesar, legitimate child of Conrad; illegitimate half-blood brother of
3. Dante, illegitimate child of Danny; Armand. In the absence of proof to the
4. Ernie, adopted child of Edward; and contrary, the law presumes that the
5. Felix, grandson of Floro. relationship is legitimate. Thus, Dante,
The net value of Armand's estate is an illegitimate child of Danny, is barred
Pl,200,000. from inheriting from Armand pursuant
to the “iron curtain rule” which
I. (1) How much do Benny and Bonnie disqualifies an illegitimate child from
stand to inherit by right of representation? inheriting ab intestao from the
(1%) legitimate children and relatives of his
P200,000
father or mother, and vice versa (Art
P300,000
992, Civil Code).
P400,000
P150,000 I. (3) How much is Ernie's share in the net
None of the above. estate . (1%)
P 0.
SUGGESTED ANSWER: P400,000.
(E) None of the above. P150,000.
If all the brothers/sisters are disqualified P200,000.
to inherit, the nephews/nieces inherit None of the above.
SUGGESTED ANSWER:

“Never Let The Odds Keep You From Pursuing What You Know In Your Heart You Were Meant To Do.”-Leroy Satchel Paige
Page 118 of 180
Civil Law Q&As (2007-2013) hectorchristopher@yahoo.com dbaratbateladot@gmail.com

(A) 0 or (E) None of the above. Should the share of insolvent debtor C be
The legal relationship created by divided only between the two other
adoption is strictly between the adopter remaining debtors, A and B? (1%)
and the adopted. It does not extend to Yes. Remission of D's share carries with it
the relatives of either party (Sayson v. total extinguishment of his obligation to the
CA, G.R. Nos. 89224-25, Jan 23, 1992). benefit of the solidary debtors.
(Note: “E. None of the above’” is another Yes. The Civil Code recognizes remission as
answer because Ernie has no share at a mode of extinguishing an obligation. This
all in the net estate). clearly applies to D.
No. The rule is that gratuitous acts should
I. (4) How much is Felix's share in the net be restrictively construed, allowing only the
estate? (1%) least transmission of rights.
P400,000. No, as the release of the share of one
P150,000. debtor would then increase the burden of
P300,000. the other debtors without their consent.
P0.
None of the above. SUGGESTED ANSWER:
(D). No, as the release of the share of one
SUGGESTED ANSWER: debtor would then increase the burden of
(D). 0. Or (E) None of the above. the other debtors without their consent.
In the collateral line, representation is When one of the solidary debtors cannot,
granted only to children of brother or because of his insolvency, reimburse his
sisters, Felix is a grandson of a share to the debtor paying the
predeceased brother. obligation, such share shall be borne by
(Note: “E. None of the above: is another all his co-debtors, in proportion to the
answer because Felix has no share at debt of each (Art 1217, Civil Code).
all in the net estate) Additionally, D was released only from
his share of P10,000.00 not from the
A, B, C and D are the solidary debtors of X solidary tie that binds him to A, B and C.
for P40,000. X released D from the payment
of his share of PI 0,000. When the Amador obtained a loan of P300,000 from
obligation became due and demandable, C Basilio payable on March25, 2012. As
turned out to be insolvent. security for the payment of his loan,
Amador constituted a mortgage on his
residential house and lot in Basilio's favor.

“Never Let The Odds Keep You From Pursuing What You Know In Your Heart You Were Meant To Do.”-Leroy Satchel Paige
Page 119 of 180
Civil Law Q&As (2007-2013) hectorchristopher@yahoo.com dbaratbateladot@gmail.com

Cacho, a good friend of Amador, guaranteed Basilio (the creditor) must first be
and obligated himself to pay Basilio, in case exhausted”)
Amador fails to pay his loan at maturity. (2) If Amador sells his residential house and
lot to Diego, can Basilio foreclose the real
(1) If Amador fails to pay Basilio his loan on estate mortgage? (1%)
March 25, 2012, can Basilio compel Cacho
to pay? (1%) Yes, Basilio can foreclose the real estate
mortgage because real estate mortgage
No, Basilio cannot compel Cacho to pay creates a real right that attaches to the
because as guarantor, Cacho can invoke property.
the principle of excussion, i.e., all the Yes, Basilio can foreclose the real estate
assets of Basilio must first be exhausted. mortgage. It is binding upon Diego as the
No, Basilio cannot compel Cacho to pay mortgage is embodied in a public
because Basilio has not exhausted the instrument.
available remedies against Amador. No, Basilio cannot foreclose the real
Yes, Basilio can compel Cacho to pay estate mortgage. The sale confers
because the nature of Cacho's undertaking ownership on the buyer, Diego, who
indicates that he has bound himself must therefore consent.
solidarily with Amador. No, Basilio cannot foreclose the real estate
Yes, Basilio can compel Cacho who bound mortgage. To deprive the new owner of
himself to unconditionally pay in case ownership and possession is unjust and
Amador fails to pay; thus the benefit of inequitable.
excussion will not apply.
SUGGESTED ANSWER:
SUGGESTED ANSWER: Yes, Basilio can foreclose the real estate
No, Basilio cannot compel Cacho to pay mortgage. It is binding upon Diego as the
because Basilio has not exhausted the mortgage is embodied in a public
available remedies against Amador. instrument.
The guarantor cannot be compelled to Since the mortgage is in a public
pay the creditor unless the latter has instrument, there is constructive notice
exhausted all the property of the debtor to Diego, who is the buyer if the
and has resorted to all the legal remedies mortgaged property.
against the debtor (Art. 2058, Civil Code)
(Note: “A” is not the correct answer ALTERNATIVE ANSWER:
because it states that “all the assets of

“Never Let The Odds Keep You From Pursuing What You Know In Your Heart You Were Meant To Do.”-Leroy Satchel Paige
Page 120 of 180
Civil Law Q&As (2007-2013) hectorchristopher@yahoo.com dbaratbateladot@gmail.com

No, Basilio cannot foreclose the real No, Jose's refusal is not justified. The
estate mortgage. The sale confers expenses he incurred are useful for the
ownership on the buyer, Diego, who preservation of the thing loaned. It is
must therefore consent. Jose's obligation to shoulder these useful
The mortgage is not registered, thus, expenses.
cannot be binding against third persons
(Art. 2125, Civil Code) SUGGESTED ANSWER:
No, Jose's refusal is not justified. The
IV. Cruz lent Jose his car until Jose expenses he incurred are useful for the
finished his Bar exams. Soon after Cruz preservation of the thing loaned. It is
delivered the car, Jose brought it to Jose's obligation to shoulder these useful
Mitsubishi Cubao for maintenance check expenses.
up and incurred costs of P8,000. Seeing the In commodatum, the bailee is obliged to
car's peeling and faded paint, Jose also had pay for the ordinary expenses for the use
the car repainted for P10,000. Answer the and preservation of the thing loaned (Art
two questions below based on these 1941, Civil Code).
common facts. The bailee, Jose, has no right of
retention on the ground that the bailor
IV. (1) After the bar exams, Cruz asked for owes him something, even if it may be
the return of his car. Jose said he would by reason of expenses. He can only
return it as soon as Cruz has reimbursed retain it if he suffers damages by reason
him for the car maintenance and repainting of a flaw or defect in the thing loaned of
costs of P 18,000. which the bailor knows (Art 1951, Civil
Is Jose's refusal justified? (1%) Code).
No, Jose's refusal is not justified. In this
kind of contract, Jose is obliged to pay for IV. (2) During the bar exam month, Jose
all the expenses incurred for the lent the car to his girlfriend, Jolie, who
preservation of the thing loaned. parked the car at the Mall of Asia's open
Yes, Jose's refusal is justified. He is obliged parking lot, with the ignition key inside the
to pay for all the ordinary and extraordinary car. Car thieves broke into and took the
expenses, but subject to reimbursement car.
from Cruz.
Yes, Jose's refusal is justified. The principle Is Jose liable to Cruz for the loss of the car
of unjust enrichment warrants the due to Jolie's negligence? (1%)
reimbursement of Jose's expenses.

“Never Let The Odds Keep You From Pursuing What You Know In Your Heart You Were Meant To Do.”-Leroy Satchel Paige
Page 121 of 180
Civil Law Q&As (2007-2013) hectorchristopher@yahoo.com dbaratbateladot@gmail.com

No, Jose is not liable to Cruz as the loss Securities and Exchange Commission,
was not due to his fault or negligence. designated L and 0 as managing partners; L
No, Jose is not liable to Cruz. In the was liable only to the extent of his capital
absence of any prohibition, Jose could lend contribution; and P was not liable for
the car to Jolie. Since the loss was due to losses.
force majeure, neither Jose nor Jolie is In 2006, the partnership earned a net profit
liable. of P800,000. In the same year, P engaged in
Yes, Jose is liable to Cruz. Since Jose a different business with the consent of all
lent the car to Jolie without Cruz's the partners. However, in 2007, the
consent, Jose must bear the consequent partnership incurred a net loss of
loss of the car. P500,000. In 2008,the partners dissolved
Yes, Jose is liable to Cruz. The contract the partnership. The proceeds of the sale of
between them is personal in nature. Jose partnership assets were insufficient to
can neither lend nor lease the car to a third settle its obligation. After liquidation, the
person. partnership had an unpaid liability
ofP300,000.
SUGGESTED ANSWER:
Yes, Jose is liable to Cruz. Since Jose V. (l) Assuming that the just and equitable
lent the car to Jolie without Cruz's share of the industrial partner, P, in the
consent, Jose must bear the consequent profit in 2006 amounted to P1 00,000, how
loss of the car. much is the share of 0, a limited partner, in
The bailee is liable for the loss of the the P800,000 net profit? (1%)
thing, even if it should be through a P160,000.
fortuitous event if he lends or leases the P175,000.
thing to a third person, who is not a P280,000.
member of his household (Art 1942, Civil P200,000.
Code). None of the above.

V. In 2005, L, M, N, 0 and P formed a SUGGESTED ANSWER:


partnership. L, M and N were capitalist (C) P280,000.
partners who contributed P500,000 each, First, deduct the share of P from the
while 0, a limited partner, contributed P1 , profits. P800,000 less P100,000 is
000,000. P joined as an industrial partner, P700,000. Next, get the share of O by
contributing only his services. The Articles following the proportion that the shares
of Partnership, registered with the of L, M, N, O is 1:1:1:2, respectively.

“Never Let The Odds Keep You From Pursuing What You Know In Your Heart You Were Meant To Do.”-Leroy Satchel Paige
Page 122 of 180
Civil Law Q&As (2007-2013) hectorchristopher@yahoo.com dbaratbateladot@gmail.com

No. P is not liable because there is a valid


V. (2) In 2007, how much is the share of 0, stipulation exempting him from losses.
a limited partner, in the net loss of Since the other partners allowed him to
P500,000? (1%) engage in an outside business activity, the
P 0. stipulation absolving P from liability is
P1 00,000. valid. For 0, it is basic that a limited
P125,000. partner is liable only up to the extent of his
P200,000. capital contribution.
None of the above. Yes. The stipulations exempting P and L
from losses are not binding upon the
SUGGESTED ANSWER: creditors. 0 is likewise liable because the
(D) P200,000 partnership was not formed in accordance
A limited partner shall not become liable with the requirements of a limited
a s a general partner unless, in addition partnership.
to the exercise of his rights and powers No. The Civil Code allows the partners to
as a limited partner, he takes part in the stipulate that a partner shall not be liable
control of the business (Art 1948, Civil for losses. The registration of the Articles of
Code). In the absence of stipulation as to Partnership embodying such stipulations
profits and losses, the share of each serves as constructive notice to the
partner in the losses shall be partnership creditors.(E) None of the above

proportionate to what he may have is completely accurate.

contributed (Art 1797). None of the above is completely

accurate.
V. (3) Can the partnership creditors hold L,
0 and Pliable after all the assets of the SUGGESTED ANSWER:
partnership are exhausted? (1%) None of the above is completely
Yes. The stipulation exempting P from accurate.
losses is valid only among the partners. L is VI. Gary is a tobacco trader and also a
liable because the agreement limiting his lending investor. He sold tobacco leaves to
liability to his capital contribution is not Homer for delivery within a month,
valid insofar as the creditors are concerned. although the period for delivery was not
Having taken part in the management of guaranteed. Despite Gary's efforts to deliver
the partnership, 0 is liable as capitalist on time, transportation problems and
partner. government red tape hindered his efforts
and he could only deliver after 30 days.

“Never Let The Odds Keep You From Pursuing What You Know In Your Heart You Were Meant To Do.”-Leroy Satchel Paige
Page 123 of 180
Civil Law Q&As (2007-2013) hectorchristopher@yahoo.com dbaratbateladot@gmail.com

Homer refused to accept the late delivery No. Homer was not justified in refusing to
and to pay on the ground that the agreed accept the tobacco leaves. There was no
term had not been complied with. term in the contract but a mixed condition.
As lending investor, Gary granted a The fulfillment of the condition did not
Pl,000,000 loan to Isaac to be paid within depend purely on Gary's will but on other
two years from execution of the contract. As factors, e.g., the shipping company and the
security for the loan, Isaac promised to government. Homer should comply with his
deliver to Gary his Toyota Innova within obligation.
seven (7) days, but Isaac failed to do so.
Gary was thus compelled to demand SUGGESTED ANSWER:
payment for the loan before the end of the No. Homer was not justified in refusing
agreed two-year term. to accept the tobacco leaves. He
consented to the terms and conditions
VI. (l) Was Homer justified in refusing to of the sale and must abide by it.
accept the tobacco leaves? (1%) Obligations arising from contract have
Yes. Homer was justified in refusing to the force of law between the contracting
accept the tobacco leaves. The delivery was parties.
to be made within a month. Gary's promise It is clear under the facts that the period
of delivery on a "best effort" basis made the of delivery of the tobacco leaves was not
delivery uncertain. The term, therefore, was guaranteed. Gary anticipated other
ambiguous. factors which may prevent him from
No. Homer was not justified in refusing making the delivery within a month.
to accept the tobacco leaves. He True enough, transportation problems
consented to the terms and conditions and government red tape did. Such
of the sale and must abide by it. slight delay was, thus, excusable.
Obligations arising from contract have Obligations arising from contract have
the force of law between the contracting the force of law between the contracting
parties. parties and should be complied with in
Yes. Homer was justified in his refusal to good faith (Art. 1160, Civil Code)
accept the delivery. The contract
contemplates an obligation with a term. VI. (2) Can Gary compel Isaac to pay his
Since the delivery was made after 30 days, loan even before the end of the two-year
contrary to the terms agreed upon, Gary period? (1%)
could not insist that Homer accept the
tobacco leaves.

“Never Let The Odds Keep You From Pursuing What You Know In Your Heart You Were Meant To Do.”-Leroy Satchel Paige
Page 124 of 180
Civil Law Q&As (2007-2013) hectorchristopher@yahoo.com dbaratbateladot@gmail.com

Yes, Gary can compel Isaac to Under Art 1198 (2) of the Civil Code, the
immediately pay the loan. Non- debtor shall lose every right to make use
compliance with the promised guaranty of the period when he does not furnish
or security renders the obligation to the creditor the guaranties or
immediately demandable. Isaac lost his securities which he has promised.
right to make use of the period.
Yes, Gary can compel Isaac to immediately VII. Lito was a commercial pilot who flew for
pay the loan. The delivery of the Toyota Pacific-Micronesian Air. In 1998, he was
Innova is a condition for the loan. Isaac's the co-pilot of the airline's Flight MA916
failure to deliver the car violated the that mysteriously disappeared two hours
condition upon which the loan was granted. after take-off from Agana, Guam,
It is but fair for Gary to demand immediate presumably over the Pacific Ocean. No trace
payment. of the plane and its 105 passengers and
No, Gary cannot compel Isaac to crew was ever found despite diligent search;
immediately pay the loan. The delivery of Lito himself was never heard of again. Lito
the car as security for the loan is an left behind his wife, Lita, and their two
accessory contract; the principal contract is children.
still the P 1,000,000 loan. Thus, Isaac can
still make use of the period. In 2008, Lita met and married Jaime. They
No, Gary cannot compel Isaac to now have a child of their own.
immediately pay the loan. Equity dictates While on a tour with her former high school
that Gary should have granted a reasonable classmates in a remote province of China in
extension of time for Isaac to deliver his 2010, Lita was surprised to see Lito or
Toyota Innova. It would be unfair and somebody who looked exactly like him, but
burdensome for Isaac to pay she was sure it was Lito because of the
the P 1,000,000 simplybecause the extreme surprise that registered in his face
promised security was not delivered. when he also saw her. Shocked, she
immediately fled to her hotel and post haste
SUGGESTED ANSWER: returned to the country the next day. Lita
Yes, Gary can compel Isaac to now comes to you for legal advice. She asks
immediately pay the loan. Non- you the following questions:
compliance with the promised guaranty
or security renders the obligation VII. (l) If Lito is alive, what is the status of
immediately demandable. Isaac lost his his marriage to Lita? (1%)
right to make use of the period.

“Never Let The Odds Keep You From Pursuing What You Know In Your Heart You Were Meant To Do.”-Leroy Satchel Paige
Page 125 of 180
Civil Law Q&As (2007-2013) hectorchristopher@yahoo.com dbaratbateladot@gmail.com

The marriage subsists because the marital The marriage is valid. After an absence of
bond has not been terminated by death. more than 10 years, Lito is already
The marriage was terminated when Lita presumed dead for all purposes.
married Jaime. The marriage is void. Lito's mere absence,
The marriage subsists because Lita's however lengthy, is insufficient to authorize
marriage to Jaime is void. Lita to contract a subsequent marriage.
The marriage is terminated because Lito is The marriage is void. If Lito is indeed alive,
presumed dead after his plane has been his marriage to Lita was never dissolved
missing for more than 4 years. and they can resume their marital relations
The marriage can be formally declared at any time.
terminated if Lito would not resurface.
SUGGESTED ANSWER:
SUGGESTED ANSWER: Any answer is correct.
The marriage subsists because Lita's Under Art 390 of the Civil Code, after an
marriage to Jaime is void. absence of seven years, it being
For the purpose of contracting the unknown whether or not the absentee
subsequent marriage under Art 41 of the still lives, he shall be presumed dead doe
Family Code, the spouse present must all purposes, except for those of
institute a summary proceeding as succession. This provision was not
provided in the Family Code for the repealed by the present Family Code.
declaration of presumptive death of the Applying this to the problem, (A) may be
absentee, without prejudice to the effect correct. (B) may also be correct. (C) and
of the reappearance of the absent may also be correct under Art 41 of the
spouse. Family Code.

VIII.Which of the following actions or


VII. (2) If Lito is alive, what is the status of defenses are meritorious: (1%)
Lita's marriage to Jaime? (1%)
An action for recovery of down payment
The marriage is valid because Lita's paid under a rescinded oral sale of real
marriage to Lito was terminated upon Lito's property.
disappearance for more than seven years. A defense in an action for ejectment that
the lessor verbally promised to extend or
renew the lease.

“Never Let The Odds Keep You From Pursuing What You Know In Your Heart You Were Meant To Do.”-Leroy Satchel Paige
Page 126 of 180
Civil Law Q&As (2007-2013) hectorchristopher@yahoo.com dbaratbateladot@gmail.com

An action for payment of sum of money


filed against one who orally promised to ALTERNATIVE ANSWER:
answer another's debt in case the latter (E) None of the above.
defaults. The recovery of the down payments
A defense in an action for damages that the should be made in the same action for
debtor has sufficient, but unliquidated rescission. Otherwise, it would be a
assets to satisfy the credit acquired when it ground for dismissal under Rule 2, Sec 4
becomes due. of Rules of Court.
None of the above. Lease of a real property is covered by the
Statute of Frauds. Furthermore, it also
SUGGESTED ANSWER: consists of interest in real property.
An action for recovery of down payment Hence, it must be in writing. (Art 1403,
paid under a rescinded oral sale of real Civil Code)
property. A contract of guaranty is a promise to
An oral sale of real property is an answer for the debt of another and
unenforceable contract under the hence, it is also covered by the Statute
Statute of Frauds. Since, in the problem, of Frauds. It must be in writing before it
the vendee paid down payment, it takes can be enforced in a court action. (Art
it out of the ambit of Statute of Frauds. 1403, Civil Code)
The rescission here must be in the sense The fact that a debtor has unliquidated
of resolution of the reciprocal obligation assets does not excuse him from paying
arising from the contract of sale. If his debt.
rescinded (resolved) by the vendee on In the technical meaning of rescission
account of the vendors’ failure to deliver under Art 1191 of the Civil Code will be
the thing sold, the parties will go back to adhered to, then there is no absolutely
their status prior to the contract. If the correct answer. Hence, letter E is also a
vendor refuses to return the down possible answer.
payment, then the vendee can file an
action to recover the down payment. IX. Betty entrusted to her agent, Aida,
If, on the other hand, the vendor and the several pieces of jewelry to be sold on
vendee mutually agree to rescind i.e. commission with the express obligation to
cancel the contract, the vendee likewise turn over to Betty the proceeds of the sale,
can file an action for the recovery of the or to return the jewelries if not sold in a
down payment on the basis of solution month's time. Instead of selling the
indebiti. jewelries, Aida pawned them with the

“Never Let The Odds Keep You From Pursuing What You Know In Your Heart You Were Meant To Do.”-Leroy Satchel Paige
Page 127 of 180
Civil Law Q&As (2007-2013) hectorchristopher@yahoo.com dbaratbateladot@gmail.com

Tambunting Pawnshop, and used the I will rule in favor of Tambunting. Its good
money for herself. Aida failed to redeem the faith takes precedence over the right of
pawned jewelries and after a month, Betty Betty to recover the jewelries.
discovered what Aida had done. Betty I will rule in favor of Tambunting. Good
brought criminal charges which resulted in faith is always presumed. Tambunting's
Aida's conviction for estafa. lawful acquisition in the ordinary course of
business coupled with good faith gives it
Betty thereafter filed an action against legal right over the jewelries.
Tambunting Pawnshop for the recovery of
the jewelries. Tambunting raised the SUGGESTED ANSWER:
defense of ownership, additionally arguing I will rule in favor of Betty. My ruling is
that it is duly licensed to engage in the based on the Civil Code provision that
pawnshop and lending business, and that it one who has lost any movable or has
accepted the mortgage of the jewelry in been unlawfully deprived thereof may
good faith and in the regular course of its recover it from the person in possession
business. of the same. Tam bunting's claim of good
faith is inconsequential.
If you were the judge, how will you decide
the case? (1%) Although possession of movable property
acquired in good faith is equivalent to a
I will rule in favor of Betty. My ruling is title, nevertheless, one who has lost any
based on the Civil Code provision that movable or has been unlawfully deprived
one who has lost any movable or has thereof may recover it from the person
been unlawfully deprived thereof may in possession of the same. Betty has
recover it from the person in possession been deprived unlawfully of her jewelries
of the same. Tam bunting's claim of good by the estafa committed by Aida. The
faith is inconsequential. pledge of the said jewelries by Aida to
I will rule in favor of Betty. Tambunting's Tambunting pawnshop is void because
claim of good faith pales into insignificance the pledgor is not the owner (Art 2085
in light of the unlawful deprivation of the (2), Civil Code). Tambunting’s claim of
jewelries. However, equity dictates that good faith is inconsequential, because,
Tambunting must be reimbursed for the aside from good faith, Tambunting must
pawn value of the jewelries. prove also that it acquired the jewelries
at a public sale in order to be able to
retain the jewelries until reimbursed by

“Never Let The Odds Keep You From Pursuing What You Know In Your Heart You Were Meant To Do.”-Leroy Satchel Paige
Page 128 of 180
Civil Law Q&As (2007-2013) hectorchristopher@yahoo.com dbaratbateladot@gmail.com

Betty the amount of loan including If you were the judge, would you grant
interest (Art 559, Civil Code). Arlene's motion? (1%)

The only exception the law allows is Yes, I will grant the motion because the
when there is acquisition in good faith of lease contract between Arlene and Janet
the possessor at a public sale, in which was not in writing, hence, Janet may not
case, the owner cannot obtain its return enforce any right arising from the same
without reimbursing the price (Dizon v. contract.
Suntay, 47 SCRA 160, Sept 29, 1972). No, I will not grant the motion because to
allow Arlene to retain the advance
X. Arlene owns a row of apartment houses payments would amount to unjust
in Kamuning, Quezon City. She agreed to enrichment.
lease Apartment No. 1 to Janet for a period Yes, I will grant the motion because the
of 18 months at the rate of P10,000 per action for recovery is premature; Janet
month. The lease was not covered by any should first secure a judicial rescission of
contract. Janet promptly gave Arlene two the contract of lease.
months deposit and 18 checks covering the No. I will not grant the motion because
rental payment for 18 months. This show of the cause of action does not seek to
good faith prompted Arlene to promise enforce any right under the contract of
Janet that should Arlene decide to sell the lease.
property, she would give Janet the right of
first refusal. SUGGESTED ANSWER:
X. (1) Not long after Janet moved in, she No. I will not grant the motion because
received news that her application for a the cause of action does not seek to
Master of Laws scholarship at King's enforce any right under the contract of
College in London had been approved. lease.
Since her acceptance of the scholarship Janet is not asking for the continued use
entailed a transfer of residence, Janet of the leased premises. Moreover, the
asked Arlene to return the advance rental contract is aside the ambit of the
payments she made. Arlene refused, Statute of Frauds as the same has
prompting Janet to file an action to recover already been partially performed.
the payments. Arlene filed a motion to
dismiss, claiming that the lease on which X. (2)Assume that Janet decided not to
the action is based, is unenforceable. accept the scholarship and continued
leasing Apartment No. 1. Midway through

“Never Let The Odds Keep You From Pursuing What You Know In Your Heart You Were Meant To Do.”-Leroy Satchel Paige
Page 129 of 180
Civil Law Q&As (2007-2013) hectorchristopher@yahoo.com dbaratbateladot@gmail.com

the lease period, Arlene decided to sell Frauds under Art 1403 (2)(e) of the Civil
Apartment No. 1 to Jun in breach of her Code. It must be in writing in order to be
promise to Janet to grant her the right of enforceable.
first refusal. Thus, Janet filed an action
seeking the recognition of her right of first
refusal, the payment of damages for the 2012 Taxation Law Exam
violation of this right, and the rescission of
the sale between Arlene and Jun.
MCQ (October 14, 2012)

Which of the following is NOT included in


Is Janet's action meritorious? (1%)
the attributes of juridical capacity?
Juridical capacity is inherent in
Yes, under the Civil Code, a promise to buy
every natural person, and therefore
and sell a determinate thing is reciprocally
it is not acquired.
demandable.
Juridical capacity is lost only
No, the promise to buy and sell a
through death.
determinate thing was not supported by a
Juridical capacity is the fitness to be
consideration.
the subject of legal relations.
Yes, Janet's right of first refusal was clearly
Juridical capacity cannot exist
violated when the property was not offered
without capacity to act.
for sale to her before it was sold to Jun.
No, a right of first refusal involves an
Which of the following is NOT a restriction
interest over real property that must be
on one’s capacity to act?
embodied in a written contract to be
Minority
enforceable.
Marriage
None of the above.
Deaf-mute
Civil Interdiction
SUGGESTED ANSWER:
No, a right of first refusal involves an
SUGGESTED ANSWER:
interest over real property that must be
This question should be disregarded.
embodied in a written contract to be
(NOTE: There is no correct answer among
enforceable.
the choices given. All choices are restrictions
The right of first refusal involves a
on one’s capacity to act. While Marriage is
transfer of interest in the real property.
the only one not mentioned in Articles 38
As such, it is covered by the Statute of
and 39 of the NCC as a restriction on

“Never Let The Odds Keep You From Pursuing What You Know In Your Heart You Were Meant To Do.”-Leroy Satchel Paige
Page 130 of 180
Civil Law Q&As (2007-2013) hectorchristopher@yahoo.com dbaratbateladot@gmail.com

capacity to act, it restricts the capacity of a Yes, the will is not valid under
married person in cases of adoption.) Philippine law.
No, attestation clause is not an
This attribute or incident of a case act of the testator.
determine whether it is a conflict-of-laws No, the governing law is Spanish
case or one covered by domestic law. law.
Cause of action
Foreign element Note: The facts do not state the Law
Jurisdiction observed by the testator in executing his
Forum non conveniens will. He could have observed Spanish Law
or Philippine Law (see comment of Tolentino
The capacity of an heir to succeed shall be to Art. 815 NCC in 3Tolentino117, 1992). If
governed by the: he observed Spanish Law, the opposition is
national law of the decedent’s heirs not correct because the will is valid under
law of the country where the Spanish Law, hence choice (d) is the correct
decedent was a resident at the time answer. If he observed Philippine Law, the
of his death opposition is still not correct because
national law of the person who Philippine Law does not require the testator
died to sign the Attestation Clause of his will,
law of the country where the said clause not being his act. In such case,
properties of the decedent are choice (c) is the correct answer).
located.
Ramon, a Filipino, executed a will in
Atty. BUKO, a Filipino, executed a will Manila, where he left his house and located
while he was in Spain. The attestation in BP Homes Parañaque in favor of his
clause of the said will does not contain Filipino son, Ramgen. Ramon’s other
Buko’s signature. It is valid under Spanish children RJ and Ramona, both Turkish
law. At its probate in Manila, it is being nationals, are disputing the bequest to
opposed on the ground that the attestation Ramgen. They plotted to kill Ramgen.
clause does not contain BUKO’s signature. Ramon learned of the plot, so he tore his
Is the opposition correct? Choose the best will in two pieces out of anger. Which
answer.. statement is most accurate?
Yes, because it is a fatal defect. The mere act of Ramon Sr. is
immaterial because the will is still
readable.

“Never Let The Odds Keep You From Pursuing What You Know In Your Heart You Were Meant To Do.”-Leroy Satchel Paige
Page 131 of 180
Civil Law Q&As (2007-2013) hectorchristopher@yahoo.com dbaratbateladot@gmail.com

The mere act of tearing the will


amounts to revocation. A Japanese national and a Filipino national
The tearing of the will may entered into a contract for services in
amount to revocation if coupled Thailand. The services will be rendered in
with intent of revoking it. Singapore. In case of breach, what law will
The act of tearing the will is govern?
material. Thailand law
Philippine law
Even if the applicable law is a foreign law, a Singapore law
count in the Philippines may be Japanese law
constrained to apply Philippine law under
any of the following instances, except: Pedro (Filipino) and his wife Jane
when the foreign law, judgment or (American) executed a joint will in Canada,
contract is contrary to a sound and where such joint will is valid. In case the
important public policy of the joint will is probated in Japan, what law
forum; will govern the formalities of the joint will?
when the property subject of the American law
case is located outside of the Philippine law
Philippines; Canadian law
when the foreign law or judgment Japanese law
is penal in nature;
when the foreign law is procedural A French national revokes his will in Japan
in nature. where he is domiciled. He then changed his
domicile to the Philippines where he died.
If a will is executed by a testator who was The revocation of his will in Japan is valid
born a Filipino citizen but became under Japanese law but invalid under
naturalized Japanese citizen at the time of Philippine law. The affected heir is a
his death, what law will govern its Malaysian national residing in the
testamentary provisions if the will is Philippines. What law will apply?
executed in China and the property being Japanese law
disposed is located in Indonesia? Philippine law
Chinese law French law
Philippine law Malaysian law
Indonesia law
Japanese law

“Never Let The Odds Keep You From Pursuing What You Know In Your Heart You Were Meant To Do.”-Leroy Satchel Paige
Page 132 of 180
Civil Law Q&As (2007-2013) hectorchristopher@yahoo.com dbaratbateladot@gmail.com

In the absence of contrary stipulation in a obligation, but Bill fails or refuses to pay,
marriage settlement, property relations of what law will govern?
Filipino spouses shall be governed by --- American law
Philippines laws Philippine law
Law of the place where the spouses Australian law
reside Japanese law
Law of the place where the (Facts for item numbers 15-18)
properties are situated In 1989, Charice (Filipina) and Justine
Law of the place where they were (American), were married in the Philippines.
married. In 1990, they separated and Justine went
to Las Vegas where he obtained a divorce in
The will of a Filipino executed in a foreign the same year. He then married another
country --- Filipina, Lea, in Canada on January 1,
cannot be probated in the 1992. They had two (2) sons, James and
Philippines; John (who were both born in 1992). In
may be probated in the Philippines 1993, after failing to hear from Justine,
provided that properties in the Charice married Bugoy (a Filipino), by
estate are located in the Philippines; whom she had a daughter, Regine. In 2009,
cannot be probated before the death Regine married James (son of Justine with
of the testator; Lea) in California, where such marriage is
may be probated in the valid.
Philippines provided it was
executed in accordance with the What is the current status of the marriage
laws of the place where the will of Charice and Justine under Philippine
was executed. laws?
Valid
Pedro (Filipino and Bill (American) entered Void
into a contract in Australia, whereby it was Voidable
agreed that Pedro will build a commercial Dissolved
building for Bill in the Philippines, and in (Note: While Art 26 of the FC does not
payment for the construction, Bill will categorically provide that the first marriage
transfer and convey his cattle ranch located is dissolved by the divorce obtained by the
in Japan in favor of Pedro. In case Pedro foreign spouse abroad, but provides that
performs his such divorce merely gives the Filipino
spouse the capacity to contract a second
marriage, it

“Never Let The Odds Keep You From Pursuing What You Know In Your Heart You Were Meant To Do.”-Leroy Satchel Paige
Page 133 of 180
Civil Law Q&As (2007-2013) hectorchristopher@yahoo.com dbaratbateladot@gmail.com

is believed that the dissolution of the first examinations unless he marries Princess.
marriage us the necessary consequence of As a consequence of the threat, Ricky
the foreign divorce.) married Princess. Can the marriage be
annulled on the ground of intimidation
What id the status of the marriage between under Article 45 of the Family Code?
Charice and Bugoy under Philippine laws? Choose the best answer.
Valid Yes, because without the threat,
Void Ricky would not have married
Voidable Princess.
Unenforceable Yes, because the threat to enforce
the claim of Princess vitiates the
What is the status of the marriage between consent of Ricky in contracting the
Charice and Bugoy under Philippine laws? marriage.
Valid No, because the threat made by
Void Marforth is just and legal.
Voidable No, because Marforth is not a party
Unenforceable to the contract of marriage between
Princess and Ricky.
What is the status of the marriage between
Regine and James under Philippine laws? Audrey, single, bought a parcel of land in
Valid Malolos City from Franco for P 1Million. A
Void contract was executed between them which
Voidable already vested upon Audrey full ownership
Unenforceable of the property, although payable in
monthly installments for a period of four (4)
Ricky and Princess were sweethearts. years. One (1) year after the execution of
Princess became pregnant. Knowing that the contract, Audrey got married to Arnel.
Ricky is preparing for the examinations, They executed a marriage settlement
Marforth, a lawyer and cousin of Princess, whereby they agreed that their properties
threatened Ricky with the filing of a shall be governed by the regime of conjugal
complaint for immorality in the Supreme partnership of gains. Thereafter,
Court, thus preventing him from taking subsequent installments were paid from the
conjugal partnership funds. Is the land
conjugal or paraphernal?

“Never Let The Odds Keep You From Pursuing What You Know In Your Heart You Were Meant To Do.”-Leroy Satchel Paige
Page 134 of 180
Civil Law Q&As (2007-2013) hectorchristopher@yahoo.com dbaratbateladot@gmail.com

The land is conjugal because the the laws of such consular official. Under
installments were paid from the Philippine law, what is the status of the
conjugal partnership funds. marriage of Agay and Topacio? Choose the
The land is paraphernal because best answer.
ownership thereof was acquired Void, because the consular official
before the marriage. only has authority to solemnize
The land is both conjugal and marriages between Filipinos.
paraphernal funds of installments Valid, because according to the laws
were paid from both the personal of Australia, such consular official
funds of Audrey and the conjugal has authority to celebrate the
partnership funds. marriage.
The land is paraphernal because it Voidable, because there is an
was Audrey who purchased the irregularity in the authority of the
same. consular official to solemnize
marriages.
Ernesto donated a mobile phone worth P Valid, because such marriage is
32,000 to Hubert orally and delivered the recognized as valid in the place
unit to Hubert who accepted. Which where it was celebrated.
statement is most accurate?
The donation is void and Ernesto (Note: The issues in the problem is whether
may get mobile phone back. or not the fact that one of the parties to the
The donation is void but Ernesto marriage was an alien constituted absence
cannot get the mobile phone back. of authority or mere irregularity of authority.
The donation is voidable and may be The problem only give the choice, letter (a),
anulled. in case it is interpreted as absence of
The donation is valid. authority. The problem does not give a
choice in case it is interpreted as an
Agay, a Filipino citizen and Topacio, an irregularity thereby making all the other
Australian citizen, got married in the answers wrong).
consular office of the Philippines in
Australia. According to the laws of
Australia, a marriage solemnized by a Separation of property between spouses
consular official is valid, provided that such during the marriage may take place only:
marriage is celebrated in accordance with by agreement of the spouses.

“Never Let The Odds Keep You From Pursuing What You Know In Your Heart You Were Meant To Do.”-Leroy Satchel Paige
Page 135 of 180
Civil Law Q&As (2007-2013) hectorchristopher@yahoo.com dbaratbateladot@gmail.com

If one of the spouses has given No trial shall be held without the 6-
ground for legal separation. month cooling off period being
Upon order of the court. observed.
If one spouse has abandoned the The spouses will be entitled to live
other. separately upon the start of the
trial.
The husband may impugn the legitimacy of The prosecuting attorney has to
his child but not on the ground that: conduct his own investigation.
the wife is suspected of infidelity.
the husband had a serious illness A husband by chance discovered hidden
that prevented him from engaging in treasure on the paraphernal property of his
sexual intercourse. wife. Who owns the discovered treasure?
they were living apart. The half pertaining to the husband
he is physically incapable of sexual (finder) belongs to the conjugal
intercourse. partnership.
The half pertaining to the wife (as
A marriage is void if: owner) belongs to the conjugal
solemnized with a marriage license partnership.
issued without complying with the One half shall belong to the
required 10-day posting. husband as finder and the other
solemnized by a minister whom the half shall belong to the wife as
parties believe to have the authority. owner of the property.
between parties both 23 years of age a and b
but without parental advice.
none of the above Which of the following marriages is void for
reasons of public policy?
In legal separation, which is not correct? Between brothers and sisters,
The aggrieved spouse may file the whether of the full or half blood.
action within five (5) years from the Between step-parents and step
time of the occurrence of the cause. children.
Between parents-in-law and
children-in-law.
b and c

“Never Let The Odds Keep You From Pursuing What You Know In Your Heart You Were Meant To Do.”-Leroy Satchel Paige
Page 136 of 180
Civil Law Q&As (2007-2013) hectorchristopher@yahoo.com dbaratbateladot@gmail.com

The following constitute the different Children conceived or born outside


circumstances or case of fraud which will a valid marriage.
serves as ground for the annulment of a Children born under a valid
marriage, except? marriage, which was later declared
Non-disclosure of the previous void because of the psychological
conviction by final judgment of the incapacity of either or both of the
other party of a crime involving spouses.
moral turpitude. Children conceived and born
Concealment of a sexually- outside a valid marriage.
transmissible disease, regardless of Children born under a valid
its nature, existing at the time of the marriage, but the parents later
marriage. obtained a legal separation.
Concealment of drug addiction,
habitual alcoholism, homosexuality An illegitimate child may use the surname
or lesbianism existing at the time of of his father when his filiation is
marriage. established in any of the following
Concealment by the wife or the instances, except:
husband of the fact of sexual Filiation has been recognized by the
relations prior to the marriage. father through the record of birth
appearing in the civil register
Which of the following is not a requisite for Admission of filiation by the father
a valid donation propter nuptias? in a public document.
The donation must be made before Private handwritten instrument is
the celebration of the marriage. made by the father acknowledging
The donation shall be his filiation.
automatically revoked in case of Affidavit by the mother stating
non-celebration of the marriage. the name of his true father.
The donation must be made in
consideration of the marriage. Under RA 8043, an adopter is required to
The donation must be made in favor be at least ____ years old and ____ years
of one or both of the future spouses. older than the child to be adopted at the
time of the application unless the adopter is
Who are illegitimate children? the parent by nature of the child.
30 and 15
27 and 16

“Never Let The Odds Keep You From Pursuing What You Know In Your Heart You Were Meant To Do.”-Leroy Satchel Paige
Page 137 of 180
Civil Law Q&As (2007-2013) hectorchristopher@yahoo.com dbaratbateladot@gmail.com

50 and 10 Subjects the child or allows him to


18 and 15 be subjected to acts of
lasciviousness.
Under RA 8043, a child qualified to be
adopted is any person below _____ years 37. Which of the following statements is
old. wrong?
18 The possessor in bad faith shall
21 reimburse the fruits received and
15 those which the legitimate possessor
16 could have received.
The possessor in bad faith has
Which of the following DOES NOT result in right of reimbursement for
permanent termination of parental necessary expenses and those for
authority? the production, gathering and
Death of the parents. preservation of the fruits.
Death of the child. The possessor in bad faith is not
Emancipation of the child. entitled to a refund of ornamental
Conviction of the parents of a expenses.
crime which carries with it the The possessor in bad faith is

penalty of civil interdiction. entitled to a refund of useful


expenses.
The court, in an action filed for the
purpose, may suspend parental authority if Which phrase most accurately completes
the parent or the person exercising parental the statement – The expenses incurred in
authority commits any of the following acts, improvements for the luxury or mere
except: pleasure shall not be refunded to thew
Treats the child with excessive possessor in bad faith:
harshness or cruelty. but he may remove the objects for
Gives the child corrupting orders, which such expenses have been
counsel or example. incurred, provided that the thing
Compels the child to take up a suffers no injury thereby, and
course in college against his/her that the lawful possessor does not
will. prefer to retain them.

“Never Let The Odds Keep You From Pursuing What You Know In Your Heart You Were Meant To Do.”-Leroy Satchel Paige
Page 138 of 180
Civil Law Q&As (2007-2013) hectorchristopher@yahoo.com dbaratbateladot@gmail.com

and he may not remove the objects of the dominion over the property as owner.
for which such expenses have been What action is being referred to?
incurred. Accion publiciana
and he may not remove the objects Accion reinvindicatoria
for which such expenses have been Accion interdictal
incurred, unless he pays the value Quieting of Title
they may have at the time he
entered into possession. A summary action to recover physical or
but he may remove the objects for material possession only and must be
which such expenses have been brought within one (1) year from the time
incurred. the cause of action arises. What action is
being referred to?
The following are the limitations on the Accion publiciana
right of ownership imposed by the owner Accion reinvindicatoria
himself, except: Accion interdictal
Will/Succession Quieting of Title
Mortgage
Pledge The following things are property of public
Lease dominion, except:
ports and bridges constructed by
A plenary action for the recovery of the the State.
possession of real estate, upon mere vehicles and weapons of the Armed
allegation and proof of a better right Forces of the Philippines.
thereto, and without allegation of proof of rivers.
title. This action can only be brought after lands reclaimed by the state from
the expiration of one (1) year. What action the sea.
is being referred to?
Accion publiciana SUGGESTED ANSWER:
Accion reinvindicatoria This question should be disregarded
Accion interdictal because there is no correct answer.
Quieting of Title (Note: At first glance, one gets the
impression that vehicles and weapons of the
Action to recover real property based on AFP are not property of the public domain.
ownership. Here, the object is the recovery But they are actually property of the public
dominion under the second paragraph of Art
420 of

“Never Let The Odds Keep You From Pursuing What You Know In Your Heart You Were Meant To Do.”-Leroy Satchel Paige
Page 139 of 180
Civil Law Q&As (2007-2013) hectorchristopher@yahoo.com dbaratbateladot@gmail.com

the NCC. Property of the state which are not the donee accepts the donation.
for public use but are intended for some the donor executes the deed of
public service are properties of the public donation.
dominion. While the vehicles and weapons the donor knows of the donee’s
of the AFP are not for public use, they are acceptance even if the latter has
used for the defense of the State which is a not received the copy of the deed
public service.) of donation.
the donee confirms that the donor
44. Which of the following statements is has learned the former’s acceptance.
wrong?
patrimonial property of the state, The following are the elements of an
when no longer intended for obligation, except:
public use or for public service, Juridical/Legal Tie
shall become property of public Active subject
dominion. Passive subject
all property of the State, which is Consideration
not of public dominion, is
patrimonial property. It is a conduct that may consist of giving,
The property of provinces, cities and doing, or not doing something.
municipalities is divided into Obligation
property for public use and Juridical necessity
patrimonial property. Prestation
Property is either of public dominion Contract
or of private ownership. It is a juridical relation arising from lawful,
voluntary and unilateral acts based on the
The following cannot ask for the reduction principle that no one should unjustly
of inofficious donation, except: enrich himself at the expense of another.
Creditors of the deceased Quasi-contract
Devisees or legatees Quasi-delict
Compulsory heirs of the donor Cotract
The surviving spouse of the donee. Delict

Donation is perfected from the moment The following are the elements of quasi-
--- delict, except:

“Never Let The Odds Keep You From Pursuing What You Know In Your Heart You Were Meant To Do.”-Leroy Satchel Paige
Page 140 of 180
Civil Law Q&As (2007-2013) hectorchristopher@yahoo.com dbaratbateladot@gmail.com

Act or omission It is an international evasion of the faithful


Fault/negligence performance of the obligation.
Damage/injury Negligence
Pre-existing contract Fraud
Delay
A debtor is liable for damages in case of Mistake
delay if he is guilty of any of the following,
except: The following are the requisites of
default (mora) fortuitous event, except:
mistake Cause is independent of the will of
negligence (culpa) the debtor.
breach through contravention of the b) The event
tenor thereof is unforeseeable/unavoidable.
Occurrence renders it absolutely
This term refers to a delay on the part of impossible for the debtor to fulfill his
both the debtor and creditor in reciprocal obligation in a normal manner;
obligations. impossibility must be absolute not
Mora accipiendi partial, otherwise not force majeure.
Mora solvendi Debtor contributed to the
Compensation morae aggravation of the injury to the
Solution indibiti creditor.

The following are the requisites of mora A debtor may still be held liable for loss or
solvendi, except: damages even if it was caused by a
Obligation pertains to the debtor fortuitous event in any of the following
and is determinate, due, instances, except:
demandable, and liquidated. The debtor is guilty of dolo, malice
Obligation was performed on its or bad faith, has promised the same
maturity date. thing to two or more persons who do
There is judicial or extrajudicial not have the same interest.
demand by the creditor. The debtor contributed to the loss.
Failure of the debtor to comply with The thing to be delivered is
such demand. generic.

“Never Let The Odds Keep You From Pursuing What You Know In Your Heart You Were Meant To Do.”-Leroy Satchel Paige
Page 141 of 180
Civil Law Q&As (2007-2013) hectorchristopher@yahoo.com dbaratbateladot@gmail.com

The creditor is guilty of fraud, No, because a motion to dismiss is a


negligence or delay or if he prohibited pleading.
contravened the tenor of the Yes, because Fermin and Toti
obligation. should also pay their share of the
obligation.
Buko, Fermin and Toti bound themselves
solidarily to pay Ayee the amount of P Buko, Fermin and Toti are solidarily
5,000.00. Suppose Buko paid the debtors of Ayee. Twelve (12) years after the
obligation, what is his right as against his obligation became due and demandable,
co-debtors? Buko paid Ayee and later on asked for
Buko cas ask for reimbursement reimbursement of Fermin’s and Toti’s
from Fermin and Toti. shares. Is Buko correct? Why?
Buko can sue Fermin and Toti for No, because the obligation has
damages. already prescribed.
Buko can sue for rescission. Yes, because the obligation is
Buko can claim a refund from Ayee. solidary.
No, because in solidary obligation
Buko, Fermin and Toti bound themselves any one of the solidary debtors can
solidarily to pay Ayee the sum of P pay the entire debt.
10,000.00. When the obligation became due Yes, because Fermin and Toti will be
and demandable, Ayee sued Buko for the unduly enriched at the expense of
payment of the P 10,000.00. Buko moved to Buko.
dismiss on the ground that there was
failure to implead Fermin and Toti who are Buko, Fermin and Toti are solidary
indispensable parties. Will the motion to debtors under a loan obligation of P
dismiss prosper? Why? 300,000.00 which has fallen due. The
Yes, because Fermin and Toti creditor has, however, condoned Fermin’s
should have been impleaded as their entire share in the debt. Since Toti has
obligation is solidary. become insolvent, the creditor makes a
No, because the creditor may demand on Buko to pay the debt. How
proceed against any one of the much, if any, may Buko be compelled to
solidary debtors or some or all of pay?
them simultaneously. P 200.000.00
P 300,000.00
P 100,000.00

“Never Let The Odds Keep You From Pursuing What You Know In Your Heart You Were Meant To Do.”-Leroy Satchel Paige
Page 142 of 180
Civil Law Q&As (2007-2013) hectorchristopher@yahoo.com dbaratbateladot@gmail.com

P 150,000.00 That they be liquidated and


demandable.
Dina bought a car from Jai and delivered a
check in payment of the same. Has Dina 63. Which of the following statements is
paid the obligation? Why? correct?
No, not yet. The delivery of All contracts are perfected by mere
promissory notes payable to consent.
order, or bills of exchange or All contracts are perfected by
other mercantile documents shall delivery of the object.
produce the effect of payment All contracts are required to be in
only when they have been cashed, writing.
or when through the fault of the All contracts are required to have
creditor they have been impaired. a valid consideration.
Yes, because a check is a valid legal
tender of payment. It is a principle which holds that parties are
It depends. If the check is a bound not only by what has been expressly
manager’s check or cashier’s check provided for in the contract but also to the
it will produce the effect of payment. natural consequences that flow out of such
If it’s an ordinary check, no agreement.
payment. Obligatory force of contracts
Yes, because a check is as good as Mutuality of contracts
cash. Autonomy of contracts
The following are the requisites of legal Relativity of contracts
compensation, except:
That each of the obligors is bound It is a principle which holds that contracts
principally and that he be the same must be binding to both parties and its
time a principal creditor of the validity and effectivity can never be left to
other. the will of one of the parties.
That both debts consist in a sum of Obligatory force of contracts
money, or if the things due are Mutuality of contracts
consumable, they be the same kind, Autonomy of contracts
and also of the same quality if the Relativity of contracts
latter has been stated.
That the two (2) debts are not yet It refers to the rule that a contract
due. is binding not only between

“Never Let The Odds Keep You From Pursuing What You Know In Your Heart You Were Meant To Do.”-Leroy Satchel Paige
Page 143 of 180
Civil Law Q&As (2007-2013) hectorchristopher@yahoo.com dbaratbateladot@gmail.com

parties but extends to the heirs, successors Acceptance of the offer by the
in interest, and assignees of the parties, offeree.
provided that the contract involved Qualified/conditional acceptance of
transmissible rights by their nature, or by the offer, which becomes counter-
stipulation or by law. offer.
Obligatory force of contracts Subject matter becomes
Mutuality of contracts illegal/impossible before acceptance
Autonomy of contracts is communicated.
Relativity of contracts
70. Which of the following statements is
It is rule which holds that the freedom of correct?
the parties to contract includes the freedom Offers in interrelated contracts are
to stipulate, provided the stipulations are perfected upon consent.
not contrary to law, morals, good customs, Offers in interrelated contracts
public order or public policy. require a single acceptance.
Obligatory force of contracts c) Business advertisements
Mutuality of contracts are definite offers that require
Autonomy of contracts specific acceptance.
Relativity of contracts Advertisements for Bidders are
only invitations to make
The following are the ways by which proposals and the advertiser is
innominate contracts are regulated, not bound to accept the
except: highest/lowest bidder, unless it
By the stipulation of the parties. appears otherwise.
By the general principles of quasi-
contracts and delicts 71. The following are solemn contracts
By the rules governing the most (Contracts which must appear in writing),
analogous nominate contracts. except:
By the customs of the place. Donations of real estate or of
An offer becomes ineffective on any of the movables if the value exceeds P
following grounds, except: 5,000.00.
Death, civil interdiction, Stipulation to pay interest in loans.
insanity/insolvency of either party Sale of land through an agent
before acceptance is conveyed. (authority must be in writing).

“Never Let The Odds Keep You From Pursuing What You Know In Your Heart You Were Meant To Do.”-Leroy Satchel Paige
Page 144 of 180
Civil Law Q&As (2007-2013) hectorchristopher@yahoo.com dbaratbateladot@gmail.com

Construction contract of a The following are the characteristics of a


building. voidable contract, except:
Effective until set aside.
The following are rescissible contracts, May be assailed/attacked only in an
except: action for that purpose.
Entered into by guardian whenever Can be confirmed or ratified.
ward suffers damage more than ¼ of Can be assailed only by either
value of property. party.
Agreed upon in representation of
absentees, if absentee suffers lesion The following are void contracts, except:
by more than ¼ of value of property. Pactum commissorium
Contracts where fraud is committed Pactum de non alienando
on creditor (accion pauliana). Pactum leonina
Contracts entered into by minors. Pacto de retro

The following are the requisites before a The borrower in a contract of loan or
contract entered into in fraud of creditors mutuum must pay interest to the lender.
may be rescinded, except: If there is an agreement in writing
There must be credited existing to the effect.
prior to the celebration of the As a matter of course.
contract. If the amount borrowed is very
There must be fraud, or at least, the large.
intent to commit fraud to the If the lender so demands at the
prejudice of the creditor seeking maturity date.
rescission.
The creditor cannot in any legal The liability of the school, its administrators
manner collect his credit (subsidiary and teachers, or the individual, entity or
character of rescission) institution engaged in child care over the
The object of the contract must minor child or damage caused by the acts or
be legally in the possession of a omissions of the unemancipated minor while
3rd person in good faith. under their supervision, instruction or
custody shall be:
Joint and subsidiary
Principal and solidary

“Never Let The Odds Keep You From Pursuing What You Know In Your Heart You Were Meant To Do.”-Leroy Satchel Paige
Page 145 of 180
Civil Law Q&As (2007-2013) hectorchristopher@yahoo.com dbaratbateladot@gmail.com

Principal and joint quasi-contract


Subsidiary and solidary. civil
natural
The creditor has the right to the fruits of
the thing from the time: Consent was given by one in representation
the thing is delivered. of another but without authority. The
the obligation to deliver the contract is:
things arises. voidable
the contract is perfected. rescissible
the fruits are delivered. void
unenforceable
If one of the parties to the contract is Michael Fermin, without the authority of
without juridical capacity, the contract is: Pascual Lacas, owner of a car, sold the
voidable same car in the name of Mr. Lacas to Atty.
rescissible Buko. The contract between Atty. Buko and
void Mr. Lacas is ---
unenforceable void because of the absence of
consent from the owner, Mr. Lacas.
When both parties to the contract are valid because all of the essential
minors, the contract is: requisites of a contract are present.
voidable unenforceable because Michael
rescissible Fermin had no authority but he
void sold the car in the name of Mr.
unenforceable Lacas, the owner.
rescissible because the contract
When the consent of one of the parties was caused lesion to Atty. Buko.
vitiated, the contract is:
voidable Which of the following contracts is void?
rescissible An oral sale of a parcel of land.
void A sale of land by an agent in a
unenforceable public instrument where his
authority from the principal is
An obligation which is based on equity and oral.
natural law is known as: A donation of a wrist watch worth P
pure 4,500.00.

“Never Let The Odds Keep You From Pursuing What You Know In Your Heart You Were Meant To Do.”-Leroy Satchel Paige
Page 146 of 180
Civil Law Q&As (2007-2013) hectorchristopher@yahoo.com dbaratbateladot@gmail.com

A relatively simulated contract for fulfillment of the obligation even


if he has not tendered payment of
Which of the following expresses a correct the purchase price.
principle of law? Choose the best answer. The contract between the parties is
Failure to disclose facts when there rescissible.
is a duty to reveal them, does not The contract between the parties
constitute fraud. is subject to ratification by the
Violence or intimidation does not parties.
render a contract annullable if
employed not by a contracting party 88. Which of the following statements is
but by a third person. wrong?
A threat to enforce one’s claim Creditors are protected in cases of
through competent authority, if the contracts intended to defraud them.
claim is legal or just, does not vitiate Contracts take effect only between
consent. the parties, their assign and heirs,
Absolute simulation of a contract except in case where the rights and
always results in a void contract. obligations arising from the contract
are not transmissible by their
Aligada orally offered to sell his two-hectare nature, or by stipulation or by
rice land to Balane for P 10Million. The provision of law.
offer was orally accepted. By agreement, the If a contract should contain some
land was to be delivered (through execution stipulation in favor of a third
of a notarized Deed of Sale) and the price person, he may demand its
was to be paid exactly one-month from fulfillment provided he
their oral agreement. Which statement is communicated his acceptance to the
most accurate? obligor before its revocation.
If Aligada refuses to deliver the land In contracts creating real rights,
on the agreed date despite payment third persons who come into
by Balane, the latter may not possession of the object of the
successfully sue Aligada because contract are not bound thereby.
the contract is oral.
If Aligada refused to deliver the land, Which phrase most accurately completes
Balane may successfully sue the statement – Any third person who
induces another to violate his contract:

“Never Let The Odds Keep You From Pursuing What You Know In Your Heart You Were Meant To Do.”-Leroy Satchel Paige
Page 147 of 180
Civil Law Q&As (2007-2013) hectorchristopher@yahoo.com dbaratbateladot@gmail.com

shall be liable for damages only if he


is a party to the same contract. The attestation clause contains the
shall be liable for damages to the following, except:
other contracting party. the number of pages used;
shall not be liable for damages to that the testator signed or caused
the other contracting party. another to sign the will and every
shall not be liable for damages if the page thereof in the presence of the
parties are in pari delicto. instrumental witnesses;
notary public;
The requisites of succession are as follows, the instrumental witnesses
except: witnessed and signed the will and
Death of decedent all the pages thereof in the presence
Transmissible estate of the testator and one another.
Existence and capacity of successor,
designated by decedent or law The following are the formalities required in
Payment of Taxes the execution of holographic will, except:
Entirely written;
The characteristics of succession are as Dated;
follows, except: Signed by testator himself
It is a legal contract. Notarized by a notary public.
Only property, rights and
obligations to the extent of the value The following are the grounds for
of the inheritance are transmitted. disallowance of wills, except:
The transmission takes place only at The formalities required by law have
the time of death. not been complied with.
The transmission takes place either The testator was insane or mentally
by will or by operation of law. incapable of making will.
The will was executed through force
The following rights are extinguished by or under duress, or influence of fear
death, except: or threats.
Legal support The will contains an attestation
Parental authority clause.
Right to inherit
Agency

“Never Let The Odds Keep You From Pursuing What You Know In Your Heart You Were Meant To Do.”-Leroy Satchel Paige
Page 148 of 180
Civil Law Q&As (2007-2013) hectorchristopher@yahoo.com dbaratbateladot@gmail.com

It is the omission in the testator’s will of consideration, which gives him the right to
one, some or all of the compulsory heirs in buy certain merchandise or specified
direct line, whether living at the time of property, from another person, at anytime
execution of the will or born after the death within the agreed period, at a fixed price.
of the testator. What principle is being What contract is being referred to?
referred to? Option Contract
reserva troncal Contract to Sell
preterition Contract of Sale
fideicommissary Lease
disposicion captatoria
Any disposition made upon the condition Which of the following contracts of sale is
that the heir shall make some provision in void?
his will in favor of the testator or of any Sale of EGM’s car by KRP, EGM’s
other person shall be void. Here, both the agent, whose authority is not
condition and the disposition are void. reduced into writing.
What principle is being referred to? Sale of EGM’s piece of land by
reserva troncal KRP, EGM’s agent, whose
preterition authority is not reduced into
fideicommissary writing.
disposicion captatoria Sale of EGM’s car by KRP, a person
stranger to EGM, without
Which phrase most accurately completes EGM’s consent or authority.
the statement – If at the time the contract Sale of EGM’s piece of land by
of sale is perfected, the thing which is the KRP, a person stranger to EGM,
object of the contract has been entirely lost: without EGM’s consent or authority.
the buyer bears the risk of loss.
the contract shall be without any
effect. 2011 Taxation Law Exam
the seller bears the risk of loss.
the buyer may withdraw from the
MCQ (November 13, 2011)
contract. (1)When does a declaration of absence of a
missing person take effect?
A contract granting a privilege to a person,
Immediately from the issuance of
for which he has paid a
the declaration of absence.

“Never Let The Odds Keep You From Pursuing What You Know In Your Heart You Were Meant To Do.”-Leroy Satchel Paige
Page 149 of 180
Civil Law Q&As (2007-2013) hectorchristopher@yahoo.com dbaratbateladot@gmail.com

3 months after the publication of accepted the substitution. Later, however,


the declaration of absence. the new debtor became insolvent and
6 months after the publication of defaulted in his obligation. What is the
the declaration of absence. effect of the new debtor’s default upon the
15 days from the issuance of the original debtor?
declaration of absence. The original debtor is freed of
liability since novation took place
The authority that school administrators and this relieved him of his
exercise over school children under their obligation.
supervision, instruction, or custody is The original debtor shall pay or
called perform the obligation with recourse
legal parental authority. to the new debtor.
substitute parental authority. The original debtor remains liable
ordinary parental authority. since he gave no consent to the
special parental authority. substitution.
The original debtor shall pay or
Can future inheritance be the subject of a perform 50% of the obligation to
contract of sale? avoid unjust enrichment on his
No, since it will put the predecessor part.
at the risk of harm from a tempted
buyer, contrary to public policy. Lennie bought a business class ticket from
Yes, since the death of the decedent Alta Airlines. As she checked in, the
is certain to occur. manager downgraded her to economy on
No, since the seller owns no the ground that a Congressman had to be
inheritance while his predecessor accommodated in the business class.
lives. Lennie suffered the discomfort and
Yes, but on the condition that the embarrassment of the downgrade. She sued
amount of the inheritance can only the airlines for quasi-delict but Alta Airlines
be ascertained after the obligations countered that, since her travel was
of the estate have been paid. governed by a contract between them, no
Upon the proposal of a third person, a new quasi-delict could arise. Is the airline
debtor substituted the original debtor correct?

without the latter’s consent. The creditor No, the breach of contract may in
fact be tortious as when it is
tainted as in this case with

“Never Let The Odds Keep You From Pursuing What You Know In Your Heart You Were Meant To Do.”-Leroy Satchel Paige
Page 150 of 180
Civil Law Q&As (2007-2013) hectorchristopher@yahoo.com dbaratbateladot@gmail.com

arbitrariness, gross bad faith, and No, since the marriage did not take
malice. place.
No, denying Lennie the comfort and Yes, since all the requisites of a
amenities of the business class as donation of an immovable are
provided in the ticket is a tortious present.
act. No, since the donation and its
Yes, since the facts show a breach of acceptance are not in a public
contract, not a quasi-delict. instrument.
Yes, since quasi-delict presupposes Yes, since X freely donated the
the absence of a pre- property to Y who became its owner.
existing contractual relation
between the parties. Rene and Lily got married after a brief
courtship. After one month, Lily discovered
Which of the following is an indispensable that while Rene presented himself as a
requirement in an action for "quieting of macho man he was actually gay. He would
title" involving real property? The plaintiff not go to bed with her. He kept obscene
must magazines of nude men and always sought
be in actual possession of the the company of handsome boys. What legal
property. remedy does Lily have?
be the registered owner of the She can file an action for
property. annulment of marriage on ground
have legal or equitable title to the of fraud.
property. She can seek a declaration of nullity
be the beneficial owner of the of the marriage based on
property. Rene’s psychological incapacity.
She can go abroad and file for
X and Y were to marry in 3 months. divorce in a country that can grant
Meantime, to express his affection, X it.
donated a house and lot to Y, which She has none since she had the
donation X wrote in a letter to Y. Y wrote opportunity to examine the goods
back, accepting the donation and took and freely entered into the marriage.
possession of the property. Before the
wedding, however, Y suddenly died of heart Lucio executed a simple deed of donation of
attack. Can Y’s heirs get the property? P50 million on time deposit with a bank in
favor of A, B, C, D, and E,

“Never Let The Odds Keep You From Pursuing What You Know In Your Heart You Were Meant To Do.”-Leroy Satchel Paige
Page 151 of 180
Civil Law Q&As (2007-2013) hectorchristopher@yahoo.com dbaratbateladot@gmail.com

without indicating the share of each donee. No, since there was no impediment
All the donees accepted the donation in to Raul selling his inheritance to a
writing. A, one of the donees, died. Will B, stranger.
C, D, and E get A’s share in the money?
Yes, accretion will automatically When one exercises a right recognized by
apply to the joint-donees in equal law, knowing that he thereby causes an
shares. injustice to another, the latter is entitled to
Yes, since the donor’s intention is to recover damages. This is known as the
give the whole of P50 million to the principle of
jointdonees in equal shares. res ipsa loquitur.
No, A"s share will revert to the donor damnum absque injuria.
because accretion applies only if the vicarious liability.
joint-donees are spouses. abuse of rights.
No, A’s share goes to his heirs
since the donation did not Which of the following is NOT a basis for

provide for reversion to donor. rendering a disinheritance defective or


imperfect?
Raul, Ester, and Rufus inherited a 10- Its cause comes from the guilt of
hectare land from their father. Before the a spouse in a legal separation
land could be partitioned, however, Raul case, the innocent-spouse having
sold his hereditary right to Raffy, a stranger died.
to the family, for P5 million. Do Ester and The truth of its cause is denied and
Rufus have a remedy for keeping the land not sufficiently proved by evidence.
within their family? Its cause is not authorized by the
Yes, they may be subrogated to law.
Raffy’s right by reimbursing to Its cause is not specified.
him within the required time
what he paid Raul. Manuel came to Manila and married
Yes, they may be subrogated to Marianne. Unknown to Marianne, Manuel
Raffy’s right provided they buy him had been previously convicted in Palawan
out before he registers the sale. of theft and served time for it. After
No, they can be subrogated to Marianne learned of his previous
Raffy’s right only with his conviction, she stopped living with him.
conformity. Can Marianne seek the annulment of the

“Never Let The Odds Keep You From Pursuing What You Know In Your Heart You Were Meant To Do.”-Leroy Satchel Paige
Page 152 of 180
Civil Law Q&As (2007-2013) hectorchristopher@yahoo.com dbaratbateladot@gmail.com

marriage based on Manuel’s nondisclosure since Philippine law does not


of his previous crime? recognize divorce.
No, since the assumption is that All the children are legitimate since
marriage forgives all past wrongs. they were born of the same father
Yes, since the non-disclosure of and mother.
that crime is the equivalent of
fraud, which is a ground for Who can make a donation?
annulment. All persons who can enter into
No, in case of doubt, the law must contracts and dispose of their
be construed to preserve the property.
institution of marriage. All persons who are of legal age and
No, since Manuel already served the suffer from no civil interdiction.
penalty for his crime. All persons who can make a last will
and testament.
Arthur and Helen, both Filipinos, got All persons, whether natural or
married and had 2 children. Arthur later artificial, who own property.
worked in Rome where he acquired Italian
citizenship. He got a divorce from Helen in The liability of the partners, including
Rome but, on returning to the Philippines, industrial partners for partnership
he realized his mistake, asked forgiveness contracts entered into in its name and for
of his wife, and resumed living with her. its account, when all partnership assets
They had 2 more children. What is the have been exhausted is
status of their 4 children? Pro-rata.
The children born before the Joint.
divorce are legitimate but those Solidary.
born after it are not since Arthur Voluntary.
got the divorce when he had
ceased to be a Filipino. When can a missing person who left
The divorce rendered illegitimate the someone to administer his property be
children born before it since the declared an absentee by the court? When
marriage that begot them had been he has been missing for
nullified. 2 years from the receipt of the last
The children born before and after news about him.
the divorce are all legitimate 7 years from the receipt of the last
news about him.

“Never Let The Odds Keep You From Pursuing What You Know In Your Heart You Were Meant To Do.”-Leroy Satchel Paige
Page 153 of 180
Civil Law Q&As (2007-2013) hectorchristopher@yahoo.com dbaratbateladot@gmail.com

10 years from the receipt of the last principal two days after the principal died,
news about him. an event that neither the agent nor the
5 years from the receipt of the buyer knew at the time of the sale. What is
last news about him. the standing of the sale?
Voidable.
Which of the following claims against the Valid.
debtor enjoys preference over the others Void.
with respect to his specific immovable Unenforceable.
property and real rights?
Unpaid price of real property sold, Spouses A and B leased a piece of land
upon the immovable property. belonging to B's parents for 25 years. The
Mortgage credits recorded in the spouses built their house on it worth
registry of property, upon the P300,000.00. Subsequently, in a case that
mortgaged real estate. C filed against A and B, the court found the
Taxes due, upon the land or latter liable to C for P200,000.00. When the
building. sheriff was attaching their house for the
Expenses for the preservation and satisfaction of the judgment, A and B
improvement of property, when the claimed that it was exempt from execution,
law authorizes reimbursement, being a family home. Is this claim correct?
upon the preserved or improved Yes, because while B’s parents own
immovable. the land, they agreed to have their
daughter build her family home on
When bilateral contracts are vitiated with it.
vices of consent, they are rendered No, because there is no judicial
rescissible. declaration that it is a family home.
void. No, since the land does not belong
unenforceable. to A and B, it cannot qualify as a
voidable. family home.
Yes, because the A and B’s family
An agent, authorized by a special power of actually lives in that house.
attorney to sell a land belonging to the
principal succeeded in selling the same to a Solomon sold his coconut plantation to
buyer according to the instructions given Aragon, Inc. for P100 million, payable in
the agent. The agent executed the deed of installments of P10 million per month with
absolute sale on behalf of his 6% interest per annum. Solomon married

“Never Let The Odds Keep You From Pursuing What You Know In Your Heart You Were Meant To Do.”-Leroy Satchel Paige
Page 154 of 180
Civil Law Q&As (2007-2013) hectorchristopher@yahoo.com dbaratbateladot@gmail.com

Lorna after 5 months and they chose When A and B married, they chose conjugal
conjugal partnership of gains to govern partnership of gains to govern their
their property relations. When they property relations. After 3 years, B
married, Aragon had an unpaid balance of succeeded in getting her marriage to A
P50 million plus interest in Solomon’s annulled on ground of the latter’s
favor. To whom will Aragon’s monthly psychological incapacity. What liquidation
payments go after the marriage? procedure will they follow in disposing of
The principal shall go to the their assets?
conjugal partnership but the They will follow the rule governing
interests to Solomon. the liquidation of a conjugal
Both principal and interests shall go partnership of gains where the party
to Solomon since they are his who acted in bad faith forfeits his
exclusive properties. share in the net profits.
Both principal and interests shall go Since the marriage has been
to the conjugal partnership since declared void, the rule for
these become due after the liquidation of absolute community of
marriage. property shall be followed.
The principal shall go to Solomon The liquidation of a co-ownership
but the interests to the conjugal applies since the annulment

partnership. brought their property relation


under the chapter on
X and Y, although not suffering from any property regimes without
impediment, cohabited as husband and marriage.
wife without the benefit of marriage. The law on liquidation of
Following the birth of their child, the couple partnerships applies.
got married. A year after, however, the X and Y agreed verbally before their
court annulled the marriage and issued a marriage (a) on the paternity of the
decree of annulment. What is the present illegitimate child of Y and (b) on the
status of the child? economic regime that will govern X and Y’s
Legitimated. property relations. Is the verbal agreement
Illegitimate. valid?
Natural child. No, because a marriage
Legitimate. settlement to be valid should be
in writing.

“Never Let The Odds Keep You From Pursuing What You Know In Your Heart You Were Meant To Do.”-Leroy Satchel Paige
Page 155 of 180
Civil Law Q&As (2007-2013) hectorchristopher@yahoo.com dbaratbateladot@gmail.com

Yes, since ante-nuptial agreements Illegitimate, because by the color of


need not be in writing. its skin, the child could not possibly
No, because a marriage settlement be that of Fidel.
cannot include an agreement on the Legitimate, because the child was
paternity of an illegitimate child. born within a valid marriage.
Yes, since even if it is not a valid Legitimate, because Fidel agreed to
marriage settlement, it is a valid treat the child as his own after
verbal contract. Gloria told him who the father was.

Spouses X and Y have a minor daughter, Z, The husband’s acts of forcibly ejecting his
who needs support for her education. Both wife without just cause from the conjugal
X and Y, who are financially distressed, dwelling and refusing to take her back
could not give the needed support to Z. As constitutes
it happens, Z’s other relatives are desertion.
financially capable of giving that support. recrimination.
From whom may Z first rightfully demand constructive abandonment.
support? From her de facto separation.
grandfather.
brother. In his will, the testator designated X as a
uncle. legatee to receive P2 million for the purpose
first cousin. of buying an ambulance that the residents
of his Barangay can use. What kind of
Fidel, a Filipino with fair complexion, institution is this?
married Gloria. Before the marriage, Gloria a fideicomissary institution.
confessed to Fidel that she was two-month a modal institution.
pregnant with the child of a black African a conditional institution.
who had left the country for good. When the a collective institution.
child was born, Fidel could not accept it
being too black in complexion. What is the X insured himself for P5 million,
status of the child? designating Y, his wife, as his sole
Illegitimate, because Gloria beneficiary. The designation was
confessed that the child is not irrevocable. A few years later, X had their
Fidel’s. marriage annulled in court on the ground
that Y had an existing prior marriage. X

“Never Let The Odds Keep You From Pursuing What You Know In Your Heart You Were Meant To Do.”-Leroy Satchel Paige
Page 156 of 180
Civil Law Q&As (2007-2013) hectorchristopher@yahoo.com dbaratbateladot@gmail.com

subsequently died, Is Y entitled to the Marlon gets 1/2 and Cecilia gets
insurance benefits? 1/2.
Yes, since the insurance was not Marlon gets 3/4 and Cecilia 1/4.
dependent on the marriage.
Yes, since her designation as Contracts take effect only between the
beneficiary was irrevocable. parties or their assigns and heirs, except
No, X’s designation of Y is where the rights and obligations arising
revoked by operation of law upon from the contract are not transmissible by
the annulment of their marriage their nature, by stipulation, or by provision
based on Y’s fault. of law. In the latter case, the assigns or the
Yes, since without judicial heirs are not bound by the contracts. This
revocation, X’s designation of Y is known as the principle of
remains valid and binding. Relativity of contracts.
May a spouse freely donate communal or Freedom to stipulate.
conjugal property without the consent of Mutuality of contracts.
the other? Obligatory force of contracts.
Absolutely not, since the spouses
co-own such property. A buyer ordered 5,000 apples from the
Yes, for properties that the family seller at P20 per apple. The seller delivered
may spare, regardless of value. 6,000 apples. What are the rights and
Yes, provided the donation is obligations of the buyer?
moderate and intended for charity He can accept all 6,000 apples
or family rejoicing. and pay the seller at P20 per
Yes, in a donation mortis causa that apple.
the donor may still revoke in his He can accept all 6,000 apples and
lifetime. pay a lesser price for the 1,000
excess apples.
The decedent died intestate leaving an He can keep the 6,000 apples
estate of P10 million. He left the following without paying for the 1,000 excess
heirs: a) Marlon, a legitimate child and b) since the seller delivered them
Cecilia, the legal spouse. Divide the estate. anyway.
Marlon gets 1/4 and Cecilia gets He can cancel the whole transaction
3/4. since the seller violated the terms of
Marlon gets 2/3 and Cecilia 1/3. their agreement.

“Never Let The Odds Keep You From Pursuing What You Know In Your Heart You Were Meant To Do.”-Leroy Satchel Paige
Page 157 of 180
Civil Law Q&As (2007-2013) hectorchristopher@yahoo.com dbaratbateladot@gmail.com

Lino entered into a contract to sell with suffered from poisoning caused by a
Ramon, undertaking to convey to the latter noxious substance found in the sardines.
one of the five lots he owns, without Mylene filed a case for damages against
specifying which lot it was, for the price of Acme. Which of the following defenses will
P1 million. Later, the parties could not hold?
agree which of five lots he owned Lino The expiry date of the "Sards" was
undertook to sell to Ramon. What is the clearly printed on its can, still the
standing of the contract? store sold and Mylene bought it.
Unenforceable. Mylene must have detected the
Voidable. noxious substance in the sardines
Rescissible. by smell, yet she still ate it.
Void. Acme had no transaction with
Mylene; she bought the "Sards" from
Knowing that the car had a hidden crack in a store, not directly from Acme.
the engine, X sold it to Y without informing Acme enjoys the presumption of
the latter about it. In any event, the deed of safeness of its canning procedure
sale expressly stipulated that X was not and Mylene has not overcome such
liable for hidden defects. Does Y have the presumption.
right to demand from X a reimbursement of
what he spent to repair the engine plus Fernando executed a will, prohibiting his
damages? wife Marina from remarrying after his
Yes. X is liable whether or not he death, at the pain of the legacy of P100
was aware of the hidden defect. Million in her favor becoming a nullity. But
Yes, since the defect was not a year after Fernando’s death, Marina was
hidden; X knew of it but he acted so overwhelmed with love that she married
in bad faith in not disclosing the another man. Is she entitled to the legacy,
fact to Y. the amount of which is well within the
No, because Y is in estoppel, having capacity of the disposable free portion of
changed engine without prior Fernando’s estate?
demand. Yes, since the prohibition against
No, because Y waived the warranty remarrying is absolute, it is deemed
against hidden defects. not written.
Acme Cannery produced sardines in cans Yes, because the prohibition is
known as "Sards." Mylene bought a can of inhuman and oppressive and
Sards from a store, ate it, and

“Never Let The Odds Keep You From Pursuing What You Know In Your Heart You Were Meant To Do.”-Leroy Satchel Paige
Page 158 of 180
Civil Law Q&As (2007-2013) hectorchristopher@yahoo.com dbaratbateladot@gmail.com

violates Marina’s rights as a free It is an ordinary donation since it


woman. was not given to the bride or
No, because the nullity of the groom.
prohibition also nullifies the legacy. It is donation propter nuptias since
No, since such prohibition is it was given with the marriage in
authorized by law and is not mind.
repressive; she could remarry but It is an indirect donation propter
must give up the money. nuptias since the bride would
eventually inherit the property from
X, the owner, constituted a 10-year her parents.
usufruct on his land as well as on the It is a remunatory donation.
building standing on it in Y’s favor. After
flood totally destroyed the building 5 years X and Y, both Filipinos, were married and
later, X told Y that an act of God terminated resided in Spain although they intend to
the usufruct and that he should vacate the return to the Philippines at some future
land. Is X, the owner of the land, correct? time. They have not executed any marriage
No, since the building was destroyed settlements. What law governs their
through no fault of Y. property relations?
No, since Y still has the right to They may choose between Spanish
use the land and the materials law and Philippine law.
left on it. Philippine law since they are both
Yes, since Y cannot use the land Filipinos.
without the building. No regime of property relations will
Yes, since the destruction of the apply to them.
building without the X’s fault Spanish law since they live in Spain.
terminated the usufruct.
Birth determines personality. Death
In gratitude, the groom’s parents made a extinguishes it. Under what circumstances
donation of a property in writing to the may the personality of a deceased person
bride’s parents shortly before their continue to exist?
children’s wedding. The donation was In case of re-appearance of a
accepted. What is the nature of the missing person presumed dead.
donation?

“Never Let The Odds Keep You From Pursuing What You Know In Your Heart You Were Meant To Do.”-Leroy Satchel Paige
Page 159 of 180
Civil Law Q&As (2007-2013) hectorchristopher@yahoo.com dbaratbateladot@gmail.com

In protecting the works of a the brother or sister in need stops


deceased under intellectual property schooling without valid reason.
laws. the need for support of a brother
In case of declaration of or sister, already of age, is due to
presumptive death of a missing the latter's fault.
spouse.
In the settlement of the estate of Virgilio owned a bare and simple swimming
a deceased person. pool in his garden. MB, a 7-year old child,
surreptitiously entered the garden and
Six tenants sued X, the landowner, for merrily romped around the ledges of the
willfully denying them water for their farms, pool. He accidentally tripped, fell into the
which water happened to flow from land pool, and drowned. MB’s parents sued
under X’s control, his intention being to Virgilio for damages arising from their
force them to leave his properties. Is X child’s death, premised on the principle of
liable for his act and why? "attractive nuisance". Is Virgilio liable for
No, because the tenants must be the death of MB?
content with waiting for rainfall for No, the child was 7 years old and
their farms. knew the dangers that the pool
No, since X owns both the land and offered.
the water. Yes, being an attractive nuisance,
Yes, because the tenants’ farms Virgilio had the duty to prevent
have the natural right of access to children from coming near it.
water wherever it is located. No, since the pool was bare and
Yes, since X willfully caused had no enticing or alluring
injury to his tenants contrary to gadgets, floats, or devices in it
morals, good customs or public that would attract a 7-year old
policy. child.
Yes, since Virgilio did not cover the
Illegitimate brothers and sisters, whether of swimming pool while not in use to
full or half-blood, are bound to support prevent children from falling into it.
each other, EXCEPT when
the brother or sister who needs
support lives in another place.
such brothers and sisters are not
recognized by their father.

“Never Let The Odds Keep You From Pursuing What You Know In Your Heart You Were Meant To Do.”-Leroy Satchel Paige
Page 160 of 180
Civil Law Q&As (2007-2013) hectorchristopher@yahoo.com dbaratbateladot@gmail.com

The term of a 5-year lease contract between Yes, the donation is not deemed
X the lessor and Y the lessee, where rents made until the suspensive condition
were paid from month to month, came to an has been fulfilled.
end. Still, Y continued using the property
with X’s consent. In such a case, it is Illegitimate children, those not recognized
understood that they impliedly renewed the by their biological fathers, shall use the
lease surname of their
from month to month under the biological father subject to no
same conditions as to the rest. condition.
under the same terms and mother or biological father, at the
conditions as before. mother’s discretion.
under the same terms except the mother.
rent which they or the court must biological father unless he judicially
fix. opposes it.
for only a year, with the rent raised
by 10% pursuant to the rental Asiong borrowed P1 million from a bank,
control law. secured by a mortgage on his land. Without
his consent, his friend Boyong paid the
Rex, a philanthropist, donated a valuable lot whole loan. Since Asiong benefited from the
to the municipality on the condition that it payment, can Boyong compel the bank to
will build a public school on such lot within 2 subrogate him in its right as mortgagee of
years from its acceptance of the donation. The Asiong's land?
municipality properly accepted the donation No, but the bank can foreclose and
but did not yet build the public school after 2 pay Boyong back.
years. Can Rex revoke the donation? No, since Boyong paid for
Yes, since the donation is subject Asiong’s loan without his
to a resolutory condition which approval.
was not fulfilled. Yes, since a change of creditor took
No, but Rex is entitled to recover the place by novation with the bank’s
value of the land from the consent.
municipality. Yes, since it is but right that Boyong
No, the transfer of ownership has be able to get back his money and, if
been completed. not, to foreclose the mortgage in the
manner of the bank.

“Never Let The Odds Keep You From Pursuing What You Know In Your Heart You Were Meant To Do.”-Leroy Satchel Paige
Page 161 of 180
Civil Law Q&As (2007-2013) hectorchristopher@yahoo.com dbaratbateladot@gmail.com

Congress passed a law imposing taxes on Yes, since Fernando was a solidary
income earned out of a particular activity creditor, payment to him
that was not previously taxed. The law, extinguished the obligation.
however, taxed incomes already earned
within the fiscal year when the law took What happens to the property regimes that
effect. Is the law valid? were subsisting under the New Civil Code
No, because laws are intended to be when the Family Code took effect?
prospective, not retroactive. The original property regimes are
No, the law is arbitrary in that it immutable and remain effective.
taxes income that has already been Those enjoying specific regimes
spent. under the New Civil Code may adopt
Yes, since tax laws are the lifeblood the regime of absolute community of
of the nation. property under the Family Code.
Yes, tax laws are an exception; Those that married under the New
they can be given retroactive Civil Code but did not choose any of
effect. its regimes shall now be governed by
the regime of absolute community of
Rudolf borrowed P1 million from Rodrigo property.
and Fernando who acted as solidary They are superseded by the Family
creditors. When the loan matured, Rodrigo Code which has retroactive effect.
wrote a letter to Rudolf, demanding The testator executed a will following the
payment of the loan directly to him. Before formalities required by the law on
Rudolf could comply, Fernando went to see succession without designating any heir.
him personally to collect and he paid him. The only testamentary disposition in the
Did Rudolf make a valid payment? will is the recognition of the testator's
No, since Rudolf should have split illegitimate child with a popular actress. Is
the payment between Rodrigo and the will valid?
Fernando. Yes, since in recognizing his
No, since Rodrigo, the other illegitimate child, the testator has
solidary creditor, already made a made him his heir.
prior demand for payment from No, because the non-designation of
Rudolf. heirs defeats the purpose of a will.
Yes, since the payment covers the
whole obligation.

“Never Let The Odds Keep You From Pursuing What You Know In Your Heart You Were Meant To Do.”-Leroy Satchel Paige
Page 162 of 180
Civil Law Q&As (2007-2013) hectorchristopher@yahoo.com dbaratbateladot@gmail.com

No, the will comes to life only when Voidable, because the Judge
the proper heirs are instituted. acted beyond his territorial
Yes, the recognition of an jurisdiction and is administratively
illegitimate heir is an ample liable for the same.
reason for a will. Void, because the Judge did not
solemnize the marriage within the
A left B, his wife, in the Philippines to work premises of his court.
in Egypt but died in that country after a
year’s continuous stay. Two months after X and Y, Filipinos, got married in Los
A’s death, B gave birth to a child, claiming Angeles, USA, using a marriage license
it is A’s child. Who can assail the legitimacy issued by the Philippine consul in Los
of the child? Angeles, acting as Civil Registrar. X and Y
A’s other heirs apart from B. did not know that they were first cousins
The State which has interest in the because their mothers, who were sisters,
welfare of overseas contract were separated when they were quite
workers. young. Since X did not want to continue
Any one who is outraged by B’s with the relation when he heard of it, he left
claim. Y, came to the Philippines and married Z.
No one since A died. Can X be held liable for bigamy?
No since X’s marriage to Y is void ab
QR and TS who had a marriage license initio or did not exist.
requested a newly appointed Judge in No since X acted in good faith,
Manila to marry them on the beach of conscious that public policy did not
Boracay. Since the Judge maintained approve of marriage between first
Boracay as his residence, he agreed. The cousins.
sponsors were all public officials. What is Yes since he married Z without
the status of the marriage. first securing a judicial
Valid, since the improper venue is declaration of nullity of his
merely an irregularity; all the marriage to Y.
elements of a valid marriage are Yes since his first marriage to Y in
present. Los Angeles is valid.
Void, because the couple did not get
local permit for a beach wedding. Allan bought Billy’s property through
Carlos, an agent empowered with a special
power of attorney (SPA) to sell the same.

“Never Let The Odds Keep You From Pursuing What You Know In Your Heart You Were Meant To Do.”-Leroy Satchel Paige
Page 163 of 180
Civil Law Q&As (2007-2013) hectorchristopher@yahoo.com dbaratbateladot@gmail.com

When Allan was ready to pay as scheduled, No, the buyer is entitled to a
Billy called, directing Allan to pay directly customary 30-day extension of his
to him. On learning of this, Carlos, Billy's obligation to take delivery of the
agent, told Allan to pay through him as his goods.
SPA provided and to protect his No, since there was no express
commission. Faced with two claimants, agreement regarding automatic
Allan consigned the payment in court. Billy rescission.
protested, contending that the consignation No, the seller should first determine
is ineffective since no tender of payment that Y was not justified in failing to
was made to him. Is he correct? appear.
No, since consignation without The wife filed a case of legal separation
tender of payment is allowed in against her husband on the ground of
the face of the conflicting claims sexual infidelity
on the plaintiff. without previously exerting earnest efforts
Yes, as owner of the property sold, to come to a compromise with him. The
Billy can demand payment directly judge dismissed the case for having been
to himself. filed without complying with a condition
Yes, since Allan made no precedent. Is the dismissal proper?
announcement of the tender. No, efforts at a compromise will only
Yes, a tender of payment is required deepen the wife’s anguish.
for a valid consignation. No, since legal separation like
validity of marriage is not subject
X sold Y 100 sacks of rice that Y was to to compromise agreement for
pick up from X’s rice mill on a particular purposes of filing.
date. Y did not, however, appear on the Yes, to avoid a family feud that is
agreed date to take delivery of the rice. After hurtful to everyone.
one week, X automatically rescinded the Yes, since the dispute could have
sale without notarial notice to Y. Is the been settled with the parties
rescission valid? agreeing to legal separation.
Yes, automatic rescission is
allowed since, having the An Australian living in the Philippines
character of movables and acquired shares of stock worth P10 million
consumables, rice can easily in food manufacturing companies. He died
deteriorate. in Manila, leaving a legal wife and a child in
Australia and a live-in partner with whom

“Never Let The Odds Keep You From Pursuing What You Know In Your Heart You Were Meant To Do.”-Leroy Satchel Paige
Page 164 of 180
Civil Law Q&As (2007-2013) hectorchristopher@yahoo.com dbaratbateladot@gmail.com

he had two children in Manila. He also left Yes, after full payment, the action
a will, done according to Philippine laws, became imprescriptible.
leaving all his properties to his live-in
partner and their children. What law will A court declared Ricardo, an old bachelor,
govern the validity of the disposition in the an absentee and appointed Cicero
will? administrator of his property. After a year,
Australia law since his legal wife it was discovered that Ricardo had died
and legitimate child are Australians abroad. What is the effect of the fact of his
and domiciled in Australia. death on the administration of his
Australian law since the intrinsic property?
validity of the provisions of a will With Ricardo no longer an absentee
is governed by the decedent’s but a deceased person, Cicero will
national law. cease to be administrator of his
Philippine law since the decedent properties.
died in Manila and he executed his The administration shall be given
will according to such law. by the court having jurisdiction
Philippine law since the decedent’s over the intestate
properties are in the proceedings to a new
Philippines. administrator whom it will
appoint.
X bought a land from Y, paying him cash. Cicero automatically becomes
Since they were friends, they did not execute administrator of Ricardo’s estate
any document of sale. After 7 years, the heirs until judicially relieved.
of X asked Y to execute a deed of absolute Cicero’s alienations of Ricardo's
sale to formalize the verbal sale to their property will be set aside.
father. Unwilling to do so, X’s heirs filed an
action for specific performance against Y. Will Baldo, a rejected suitor, intimidated Judy
their action prosper? into marrying him. While she wanted to
No, after more than 6 years, the question the validity of their marriage two
action to enforce the verbal years after the intimidation ceased, Judy
agreement has already elapsed. decided in the meantime to freely cohabit
No, since the sale cannot under the with Baldo. After more than 5 years
Statute of Frauds be enforced. following their wedding, Judy wants to file a
Yes, since X bought the land and case for annulment of marriage against
paid Y for it.

“Never Let The Odds Keep You From Pursuing What You Know In Your Heart You Were Meant To Do.”-Leroy Satchel Paige
Page 165 of 180
Civil Law Q&As (2007-2013) hectorchristopher@yahoo.com dbaratbateladot@gmail.com

Baldo on ground of lack of consent. Will her all heirs in the higher level are disqualified
action prosper? or unable to inherit?
Yes, the action for annulment is Nephews and nieces.
imprescriptible. Brothers and sisters.
No, since the marriage was merely State.
voidable and Judy ratified it by Other collateral relatives up to the
freely cohabiting with Baldo after 5th degree of consanguinity.
the force and intimidation had
ceased. Roy and Carlos both undertook a contract
No, since the action prescribed 5 to deliver to Sam in Manila a boat docked in
years from the date of the Subic. Before they could deliver it, however,
celebration of the marriage. the boat sank in a storm. The contract
Yes, because the marriage was provides that fortuitous event shall not
celebrated without Judy's consent exempt Roy and Carlos from their
freely given. obligation. Owing to the loss of the motor
boat, such obligation is deemed converted
Is the wife who leaves her husband without into one of indemnity for damages. Is the
just cause entitled to support? liability of Roy and Carlos joint or solidary?
No, because the wife must always be Neither solidary nor joint since they
submissive and respectful to the cannot waive the defense of
husband. fortuitous event to which they are
Yes. The marriage not having been entitled.
dissolved, the husband continues to Solidary or joint upon the discretion
have an obligation to support his of Sam.
wife. Solidary since Roy and Carlos failed
No, because in leaving the to perform their obligation to deliver
conjugal home without just cause, the motor boat.
she forfeits her right to support. Joint since the conversion of
Yes, since the right to receive their liability to one of indemnity
support is not subject to any for damages made it joint.
condition.
Joanne married James, a person with no
In the order of intestate succession where known relatives. Through James' hard
the decedent is legitimate, who is the last work, he and his wife Joane prospered.
intestate heirs or heir who will inherit if When James died, his estate alone

“Never Let The Odds Keep You From Pursuing What You Know In Your Heart You Were Meant To Do.”-Leroy Satchel Paige
Page 166 of 180
Civil Law Q&As (2007-2013) hectorchristopher@yahoo.com dbaratbateladot@gmail.com

amounted to P100 million. If, in his will, Ric and Josie, Filipinos, have been
James designates Joanne as his only heir, sweethearts for 5 years. While working in a
what will be the free portion of his estate. European country where the execution of
Joanne gets all; estate has no free joint wills are allowed, the two of them
portion left. executed a joint holographic will where they
Joanne gets 1/2; the other half is named each other as sole heir of the other
free portion. in case either of them dies. Unfortunately,
Joanne gets 1/3; the remaining 2/3 Ric died a year later. Can Josie have the
is free portion. joint will successfully probated in the
Joanne gets 1/4; the remaining 3/4 Philippines?
is free portion. Yes, in the highest interest of comity
of nations and to honor the wishes
A warranty inherent in a contract of sale, of the deceased.
whether or not mentioned in it, is known as No, since Philippine law prohibits
the the execution of joint wills and
warranty on quality. such law is binding on Ric and
warranty against hidden defects. Josie even abroad.
warranty against eviction. Yes, since they executed their joint
warranty in merchantability. will out of mutual love and care,
values that the generally accepted
The doctrine of stare decisis prescribes principles of international law
adherence to precedents in order to accepts.
promote the stability of the law. But the Yes, since it is valid in the country
doctrine can be abandoned where it was executed, applying the
When adherence to it would result principle of "lex loci celebrationis."
in the Government’s loss of its case.
When the application of the ML inherited from his father P5 million in
doctrine would cause great legitime but he waived it in a public
prejudice to a foreign national. instrument in favor of his sister QY who
When necessary to promote the accepted the waiver in writing. But as it
passage of a new law. happened, ML borrowed P6 million from PF
When the precedent has ceased to before the waiver. PF objected to the waiver
be beneficial and useful. and filed an action for its rescission on the
ground that he had the right to ML’s P5

“Never Let The Odds Keep You From Pursuing What You Know In Your Heart You Were Meant To Do.”-Leroy Satchel Paige
Page 167 of 180
Civil Law Q&As (2007-2013) hectorchristopher@yahoo.com dbaratbateladot@gmail.com

million legitime as partial settlement of (B) Yes, insofar as Arnold


what ML owed him since ML has proved to acknowledged Mary as his
be insolvent. Does PF, as creditor, have the illegitimate child.
right to rescind the waiver? None, since the marriage did not
No, because the waiver in favor of take place.
his sister QY amounts to a donation Yes, if they acquired properties
and she already accepted it. while living together as husband
Yes, because the waiver is and wife.
prejudicial to the interest of a Joseph, a 17-year old Filipino, married
third person whose interest is Jenny, a 21-year old American in Illinois,
recognized by law. USA, where the marriage was valid. Their
No, PF must wait for ML to become parents gave full consent to the marriage of
solvent and, thereafter, sue him for their children. After three years, Joseph
the unpaid loan. filed a petition in the USA to promptly
Yes, because a legitime cannot be divorce Jenny and this was granted. When
waived in favor of a specific heir; it Joseph turned 25 years, he returned to the
must be divided among all the other Philippines and married Leonora. What is
heirs. the status of this second marriage?
Void, because he did not cause
While engaged to be married, Arnold and the judicial issuance of
Josephine agreed in a public instrument to declaration of the nullity of his
adopt out the economic regime of absolute first marriage to Jenny before
community of property. Arnold marrying Leonora.
acknowledged in the same instrument that Valid, because Joseph's marriage to
Josephine’s daughter Jenny is void, he being only 17
Mary, is his illegitimate child. But years of age when he married her.
Josephine died before the marriage could Valid, because his marriage to
take place. Does the marriage settlement Leonora has all the elements of a
have any significance? valid marriage.
None, since the instrument Void, because Joseph is still
containing the marriage settlement considered married to Jenny since
is essentially void for containing an the Philippines does not recognize
unrelated matter. divorce.

“Never Let The Odds Keep You From Pursuing What You Know In Your Heart You Were Meant To Do.”-Leroy Satchel Paige
Page 168 of 180
Civil Law Q&As (2007-2013) hectorchristopher@yahoo.com dbaratbateladot@gmail.com

T died intestate, leaving an estate of X, who was abroad, phoned his brother, Y,
P9,000,000. He left as heirs three legitimate authorizing him to sell X’s parcel of land in
children, namely, A, B, and C. A has two Pasay. X sent the title to Y by courier
children, D and E. Before he died, A service. Acting for his brother, Y executed a
irrevocably repudiated his inheritance from notarized deed of absolute sale of the land
T in a public instrument filed with the to Z after receiving payment. What is the
court. How much, if any, will D and E, as status of the sale?
A’s children, get from T’s estate? Valid, since a notarized deed of
Each of D and E will get absolute sale covered the
P1,500,000 by right of transaction and full payment was
representation since their father made.
repudiated his inheritance. Void, since X should have
Each of D and E will get P2,225,000 authorized agent Y in writing to
because they will inherit from the sell the land.
estate equally with B and C. Valid, since Y was truly his brother
D and E will get none because of X’s agent and entrusted with the
the repudiation; "B" and "C" will title needed to effect the sale.
get A’s share by right of Valid, since the buyer could file an
accretion. action to compel X to execute a deed
Each of D and E will get P2,000,000 of sale.
because the law gives them some
advantage due to the demise of "A". In a true pacto de retro sale, the title and
ownership of the property sold are
No decree of legal separation can be issued immediately vested in the vendee a retro
unless the children’s welfare is subject only to the resolutory condition of
attended to first. repurchase by the vendor a retro within the
without prior efforts at stipulated period. This is known as
reconciliation shown to be futile. equitable mortgage.
unless the court first directs conventional redemption.
mediation of the parties. legal redemption.
without prior investigation equity of redemption.
conducted by a public prosecutor. A natural obligation under the New Civil
Code of the Philippines is one which

“Never Let The Odds Keep You From Pursuing What You Know In Your Heart You Were Meant To Do.”-Leroy Satchel Paige
Page 169 of 180
Civil Law Q&As (2007-2013) hectorchristopher@yahoo.com dbaratbateladot@gmail.com

the obligor has a moral obligation to When the donor gives donations without
do, otherwise entitling the obligee to reserving sufficient funds for his support or
damages. for the support of his dependents, his
refers to an obligation in writing to donations are
do or not to do. Rescissible, since it results in
the obligee may enforce through the economic lesion of more than 25% of
court if violated by the obligor. the value of his properties.
cannot be judicially enforced but Voidable, since his consent to the
authorizes the obligee to retain donation is vitiated by mindless
the obligor’s payment or kindness.
performance. Void, since it amounts to wanton
expenditure beyond his means.
The husband assumed sole administration Reducible to the extent that the
of the family’s mango plantation since his donations impaired the support
wife worked abroad. due to himself and his
Subsequently, without his wife’s knowledge, dependents.
the husband entered into an antichretic
transaction with a company, giving it Anne owed Bessy P1 million due on October
possession and management of the 1, 2011 but failed to pay her on due date.
plantation with power to harvest and sell Bessy sent a demand letter to Anne giving
the fruits and to apply the proceeds to the her 5 days from receipt within which to
payment of a loan he got. What is the pay. Two days after receipt of the letter,
standing of the contract? Anne personally offered to pay Bessy in
It is void in the absence of the wife’s manager's check but the latter refused to
consent. accept the same. The 5 days lapsed. May
It is void absent an authorization Anne’s obligation be considered
from the court. extinguished?
The transaction is void and can Yes, since Bessy’s refusal of the
neither be ratified by the wife nor manager’s check, which is
authorized by the court. presumed funded, amounts to a
It is considered a continuing offer satisfaction of the obligation.
by the parties, perfected only No, since tender of payment even
upon the wife’s acceptance or the in cash, if refused, will not
court’s authorization. discharge the obligation without
proper consignation in court.

“Never Let The Odds Keep You From Pursuing What You Know In Your Heart You Were Meant To Do.”-Leroy Satchel Paige
Page 170 of 180
Civil Law Q&As (2007-2013) hectorchristopher@yahoo.com dbaratbateladot@gmail.com

Yes, since Anne tendered payment The owner of a thing cannot use it in a way
of the full amount due. that will injure the right of a third person.
No, since a manager’s check is not Thus, every building or land is subject to
considered legal tender in the the easement which prohibits its proprietor
Philippines. or possessor from committing nuisance like
noise, jarring, offensive odor, and smoke.
The residents of a subdivision have been This principle is known as
using an open strip of land as passage to Jus vindicandi.
the highway for over 30 years. The owner of Sic utere tuo ut alienum non
that land decided, however, to close it in laedas.
preparation for building his house on it. Jus dispondendi.
The residents protested, claiming that they Jus abutendi.
became owners of the land through
acquisitive prescription, having been in Janice and Jennifer are sisters. Janice sued
possession of the same in the concept of Jennifer and Laura, Jennifer’s business
owners, publicly, peacefully, and partner for recovery of property with
continuously for more than 30 years. Is this damages. The complaint did not allege that
claim correct? Janice exerted earnest efforts to come to a
No, the residents have not been in compromise with the defendants and that
continuous possession of the land such efforts failed. The judge dismissed the
since they merely passed through complaint outright for failure to comply
it in going to the highway. with a condition precedent. Is the dismissal
No, the owner did not abandon his in order?
right to the property; he merely No, since Laura is a stranger to
tolerated his neighbors’ use of it for the sisters, Janice has no moral
passage. obligation to settle with her.
Yes, residents of the subdivision Yes, since court should promote
have become owners by acquisitive amicable settlement among
prescription. relatives.
Yes, community ownership by Yes, since members of the same
prescription prevails over private family, as parties to the suit, are
claims. required to exert earnest efforts to
settle their disputes before coming
to court.

“Never Let The Odds Keep You From Pursuing What You Know In Your Heart You Were Meant To Do.”-Leroy Satchel Paige
Page 171 of 180
Civil Law Q&As (2007-2013) hectorchristopher@yahoo.com dbaratbateladot@gmail.com

No, the family council, which would When does the regime of conjugal
ordinarily mediate the dispute, has partnership of gains begin to exist?
been eliminated under the Family At the moment the parties take
Code. and declare each other as
husband and wife before
X borrowed money from a bank, secured by officiating officer.
a mortgage on the land of Y, his close At the time the spouses acquire
friend. When the loan matured, Y offered to properties through joint efforts.
pay the bank but it refused since On the date the future spouses
Y was not the borrower. Is the bank’s action executed their marriage settlements
correct? because this is the starting point of
Yes, since X, the true borrower, did their marital relationship.
not give his consent to Y’s offer to On the date agreed upon by the
pay. future spouses in their marriage
No, since anybody can discharge settlements since their agreement is
X’s obligation to his benefit. the law between them.
No, since Y, the owner of the
collateral, has an interest in the Josie, 18, married Dante, 25, without her
payment of the obligation. parents’ knowledge and consent, and lived
Yes, since it was X who has an with him. After a year, Josie returned to her
obligation to the bank. parents’ home, complained of the
unbearable battering she was getting from
The right of a mortgagor in a judicial Dante, and expressed a desire to have her
foreclosure to redeem the mortgaged marriage with him annulled. Who may
property after his default in the bring the action?
performance of the conditions of the Dante.
mortgage but before the sale of the Her parents.
mortgaged property or confirmation of the Josie herself.
sale by the court, is known as The State.
accion publiciana.
equity of redemption. X, a married man, cohabited with Y, an
pacto de retro. unmarried woman. Their relation bore them
right of redemption. BB, a baby boy. Subsequently, after X
became a widower, he married Y. Was BB
legitimated by that marriage?

“Never Let The Odds Keep You From Pursuing What You Know In Your Heart You Were Meant To Do.”-Leroy Satchel Paige
Page 172 of 180
Civil Law Q&As (2007-2013) hectorchristopher@yahoo.com dbaratbateladot@gmail.com

Yes, since his parents are now Yes, as long as they leave sufficient
lawfully married. property for themselves and for their
Yes, since he is an innocent party dependents.
and the marriage rectified the wrong
done him. X owed Y P1.5 million. In his will, X gave Y
No, since once illegitimate, a legacy of P1 million but the will provided
child shall always remain that this legacy is to be set off against the
illegitimate. P1.5 million X owed Y. After the set off, X
No, since his parents were not still owed Y P500,000. Can Y still collect
qualified to marry each other this amount?

when he was conceived. Yes, because the designation of Y


as legatee created a new and
The presence of a vice of consent vitiates separate juridical relationship
the consent of a party in a contract and this between them, that of testator-
renders the contract legatee.
Rescissible. It depends upon the discretion of
Unenforceable. the probate court if a claim is filed
Voidable. in the testate proceedings.
Void. No, because the intention of the
testator in giving the legacy is to
Can common-law spouses donate abrogate his entire obligation to Y.
properties of substantial value to one No, because X had no instruction in
another? his will to deliver more than the
No, they are only allowed to give legacy of P1 million to Y.
moderate gifts to each other
during family rejoicing. Josie owned a lot worth P5 million prior to
No, they cannot give anything of her marriage to Rey. Subsequently, their
value to each other to prevent conjugal partnership spent P3 million for
placing their legitimate relatives at a the construction of a house on the lot. The
disadvantage. construction resulted in an increase in the
Yes, unlike the case of legally value of the house and lot to P9 million.
married spouses, such donations Who owns the house and the lot?
are not prohibited.

“Never Let The Odds Keep You From Pursuing What You Know In Your Heart You Were Meant To Do.”-Leroy Satchel Paige
Page 173 of 180
Civil Law Q&As (2007-2013) hectorchristopher@yahoo.com dbaratbateladot@gmail.com

(A) Josie and the conjugal When fortuitous circumstances


partnership of gains will own both prevented the plaintiff from filing the
on a 50-50 basis. case sooner.
Josie will own both since the When the plaintiff is in possession
value of the house and the of the property.
increase in the property’s value is
less than her lot’s value; but she Conrad and Linda, both 20 years old,
is to reimburse conjugal applied for a marriage license, making it
partnership expenses. appear that they were over 25. They
Josie still owns the lot, it being her married without their parents’ knowledge
exclusive property, but the house before an unsuspecting judge. After the
belongs to the conjugal partnership. couple has been in cohabitation for 6 years,
The house and lot shall both belong to Linda’s parents filed an action to annul the
the conjugal partnership, with Josie marriage on ground of lack of parental
entitled to reimbursement for the consent. Will the case prosper?
value of the lot. No, since only the couple can
question the validity of their
An action for reconveyance of a registered marriage after they became 21 of
piece of land may be brought against the age; their cohabitation also
owner appearing on the title based on a convalidated the marriage.
claim that the latter merely holds such title No, since Linda’s parents made no
in trust for the plaintiff. The action allegations that earnest efforts have
prescribes, however, within 10 years from been made to come to a compromise
the registration of the deed or the date of with Conrad and Linda and which
the issuance of the certificate of title of the efforts failed.
property as long as the trust had not been Yes, since the marriage is voidable,
repudiated. What is the exception to this the couple being below 21 years of
10-year prescriptive period? age when they married.
When the plaintiff had no notice of Yes, since Linda’s parents never
the deed or the issuance of the gave their consent to the marriage.
certificate of title.
When the title holder concealed the Pepito executed a will that he and 3
matter from the plaintiff. attesting witnesses signed following the
formalities of law, except that the Notary
Public failed to come. Two days later, the

“Never Let The Odds Keep You From Pursuing What You Know In Your Heart You Were Meant To Do.”-Leroy Satchel Paige
Page 174 of 180
Civil Law Q&As (2007-2013) hectorchristopher@yahoo.com dbaratbateladot@gmail.com

Notary Public notarized the will in his law Separate since their property
office where all signatories to the will relations with their legal spouses
acknowledged that the testator signed the are still subsisting.
will in the presence of the witnesses and Co-ownership since they agreed to
that the latter themselves signed the will in work for their mutual benefit.
the presence of the testator and of one Communal since they earned the
another. Was the will validly notarized? same as common-law spouses.
No, since it was not notarized on the
occasion when the signatories What is the prescriptive period for filing an
affixed their signatures on the will. action for revocation of a donation based on
Yes, since the Notary Public has acts of ingratitude of the donee?
to be present only when the 5 years from the perfection of the
signatories acknowledged the acts donation.
required of them in relation to 1 year from the perfection of the
the will. donation.
Yes, but the defect in the mere 4 years from the perfection of the
notarization of the will is not fatal to donation.
its execution. Such action does not prescribe.
No, since the notary public did not
require the signatories to sign their Before Karen married Karl, she inherited P5
respective attestations again. million from her deceased mother which
amount she brought into the marriage. She
Venecio and Ester lived as common-law later used part of the money to buy a new
spouses since both have been married to Mercedes Benz in her name, which Karen
other persons from whom they had been and her husband used as a family car. Is
separated in fact for several years. the car a conjugal or Karen’s exclusive
Hardworking and bright, each earned property?
incomes from their respective professions It is conjugal property since the
and enterprises. What is the nature of their spouses use it as a family car.
incomes? It is Karen’s exclusive property since
Conjugal since they earned the it is in her name.
same while living as husband and It is conjugal property having been
wife. bought during the marriage.

“Never Let The Odds Keep You From Pursuing What You Know In Your Heart You Were Meant To Do.”-Leroy Satchel Paige
Page 175 of 180
Civil Law Q&As (2007-2013) hectorchristopher@yahoo.com dbaratbateladot@gmail.com

(D) It is Karen’s should the lessees sue for damages? (1%)


exclusive
(1). A, the owner
property since she bought it with

her own money. (2). B, the engineer

Because of X’s gross negligence, Y suffered (3). both A & B


injuries that resulted in the abortion of the
SUGGESTED ANSWER:
foetus she carried. Y sued X for, among
other damages, P1 million for the death of a 3. Both A & B.
family member. Is Y entitled to indemnity
for the death of the foetus she carried? The lessee may proceed against A for

Yes, since the foetus is already breach of contract, and against B for tort

regarded as a child from conception, or statutory liability. Under Article 1654

though unborn. of the New Civil Code, the lessor is

No, since X’s would not have known obliged to make all the necessary repairs
that the accident would result in Y’s in order to keep the leased property
abortion. suitable for the use to which it has been
No, since birth determines devoted. Consequently, under Article
personality, the accident did not 1659 NCC, the proprietor of a building or
result in the death of a person. structure is responsible for the damages
Yes, since the mother believed in her resulting from its total or partial
heart that she lost a child. collapse, if it is due to lack of necessary
repairs.

Under Article 1723, NCC, the engineer


2010 Civil Law Exam MCQ or architect who drew up the plans and

(September 12, 2010) specifications for a building is liable for


damage if 15 years from the completion
No.II. Multiple choice. of the structure the same should
collapse by a reason of a defect by those
(A). A had a 4-storey building which was plans and specifications, or due to the
constructed by Engineer B. After five years, defects in the ground. This liability
the building developed cracks and its maybe enforced against the architect or
stairway eventually gave way and collapsed, engineer even by a third party who has
resulting to injuries to some lessees. Who no privity of contract with the architect
or engineer under Article 2192, NCC.

“Never Let The Odds Keep You From Pursuing What You Know In Your Heart You Were Meant To Do.”-Leroy Satchel Paige
Page 176 of 180
Civil Law Q&As (2007-2013) hectorchristopher@yahoo.com dbaratbateladot@gmail.com

ALTERNATIVE ANSWER: O, owner of Lot A, learning that Japanese


soldiers may have buried gold and other
No.1. A , the owner .
treasures at the adjoining vacant Lot B
The lessee can sue only the lessor for belonging to spouses X & Y, excavated in
breach of contract under Article 1659 in Lot B where she succeeded in unearthing
relation to Article 1654, NCC. The lessee gold and precious stones. How will the
cannot sue the architect or the engineer treasures found by O be divided? (1%)

because there was no privity of contracts


(1). 100% to O as finder
between them. When sued, however, the
lessor may file a third party claim
(2). 50% to O and 50% to the spouses X
against the architect or the engineer.
and Y
ANOTHER ALTERNATIVE ANSWER:
(3). 50% to O and 50% to the state
No. 2. B, the Engineer .
(4). None of the above.
Under Article 1723 the engineer or
architect who drew up the plans and SUGGESTED ANSWER:
specifications for a building is liable for
No. 4. None of the above.
damages if within 15 years from the
completion of the structure, the same
The general rule is that the treasure
should collapse by reason of a defect in
shall belong to the spouses X and Y, the
those plans and specifications, or due to
owner of Lot B. Under Article 438 (NCC),
the defects in the ground. Under Article
the exception is that when the discovery
2192 (NCC), however, if the damages
of a hidden treasure is made on the
should be the result of any of the defects
property of another and by chance, one-
in the construction mentioned in Art
half thereof shall belong to the owner of
1723, NCC, the third person suffering
the land and the other one-half is
damages may proceed only against the
allowed to the finder. In the problem,
engineer or architect or contractor
the finding of the treasure was not by
within the period fixed therein. The
chance because O knew that the treasure
damages suffered by the lessee in the
was in Lot B. While a trespasser is also
problem are clearly those resulting from
not entitled to any share, and there is no
defects in the construction plans or
indication in the problem whether or not
specifications.
O was a trespasser, O is not entitled to a

“Never Let The Odds Keep You From Pursuing What You Know In Your Heart You Were Meant To Do.”-Leroy Satchel Paige
Page 177 of 180
Civil Law Q&As (2007-2013) hectorchristopher@yahoo.com dbaratbateladot@gmail.com

share because the finding was not “by or mother alone, even in a public

chance.” document, is not sufficient because the


father and mother did not have a special
power of attorney for the purpose. Under
Article 745 (NCC), the donee must accept
A executed a Deed of Donation in favor of
the donation personally, or through an
B, a bachelor, covering a parcel of land
authorized person with a special power of
valued at P1 million. B was, however, out of
attorney for the purpose; otherwise, the
the country at the time. For the donation to
donation shall be void.
be valid, (1%)
No.3 is also false. B cannot accept the
(1). B may e-mail A accepting the donation. donation anytime at his convenience.
Under Article 749 NCC, the donee may
(2). The donation may be accepted by B’s
accept the donation only during the
father with whom he lives.
lifetime of the donor.

(3). B can accept the donation anytime


A executed a 5-page notarial will before a
convenient to him.
notary public and three witnesses. All of them
signed each and every page of the will.
(4). B’s mother who has a general power of
attorney may accept the donation for him.
One of the witnesses was B, the father of
one of the legatees to the will. What is the
(5). None of the above is sufficient to make
effect of B being a witness to the will? (1%)
B’s acceptance valid

(1). The will is invalidated


SUGGESTED ANSWER:

No. 5 None of the above is sufficient to (2). The will is valid and effective

make B's acceptance valid .


(3). The legacy given to B’s child is not valid
Since the donation covered an
immovable property, the donation and SUGGESTED ANSWER:

the acceptance must be in public


No. 3. The legacy given to B's child is not
document and e-mail is not a public
valid.
document. Hence, No.1 is false.
The validity of the will is not affected by
No. 2 and No.4 are both false. The
the legacy in favor of the son of an
acceptance by the donee’s father alone

“Never Let The Odds Keep You From Pursuing What You Know In Your Heart You Were Meant To Do.”-Leroy Satchel Paige
Page 178 of 180
Civil Law Q&As (2007-2013) hectorchristopher@yahoo.com dbaratbateladot@gmail.com

attesting witness to the will. However, ALTERNATIVE ANSWER:


the said legacy is void under Article 823
(D). all the above
NCC.

ALTERNATIVE ANSWER: (2). A deposit made in compliance with a


legal obligation is:
No. 2 .The will is valid and effective.

Under Article 823 ( NCC ),the legacy (A). an extrajudicial deposit;

given in favor of the son of an


(B). a voluntary deposit;
instrumental witness to a will has no
effect on the validity of the will. Hence, (C). a necessary deposit;
the will is valid and effective.
(D). a deposit with a warehouseman;

(E). letters a and b


2007 Civil Law Exam MCQ
SUGGESTED ANSWER:
(September 09, 2007)
(C). a necessary deposit
No.IX. Multiple choice: Choose the right
answer. (2% each) (3). A contract of antichresis is always:

(1). The parties to a bailment are the: (A). a written contract;

(A). bailor; (B). a contract, with a stipulation that the


debt will be paid through receipt of the
(B). bailee;
fruits of an immovable;

comodatario;
(C). Involves the payment of interests, if
owing;
(D). all the above;

(D). All of the above;


(E). letters a and b

(E). Letters a and b


SUGGESTED ANSWER:

SUGGESTED ANSWER:
(E). letters a and b

“Never Let The Odds Keep You From Pursuing What You Know In Your Heart You Were Meant To Do.”-Leroy Satchel Paige
Page 179 of 180
Civil Law Q&As (2007-2013) hectorchristopher@yahoo.com dbaratbateladot@gmail.com

(D). All of the above; (D). 1/3 of the total debts must be
represented by the approving creditors;
(4). An, assignee in a proceeding under the
Insolvency Law does not have the duty of: (E). Letters a and b

(A). suing to recover the properties of the SUGGESTED ANSWER:


state of the insolvent debtor;
(C). 3/5 of the number of creditors
(B). selling property of the insolvent debtor; should agree to the settlement;

(C). ensuring that a debtor corporation [Note: Items 4&5 on Insolvency Law are
operate the business efficiently and not included within the coverage of Civil
effectively while the proceedings are Law but Commercial Law. It is therefore
pending; suggested that the examinees be given
full credit for the two items regardless of
(D). collecting and discharging debts owed
their answers.]
to the insolvent debtor.

SUGGESTED ANSWER:
References:
(C). ensuring that a debtor corporation
operate the business efficiently and ¾ Answers to Bar Examination
effectively while the proceedings are Questions by the UP LAW COMPLEX
pending; (2007, 2009, 2010)

(5). In order to obtain approval of the


UP LAW REVIEW
proposed settlement of the debtor in an
insolvency proceeding.
PHILIPPINE ASSOCIATION OF LAW

(A). the court must initiate the proposal SCHOOLS (2008)

(B). 2/3 of the number of creditors should lawphil.net

agree to the settlement;

(C). 3/5 of the number of creditors

should agree to the settlement;

“Never Let The Odds Keep You From Pursuing What You Know In Your Heart You Were Meant To Do.”-Leroy Satchel Paige
Page 180 of 180
UNIVERSITY OF SANTO TOMAS
SUGGESTED ANSWERS IN CIVIL LAW
BAR EXAMINATIONS 2013
By: Assoc. Dean Viviana M. Paguirigan

QUESTION NO. I.

You are a Family Court judge and before you is a Petition for the Declaration of Nullity of
Marriage (under Article 36 of the Family Code) filed by Maria against Neil. Maria claims that
Neil is psychologically incapacitated to comply with the essential obligations of marriage
because Neil is a drunkard, a womanizer, a gambler, and a mama's boy- traits that she never
knew or saw when Neil was courting her. Although summoned, Neil did not answer Maria's
petition and never appeared in court.

To support her petition, Maria presented three witnesses- herself, Dr. Elsie Chan, and
Ambrosia. Dr. Chan testified on the psychological report on Neil that she prepared. Since Neil
never acknowledged n9r responded to her invitation for interviews, her report is solely based
on her interviews with Maria and the spouses' minor children. Dr. Chan concluded that Neil is
suffering from Narcissistic Personality Disorder, an ailment that she found to be already
present since Neil's early adulthood and one that is grave and incurable. Maria testified on the
specific instances when she found Neil drunk, with another woman, or squandering the
family's resources in a casino. Ambrosia, the spouses' current household help, corroborated
Maria's testimony.

On the basis of the evidence presented, will you grant the petition? (8%)

SUGGESTED ANSWER:

If I were the judge, I will not grant the petition. Although psychological incapacity has not been
defined by the Family Code, the Supreme Court in several cases (Republic vs. San Jose - February
28, 2007; Zamora v. CA an Zamora G.R. No. 141917 February 7, 2007; Benjamin Ting v. Carmen
Ting G.R. No. 166562; March 31, 2009) has ruled that the intendment of the law is to confine
psychological incapacity to the most serious cases of personality disorders clearly demonstrative of
an utter insensitivity or inability to give meaning and significance to the marriage. What the law
requires is downright incapacity and not refusal or neglect or difficulty but a failure to perform
essential marital obligations due to causes psychological in nature.

Further, the presentation of expert proof presupposes a thorough and in-depth assessment of
the parties by the psychologist or expert for a conclusive diagnosis of grave, severe, and
incurable presence of psychological incapacity. (Paz vs. Paz – February 18, 2010) In this case,
the report of Dr. Chan is solely based on her interviews with Maria and the children. She did
not actually hear, see and evaluate Neil. Hence, the report cannot constitute a reasonable
basis to reach a conclusion as to Neil’s psychological incapacity.

QUESTION II.

A collision occurred at an intersection involving a bicycle and a taxicab. Both the bicycle rider
(a businessman then doing his morning exercise) and the taxi driver claimed that the other
was at fault. Based on the police report, the bicycle crossed the intersection first but the
taxicab, crossing at a fast clip from the bicycle's left, could not brake in time and hit the
bicycle's rear wheel, toppling it and throwing the bicycle rider into the sidewalk 5 meters away.

1
The bicycle rider suffered a fractured right knee, sustained when he fell on his right side on
the concrete side walk. He was hospitalized and was subsequently operated on, rendering him
immobile for 3 weeks and requiring physical rehabilitation for another 3 months. In his
complaint for damages, the rider prayed for the award ofP1,000,000 actual damages,P200,000
moral damages, P200,000 exemplary damages, P1 00,000 nominal damages and P50,000
attorney's fees.

Assuming the police report to be correct and as the lawyer for the bicycle rider, what evidence
(documentary and testimonial) and legal arguments will you present in court to justify the
damages that your client claims? (8%)

SUGGESTED ANSWER:

As lawyer for the bicycle rider, I will present in addition to the police report, the medical
abstract as to the injuries sustained by my client as well as copies of receipts of expenses
incurred in connection with the treatment of his injuries. I will also present the testimony of
my client and perhaps a bystander who witnessed the incident as to the circumstances
surrounding the accident.

As for the legal argument, I will rebut the claim of negligence on my client’s part by presenting
evidence that my client has actually crossed the intersection ahead of the taxicab and it was the
taxicab driver who rapidly cut the path of the bicycle which caused the collision. Also, even
assuming that there was contributory negligence on the part of my client, I will argue that it will
not preclude the recovery of damages but may only mitigate the damages to which he is entitled.

QUESTION III.

Sergio is the registered owner of a 500-square meter land. His friend, Marcelo, who has long
been interested in the property, succeeded in persuading Sergio to sell it to him. On June 2,
2012, they agreed on the purchase price of P600,000 and that Sergio would give Marcelo up
to June30, 2012 within which to raise the amount. Marcelo, in a light tone usual between
them, said that they should seal their agreement through a case of Jack Daniels Black and
P5,000 "pulutan" money which he immediately handed to Sergio and which the latter
accepted. The friends then sat down and drank the first bottle from the case of bourbon.

On June 15, 2013, Sergio learned of another buyer, Roberto, who was offering P800,000 in
ready cash for the land. When Roberto confirmed that he could pay in cash as soon as Sergio
could get the documentation ready, Sergio decided to withdraw his offer to Marcelo, hoping to
just explain matters to his friend. Marcelo, however, objected when the withdrawal was
communicated to him, taking the position that they have a firm and binding agreement that
Sergio cannot simply walk away from because he has an option to buy that is duly supported
by a duly accepted valuable consideration.

Does Marcelo have a cause of action against Sergio? (5%)

Can Sergio claim that whatever they might have agreed upon cannot be enforced because any
agreement relating to the sale of real property must be supported by evidence in writing and
they never reduced their agreement to writing? (3%)

2
SUGGESTED ANSWER:

Yes, Marcelo has a cause of action against Sergio. As a rule, an offer can be withdrawn at any
time before acceptance by communicating such withdrawal (Art. 1324) except when the option
is founded upon a consideration as something paid or promised. In this case, although there
was no separate consideration for the option, the offer had already been accepted and thus, it
resulted into a perfected contract of sale between Marcelo and Sergio. Sale being a consensual
contract is perfected by mere consent.

No, Sergio cannot claim that the agreement cannot be enforced because it was not reduced into
writing. Contracts shall be obligatory, in whatever form they may have been entered into,
provided all the essential requisites for their validity are present. (Art. 1356) In fact when the
law requires a document or other special form, as in the acts and enumerated by law, the
contracting parties may compel each other to observe that form, once the contract has been
perfected, and this right may be exercised simultaneously with the action upon the contract.
(Art. 1357) Even an oral sale of a parcel of land is valid between the parties (Campillo vs. CA
129 SCRA 513; Zaide v. CA 163 SCRA 71)

QUESTION IV.

Anselmo is the registered owner of a land and a house that his friend Boboy occupied for a
nominal rental and on the condition that Boboy would vacate the property on demand. With
Anselmo's knowledge, Boboy introduced renovations consisting of an additional bedroom, a
covered veranda, and a concrete block fence, at his own expense.

Subsequently, Anselmo needed the property as his residence and thus asked Boboy to vacate
and turn it over to him. Boboy, despite an extension, failed to vacate the property, forcing
Anselmo to send him a written demand to vacate.

In his own written reply, Boboy signified that he was ready to leave but Anselmo must first
reimburse him the value of the improvements he introduced on the property as he is a builder
in good faith. Anselmo refused, insisting that Boboy cannot ask for reimbursement as he is a
mere lessee. Boboy responded by removing the improvements and leaving the building in its
original state.

Resolve Boboy's claim that as a builder in good faith, he should be reimbursed the value of the
improvements he introduced. (4%)
Can Boboy be held liable for damages for removing the improvements over Anselmo's
objection? (4%)

SUGGESTED ANSWER:

Boboy’s claim that he is a builder in good faith is without merit. The contract between the parties
remains to be a lease despite the nominal rentals paid by Boboy. As such, Boboy’s right with
regard to the improvements he introduced on the property should not be resolved on the basis
of the provisions of the Civil Code on builder in good faith under Article 448 but by the
provision on lease, particularly Article 1678. A lessee who makes improvements on the
property cannot be considered a builder in good faith for he knows that he does not own the
property and his possession is merely temporary. Boboy may only claim one-half of the value
of the improvements from Anselmo but if the latter refuses to reimburse him, Boboy may
remove the improvements even if it may cause damage to the property.

3
No, Boboy cannot be held liable for damages except if he caused unnecessary impairment to the
property leased. Since Anselmo refused to appropriate the improvements and to reimburse
Boboy, the latter may exercise his right to remove the improvements provided he shall not
cause any more impairment to the property leased than is necessary.

QUESTION V.

Josefa executed a deed of donation covering a one-hectare rice land in favor of her daughter,
Jennifer. The deed specifically provides that:

"For and in consideration of he love and service Jennifer has shown and given to
me, I hereby freely, voluntarily and irrevocably donate to her my one-hectare
rice land covered by TCT No. 11550, located in San Fernando, Pampanga. This
donation shall take effect upon my death."

The deed also contained Jennifer's signed acceptance, and an attached notarized declaration
by Josefa and Jennifer that the land will remain in Josefa's possession and cannot be
alienated, encumbered, sold or disposed of while Josefa is still alive.

Advise Jennifer on whether the deed is a donation inter vivos or mortis causa and explain the
reasons supporting your advice. (8%)

SUGGESTED ANSWER:

I will advise Jennifer that the deed of donation executed in her favor by Josefa is a donation
inter vivos. An inter vivos donation is generally irrevocable once accepted, and the law
requires that if it involves immovable property, it must be in a public document and there must
be a deed of acceptance which must be in the same deed of donation. If the acceptance is in
a separate instrument, it has to be noted in both instruments. (Art. 749) In this case, the deed
of acceptance clearly signifies that it is a donation inter vivos because a donation mortis causa
need not be accepted by the donee during the lifetime of the donor although the donee in the
case of mortis causa donation is free to accept or repudiate it after the death of the donor.

Also, the prohibition on alienation during Josefa’s lifetime all the more indicates that the
donation is inter vivos because the fact that Josefa reserved the lifetime usufruct of the land
shows that her intent is to transfer the ownership of the donated property to Jennifer or else
there would have been no need for her to reserve the lifetime usufruct thereof if it were a
donation mortis causa. (Gestopa v. CA 342 SCRA 105 citing Reyes vs. Mosqueda, 187 SCRA
661, 671 (1990); Concepcion vs. Concepcion, 91 Phil. 823, 827 (1952).)

QUESTION VI.

Lito obtained a loan of P1,000,000 from Ferdie, payable within one year. To secure payment,
Lito executed a chattel mortgage on a Toyota Avanza and a real estate mortgage on a 200-
square meter piece of property.

Would it be legally significant - from the point of view of validity and enforceability - if the loan
and the mortgages were in public or private instruments? (6%)

Lito's failure to pay led to the extra-judicial foreclosure of the mortgaged real property. Within a
year from foreclosure, Lito tendered a manager's check to Ferdie to redeem the property. Ferdie
refused to accept payment on the ground that he wanted payment in cash: the

4
check does not qualify as legal tender and does not include the interest payment. Is Ferdie's
refusal justified? (4%)

SUGGESTED ANSWER:

With respect to the loan, the same is both valid and enforceable regardless of whether it is in a
private or public document because as a rule, contracts shall be obligatory in whatever form
they may have been entered into provided all the essential requisites for their validity are
present. A loan is a contract which the law does not require to be in a particular form in order
that it may be valid or enforceable.

However, with regard to the chattel mortgage, since the law (Act 1508) requires an affidavit of
good faith stating that the chattel mortgage is supposed to stand as security for the loan, it is
submitted that for validity of the chattel mortgage, it must be in a public document. A real
estate mortgage under the provisions of Article 2125 requires that in order that a mortgage
may be validly constituted that the document in which it appears must be recorded. If it is not
recorded, the mortgage is nevertheless valid and binding between the parties. Hence, for
validity both chattel and real estate mortgages must be in a public document. But for purposes
of enforceability, it is submitted that the form of the contract whether in a public or private
document would be immaterial. (Mobil Oil vs. Diocares 29 SCRA 656).

Ferdie’s refusal to accept the check on the ground that it does not qualify as legal tender is correct
because a check, whether a manager's check or ordinary check, is not legal tender, and an offer of
a check in payment of a debt is not a valid tender of payment and may be refused receipt by the
obligee or creditor. (Philippine Airlines vs. CA and Amelia Tan – January 30, 1990) Mere delivery of
checks does not discharge the obligation under a judgment. The obligation is not extinguished and
remains suspended until the payment by commercial document is actually realized (Art. 1249, Civil
Code, par. 3). Also, redemption within the period allowed by law is not a matter of intent but a
question of payment or valid tender of full redemption price within the said period. Whether the
redemption is being made under Act 3135 or under the General Banking Law, the mortgagor or his
assignee is required to tender payment to make said redemption valid. (Heirs of Quisumbing vs.
PNB aand SLDC –G.R. No. 178242 January 20, 2009)

QUESTION VII.

In 2005, Andres built a residential house on a lot whose only access to the national highway
was a pathway crossing Brando's property. Andres and others have been using this pathway
(pathway A) since 1980.

In 2006, Brand0 fenced off his property, thereby blocking Andres' access to the national
highway. Andres demanded that part of the fence be removed to maintain his old access route
to the highway (pathway A), but Brando refused, claiming that there was another available
pathway (pathway B) for ingress and egress to the highway. Andres countered that pathway B
has defects, is circuitous, and is extremely inconvenient to use.

To settle their dispute, Andres and Brando hired Damian, a geodetic and civil engineer, to survey
and examine the two pathways and the surrounding areas, and to determine the shortest and the
least prejudicial way through the servient estates. After the survey, the engineer concluded that
pathway B is the longer route and will need improvements and repairs, but will not significantly
affect the use of Brando's property. On the other hand, pathway A that had long been in place, is
the shorter route but would significantly affect the use of Brando's property.

5
In light of the engineer's findings and the circumstances of the case, resolve the parties' right
of way dispute. (6%)

SUGGESTED ANSWER:

I will rule in favor of Brando. The easement of right of way should be established at a point least
prejudicial to the servient estate where the distance from the dominant estate to the public
highway may be the shortest. (Art. 650) If these two conditions do not concur in one estate, the
criterion of least prejudice prevails over shortest distance. (Anastacia Quimen vs. CA and Yolanda
Oliveros May 29, 1996) In this case, to establish the easement on the property of Brando would
significantly affect his use of his property whereas while Pathway B may prove to be the longer
route, it will cause least prejudice to Brando. Andres’ argument that Pathway B
is circuitous and inconvenient to use should not be given weight because the true test of the
establishment of an easement is adequacy. Convenience of the dominant estate has never
been the gauge for the establishment of the easement. (Costabella Corporation v. CA 193
SCRA 333; Cristobal vs. Ledesma 291 SCRA 122)

QUESTION VIII.

Ciriaco Realty Corporation (CRC) sold to the spouses Del a Cruz a500-square meter land (Lot
A) in Paranaque. The land now has a fair market value of Pl,200,000. CRC likewise sold to the
spouses Rodriguez, a 700-square meter land (Lot B) which is adjacent to Lot A. Lot B has a
present fair market value of P1,500,000.

The spouses Dela Cruz constructed a house on Lot B, relying on there presentation of the CRC
sales agent that it is the property they purchased. Only upon the completion of their house did
the spouses Dela Cruz discover that they had built on Lot B owned by the spouses Rodriguez,
not on Lot A that they purchased. They spent P 1 000,000 for the house.

As their lawyer, advise the spouses Dela Cruz on their rights and obligations under the given
circumstances, and the recourses and options open to them to protect their interests. (8%)

SUGGESTED ANSWER:

I will advise Spouses Dela Cruz that they have the right to retain possession of the premises
until Rodriguez exercises any of the options under Article 448 of the Civil Code. (Tecnogas
Manufacturing vs. CA February 10, 1997) Spouses Dela Cruz are builders in good faith because
before constructing the house, they exercised due diligence by asking the agent of CRC the
location of Lot A and they relied on the information given by the agent who is presumed to
know the identity of the lot purchased by the Dela Cruz. (Pleasantville vs. CA 253 SCRA 10)
The owner of the land on which anything has been built in good faith by another has the right
to appropriate as his own the works, sowing or planting after payment of the indemnity or to
oblige the builder to pay the price of the land if its value is not considerably higher than the
building or trees, or to ask the sower to pay proper rent. I will also advise my clients that
Rodriguez may not compel them to remove the improvements because it is not one of the
options granted to the landowner if the builder is in good faith.

6
QUESTION IX.

Rica petitioned for the annulment of her ten-year old marriage to Richard. Richard hired Atty.
Cruz to represent him in the proceedings. In payment for Atty. Cruz's acceptance and legal
fees, Richard conveyed to Atty. Cruz a parcel of land in Taguig that he recently purchased with
his lotto winnings. The transfer documents were duly signed and Atty. Cruz immediately took
possession by fencing off the property's entire perimeter.

Desperately needing money to pay for his mounting legal fees and his other needs and despite
the transfer to Atty. Cruz, Richard offered the same parcel of land for sale to the spouses
Garcia. After inspection of the land, the spouses considered it a good investment and
purchased it from Richard. Immediately after the sale, the spouses Garcia commenced the
construction of a three-story building over the land, but they were prevented from doing this
by Atty. Cruz who claimed he has a better right in light of the prior conveyance in his favor.

Is Atty. Cruz's claim correct? (8%)

SUGGESTED ANSWER:

No, Atty. Cruz is not correct. At first glance, it may appear that Atty. Cruz is the one who has a
better right because he first took possession of the property. However, as a lawyer of Richard he is
prohibited under Article 1491 from acquiring the property and rights which may the object of any
litigation in which they may take part by virtue of their profession. While the suit is for annulment
of marriage and it may be argued that the land itself is not the object of the litigation, the
annulment of marriage if granted, will carry with it the liquidation of the absolute community or
conjugal partnership of the spouses as the case may be (Article 50 in relation to Article 43 of the
Family Code). Richard purchased the land with his lotto winnings during the pendency of the suit
for annulment and on the assumption that the parties are governed by the regime of absolute
community or conjugal partnership, winnings from gambling or betting will form part thereof. Also,
since the land is part of the absolute community or conjugal partnership of the Richard and Rica it
may not be sold or alienated without the consent of the latter and any disposition or encumbrance
of the property of the community or conjugal property without the consent of the other spouse is
void. (Article 96 and Article 124, Family Code).

QUESTION X.

Manuel was born on 12 March 1940 in a 1 000-square meter property where he grew up
helping his father, Michael, cultivate the land. Michael has lived on the property since the land
was opened for settlement at about the time of the Commonwealth government in 193 5, but
for some reason never secured any title to the property other than a tax declaration in his
name. He has held the property through the years in the concept of an owner and his stay
was uncontested by others. He has also conscientiously and continuously paid the realty taxes
on the land.

Michael died in 2000 and Manuel - as Michael’s only son and heir -now wants to secure and
register title to the land in his own name. He consults you for legal advice as he wants to
perfect his title to the land and secure its registration in his name.

What are the laws that you need to consider in advising Manuel on how he can perfect his title
and register the land in his name? Explain the relevance of these laws to your projected
course of action. (4%)

7
What do you have to prove to secure Manuel's objectives and what documentation are
necessary? (4%)

SUGGESTED ANSWER:

For purposes of confirmation of imperfect title, I will have to consider the provisions of
Commonwealth Act No. 141 as well as the Property Registration Decree or P.D. 1529 in giving my
advice to Manuel. C.A. No. 141 which amended the second Public Land Act (Act 2874) provides
that there are two requisites for judicial confirmation of imperfect title namely:1) open and
continuous, exclusive and notorious possession and occupation of the land by himself or through
his predecessor in interest under bona fide claim of ownership since June 12, 1945; and 2) the
classification of the land as alienable and disposable land of the public domain. (Secretary of DENR
v. Yap -G.R. NO. 167707, October 8, 2008) The Property Registration Decree or P.D. 1529
1
provides who may file an application for registration of title to the land under Section 14 thereof
which provides that those who by themselves or their predecessors-in-interest have been in open,
continuous, exclusive and notorious possession and occupation of alienable and disposable lands
for the public domain under a bona fide claim of ownership since June 12, 1945 or earlier. Since
Manuel’s father Michael had been in open, continuous, exclusive and notorious possession of the
land since 1935, and that the land was declared alienable in the same year, his possession has
ripened into ownership which entitles him or his successor Manuel to file an application for judicial
confirmation of imperfect title.

I have to prove that the land was already declared alienable at the time that Manuel or his father
Michael took possession of the land and that their possession was open, continuous, exclusive and
notorious which started prior to or on June 12, 1945 as required by C.A. No. 141. To prove the
first requisite, the original classification of the land as approved by the DENR Secretary (Republic
v. T.A. N. Properties 555 SCRA 4777 (2008) or in lieu thereof, a Certification by the DENR Regional
office attesting to the alienable and disposable character of the land (Republic v. Serrano G.R. No.
183063 – February 24, 2010) must have to be submitted. I also have to file together with the
application for registration all original muniments of title or copies thereof and a survey plan of the
2
land approved by the Bureau of Lands in accordance with Section 17 of P.D. 1529. Manuel may
also submit the tax declarations and tax payment receipts which have been ruled to be good
indications of possession in the concept of owner (Republic vs. Candy Maker, Inc. G.R. No.
163766, June 22, 2006).

Section 14. Who may apply. The following persons may file in the proper Court of First Instance an application for
registration of title to land, whether personally or through their duly authorized representatives:

Those who by themselves or through their predecessors-in-interest have been in open, continuous, exclusive and
notorious possession and occupation of alienable and disposable lands of the public domain under a bona fide claim
of ownership since June 12, 1945, or earlier.

Those who have acquired ownership of private lands by prescription under the provision of existing laws.

Those who have acquired ownership of private lands or abandoned river beds by right of accession or accretion
under the existing laws.

Those who have acquired ownership of land in any other manner provided for by law.

Section 17. What and where to file. The application for land registration shall be filed with the Court of First Instance of the
province or city where the land is situated. The applicant shall file together with the application all original muniments of titles
or copies thereof and a survey plan of the land approved by the Bureau of Lands.

The clerk of court shall not accept any application unless it is shown that the applicant has furnished the Director of Lands
with a copy of the application and all annexes.

8
2014 BAR EXAMINATIONS UNIVERSITY of the PHILIPPINES LAW CENTER SUGGESTED
ANSWERS IN CIVIL LAW Assoc. Dean Viviana M. Paguirigan

I.

Ariz and Paz were officemates at Perlas ng Silangan Bank (PSB). They fell in love with each other and
had a civil and church wedding. Meanwhile, Paz rapidly climbed the corporate ladder of PSB and
eventually became its Vice President, while Ariz remained one of its bank supervisors, although he was
short of 12 units to finish his Masters of Business Administration (MBA) degree.

Ariz became envious of the success of his wife. He started to drink alcohol until he became a drunkard.
He preferred to join his "barkadas"; became a wifebeater; would hurt his children without any reason;
and failed to contribute to the needs of the family. Despite rehabilitation and consultation with a
psychiatrist, his ways did not change.

After 19 years of marriage, Paz, a devout Catholic, decided to have their marriage annulled by the
church. Through the testimony of Paz and a psychiatrist, it was found that Ariz was a spoiled brat in his
youth and was sometimes involved in brawls. In his teens, he was once referred to a psychiatrist for t
reatment due to his violent tendencies. In due time, the National Appellate Matrimonial Tribunal
(NAMT) annulled the union of Ariz and Paz due to the failure of Ariz to perform and fulfill his duties as
a husband and as a father to their children. The NAMT concluded that it is for the best interest of Paz,
Ariz and their children to have the marriage annulled.

In view of the NAMT decision, Paz decided to file a Petition for Declaration of Nullity of Marriage of
their civil wedding before the Regional Trial Court (RTC) of Makati City using the NAMT decision and
the same evidence adduced in the church annulment proceedings as basis.

If you are the judge, will you grant the petition? Explain. (5%)

SUGGESTED ANSWER:
If I were the judge, I will not grant the petition. While the decision of the church tribunal annulling the
marriage of the parties may be persuasive, it is not however, binding upon the civil courts. For
psychological incapacity to be a ground for nullity, it must be shown that it was rooted in the history of
the party alleged to be suffering from it, must be grave and serious, and incurable such that it renders
the person incapacitated to perform the essential marital obligations due to causes psychological in
nature. In the case presented, it appears that Ariz fulfilled his marital obligations at the beginning and
it was only after feeling envious about the success of Paz that he started exhibiting violent tendencies
and refused to comply with marital obligations. Psychological incapacity is not mere refusal but outright
incapacity to perform marital obligations which does not appear to be present in the case of Ariz.
(Marcos v. Marcos G.R. No. 136490- October 19, 2000)

II.

Crispin died testate and was survived by Alex and Josine, his children from his first wife; Rene and
Ruby, his children from his second wife; and Allan, Bea, and Cheska, his children from his third wife.

One important provision in his will reads as follows:

"Ang lupa at bahay sa Lungsod ng Maynila ay ililipat at ilalagay sa pangalan nila Alex at Rene hindi
bilang pamana ko sa kanila kundi upang pamahalaan at pangalagaan lamang nila at nang ang sinuman
sa aking mga anak, sampu ng aking mga apo at kaapuapuhan ko sa habang panahon, ay may
tutuluyan kung magnanais na mag-aral sa Maynila o sa kalapit na mga lungsod."

Is the provision valid? (4%)

9
SUGGESTED ANSWER:
No, the provision is not valid. At first glance, the provision may appear valid as it provides for the
transfer of title in favor of Alex and Rene over the parcel of land. A legacy or devise is to be construed
as a donation effective mortis causa, and it is intended to transfer ownership to the legatee or devisee.
Since the ownership is legally transferred to the Alex and Rene, they cannot be prohibited by the
testator from alienating or partitioning the same perpetually. The dispositions of the testator declaring
all or part of the estate inalienable for more than twenty years are void. (Article 870)

III.

The Roman Catholic Church accepted a donation of a real property located in Lipa City. A deed of
donation was executed, signed by the donor, Don Mariano, and the donee, the Church, as represented
by Fr. Damian. Before the deed could be notarized, Don Mariano died. Is the donation valid? (4%)

SUGGESTED ANSWER:
The donation is void. The donation of an immovable property must be in a public instrument in order
for it to be valid. In this case, the donor died even before the notarization of the deed of donation.
Hence, it does not satisfy the requirement of being in a public instrument for the donation to be valid.

IV.

Nante, a registered owner of a parcel of land in Quezon City, sold the property to Monica under a deed
of sale which reads as follows:

"That for and in consideration of the sum of P500,000.00, value to be paid and delivered to me, and
receipt of which shall be acknowledged by me to the full satisfaction of Monica, referred to as Vendee,
I hereby sell, transfer, cede, convey, and assign, as by these presents, I do have sold, transferred,
ceded, conveyed and assigned a parcel of land covered by TCT No. 2468 in favor of the Vendee."

After delivery of the initial payment of P100,000.00, Monica immediately took possession of the
property. Five (5) months after, Monica failed to pay the remaining balance of the purchase price.
Nante filed an action for the recovery of possession of the property. Nante alleged that the agreement
was one to sell,which was not consummated as the full contract price was not paid. Is the contention
of Nante tenable?

SUGGESTED ANSWER:
The contention of Nante is not tenable. The deed itself states that for consideration received, he sells,
transfers, and conveys the land to Monica and there was delivery of the property to the latter. The
contract is clearly one of sale as there was no reservation of ownership on the part of the seller Nante.
The non-payment of the price in a contract of sale would only entitle the seller to rescind the contract
but it does not thereby prevent the transfer of ownership particularly so as in this case, where there
was already delivery to the buyer.

V.

What is the effect of preterition ? (1%)

It annuls the devise and legacy

It annuls the institution of heir

It reduces the devise and legacy

It partially annuls the institution of heir

10
Answer is letter B (preterition annuls the institution of heirs)

VI.

Miko and Dinah started to live together as husband and wife without the benefit of marriage in 1984.
Ten (10) years after, they separated. In 1996, they decided to live together again, and in 1998, they
got married.

On February 17, 2001, Dinah filed a complaint for declaration of nullity of her marriage with Miko on
the ground of psychological incapacity under Article 36 of the Family Code. The court rendered the
following decision:

"1. Declaring the marriage null and void;

Dissolving the regime of absolute community of property; and

Declaring that a decree of absolute nullity of marriage shall only be issued after liquidation, partition
and distribution of the parties’ properties under Article 147 of the Family Code."

Dinah filed a motion for partial reconsideration questioning the portion of the decision on the issuance
of a decree of nullity of marriage only after the liquidation, partition and distribution of properties
under Article 147 of the Code.

If you are the judge, how will you decide petitioner’s motion for partial reconsideration? Why? (4%)

SUGGESTED ANSWER:
I will grant partial reconsideration. If the marriage is declared void under Article 36, the provisions of
the Family Code on liquidation, partition, and distribution of the properties on absolute community or
conjugal partnership will not apply but rather Article 147 or Article 148 depending on the presence or
3
absence of a legal impediment between them. In Dino v. Dino, the SC ruled that Art. 50 of the Family
Code and Section 19 of the Rules on Declaration of Nullity applies only to marriages which are declared
void ab initio or annulled by final judgment under Articles 40 and 45 of the Family. In short, Art. 50 of
the Family Code does not apply to marriages which are declared void ab initio under Art. 36 of the FC
which should be declared void without waiting for the liquidation of the properties of the parties.

VII.

Due to the continuous heavy rainfall, the major streets in Manila became flooded. This compelled Cris
to check-in at Square One Hotel. As soon as Crisgot off from his Toyota Altis, the Hotel’s parking
attendant got the key of his car and gave him a valet parking customer’s claim stub. The attendant
parked his car at the basement of the hotel. Early in the morning, Cris was informed by the hotel
manager that his car was carnapped. (4%)

What contract, if any, was perfected between Cris and the Hotel when Cris surrendered the key of his
car to the Hotel’s parking attendant?

What is the liability, if any, of the Hotel for the loss of Cris’ car?

Alain Dino vs. Ma. Caridad Dino G.R. No. 178044, January 19, 2011

11
SUGGESTED ANSWER:

The contract between Cris and Square One Hotel is one of necessary deposit. Deposit of effects
4
made by travelers or guests in hotels or inns is considered a necessary deposit. This includes
not only the personal effects brought inside the hotel premises but also vehicles or animals and
articles which have been introduced or placed in the annexes of the hotel.

5
In the case of Durban Apartments vs. Pioneer Insurance, the Supreme Court held the hotel liable
for the loss of the vehicle of the guest after its valet parking attendant parked the vehicle in
front of a bank near the hotel premises. The court ruled that the bank’s parking area became
an annex of the hotel when the management of the bank allowed the hotel to park vehicles
there on the night in question. The contract of deposit was perfected when the guest
surrendered the keys to his vehicle to the parking attendant and the hotel is under obligation of
safely keeping and returning it. Ultimately, Square One Hotel is liable for the loss of the vehicle.

VIII.

Tess leased her 1,500 sq. m. lot in Antipolo City to Ruth for a period of three (3) years, from
January 2010 to February 2013.

On March 19, 2011, Tess sent a letter to Ruth, part of which reads as follows:

"I am offering you to buy the property you are presently leasing at P5,000.00 per sq. m. or for a total of
P7,500,000.00. You can pay the contract price by installment for two (2) years without interest.

I will give you a period of one (1) year from receipt of this letter to decide whether you will buy the
property."

After the expiration of the lease contract, Tess sold the property to her niece for a total
consideration of P4 million.

Ruth filed a complaint for the annulment of the sale, reconveyance and damages against Tess and
her niece. Ruth alleged that the sale of the leased property violated her right to buy under the
principle of right of first refusal.

Is the allegation of Ruth tenable? (4%)

SUGGESTED ANSWER:
No, the allegation of Ruth is not tenable. The letter written by Tess did not grant a right of first
refusal to Ruth. At most, it is to be construed as an option contract whereby Ruth was given the
right to buy or not to buy the leased property. An option is itself not a purchase but it merely
secures the privilege to buy. However, the option is not valid because it was not supported by a
cause or consideration distinct from the price of the property. (Article 1479) Also, Ruth does not
appear to have exercised her option before the offer was withdrawn by the subsequent sale of the
property to the niece of Tess.

IX.

Article 1998, Civil Code

5
G.R. No. 179419 January 12, 2011

12
Spouses Macario and Bonifacia Dakila entered into a contract to sell with Honorio Cruz over a
parcel of industrial land in Valenzuela, Bulacan for a price of Three Million Five Hundred Thousand
Pesos (P3,500,000.00). The spouses would give a downpayment of Five Hundred Thousand Pesos
(P500,000.00) upon the signing of the contract, while the balance would be paid for the next three
consecutive months in the amount of One Million Pesos (P1,000,000.00) per month. The spouses
paid the first two (2) installments but not the last installment. After one (1) year, the spouses
offered to pay the unpaid balance which Honorio refused to accept.

The spouses filed a complaint for specific performance against Honorio invoking the application of
the Maceda Law. If you are the judge, how will you decide the case? (4%)

SUGGESTED ANSWER:
I will rule in favor of Honorio. The invocation of the Maceda Law is misplaced. The law applies only to
sale or financing of realty on installment payments including residential units or residential condominium
apartments and does not apply to sales of industrial units or industrial lands like in the case presented.
Another reason why the Maceda law will not apply is that, the sale in the case at bar is not the sale on
installment as contemplated by the law. The sale on installment covered by the Maceda Law is one
where the price is paid or amortized over a certain period in equal installments. The sale to the Spouses
Dakila is not a sale on installment but more of a straight sale where a down payment is to be made and
the balance to be paid in a relatively short period of three months.

X.

Dorotea leased portions of her 2,000 sq. m. lot to Monet, Kathy, Celia, and Ruth for five (5) years.
Two (2) years before the expiration of the lease contract, Dorotea sold the property to PM Realty
and Development Corporation. The following month, Dorotea and PM Realty stopped accepting
rental payments from all the lessees because they wanted to terminate the lease contracts.

Due to the refusal of Dorotea to accept rental payments, the lessees , Ruth, et al., filed a complaint
for consignation of the rentals before the Regional Trial Court (RTC) of Manila without notifying
Dorotea.

Is the consignation valid? (4%)

SUGGESTED ANSWER:
No, the consignation is not valid. For consignation of the thing or sum due to be proper, there must
be prior notice to the creditor that the debtor is going to consign the payment in court. This notice
is intended to give the creditor the opportunity to accept payment and thus avoid liability for costs
in case it is found that the act of consignation was properly made. Even on the assumption that
Dorotea was no longer the creditor as she had already sold the property to DM Realty, the facts do
not state that the realty corporation was also given notice before filing the case for consignation.

XI.

An easement that can be acquired by prescription: (1%)

Right of way

Watering of an animal

Lateral and subjacent support

Light and view

Correct answer – letter D – only continuous and apparent easements may


be acquired by prescription

13
XII.

J.C. Construction (J.C.) bought steel bars from Matibay Steel Industries (MSI) which is owned by
Buddy Batungbacal. J.C. failed to pay the purchased materials worth P500,000.00 on due date. J.C.
persuaded its client Amoroso with whom it had receivables to pay its obligation to MSI. Amoroso
agreed and paid MSI the amount of P50,000.00. After two (2) other payments, Amoroso stopped
making further payments.

Buddy filed a complaint for collection of the balance of the obligation and damages against J.C. J.C.
denied any liability claiming that its obligation was extinguished by reason of novation which took
place when MSI accepted partial payments from Amoroso on its behalf.

Was the obligation of J.C. Construction to MSI extinguished by novation? Why? (4%)

SUGEGSTED ANSWER:
No, the obligation of JC was not extinguished by novation. Novation may either be objective or
subjective. Subjective novation takes place by the substitution of debtor or subrogation of a third
person to the rights of the creditor. Novation by substituting a new debtor may take place even without
the knowledge or against the will of the original debtor but not without the consent of the creditor.
Moreover, novation must be expressed and it cannot be implied and there must be an agreement that
the old obligation is extinguished. In the case of JC, it does not appear that MSI had agreed to release
JC from the obligation. Hence, the obligation of JC was not extinguished.

XIII.

Esteban and Martha had four (4) children: Rolando, Jun, Mark, and Hector. Rolando had a daughter,
Edith, while Mark had a son, Philip. After the death of Esteban and Martha, their three (3) parcels of
land were adjudicated to Jun. After the death of Jun, the properties passed to his surviving spouse
Anita, and son Cesar. When Anita died, her share went to her son Cesar. Ten (10) years after, Cesar
died intestate without any issue. Peachy, Anita’s sister, adjudicated to herself the properties as the only
surviving heir of Anita and Cesar. Edith and Philip would like to recover the properties claiming that they
should have been reserved by Peachy in their behalf and must now revert back to them.

Is the contention of Edith and Philip valid? (4%)

SUGGESTED ANSWER:
No, the contention is not valid. The property adjudicated to Jun from the estate of his parents which he in
turn left to Anita and Cesar is not subject to reservation in favor of Edith and Philip. In Mendoza et. al.
6
vs.Policarpio, et. al. the court ruled that lineal character of the reservable property is reckoned from the
ascendant from whom the propositus received the property by gratuitous title. The ownership should be
reckoned only from Jun, as he is the ascendant from where the first transmission occurred or from whom
Cesar inherited the properties. Moreover, Article 891 provides that the person obliged to reserve the
property should be an ascendant. Peachy is not Cesar’s ascendant but a mere collateral relative. On the
assumption that the property is reservable, Edith and Philip being first cousins of Cesar who is the
propositus are disqualified to be reservatarios as they are not third degree relatives of Cesar.

XIV.

G.R. NO. 176422 -March 20, 2013

14
A pedestrian, who was four (4) months pregnant, was hit by a bus driver while crossing the street.
Although the pedestrian survived, the fetus inside her womb was aborted. Can the pedestrian recover
damages on account of the death of the fetus? (1%)

Yes, because of Article 2206 of the Civil Code which allows the surviving heirs to demand damages
for mental anguish by reason of the death of the deceased.

Yes, for as long as the pedestrian can prove that she was not at fault and the bus driver was the
one negligent.

No, because a fetus is not a natural person.

No, if the fetus did not comply with the requirements under Article 41 of the Civil Code.

Correct Answer is letter D – Article 41 of the Civil Code requires that to be considered a person, a
fetus with an intrauterine life of less than seven months must survive for the full twenty-four hours from
complete separation from the mother’s womb.

XV.

Mr. Bong owns several properties in Pasig City. He decided to build a condominium named Flores de
Manila in one of his lots. To fund the project, he obtained a loan from the National Bank (NB) secured
by a real estate mortgage over the adjoining property which he also owned.

During construction, he built three (3) pumps on the mortgaged property to supply water to the
condominium. After one (1) year, the project was completed and the condominium was turned over to
the buyers. However, Mr. Bong failed to pay his loan obligation to NB. Thus, NB foreclosed the
mortgaged property where the pumps were installed. During the sale on public auction of the
mortgaged property, Mr. Simon won in the bidding. When Mr. Simon attempted to take possession of
the property, the condominium owners, who in the meantime constituted themselves into Flores de
Manila Inc. (FMI), claimed that they have earlier filed a case for the declaration of the existence of an
easement before the Regional Trial Court (RTC) of Pasig City and prayed that the easement be
annotated in the title of the property foreclosed by NB. FMI further claimed that when Mr. Bong installed
the pumps in his adjoining property, a voluntary easement was constituted in favor of FMI.

Will the action prosper? (4%)

SUGGESTED ANSWER:
No, the action will not prosper. The essence of a mortgage is that it immediately subjects the property upon
which it is imposed, and whoever the possessor may be, to the fulfillment of the obligation for whose security
7
it was constituted. There was no voluntary easement in this case because at the time the water pumps were
constructed, the subject lot where the water pumps were constructed and the condominium belong to the
same person. No one can have an easement over his own property. (Bogo- Medellin vs. CA G.R. 124699, July
31, 2003.) Even of the assumption that an easement was created in favor of FMI that alone will not defeat
the right of the mortgagee to enforce the security if the debtor defaults.

XVI.

A congregation for religious women, by way of commodatum, is using the real property owned and
registered in the name of Spouses Manuel as a retreat house.

Maria, a helper of the congregation discovered a chest in the backyard. When she opened the chest, it
contained several pieces of jewelry and money. (4%)

Article 2126

15
Can the chest containing the pieces of jewelry and money be considered as hidden treasure?

Who has the right to claim ownership of it?

SUGGESTED ANSWER:

No, for property to be considered hidden treasure it must consist of money, jewelry or other
precious objects, the lawful ownership of which does not appear. In the case at bar, the chest
was just lay in the backyard and the real property where it was found belongs to the Spouses
Manuel. They are thus presumed the owner of the chest where the jewelry was found.

Since it does not come within the purview of hidden treasure, the spouses Manuel have the right to
claim ownership over the chest as well as its contents.

XVII.

On March 30, 2000, Mariano died intestate and was survived by his wife, Leonora, and children, Danilo
and Carlito. One of the properties he left was a piece of land in Alabang where he built his residential
house.

After his burial, Leonora and Mariano’s children extrajudicially settled his estate. Thereafter, Leonora
and Danilo advised Carlito of their intention to partition the property. Carlito opposed invoking Article
159 of the Family Code. Carlito alleged that since his minor child Lucas still resides in the premises, the
family home continues until that minor beneficiary becomes of age.

Is the contention of Carlito tenable? (4%)

SUGGESTED ANSWER:
8
No, the contention of Carlito is not tenable. In the case of Patricio v. Dario, with similar facts to the
case at bar, the court ruled that to qualify as beneficiary of the family home the person must be among
those mentioned in Article 154, he/she must be actually living in the family home and must be
dependent for legal support upon the head of the family. While Lucas, the son of Carlito satisfies the
first and second requisites, he cannot however, directly claim legal support from his grandmother,
Leonora because the person primarily obliged to give support to Lucas is his father, Carlito. Thus,
partition may be successfully claimed by Leonora and Danilo.

XVIII.

Spouses Magtanggol managed and operated a gasoline station on a 1,000 sq.m. lot which they leased
from Francisco Bigla -awa. The contract was for a period of three (3) years. When the contract expired,
Francisco asked the spouses to peacefully vacate the premises. The spouses ignored the demand and
continued with the operation of the gasoline station.

One month after, Francisco, with the aid of a group of armed men, caused the closure of the gasoline
station by constructing fences around it.

Was the act of Francisco and his men lawful? Why? (4%)

SUGGESTED ANSWER:
No, the act was not lawful. Even if the lessee’s right to occupy the premises has expired, the lessor cannot
physically oust the lessee from the leased premises if the latter refuses to vacate. The lessor must go
through the proper channels by filing an appropriate case for unlawful detainer or recovery of possession.
Every possessor has a right to be respected in his possession (Article 539) and in no case my possession be
acquired through force or intimidation as long as there is a possessor who objects thereto. (Article

G.R. No. 170829 November 20, 2006

16
The act of Francisco is an abuse of rights because even if he has the right to recover possession of his
property, he must act with justice and give the lessees their day in court and observe honesty and good
faith.

XIX.

Who enjoys the Right of Retention? (1%)

Depositary until full payment of what may be due him in deposit.

Lessee if he advances the expenses for the repair of the leased premises.

Bailee if bailor owes him something.

Builder in bad faith for the recovery of necessary and useful expenses.

Correct answer is letter A – depositary (Article 1994)

XX.

Mabuhay Elementary School organized a field trip for its Grade VI students in Fort Santiago,
Manila Zoo, and Star City. To be able to join, the parents of the students had to sign a piece of
paper that reads as follows:

"I allow my child (name of student), Grade – Section, to join the school’s field trip on February
14, 2014.

I will not file any claim against the school, administrator or teacher in case something happens
to my child during the trip."

Joey, a 7-year-old student of Mabuhay Elementary School was bitten by a snake while the
group was touring Manila Zoo. The parents of Joey sued the school for damages. The school,
as a defense, presented the waiver signed by Joey’s parents.

Was there a valid waiver of right to sue the school? Why? (4%)

SUGGESTED ANSWER:
No, there was no valid waiver of the right to sue the school. A waiver to be valid must have three
requisites 1) existence of the right; 2) legal capacity of the person waiving the right and 3) the waiver
must not be contrary to law, morals, good customs, public order or public policy or prejudicial to a third
person with a right recognized by law. In the case presented, the waiver may be considered contrary to
public policy as it exonerates the school from liability for future negligence. The waiver in effect allows
the school to not exercise even ordinary diligence.

XXI.

A delayed accession is: (1%)

formation of an island

avulsion

alluvium

17
(D) change in the course of the riverbed

Correct answer is letter B (Article 459 Civil Code)

XXII.

On March 27, 1980, Cornelio filed an application for land registration involving a parcel of
agricultural land that he had bought from Isaac identified as Lot No. 2716 with an area of one
hectare. During the trial, Cornelio claimed that he and his predecessors-in-interest had been in
open, continuous, uninterrupted, public and adverse possession and occupation of the land for
more than thirty (30) years. He likewise introduced in evidence a certification dated February
12, 1981 citing a presidential declaration to the effect that on June 14, 1980, agricultural lands
of the public domain, including the subject matter of the application, were declared alienable
and disposable agricultural land. (4%)

If you are the judge, will you grant the application for land registration of Cornelio?

Can Cornelio acquire said agricultural land through acquisitive prescription, whether
ordinary or extraordinary?

SUGGESTED ANSWER:

No, I will not grant the application. To be entitled to registration of the parcel of land, the applicant
must show that the land being applied for is alienable land. At the time of the filing of the
application, the land has not yet been declared alienable by the state. (Republic v. CA, G.R. No.
144057, January 17, 2005)

Cornelio can acquire the land by acquisitive prescription only after it was declared part of alienable
land by the state by possession for the required number of years for ordinary prescription, ten
years possession in good faith with just title or extraordinary prescription by possession for thirty
years without need of any other condition. (Article 1134, Civil Code)

XXIII.

After undergoing sex reassignment in a foreign country, Jose, who is now using the name
of "Josie," married his partner Ador. Is the marriage valid? (1%)

Yes, the marriage is valid for as long as it is valid in the place where it is celebrated
following Article 17 of the Civil Code.

Yes, the marriage is valid if all the essential and formal elements of marriage under
the Family Code are present.

No, the marriage is not valid because one essential element of marriage is absent.

No, the marriage is not valid but is voidable because "Josie" concealed her real
identity.

Correct answer is letter C – not valid for lack of one essential requirement (Silverio v.
Republic G.R. No. 174689, October 22, 2007)

XXIV.

18
Ted, married to Annie, went to Canada to work. Five (5) years later, Ted became a naturalized
Canadian citizen. He returned to the Philippines to convince Annie to settle in Canada.
Unfortunately, Ted discovered that Annie and his friend Louie were having an affair. Deeply
hurt, Ted returned to Canada and filed a petition for divorce which was granted. In December
2013, Ted decided to marry his childhood friend Corazon in the Philippines. In preparation for
the wedding, Ted went to the Local Civil Registry of Quezon City where his marriage contract
with Annie was registered. He asked the Civil Register to annotate the decree of divorce on his
marriage contract with Annie. However, he was advised by the National Statistics Office (NSO)
to file a petition for judicial recognition of the decree of divorce in the Philippines.

Is it necessary for Ted to file a petition for judicial recognition of the decree of divorce he
obtained in Canada before he can contract a second marriage in the Philippines? (4%)

SUGGESTED ANSWER:
Yes, a divorce decree even if validly obtained abroad cannot have effect in the Philippines unless it is
judicially recognized through an appropriate petition filed before Philippine courts. In Corpuz v. Sto.
9
Tomas, the SC ruled that the foreigner must file a petition under Rule 108 and prove therein the fact
of divorce by presenting an official copy attested by the officer having custody of the original. He must
also prove that the court which issued the divorce has jurisdiction to issue it and the law of the foreign
country on divorce.

XXV.

Mario executed his last will and testament where he acknowledges the child being conceived by
his live-in partner Josie as his own child; and that his house and lot in Baguio City be given to
his unborn conceived child. Are the acknowledgment and the donation mortis causa valid?
Why? (4%)
SUGGESTED ANSWER:
Yes, the acknowledgment is considered valid because a will (although not required to be filed by the
notary public) may still constitute a document which contains an admission of illegitimate filiation. Article
834 also provides that the recognition of an illegitimate child does not lose its legal effect even though
the will wherein it was made should be revoked. This provision by itself warrants a conclusion that a will
may be considered as proof of filiation. The donation mortis causa may be considered valid because
although unborn, a fetus has a presumptive personality for all purposes favorable to it provided it be
born under the conditions specified in Article 41.

XXVI.

Isaac leased the apartment of Dorotea for two (2) years. Six (6) months after, Isaac subleased
a portion of the apartment due to financial difficulty. Is the sublease contract valid? (1%)

Yes, it is valid for as long as all the elements of a valid sublease contract are present.

Yes, it is valid if there is no express prohibition for subleasing in the lease contract.

No, it is void if there is no written consent on the part of the lessor.

Gerbert Corpuz vs. Daisylyn Sto. Tomas G.R. No. 186571; August 11, 2010

19
(D) No, it is void because of breach of the lease contract.

Correct answer is letter B – Article 1650

XXVII.

Fe, Esperanza, and Caridad inherited from their parents a 500 sq. m. lot which they leased to
Maria for three (3) years. One year after, Fe, claiming to have the authority to represent her
siblings Esperanza and Caridad, offered to sell the leased property to Maria which the latter
accepted. The sale was not reduced into writing, but Maria started to make partial payments
to Fe, which the latter received and acknowledged. After giving the full payment, Maria
demanded for the execution of a deed of absolute sale which Esperanza and Caridad refused
to do. Worst, Maria learned that the siblings sold the same property to Manuel. This compelled
Maria to file a complaint for the annulment of the sale with specific performance and damages.

If you are the judge, how will you decide the case? (4%)

SUGGESTED ANSWER:
I will dismiss the case for annulment of the sale and specific performance filed by Maria with respect to
the shares pertaining to Esperanza and Caridad. Since the object of the sale is a co -owned property, a
co-owner may sell his undivided share or interest in the property owned in common but the sale will be
subject to the result of the partition among the co-owners. In a co-ownership there is no mutual agency
except as provided under Article 487. Thus, Fe cannot sell the shares of Esperanza and Caridad without
a special power of attorney from them and the sale with respect to the shares of the latter without their
written authority is void under Article 1874. Hence, the sale of the property to Manuel is not valid with
respect to the shares of Esperanza and Caridad. Maria can only assail the portion pertaining to Fe as the
same has been validly sold to her by Fe.

XXVIII.

Spouses Esteban and Maria decided to raise their two (2) nieces, Faith and Hope, both minors,
as their own children after the parents of the minors died in a vehicular accident.

Ten (10) years after, Esteban died. Maria later on married her boss Daniel, a British national
who had been living in the Philippines for two (2) years.

With the permission of Daniel, Maria filed a petition for the adoption of Faith and Hope. She
did not include Daniel as her co-petitioner because for Maria, it was her former husband
Esteban who raised the kids.

If you are the judge, how will you resolve the petition? (4%)

SUGGESTED ANSWER:
I will dismiss the petition for adoption. The rule is that the husband and wife must jointly adopt and there are
only three recognized exceptions to joint adoption by the husband and wife: 1) if one spouse seeks to adopt
the legitimate child of the other; 2) if one spouse seeks to adopt his or her own illegitimate child;
if the spouses are legally separated. The case of Maria and Daniel does not appear to fall under any of
the recognized exceptions, accordingly the petition filed by the wife alone should be dismissed.

XXIX

20
Timothy executed a Memorandum of Agreement (MOA) with Kristopher setting up a business
venture covering three (3) fastfood stores known as "Hungry Toppings" that will be
established at Mall Uno, Mall Dos, and Mall Tres.

The pertinent provisions of the MOA provides:

Timothy shall be considered a partner with thirty percent (30%) share in all of the
stores to be set up by Kristopher;

The proceeds of the business, after deducting expenses, shall be used to pay the
principal amount of P500,000.00 and the interest therein which is to be computed
based on the bank rate, representing the bank loan secured by Timothy;

The net profits, if any, after deducting the expenses and payments of the principal and
interest shall be divided as follows: seventy percent (70%) for Kristopher and thirty
percent (30%) for Timothy;

Kristopher shall have a free hand in running the business without any interference from
Timothy, his agents, representatives, or assigns , and should such interference happen,
Kristopher has the right to buy back the share of Timothy less the amounts already paid
on the principal and to dissolve the MOA; and

Kristopher shall submit his monthly sales report in connection with the business to
Timothy.

What is the contractual relationship between Timothy and Kristopher? (4%)

SUGGESTED ANSWER:
The contractual relationship between Timothy and Kristopher is a contract of partnership as defined
under Article 1767 of the Civil Code, since they have bound themselves to contribute money, property
or industry to a common fund, with the intention of dividing the profits of the partnership between
them. With a seed money of P500, 000.00 obtained by Timothy through a bank loan, they agreed to
divide the profits, 70% for Kristopher and 30% for Timothy.

However, to be more specific, theirs is a limited partnership as defined under Article 1843 of the Civil
Code because Timothy does not take part in the control of the business pursuant to Article 1848, Civil
Code. Nevertheless, Timothy is entitled to monthly sales reports in connection with the business, a
right enshrined in Article 1851 of the Civil Code.

XXX.

Joe Miguel, a well-known treasure hunter in Mindanao, executed a Special Power of Attorney
(SPA) appointing his nephew, John Paul, as his attorney -infact. John Paul was given the
power to deal with treasure-hunting activities on Joe Miguel’s land and to file charges against
those who may enter it without the latter’s authority. Joe Miguel agreed to give John Paul forty
percent (40%) of the treasure that may be found on the land.

Thereafter, John Paul filed a case for damages and injunction against Lilo for illegally entering
Joe Miguel’s land. Subsequently, he hired the legal services of Atty. Audrey agreeing to give
the latter thirty percent (30%) of Joe Miguel’s share in whatever treasure that may be found in
the land.

Dissatified however with the strategies implemented by John Paul, Joe Miguel unilaterally
revoked the SPA granted to John Paul.
21
Is the revocation proper? (4%)

SUGGESTED ANSWER:
10
No, the revocation was not proper. As a rule, a contract of agency may be revoked by the principal at will.
However, an agency ceases to be revocable at will if it is coupled with an interest or if it is a means of
fulfilling an obligation already contracted. (Article 1922). In the case at bar, the agency may be deemed an
agency coupled with an interest not only because of the fact that John Paul expects to receive 40% of
whatever treasure may be found but also because he also contracted the services of a lawyer pursuant to
his mandate under the contract of agency and he therefore stands to be liable to the lawyer whose services
he has contracted. (Sevilla v. Tourist World Service, G.R. No. L-41182-3 April 16, 1988)

UNIVERSITY OF SANTO TOMAS


SUGGESTED ANSWERS
2015 CIVIL LAW BAR EXAMINATIONS
By: Assoc. Dean Viviana M. Paguirigan

I.

Alden and Stela were both former Filipino citizens. They were married in the Philippines
but they later migrated to the United States where they were naturalized as American citizens.
In their union they were able to accumulate several real properties both in the US and in the
Philippines. Unfortunately, they were not blessed with children. In the US, they executed a
joint will instituting as their common heirs to divide their combined estate in equal shares, the
five siblingsand of Alden the seven siblings of Stela. Alden passed away in 2013 and a year
later, Stela also died. The siblings of Alden who were all citizens of the US instituted probate
proceedings in a US court impleading the siblings of Stela who were all in the Philippines.
Was the joint will executed by Alden and Stela who were both former Filipinos valid?
Explain with legal basis. (3%)

Can the joint will produce legal effect in the Philippines with respect to the
propertiesand of Alden Stela found here? If so, how? (3%)

Is the situation presented in Item I an example of depe9age? (2%)

SUGGESTED ANSWER:

Yes, the joint will of Alden and Stela is considered valid. Being no longer Filipino citizens at the
time they executed their joint will, the prohibition under our Civil Code on joint wills will no
longer apply to Alden and Stela. For as long as their will was executed in accordance with the
law of the place where they reside, or the law of the country of which they are citizens or even
in accordance with the Civil Code, a will executed by an alien is considered valid in the
Philippines. (Article 816)
Yes, the joint will of Alden and Stela can take effect even with respect to the properties
located in the Philippines because what governs the distribution of their estate is no longer
Philippine law but their national law at the time of their demise. Hence, the joint will produces
legal effect even with respect to the properties situated in the Philippines.

Article 1920

22
No, because depecage is a process of applying rules of different states on the basis of the
precise issue involved. It is a conflict of laws where different issues within a case may be
governed by the laws of different states. In the situation in letter (a) no conflict of laws will
arise because Alden and Stela are no longer Filipino citizens at the time of the execution of
their joint will and the place of execution is not the Philippines.

II.

Marco and Gina were married in 1989. Ten years later, or in 1999, Gina left Marco and
lived with another man, leaving their two children of school age with Marco. When Marco
needed money for their children's education he sold a parcel of land registered in his name,
without Gina's consent, which he purchased before his marriage. Is the sale by Marco valid,
void or voidable? Explain with legal basis. (4%)

SUGGESTED ANSWER:

The sale made by Marco is considered void. The parties were married in 1989 and no mention
was made whether they executed a marriage settlement. In the absence of a marriage
settlement, the parties shall be governed by absolute community of property whereby all the
properties owned by the spouses at the time of the celebration of the marriage as well as
whatever they may acquire during the marriage shall form part of the absolute community. In
ACP, neither spouse can sell or encumber property belonging to the ACP without the consent
of the other. Any sale or encumbrance made by one spouse without the consent of the other
shall be void although it is considered as a continuing offer on the part of the consenting
spouse upon authority of the court or written consent of the other spouse. (Article 96 FC)

III.

Julie had a relationship with a married man who had legitimate children. A son was born out of
that illicit relationship in 1981. Although the putative father did not recognize the child in his
certificate of birth, he nevertheless provided the with child all the support he needed and
spent time regularly with the child and his mother. When the man died in 2000, the child was
already 18 years old so he filed a petition to be recognized as an illegitimate child of the
putative father and sought to be given a share in his putative father's estate. The legitimate
family opposed, saying that under the Family Code his action cannot prosper because he did
not bring the action for recognition during the lifetime of his putative father.
If you were the judge in this case, would how you rule? (4%)

Wishing to keep the peace, the child during the pendency of the case decides to
compromise with his putative father's family by abandoning his petition in exchange for Yi
of what he would have received as inheritance if he were recognized as an illegitimate
child. As the judge, would you approve such a compromise? (2%)
SUGGESTED ANSWER:

23
If I were the judge, I will not allow the action for recognition filed after the death of the
putative father. Under the Family Code, an illegitimate child who has not been recognized
by the father in the record of birth, or in a private handwritten instrument, or in a public
document and may prove his filiation based on open and continuous possession of the
status of an illegitimate child but pursuant to Article 175, he or she must file the action for
recognition during the lifetime of the putative father. The provision of Article 285 of the
Civil Code allowing the child to file the action for recognition even after the death of the
father will not apply because in the case presented, the child was no longer a minor at the
time of death of the putative father.

No, I will not approve the compromise agreement because filiation is a matter to be decided
by law. It is not for the parties to stipulate whether a person is a legitimate or illegitimate
child of another. (De Jesus v. Estate of Dizon 366 SCRA 499) In all cases of illegitimate
children, their filiation must be duly proved. (Article 887, Civil Code)

ALTERNATIVE ANSWER: Yes, I would approve the compromise because it is no longer


considered future inheritance. What the law prohibits is a compromise with respect to future
legitime. In this case, the father is already dead so the compromise is considered valid.

IV.

Bert and Joe, both male and single, lived together as common law spouses and agreed to raise a
son of Bert's living brother as their child without legally adopting him. Bert worked while Joe took
care of their home and the boy. In their 20 years of cohabitation they were able to acquire real
estate assets registered in their names as co-owners. Unfortunately, Bert died of cardiac arrest,
leaving no will. Bert was survived by his biological siblings, Joe, and the boy.

Can Article 147 on co-ownership apply to Bert and Joe, whereby all properties they
acquired will be presumed to have been acquired by their joint industry and shall be owned
by them in equal shares? (2%)

What are the successional rights of the boy Bert Joe and raised as their son? (2%)

If Bert and Joe had decided in the early years of their cohabitation to jointly adopt the boy,
would they have been legally allowed to do so? Explain with legal basis. (3%)

SUGGESTED ANSWER:

No, Article 147 cannot apply to Bert and Joe because the law only applies to a man and a
woman who are capacitated to marry each other who live together as husband and wife
without the benefit of marriage or under a void marriage. In the case of Bert and Joe, they
are both men so the law does not apply.
Neither of the two will inherit from Bert. Joe cannot inherit because the law does not recognize the
right of a stranger to inherit from the decedent in the absence of a will. Their cohabitation will
not vest Joe with the right to inherit from Bert. The child will likewise not inherit from Bert
because of the lack of formal adoption of the child. A mere ward or “ampon” has no right to
inherit from the adopting parents. (Manuel v. Ferrer, 247 SCRA 476)

24
No, because joint adoption is allowed between husband and wife. Even if Bert and Joe are
cohabiting with each other, they are not vested with the right to jointly adopt under the
Family Code or even under the Domestic Adoption Act. (Section 7, R.A. 8552)

V.

Mrs. L was married to a ship captain who worked for an international maritime vessel. For her
and her family's support, she would claim monthly allotments from her husband's company.
One day, while en route from Hong Kong to Manila, the vessel manned by Captain L
encountered a severe typhoon at sea. The captain was able to send radio messages of distress
to the head office until all communications were lost. In the weeks that followed, the search
operations yielded debris of the lost ship but the bodies of the crew and the passengers were
not recovered. The insurance company thereafter paid out the death benefits to all the heirs of
the passengers and crew. Mrs. L filed a complaint demanding that her monthly allotments
continue for the next four years until her husband may be legally presumed dead because of
his absence. If you were the magistrate would how you rule? (3%)
SUGGESTED ANSWER:
I would rule against Mrs. L. There is no merit in her contention that the monthly allotments to
her should continue despite the presumptive death of the husband. In case of disappearance
where there is danger of death, the person shall be presumed to have died at the beginning of
the four (4) year period although his succession will be opened only at the end of the four
year period. (Article 391, Civil Code) Since the husband of Mrs. L is presumed to have died at
about the time of disappearance, he is no longer entitled to receive his salary from the day the
presumption of death arises.

VI.

Kardo met Glenda as a young lieutenant and after a whirlwind courtship, they were
married. In the early part of his military career, Kardo was assigned to different places all over
the country but Glenda refused to accompany him as she preferred to live in her hometown.
They did not live together until the 12th year of their marriage when Kardo had risen up the
ranks and was given his own command. They moved to living quarters in Fort Gregorio. One
day, while Kardo was away on official business, one of his military aides caught Glenda having
sex with the corporal assigned as Kardo's driver. The aide immediately reported the matter to
Kardo who rushed home to confront his wife. Glenda readily admitted the affair and Kardo
sentawayher in anger. Kardo would later come to know the true extent of Glenda's
unfaithfulness from his aides, his household staff, and former neighbors who informed him
that Glenda has had intimate relations with various men throughout their marriage whenever
Kardo was away on assignment.
Kardo filed a petition for declaration of nullity of marriage under Article 36. Based on
interviews from Kardo, his aide, and the housekeeper, a psychologist testified that Glenda's
habitual infidelity was due to her affliction with Histrionic Personality Disorder, an illness
characterized by excessive emotionalism and uncontrollable attention-seeking behavior rooted
in Glenda's abandonment as a child by her father. Kardo himself, his aide, and his
housekeeper also testified in court. The RTC granted the petition, relying on the liberality
espoused by Te v. Te and Azcueta v. Republic. However, the OSG filed an appeal, arguing that
sexual infidelity was only a ground for legal separation and that the RTC failed to abide by the
guidelines laid down in the Molina case. How would you decide the appeal? (5%)

25
SUGGESTED ANSWER:
I will resolve the appeal in favor of the Republic. In the case of Dedel v. Dedel, (G.R. No.
151867 January 29, 2004) the Supreme Court refused to declare the marriage of the parties
void on the ground of sexual infidelity of the wife Sharon. In case mentioned, the wife
committed infidelity with several men up to the extent of siring two illegitimate children with a
foreigner. The court, however, said that it was not shown that the sexual infidelity was a
product of a disordered personality and that it was rooted in the history of the party alleged to
be psychologically incapacitated. Also, the finding of psychological incapacity cannot be based
on the interviews conducted by the clinical psychologist on the husband or his witnesses and
the person alleged to be psychologically incapacitated must be personally examined to arrive
at such declaration. (Marcos v. Marcos, 343 SCRA 755; Agraviador v. Agraviador, G.R. No.
170729- December 8, 2010)

VII.

Mr. and Mrs. X migrated to the US with all their children. As they had no intention of coming
back, they offered their house and lot for sale to their neighbors, Mr. and Mrs. A (the buyers)
who agreed to buy the property for 128 Million. Because Mr. and Mrs. A needed to obtain a
loan from a bank first, and since the sellers were in a hurry to migrate, the latter told the
buyers that they could already occupy the house, renovate it as it was already in a state of
disrepair, and pay only when their loan is approved and released. While waiting for the loan
approval, the buyers spent .Pl Million in repairing the house. A month later, a person carrying
an authenticated special power of attorney from the sellers demanded that the buyers either
immediately pay for the property in full now or vacate it and pay damages for having made
improvements on the property without a sale having been perfected.

What are the buyers' options or legal rights with respect to the they expenses incurred
in improving the property under circumstances? (3%)

Can the buyers be made to immediately vacate on the ground that the sale was not
perfected? Explain briefly. (3%)
SUGGESTED ANSWER:
The buyers here may be deemed possessors or builders in good faith because they were made to
believe that they were allowed to make repairs or renovation by the sellers themselves. As
builders in good faith, they have the right to seek reimbursement for the value of the
improvements in case the owner decides to appropriate them. They cannot be asked to
remove the improvements because that is not one of the options given by law to the
landowner in case the builder is in good faith.

No, the buyers cannot be made to vacate on the ground that the sale was not perfected for the
fact of the matter is that a contract of sale is consensual and is perfected by mere consent.
(Article 1315, Civil Code) In this case, there was an agreement to deliver a determinate thing
for a price certain in money. When the owners made an offer to sell their property to Mr. and
Mrs. A and the latter accepted the offer, there was already a meeting of the minds between
the parties resulting in the perfection of the contract of sale.

26
VIII.

X, Y, Z are siblings who inherited a IO-storey building from their parents. They agreed in writing
to maintain it as a co -owned property for leasing out and to divide the net profits among
themselves equally for a period of 20 years. On the gth year, X wanted to get out of the co-
ownership so he could get his 1/3 share in the property. Y and Z refused, saying X is bound by
their agreement to keep the co-ownership for 20 years. Are Y and Z correct? Explain. (3%)

SUGGESTED ANSWER:
Y and Z are partly correct. The law provides that none of the co-owners shall be obliged to
remain in the co-ownership and it is the right of a co-owner to ask for partition of the co-
ownership anytime. One exception to the rule is if the co -owners agree to keep the thing
undivided which period shall not exceed ten years. In this case, the agreement to keep the
thing undivided shall be valid at the most for ten years. (Article 494, Civil Code)

IX.

Jose, single, donated a house and lot to his only niece, Maria, who was of legal age and who
accepted the donation. The donation and Maria's acceptance thereof were evidenced by a
Deed of Donation. Maria then lived in the house and lot donated to her, religiously paying real
estate taxes thereon. Twelve years later, when Jose had already passed away, a woman
claiming to be an illegitimate daughter of Jose filed a complaint against Maria. Claiming rights
as an heir, the woman prayed that Maria be ordered to reconvey the house and lot to Jose's
estate. In her complaint she alleged that the notary public who notarized the Deed of
Donation had an expired notarial commission when the Deed of Donation was executed by
Jose. Can Maria be made to reconvey the property? What can she put up as a defense? (4%)
SUGGESTED ANSWER:
No. Maria cannot be compelled to reconvey the property. The Deed of Donation was void
because it was not considered a public document. However, a void donation can trigger
acquisitive prescription. (Solis v. CA 176 SCRA 678; Doliendo v. Biarnesa 7 Phil. 232) The void
donation has a quality of titulo colorado enough for acquisitive prescription especially since 12
years had lapsed from the deed of donation.

ALTERNATIVE ANSWER: Yes, Maria can be made to reconvey the property. The law provides
that no person may give or receive by way of donation more than what he may give or receive
by will. On the assumption that the property donated to Maria is the only property of Jose, the
legitime of his illegitimate child would be impaired if Maria would be allowed to keep the entire
property. After taking into account the value of the property, Maria can be made to reconvey
the property to the extent necessary to satisfy the legitime of Jose’s illegitimate daughter
provided that the woman claiming to be Jose’s child can prove her filiation to the deceased.

Maria can set up the defense that the action has prescribed. An action for revocation of the
donation on the ground that it impaired the legitime of a compulsory heir may only be filed
within ten (10) years from the time the cause of action accrues which is at the time of the
death of Jose. The facts are not clear as to when Jose died but on the assumption that he died
ten years prior to the filing of the action, the same has clearly prescribed.

X.

X, a dressmaker, accepted clothing materials from Karla to make two dresses for her.
dayOn the X was supposed to deliver Karla's dresses, X called up Karla to say that she had an
27
urgent matter to attend to and will deliver them the next day. That night, however, a robber
broke into her shop and took everything including Karla's two dresses. X claims she is not
liable to deliver Karla's dresses or to pay for the clothing materials considering she herself was
a victim of the robbery which was a fortuitous event and over which she had no control. Do
you agree? Why? (3%)
SUGGESTED ANSWER:
No, I do not agree with the contention of X. The law provides that except when it is otherwise
declared by stipulation or when the law provides or the nature of the obligation requires the
assumption of risk, no person shall be liable for those events which could not be foreseen or
which though foreseen were inevitable. (Article 1174, Civil Code) In the case presented, X
cannot invoke fortuitous event as a defense because she had already incurred in delay at the
time of the occurrence of the loss. (Article 1165, Civil Code)

XI.

Jackie, 16, inherited a townhouse. Because she wanted to study in an exclusive school, she
sold her townhouse by signing a Deed of Sale and turning over possession of the same to the
buyer. Whenthatthe buyer discovered she was still a minor, she promised to execute another
Deed of Sale when she turns 18. When Jackie turned 25 and was already working, she wanted
to annul the sale and return the buyer's money to recover her townhouse. Was the sale
contract void, voidable or valid? Can Jackie still recover the property? Explain. (4%)
SUGGESTED ANSWER:
The contract of sale was voidable on the ground that Jackie is incapable of giving consent at
the time of the execution of the sale. (Article 1390 and Article 1327) Jackie can no longer
recover the townhouse unit because if a contract is voidable on the ground of minority, the
action to annul it must be filed within four (4) years from attainment of the age of majority.
Since Jackie was already 25 years old, the action has clearly prescribed because she should
have filed it before she reached the age of 22. (Article 1391, Civil Code)

XII.

A. Iya and Betty owed Jun P500,000.00 for advancing their equity in a corporation they
joined as incorporators. Iya and Betty bound themselves solidarily liable for the debt. Later,
Iya and Jun became sweethearts so Jun condoned the debt of P500,000.00. May lya demand
from Betty ~250,000.00 as her share in the debt? Explainlegal with basis. (2%)
B. Juancho, Don and Pedro borrowed ~150,000.00 from their friend Cita to put up an
internet cafe orally promising to pay her the full amount after one year. Because of their lack
of business know-how, their business collapsed. Juancho and Don ended up penniless but
Pedro was able to borrow money and put up a restaurant which did well. Can Cita demand
that Pedro pay the entire obligation since he, together with the two others, promised to pay
the amount in full after one year? Defend your answer. (2%)
SUGGESTED ANSWER:

No, Iya may not demand the 250,000 from Betty because the entire obligation has been
condoned by the creditor Jun. In a solidary obligation the remission of the whole obligation
obtained by one of the solidary debtors does not entitle him to reimbursement from his co-
debtors. (Article 1220, Civil Code)
28
No, Cita cannot demand that Pedro pay the entire obligation because the obligation in this case is
presumed to be joint. The concurrence of two or more creditors or of two or more debtors in
one and the same obligation does not imply that each one of the former has a right to
demand, or that each one of the latter is bound to render, entire compliance with the
prestation. (Article 1207) In a joint obligation, there is no mutual agency among the joint
debtors such that if one of them is insolvent the others shall not be liable for his share.

XIII.

A. X and Y are partners in a shop offering portrait painting. Y provided the capital and
the marketing while X was the portrait artist. They accepted the PS0,000.00 payment of Kyla
to do her portrait but X passed away without being able to do it. Can Kyla demand that Y
deliver the portrait she had paid for because she was dealing the with business establishment
and not with the artist personally? Why or why not? (3%)
B. In this jurisdiction, is a joint venture (i.e., a group of corporations contributing
resources for a specific project and sharing the profits therefrom) considered a partnership?
(3%)

SUGGESTED ANSWER:

No Kyla cannot demand that Y deliver the portrait. The death of X has the effect of dissolving the
partnership. (Article 1830, Civil Code) Also, while the obligation was contracted by the
partnership, it was X who was supposed to create the portrait for Kyla. Since X died before
creating the portrait, the obligation can no longer be complied because of impossibility of
performance. (Article 1266) In obligations to do, the debtor shall be released when the
prestation becomes legally or physically impossible without the debtor’s fault.
Yes, under Philippine law, a joint venture is understood to mean an organization formed for some
temporary purpose and is hardly distinguishable form a partnership since its elements are
similar which are: community of interest in business, sharing of profits, and losses, and a
mutual right of control. (Primelink Properties v. Lazatin June 27, 2006 citing Blackner v.
Mcdermott, 176 F. 2d 498[1949])

XIV.

A driver of a bus owned by company Z ran over a boy who died instantly. A criminal case for
reckless imprudence resulting in homicide was filed against the driver. He was convicted and
was ordered to pay P2 Million in actual and moral damages to the parents of the boy who was
an honor student and had a bright future. Without even trying to find out if the driver had
assets or means to pay the award of damages, the parents of the boy filed a civil action
against the bus company to make it directly liable for the damages.
Will their action prosper? (4%)
If the parents of the boy do not wish to file a separate civil action against. the bus
company, can they still make the bus company liable if the driver cannot' pay the award for
damages? If so, what is the nature of the employer's liability and how may civil damages be
satisfied? (3%)

SUGGESTED ANSWER:

29
Yes, the action will prosper. The liability of the employer in this case may be based on quasi-delict
and is included within the coverage of independent civil action. It is not necessary to enforce
the civil liability based on culpa aquiliana that the driver or employee be proven to be insolvent
since the liability of the employer for the quasi-delicts committed by their employees is direct
and primary subject to the defense of due diligence on their part. (Article 2176; Article 2180)
Yes, the parents of the boy can enforce the subsidiary liability of the employer in the criminal case
against the driver. The conviction of the driver is a condition sine qua non for the subsidiary
liability of the employer to attach. Proof must be shown that the driver is insolvent. (Article
103, Revised Penal Code)

XV.

Sara borrowed PS0,000.00 from Julia and orally promised to pay it within six months.
When Sara tried to pay her debt on the gth month, Julia demanded the payment of interest of
12o/o per annum because of Sara's delay in payment. Sara paid her debt and the interest
claimed by Julia. After rethinking, Sara demanded back from Julia the amount she had paid as
interest. Julia claims she has no obligation to return the interest paid by Sara because it was a
natural obligation which Sara voluntarily performed and can no longer recover. Do you agree?
Explain. (4%)

Distinguish civil and natural obligations. (2%)

SUGGESTED ANSWER:

No, the case is not one of a natural obligation because even if the contract of loan is verbal, the
delay of Julia made her liable for interest upon demand by Sara. This is not a case of a natural
obligation but a civil obligation to pay interest by way of damages by reason of delay. (Article
1956; Article 1169; Article 2209 Civil Code)
A civil obligation is based on positive law which gives a right of action to compel their
performance in case of breach. A natural obligation is based on equity and natural law and
cannot be enforced by court action but after voluntary fulfilment by the obligor, they authorize
the retention of what may have been delivered or rendered by reason thereof. (Article 1423,
Civil Code)

XVI.

Donna pledged a set of diamond ring and earrings to Jane for P200,000.00 She was
made to sign an agreement that if she cannot pay her debt within six months, Jane could
immediately appropriate the jewelry for herself. After six months, Donna failed to pay. Jane
then displayed the earrings and ring set in her jewelry shop located in a mall. A buyer, Juana,
bought the jewelry set for P300,000.00.
Was the agreement which Donna signed with Jane valid? Explain with legal basis. (2%)
Can Donna redeem the jewelry set from Juana by paying the amount she owed Jane to
Juana? Explain with legal basis. (2%)
Give an example of a pledge created by operation of law. (2%)

30
SUGGESTED ANSWER:
appropriate the jewelry upon default of Donna is considered pactum commissorium and it is
considered void by law. ( Article 2088)
No, Donna cannot redeem it from Juana because the pledge contract is between her and Jane.
Juana is not a party to the pledge contract. (Article 1311, Civil Code)
One example of a pledge created by operation of law is the right of the depositary to retain the
thing deposited until the depositor shall have paid him whatever may be due to the depositary
by reason of the deposit. (1994) Another is the right of the agent to retain the thing which is
the object of the agency until the principal reimburses him the expenses incurred in the
execution of the agency. (Article 1914, Civil Code)

XVII.

Z, a gambler, wagered and lost P2 Million in baccarat, a card game. He was pressured into
signing a Deed of Absolute Sale in favor of the winner covering a parcel ·of land with
improvements worth P20 Million. One month later, the supposed vendee of the property
demanded that he and his family vacate the property subject of the deed of sale. Was the
deed of sale valid? What can Z do? (4%)
SUGGESTED ANSWER:

The sale is valid. Being pressured to sign the deed of sale is not equivalent to vitiation of
consent. Z however, can recover his losses from the winner because the law provides that no
action can be maintained by the winner for the collection of what he has won in any game of
chance. But any loser in a game of chance may recover his loss from the winner, with legal
interests from the time he paid the amount lost. (Article 2014)

XVIII.

A lawyer was given an authority by means of a Special Power of Attorney by his client to sell a
parcel of land for the amount of P3 Million. Since the client owed the lawyer Pl Million in
attorney's fees in a prior case he handled, the client agreed that if the property is sold, the
lawyer was entitled to get 5% agent's fee plus Pl Million as payment for his unpaid attorney's
fees. The client, however, subsequently found a buyer of his own who was willing to buy the
property for a higher amount. Can the client unilaterallythe rescind authority he gave in favor
of his lawyer? Why or why not? (4%)
SUGGESTED ANSWER:

No, the agency in the case presented is one which is coupled with an interest. As a rule,
agency is revocable at will except if it was established for the common benefit of the agent
and the principal. In this case, the interest of the lawyer is not merely limited to his
commission for the sale of the property but extends to his right to collect his unpaid
professional fees. Hence, it is not revocable at will. (Article 1927)

XIX.

31
Mr. A, a businessman, put several real estate properties under the name of his eldest son X
because at that time, X was the only one of legal age among his four children. He told his son
he was to hold those assets for his siblings until they become adults themselves. X then got
married. After 5 years, Mr. A asked X to transfer the titles over three properties to his three
siblings, leaving two properties for himself. To A’s surprise, X said that he can no longer be
made to transfer the properties to his siblings because more than 5 years have passed since
the titles were registered in his name. Do you agree? Explain. ( 4%)
SUGGESTED ANSWER:

No, the transfer of the properties in the name of X was without cause or consideration and it
was made for the purpose of holding these properties in trust for the siblings of X. If the
transfer was by virtue of a sale, the same is void for lack of cause or consideration. Hence, the
action to declare the sale void is imprescriptible. (Article Heirs of Ureta vs. Ureta September
14, 2011- G.R. No. 165748 September 14, 2011
ALTERNATIVE ANSWER:

No, I do not agree. A trust was created in favor of the siblings of X when their father A
transferred the titles in his name. The facts are clear that X was to hold these assets for his
siblings until they reach the age of majority. An action to recover property based on an implied
trust prescribes in ten years from the time the title was issued in favor of the trustee. In the
case presented, only five years had lapsed from the issuance of the title hence, the action has
not yet prescribed.

XX.

Mr. and Mrs. Roman and Mr. and Mrs. Cruz filed an application for registration of a parcel of
land which after due proceedings was granted by the RTC acting registration as land court.
However, before the decree of registration could be issued, the spouses Roman and the
spouses Cruz sold the lot to Juan. In the notarized deed of sale, the sellers expressly
undertook to submit the deed of sale to the land registration court so that the title to the
property would be directly issued in Juan's name. Is such a stipulation valid? (2%)
Distinguish a direct attack from a collateral attack on a title. (2%)

If the title in Item XX.A is issued in the names of the original sellers, would a motion filed by Juan
in the same case to correct or amend the title in order to reflect his name as owner
considered be collateral attack? (2%)

SUGGESTED ANSWER:

Yes, because when one who is not the owner of the property sells or alienates it and later the
seller or grantor acquires title, such title passes by operation of law to the buyer or grantee.
(Article 1434, Civil Code)
A direct attack on a title is one where the action filed is precisely for the purpose of pointing out
the defects in the title with a prayer that it be declared void. A collateral attack is one where
the action is not instituted for the purpose of attacking the title but the nullity of the title is
raised as a defense in a different action.

32
No, because Juan is not attacking the title but merely invoking his right as transferee. Hence, it
does not involve a collateral attack on the title.

33
I.

State whether the following marital unions are valid, void, or voidable, and give the
corresponding justifications for your answer:

a. Ador and Becky’s marriage wherein Ador was afflicted with AIDS prior to the
marriage. (2%)
SUGGESTED ANSWER: Voidable. Under the Family Code, a marriage is voidable if either of
the party was afflicted with a sexually transmissible disease which is serious and incurable,
such as AIDS. Here, Ador was afflicted with AIDS at the time of the celebration of the
marriage, a sexually transmissible disease considered to be serious and incurable. [Basis:
Article 45(6), Family Code; discussed in p. 122, Vol. 1, Rabuya’s Civ Reviewer Book]
b. Carlos’ marriage to Dina which took place after Dina had poisoned her previous
husband Edu in order to free herself from any impediment in order to live with
Carlos. (2%)
SUGGESTED ANSWER: Void. Under the Family Code, a marriage is declared void by reason
of public policy when one, with the intention to marry the other, killed that other spouse or
his or her own spouse. Here, the wife killed her previous husband for the purpose of
marrying the second husband. [Basis: Article 38 (9), Family Code; discussed in p. 94, Vol. 1,
Rabuya’s Civ Reviewer Book]
c. Eli and Fely’s marriage solemnized seven years after the disappearance of Chona,
Eli’s previous spouse, after the plane she had boarded crashed in the West Philippine
Sea. (2%)
SUGGESTED ANSWER: If the marriage took place during the effectivity of the Family Code
and Chona is in fact alive, the subsequent marriage is void for being bigamous because Eli
failed to obtain a judicial declaration of presumptive death of the absentee spouse prior to
contracting the subsequent marriage. Under the Family Code, a judicial declaration of
presumptive death of the absentee is required to be obtained by the spouse present to
make the subsequent marriage valid. However, had Chona really died when the plane
crashed, the subsequent marriage of Eli is valid because the prior marriage was already
terminated. [Basis: Article 41, Family Code; Armas v. Calisterio, 330 SCRA 201 (2000);
discussed in pp. 99-100, Vol. 1, Rabuya’s Civ Reviewer Book]
But if the subsequent marriage took place during the effectivity of the Civil Code, the

marriage is valid until annulled (voidable) because no judicial declaration of presumptive

death was required under the Civil Code.


d. David who married Lina immediately the day after obtaining a judicial decree
annulling his prior marriage to Elisa. (2%)
SUGGESTED ANSWER: Void. Under the Family Code, David is required to record the
judgment of annullment and the partition and distribution of the properties of the spouses,
as well as the delivery of the presumptive legitimes of their children, in the appropriate civil
registry and registries of property prior to contracting the second marriage; otherwise, the
subsequent marriage is void. [Basis: Article 35(6), in relation to Artcicles 53 and 52, Family
Code; discussed in p. 83, Vol. 1, Rabuya’s Civ Reviewer Book]
e. Marriage of Zoren and Carmina who did not secure a marriage license prior to
their wedding, but lived together as husband and wife for 10 years without any legal
impediment to marry. (2%)
SUGGESTED ANSWER: Valid because their marriage is exceptional and exempt from the
requirement of a marriage license. Under the Family Code, the marriage of a man and
woman who lived exclusively as husband and wife for at least five years and without
impediment is exempt from the requirement of a marriage license. [Basis: Article 34, Family
Code; discussed in pp. 57-58, Vol. 1, Rabuya’s Civ Reviewer Book]
II.

In 1960, Rigor and Mike occupied two separate but adjacent tracts of land in
Mindoro. Rigor’s tract was classified as timber land while Mike’s was classified as
agricultural land. Each of them fenced and cultivated his own tract continuously for
30 years. In 1991, the Government declared the land occupied by Mike as alienable
and disposable, and the one cultivated by Rigor as no longer intended for public use
or public service.

Rigor and Mike now come to you today for legal advice in asserting their right of
ownership of their respective lands based on their long possession and occupation
since 1960.

a. What are the legal consequences of the 1991 declarations of the Government
respecting the lands? Explain your answer. (2%)
SUGGESTED ANSWER:

As to the land occupied Mike, the same remains property of the public dominion. According

to jurisprudence, the classification of the property as alienable and disposable land of the

public domain does not change its status as property of the public dominion. There must be

an express declaration by the State that the public dominion property is no longer intended
for public service or the development of the national wealth or that the property has been
converted into patrimonial. Without such express declaration, the property, even if
classified as alienable or disposable, remains property of the public dominion. [Basis: Heirs
of Mario Malabanan v. Republic, 587 SCRA 172 (2009); Heirs of Mario Malabanan v.
Republic, 704 SCRA 561 (2013); discussed in my FB wall on October 15 and 16, 2017 and
pp. 338-344, Vol. 1, Rabuya’s Civil Law Reviewer]
As to the land occupied by Rigor, the declaration that it is no longer intended for public use
or public service converted the same into patrimonial property provided that such express
declaration was in the form of a law duly enacted by Congress or in a Presidential
Proclamation in cases where the President was duly authorized by law. According to
jurisprudence, when public land is no longer intended for public use, public service or for
the development of the national wealth it is thereby effectively removed from the ambit of
public dominion and converted into patrimonnial provided that the declaration of such
conversion must be made in the form of a law duly enacted by Congress or by a Presidential
proclamation in cases where the President is duly authorized by law to that effect. [Basis:
Heirs of Mario Malabanan v. Republic, 587 SCRA 172 (2009); Heirs of Mario Malabanan v.
Republic, 704 SCRA 561 (2013); discussed in my FB wall on October 15 and 16, 2017 and
pp. 338-344, Vol. 1, Rabuya’s Civil Law Reviewer]
b. Given that, according to Section 48(b) of Commonwealth Act No. 141, in relation to
Section 14(1) of Presidential Decree No. 1529, the open, continuous, exclusive, and
notorious possession and occupation of alienable and disposable lands of the public
domain as basis for judicial confirmation of imperfect title must be from June 12,
1945, or earlier, may Mike nevertheless validly base his assertion of the right of
ownership on prescription under the Civil Code? Explain your answer. (4%)
SUGGESTED ANSWER:

No, because the land remains property of public dominion and, therefore, not susceptible to
acquisition by prescription.
According to jurisprudence, the classification of the subject property as alienable and
disposable land of the public domain does not change its status as property of the public
dominion. In order to convert the property into patrimonial, there must be an express
declaration by the State that the public dominion property is no longer intended for public
service or the development of the national wealth or that the property has been converted
into patrimonial. Without such express declaration, the property, even if classified as
alienable or disposable, remains property of the public dominion, and thus incapable of
acquisition by prescription. [Basis: Heirs of Mario Malabanan v. Republic, 587 SCRA 172
(2009); Heirs of Mario Malabanan v. Republic, 704 SCRA 561 (2013); discussed in my FB
wall on October 15 and 16, 2017 and pp. 338-344, Vol. 1, Rabuya’s Civil Law Reviewer]
Here, the declaration of the property into alienable and disposable land of the public
domain in 1991 did not convert the property into patrimonial in the absence of an express
declaration of such conversion into patrimonial in the form of a law duly enacted by
Congress or by a Presidential proclamation in cases where the President is duly authorized
by law to that effect.
c. Does Rigor have legal basis for his application for judicial confirmation of
imperfect title based on prescription as defined by the Civil Code given that, like
Mike, his open, continuous, exclusive, and notorious possession and occupation was
not since June 12, 1945, or earlier, and his tract of land was timber land until the
declaration in 1991? Explain your answer. (4%)
SUGGESTED ANSWER:

None, because Rigor’s possession was short of the period required by the Civil Code for
purposes of acquisitive prescription which requires ten (10) years of continuous
possession, if possession was in good faith and with a just title, or thirty years, in any event.
While the property may be considered converted into patrimomial because of the 1991
declaration that it is no longer intended for public use or public service (provided that the
declaration be in the form of a law of a law duly enacted by Congress or by a Presidential
proclamation in cases where the President is duly authorized by law to that effect), Rigor
failed to complete the 30-year period required by law in case of extra-ordinary
prescription. Since the property was converted into patrimonial only in 1991, the period of
presciption commenced to run beginning that year only. Rigor’s possession prior to the
conversion of the property into patrimonial cannot be counted for the purpose of
completing the prescriptive period because prescription did not operate against the State at
that time, the property then being public dominion property.
Rigor may not likewise acquire ownership by virtue of the shorter 10-year ordinary
prescription because his possession was not in good faith and without a just title.
[Basis: Heirs of Mario Malabanan v. Republic, 587 SCRA 172 (2009); Heirs of Mario

Malabanan v. Republic, 704 SCRA 561 (2013); discussed in my FB wall on October 15 and

16, 2017 and pp. 338-344, Vol. 1, Rabuya’s Civil Law Reviewer]

III.

Josef owns a piece of land in Pampanga. The National Housing Authority (NHA)
sought to expropriate the property for its socialized housing project. The trial court
fixed the just compensation for the property at P50 million. The NHA immediately
deposited the same at the authorized depository bank and filed a motion for the
issuance of a writ of possession with the trial court. Unfortunately, there was delay in
the resolution of the motion. Meanwhile, the amount deposited earned interest.

When Josef sought the release of the amount deposited, NHA argued that Josef should
only be entitled to P50 million.
Who owns the interest earned? (3%)
SUGGESTED ANSWER:

The interest earned belongs to Josef because bank interest partakes of the nature of civil
fruits under Article 442 of the Civil Code and shall belong to the owner of the principal
thing.
When the National Housing Authority deposited the P50 Million as payment for the just
compensation with an authorized depositary bank for the purpose of obtaining a writ of
possession, it is deemed to be a constructive delivery of the said amount to Josef. Since Josef
is entitled to the P50 Million and undisputably the owner of the said principal amount, the
interest yield, as accession, in a bank deposit should likewise pertain to the owner of the
money deposited. Being an attribute of ownership (jus fruendi), Josef’s right over the fruits,
that is the bank interests, must be respected. [Basis: Republic v. Holy Trinity Realty
Development Corp., G.R. No. 172410, April 14, 2008]

IV.

a. Distinguish antichresis from usufruct. (3%)


SUGGESTED ANSWER:

They are distinguished as follows:

Antichresis is always a contract while usufruct need not arise from a contract because it
may also be constituted by law or by other acts inter vivos, such as donation, or in a last will
and testament, or by prescription.
The subject matter of antichresis is always a real property while the subject matter of
usufruct may either be real property or personal property.
Antichresis is an accessory contract or contract of security while usufruct is a real right.
While in both, the fruits do not pertain to the owner, the usufructuary is entitled to enjoy

the fruits while the antichretic creditor has the obligation to apply the fruits to the payment

of the interest, if owing, and therefatre to the principal of the credit.

b. Distinguish commodatum from mutuum. (3%)


They are distinguished, as follows:

As to subject matter: The subject matter of commodatum is ordinarily non-consumable


while the subject matter of mutuum is either money or consumable;
As to compensation: Commodatum is essentially gratuitous while mutuum may be
gratuitous or with a stipulation to pay interest;
As to right in subject matter: In commodatum, there is no transmission fo ownership of the
thing loaned while in mutuum, the borrower acquires ownership of the thing borrowed.
As to duty of borrower: In commodatum, the same thing borrowed is required to be

returned while in mutuum, the borrower discharges himself, not by returning the identical

thing loaned, but by paying its equivalent in kind, quality and quantity. [Discussed in pp.

725-726, Vol. 1, Rabuya’s Civil Law Reviewer]

V.

Jacob has owned a farm land in Ramos, Tarlac. In 2012, Liz surreptitiously entered
and cultivated the property. In 2014, Jacob discovered Liz’s presence in and
cultivation of the property. Due to his being busy attending to his business in Cebu, he
tolerated Liz’s cultivation of the property. Subsequently, in December 2016, Jacob
wanted to regain possession of the property; hence, he sent a letter to Liz demanding
that she vacate the property. Liz did not vacate despite the demand.
Jacob comes to enlist your legal assistance to bring an action against Liz to recover
the possession of the property.

What remedies are available to Jacob to recover possession of his property under the
circumstances? Explain your answer. (4%)
SUGGESTED ANSWER:

The remedy available to Jacob is accion publiciana, or an action for the recovery of the

better right of possession. It also refers to an ejectment suit filed after the expiration of one
year from accrual of the cause of action or from the unalwful withholding of possession of

the realty.

Since the entry made by Liz is through stealth, Jacob could have filed an action for forcible
entry. Ordinarily, the one-year period within which to bring an action for forcible entry is
generally counted from the date of actual entry on the land, except that when the entry is
through stealth, the one -year period is counted from the time the plaintiff learned thereof.
Here, since more than one year had elapsed since Jacob learned of the entry made by Liz
through stealth, the action that may be filed by Jacob is no longer forcible entry, but an
accion publiciana. [Basis: Canlas v. Tubil, 601 SCRA 147 (2009); Valdez v. CA, 489 SCRA 369
(2006); discussed in pp. 353-354, Vol. 1, Rabuya’s Civil Law Reviewer]

VI.

Tyler owns a lot that is enclosed by the lots of Riley to the North and East, of Dylan to
the South, and of Reece to the West. The current route to the public highway is a
kilometer’s walk through the northern lot of Riley, but the route is a rough road that
gets muddy during the rainy season, and is inconvenient because it is only 2.5 meters
wide. Tyler’s nearest access to the public highway would be through the southern lot
of Dylan.

May Dylan be legally required to afford to Tyler a right of way through his property?
Explain your answer. (4%)
SUGGESTED ANSWER:

No, Dylan is not entitled to a grant of compulsory right of way because he has an adequate
outlet going to the public highway.
One of the requisites for a compulsory grant of right of way is that the estate of the claimant
of a right of way must be isolated and without adequate outlet to a public highway. The true
standard for the grant of compulsory right of way is “adequacy” of outlet going to a public
highway and not the convenience of the dominant estate.
In the case at bar, there is already an existing adquate outlet from the dominant estate to a
public highway. Even if said outlet be incovenient, the need to open up another servitude is
entirely unjustified. [Basis: Article 649, Civil Code; Dichoso, Jr. v. Marcos, 647 SCRA 495
(2011); Costabella Corp. v. CA, 193 SCRA 333 (1991); discussed in pp. 559-561, Vol. 1,
Rabuya’s Civil Law Reviewer]
VII.

Alice agreed to sell a parcel of land with an area of 500 square meters registered in
her name and covered by TCT No. 12345 in favor of Bernadette for the amount of
P900,000. Their agreement dated October 15, 2015, reads as follows:

I, Bernadette, agree to buy the lot owned by Alice covered by TCT No. 12345 for the
amount of P900,000 subject to the following schedule of payment:

Upon signing of agreement – P100,000

November 15, 2015 – P200,000

December 15, 2015 – P200,000

January 15, 2016 – P200,000

February 15, 2016 – P200,000

Title to the property shall be transferred upon full payment of P900,000 on or before
February 15, 2016.

After making the initial payment of P100,000 on October 15, 2015, and the second
installment of P200,000 on November 15, 2015, Bernadette defaulted despite
repeated demands from Alice.

In December 2016, Bernadette offered to pay her balance but Alice refused and told
her that the land was no longer for sale. Due to the refusal, Bernadette caused the
annotation of her adverse claim upon TCT No. 12345 on December 19, 2016. Later
on, Bernadette discovered that Alice had sold the property to Chona on February 5,
2016, and that TCT No. 12345 had been cancelled and another one issued (TCT No.
67891) in favor of Chona as the new owner.

Bernadette sued Alice and Chona for specific performance, annulment of sale and
cancellation of TCT No. 67891. Bernadette insisted that she had entered into a
contract of sale with Alice; and that because Alice had engaged in double sale, TCT
No. 67891 should be cancelled and another title be issued in Bernadette’s favor.

a. Did Alice and Bernadette enter into a contract of sale of the lot covered by TCT No.
12345? Explain your answer. (4%)
SUGGESTED ANSWER:
No, because in the agreement between Alice and Bernadette the ownership is reserved in

the vendor and is not to pass to the vendee until full payment of the purchase price, which

makes the contract one of contract to sell and not a contract of sale.

Distinctions between a contract to sell and a contract of sale are well-established in


jurisprudence. In a contract of sale, the title to the property passes to the vendee upon the
delivery of the thing sold; in a contract to sell, ownership is, by agreement, reserved in the
vendor and is not to pass to the vendee until full payment of the purchase price. Otherwise
stated, in a contract of sale, the vendor loses ownership over the property and cannot
recover it until and unless the contract is resolved or rescinded; whereas, in a contract to
sell, title is retained by the vendor until full payment of the price. In the latter contract,
payment of the price is a positive suspensive condition, failure of which is not a breach but
an event that prevents the obligation of the vendor to convey title from becoming effective.
[Saberon v. Ventanilla, Jr., 722 SCRA 287 (2014); Spouses Torrecampo v. Alindogan, 545
Phil. 686 (2007); discussed in pp. 363-366, Vol. 2, Rabuya’s Civil Law Reviewer]
In the case at bar, the contract entered between the parties is a contract to sell because
ownership is retained by the vendor and is not to pass to the vendee until full payment of
the purchase price.
b. Did Alice engage in double sale of the property? Explain your answer. (4%)
SUGGESTED ANSWER:

NO, because there was no previous sale of the same property prior to its sale to Chona.

Despite the earlier transaction of Alice with Bernadette, the former is not guilty of double
sale because the previous transaction with Bernadette is charactrerized as a contract to
sell. In a contract to sell, there being no previous sale of the property, a third person buying
such property despite the fulfillment of the suspensive condition such as the full payment
of the purchase price, for instance, cannot be deemed a buyer in bad faith and the
prospective buyer cannot seek the relief of reconveyance of the property. There is no double
sale in such case. Title to the property will transfer to the buyer after registration because
there is no defect in the owner-sellers title per se, but the latter, of course, may be sued for
damages by the intending buyer. [Basis: Coronel v. CA, 263 SCRA 15 (1996); discussed in pp.
363-366, Vol. 2, Rabuya’s Civil Law Reviewer]
VIII.

Pedro had worked for 15 years in Saudi Arabia when he finally decided to engage in
farming in his home province where his 10-hectare farmland valued at P2,000,000
was located. He had already P3,000,000 savings from his long stint in Saudi Arabia.

Eagerly awaiting Pedro’s arrival at the NAIA were his aging parents Modesto and
Jacinta, his common-law spouse Veneranda, their three children, and Alex, his child
by Carol, his departed legal wife. Sadly for all of them, Pedro suffered a stroke
because of his over-excitement just as the plane was about to land, and died without
seeing any of them.

The farmland and the savings were all the properties he left.

State who are Pedro’s legal heirs, and the shares of each legal heir to the estate?
Explain your answer. (4%)
SUGGESTED ANSWER:

Pedro’s legal heirs are his legitimate child, Alex, and his three illegitimate chidlren with
Veneranda. Pedro’s chidlren with Veneranda are illegitimate because they were conceived
and born outside of a valid marriage. Alex, on the other hand, is a legitimate child because
she was conceived or born inside a valid marriage.
Pedro’s surviving parents are not legal heirs because they are excluded by Alex. In intestate
succession, the legitimate ascendants do not become legal heirs if there is a surviving
legitimate descendant, such as Alex in the problem. Veneranda is not a legal heir of Pedro
because she and Pedro were not married.
Ordinarily, the share of an illegitimate child in intestate succession is one-half of the share
of the legitimate child. Considering, however, that the three illegitimate chidlren will impair
the legitime of Alex if the foregoing formula is followed, Alex is entitled instead to get his
legitime, which is ½ of the estate, or P2.5 Million, while the remaining P2.5 Million is to be
divided equally among the three illegitimate children of Pedro. Their legitimes in this case
will likewise be their shares in intestate succession. [Discussed in pp. 944, Vol. 1, Rabuya’s
Civil Law Reviewer]
Assuming that Pedro’s will is discovered soon after his funeral. In the will, he
disposed of half of his estate in favor of Veneranda, and the other half in favor of his
children and his parents in equal shares. Assuming also that the will is admitted to
probate by the proper court. Are the testamentary dispositions valid and effective
under the law on succession? Explain your answer. (4%)
SUGGESTED ANSWER:
No, because the testamentary dispositions impair the legitimes of Pedro’s compulsory

heirs.

Following the provisions of the Civil Code, only Alex and Pedro’s three illegitimate children
are Pedro’s compulsory heirs. Since Alex is Pedro’s legitimate descendant and a primary
compulsory heir, she excludes Pedro’s parents as compulsory heirs, the latter being merely
secondary compulsory heirs. However, the three illegitimate chidlren are considered
concurring compulsory heirs who are also entitled to a share of the legitime.
Under the law, the legitime of Alex, being a legitimate descendant, is ½ of Pedro’s estate, or
P2.5 Million. The legitime of each of the illegitimate children is supposed to be ½ of the
share of Alex, or P1.25 Million each. Considering, however, that the remaining portion of the
estate is no longer sufficient to cover the supposed legitimes of the three illegitimate
children, they will simply share equally in the remaining P2.5 Million. Consequently, there is
no disposable free portion that Pedro may validly give to Veneranda or to his parents.
Hence, the will is intrinsically invalid. [Discussed in pp. 859, Vol. 1, Rabuya’s Civil Law
Reviewer]

IX.

Danny and Elsa were married in 2002. In 2012, Elsa left the conjugal home and her
two minor children with Danny to live with her paramour. In 2015. Danny sold
without EIsa’s consent a parcel of land registered in his name that he had purchased
prior to the marriage. Danny used the proceeds of the sale to pay for his children’s
tuition fees.

Is the sale valid, void or voidable? Explain your answer. (3%)


SUGGESTED ANSWER:

The sale is void because the subject property is a community property which was sold
without the consent of one of the spouses.
Since the marriage of Danny and Elsa was celebrated during the effectivity of the Family
Code without a marriage settlement, their property regime is absolute community of
property, which is the property regime that applies by default under the Family Code in the
absence of a marriage settlement. Under the regime of absolute community, properties
acquired by the future spouses prior to the celebration of the marriage shall become
community property after the marriage. Hence the subject property is a community
property.
Under the regime of absolute community, the disposition or encumbrance of community
property must have the written consent of the other spouse or the authority of the court
without which the disposition or encumbrance is void Here, the sale of the absolute
community property by the husband without the consent of the wife or the authority of the
court renders the sale void, whatever may be the reason for such sale. The husband should
have obtained court authorization in selling the community property for the purpose of
using the proceeds thereof to pay his children’s tuition fees. [[Basis: Articles 75, 91 and 96,
Family Code; discussed in pp. 145, 147 and 153, Vol. 1, Rabuya’s Civil Law Reviewer].

X.

Briefly explain whether the following contracts are valid, rescissible, unenforceable,
or void:

A contract of sale between Lana and Andy wherein 16-year old Lana agreed to sell
her grand piano for 25,000.00. (2%)
SUGGESTED ANSWER: Voidable. Under the Civil Code, a contract where one of the parties is
incapable of giving consent to a contract is voidable. A minor, like Andy in this case, is
incapable of giving consent to a contract. Hence, the contract is voidable. [Basis: Articles
1390(1) and 1327, Civil Code; discussed in p. 278, Vol. 2, Rabuya’s Civil Law Reviewer]
A contract of lease of the Philippine Sea entered by and between Mitoy and Elsa. (2%)
SUGGESTED ANSWER: Void. Under the Civil Code, a contract whose cause, object or
purpose is contrary to law, morals, good customs, public order or public policy is void. The
Philippine Sea is either a property of public dominion (if within Philippine territory) or a
common thing (if outside of Philippine territory) and, therefore, outside the commerce of
men. Hence, it cannot be made the object of a contract. [Basis: Articles 1409(1) and 1347,
Civil Code; discussed in pp. 217 -218, Vol. 2, Rabuya’s Civil Law Reviewer]
(c) A barter of toys executed by 12-year old Clarence and 10-year old Czar (2%)
SUGGESTED ANSWER: Unenforceable. Under the Civil Code, a contract where both parties
are incapable of giving consent to a contract is unenforceable. Here, both parties to the
contract are minors and, therefore, incapable of giving consent to a contract. [Basis: Articles
1403(3) and 1327, Civil Code; discussed in p. 278, Vol. 2, Rabuya’s Civil Law Reviewer]
(d)A sale entered by Barri and Garri, both minors, which their parents later ratified.
(2%)
SUGGESTED ANSWER: Valid. Under the Civil Code, while both parties to the contract are

minors and, therefore, incapable of giving consent, the ratification made by the parents of

both the contracting parties shall nonetheless validate the contract from the inception.

[Basis: Article 1407, Civil Code; discussed in p. 297, Vol. 2, Rabuya’s Civil Law Reviewer]

Jenny’s sale of her car to Celestine in order to evade attachment by Jenny’s creditors.
(2%)
SUGGESTED ANSWER: Rescissible. Under the Civil Code, a contract undertaken in fraud of

creditors is rescissible when the latter cannot in any other manner collect the claims due

them. [Basis: Article 1381 (3), Civil Code; discussed in p. 256, Vol. 2, Rabuya’s Civil Law

Reviewer]

XI.

Zeny and Nolan were best friends for a long time already. Zeny borrowed 310,000.00
from Nolan, evidenced by a promissory note whereby Zeny promised to pay the loan
“once his means permit.” Two months later, they had a quarrel that broke their long-
standing friendship.

Nolan seeks your advice on how to collect from Zeny despite the tenor of the
promissory note. what will your advice be? Explain your answer. (3%)
SUGGESTED ANSWER:

I will advice Nolan to file first an action to fix the term or period because the fulfillment of
the obligation itself cannot be demanded unti after the court has fixed the period for
compliance therewith, and such period has arrived. Any action to compel performance
brought before that would be premature.
Under the Civil Code, when the debtor binds himself when his means permit to do so, the
obligation shall be deemed to be one with a period, but which period shall be fixed by the
court. In such a situation, the court is authorized to fix the period because the duration of
the period depends exclusively upon the will of the debtor. Any action filed prior to the
expiration of the period to be fixed by the court would be premature. [Basis: Articles 1180
and 1197, Civil Code; Concepcion v. People, 74 Phil. 63; Gonzales v. Jose, 66 Phil. 369;
dicussed in pp. 70-72, Vol. 2, Rabuya’s Civil Law Reviewer]
XII.

Krystal owns a parcel of land covered by TCT No. 12345 in Angeles City, Due to severe
financial constraints, Krystal was lorc based in the property to RBP Corporation, a
foreign corporation based in South Korea. Subsequently, RBP Corporation sold the
property to Gloria, one of its most valued clients.

Wanting her property back, Krystal, learning of the transfer of the property from
RBP Corporation to Gloria, sued both of them in the Regional Trial Court (RTC) for
annulment of sale and for reconveyance. She alleged that the sale by RBP Corporation
to Gloria was void because RBP Corporation was a foreign corporation prohibited by
the Constitution from acquiring and owning lands in the Philippines.

Will KrystaI’s suit for annulment of sale and reconveyance prosper? Explain your
answer. (4%)
SUGGESTED ANSWER:

No, because the flaw in the original transaction is considered cured by the subsequent
transfer of the property to a Filipino citizen who is constitutionally qualified to own land in
the Philippines.
While the Constitutuion prohibits an alien from acquiring or holding title to private lands or
to lands of the public domain in the Philippines, except only by way of hereditary
succession, jurisprudence is consistent that if land is invalidly transferred to an alien who
subsequently becomes a citizen or transfers it to a citizen, the flaw in the original
transaction is considered cured and the title of the transferee is rendered valid.
In the case at bar, the subsequent transfer of the property to Gloria, a Filipino citizen, has
the effect of curing the defect of the original transaction in favor of RBP Corporation
because the land has since become the property of a Filipino citizen who is constitutionally
qualified to own land. As such, the prior invalid transfer can no longer be assailed because
the objective of the constitutional provision -- to keep our land in Filipino hands -- has been
served. [Basis: United Church Board of World Ministries vs. Sebastian, 159 SCRA 446, 451-
452, March 30, 1988; per Cruz, J. See also Tejido vs. Zamacoma, 138 SCRA 78, August 7,
1985; Sarsosa vda. de Barsobiavs. Cuenco, 113 SCRA 547, April 16, 1982; Godinez vs. Fong
Pak Luen, 120 SCRA 223, January 27, 1983; Yap vs. Maravillas, 121 SCRA 244, March 28,
1983; De Castro vs. Tan, 129 SCRA 85, April 30, 1984]
XIII.

TRUE or FALSE – Explain your answers.

(a) All rights are considered as property. (2%)


SUGGESTED ANSWER: False, because rights which are not patrimonial in nature, such as
the right to liberty, the right to honor, family rights, etc., cannot be considered as property.
[Basis: II Tolentino, Civil Code of the Philippines, 1992 ed., pp. 4-5]
A lessee cannot bring a case for quieting of title respecting the property that he
leases. (2%)
SUGGESTED ANSWER: False, because the action may be filed by anyone who has legal or
equitable title to, or interest in, the property which is the subject matter of the action.
Hence, any holder of interest to the property or right to possession of the land, including
the interest of a lessee, may bring an action for quieting of title. [Basis: Article 477, Civil
Code]
Only the city or municipal mayor can file a civil action to abate a public nuisance.
(2%)
SUGGESTED ANSWER: False, because under the law it is the district health officer and not
the chief executive of the local government who has been authorized to file a civil action to
abate a public nuisance. [Basis: Article 700, in relation to Article 699, Civil Code; Cruz v.
Pandacan Hiker’s Club, Inc., 778 SCRA 385 (2016), discussed in p. 601, Vol. 1, Rabuya’s Civil
Law Reviewer].
Possession of a movable property is lost when the location of the said movable is
unknown to the owner. (2%)
SUGGESTED ANSWER: False, because possession of movables is not deemed lost so long as
they remain under the control of the possessor, even though for the time being he may not
know their whereabouts. [Basis: Article 556, Civil Code; discussed in pp. 485-486, Vol. 1,
Rabuya’s Civil Law Reviewer]
Continuous non-apparent easements can be acquired either through title or by
prescription. (2%)
SUGGESTED ANSWER: False, because only continuous and apparent easements can be

acquired either by virtue of a title or by prescription. [Basis: Article 620, Civil Code;

discussed in p. 533, Vol. 1, Rabuya’s Civil Law Reviewer]


XIV.

Plutarco owned land that borders on a river. After several years the action of the
water of the river caused the deposit of soil, and increased the area of Plutarco’s
property by 200 square meters.

a. If Plutarco wants to own the increase in area, what will be his legal basis for doing
so? Explain your answer. (2%)
SUGGESTED ANSWER:

Plutarco acquires ownership over the increased area by virtue of accession. According to
the Civil Code, the accretion gradually receive from the effects of the current of the waters
shall belong to the owner of the lands adjoining the banks of rivers.
In order for the above rule to apply, however, the following requisites must be present: (1)
that the deposit of soil be gradual and imperceptible; (2) that it be made through the effects
of the current of the waters; and (3) that the land where accretion takes place is adjacent to
the banks of the rivers. All foregoing requirements are present in this case. Hence, Plutarco
aquires ownership over the increased area by operation of law. [Basis: Article 457, Civil
Code; Republic v. CA, 132 SCRA 514 (1984); discussed in pp. 402-405, Vol. 1, Rabuya’s Civil
Law Reviewer]
b. On the other hand, if the river dries up, may Plutarco validly claim a right of
ownership of the dried-up river bed? Explain your answer. (2%)
SUGGESTED ANSWER:

No, because the dried-up river bed shall continue to belong to the State as its property of
public dominion. As such, it is not susceptible to private appropriation and acquisitive
prescription. Therefore, Plutarco may not validly claim a right of ownership of the dried-up
river bed. [Republic v. Santos III, 685 SCRA 51 (2012); Celestial v. Cachopero, 431 SCRA 469
(2003); 657 SCRA 499 (2011); discussed in p. 409, Vol. 1, Rabuya’s Civil Law Reviewer]

XV.

Kevin signed a loan agreement with ABC Bank. To secure payment, Kevin requested
his girlfriend Rosella to execute a document entitled “Continuing Guaranty
Agreement” whereby she expressly agreed to be solidarily liable for the obligation of
Kevin.
Can ABC Bank proceed directly against Rosella upon Kevin’s default even without
proceeding against Kevin first? Explain your answer. (3%)
SUGGESTED ANSWER:

Yes, ABC Bank may proceed directly against Rosella upon Kevin’s default even without
proceeding against Kevin first because Rosella is a surety after she bound herself solidarily
with the principal debtor.
Notwithstanding the use of the word “guaranty” circumstances may be shown which
convert the contract into one of suretyship. Under the Civil Code, when the guarantor binds
himself solidarily with the principal debtor, the contract becomes one of suretyship and not
of guaranty proper. In a contract of suretyship, the liability of the surety is direct, primary
and absolute. He is directly and equally bound with the principal debtor. Such being the
case, a creditor can go directly against the surety although the principal debtor is solvent
and is able to pay or no prior demand is made on the principal debtor. [Basis: Article 2047,
Civil Code; Ong v. PCIB, 448 SCRA 705; discussed in pp. 810-812, Vol. 2, Rabuya’s Civil Law
Reviewer]
In this case, since Rosella is a surety, ABC Bank can go directly against her even without

proceeding against the principal debtor because the surety insures the debt, regardless of

whether or not the principal debtor is financially capable to fulfil his obligation.

XVI.

Jovencio operated a school bus to ferry his two sons and five of their schoolmates
from their houses to their school, and back. The parents of the five schoolmates paid
for the service. One morning, Porfirio, the driver, took a short cut on the way to
school because he was running late, and drove across an unmanned railway crossing.
At the time, Porfirio was wearing earphones because he loved to hear loud music
while driving. As he crossed the railway tracks, a speeding PNR train loudly blared its
horn to warn Porfirio, but the latter did not hear the horn because of the loud music.
The train inevitably rammed into the school bus. The strong impact of the collision
between the school bus and the train resulted in the instant death of one of the
classmates of Jovencio’s younger son.

The parents of the fatality sued Jovencio for damages based on culpa contractual
alleging that Jovencio was a common carrier; Porfirio for being negligent; and the
PNR for damages based on culpa aquiliana.
Jovencio denied being a common carrier. He insisted that he had exercised the
diligence of a good father of a family in supervising Porfirio, claiming that the latter
had had no history of negligence or recklessness before the fatal accident.

Did his operation of the school bus service for a limited clientele render Jovencio a
common carrier? Explain your answer. (3%)
SUGGESTED ANSWER:

Yes, because a common carrier is one who is engaged in the business of carrying or
transporting passengers or goods or both, or one who holds himself or itself out to the
public as being engaged in said business.
In Perena v. Zarate [679 SCRA 208 (2012)], the Court definitively ruled that the operators of
a school bus service are common carriers even if they are catering to a limited clientele
because of the following reasons: (1) they are engaged in transporting passengers generally
as a business, not just as a casual occupation; (2) they are undertaking to carry passengers
over established roads by the method by which the business was conducted; and (3) they
are transporting students for a fee.
The Court additionally explained that despite catering to a limited clienteè le, they operate as
common carriers because they held themselves out as a ready transportation
indiscriminately to the students of a particular school living within or near where they
operated the service and for a fee. [Discussed and posted on my FB wall as early as October
23, 2017]
In accordance with your answer to the preceding question, state the degree of
diligence to be observed by Jovencio, and the consequences thereof. Explain your
answer. (3%)
SUGGESTED ANSWER:

Being a common carrier, Jovencio is required to observe extraordinary diligence, and is


presumed to be at fault or to have acted negligently in case of the loss of the effects of
passengers, or the death or injuries to passengers.
In this case, Jovencio is liable for the death of the student because, acting as a common
carrier, he is already presumed to be negligent at the time of the accident because death
had occurred to the passenger. Here, Jovencio failed to fend off liability because he failed to
prove that he observed extraordinary diligence in ensuring the safety of the passengers.
[Basis: Perena v. Zarate, 679 SCRA 208 (2012); discussed and posted on my FB wall as early
as October 23, 2017]
Assuming that the fatality was a minor of only 15 years of age who had no earning
capacity at the time of his death because he was still a student in high school, and the
trial court is minded to award indemnity, what may possibly be the legal and factual
justifications for the award of loss of earning capacity? Explain your answer. (4%)
SUGGESTED ANSWER:

The basis for the computation of the deceased’s earning capacity should be the minimum
wage in effect at the time of his death, pursuant to the ruling of the Court in Perena v. Zarate
[679 SCRA 208 (2012)]. In the same case, the Court also ruled that the computation of the
victim’s life expectancy rate should not be reckoned from his age of 15 years at the time of
his death, but on 21 years, his age when he would have graduated from college.
In the same case, the Court justified the indemnification of the victim’s loss of earning

capacity despite him having been unemployed because compensation of this nature is

awarded not for loss of time or earnings but for loss of the deceased’s power or ability to

earn money.

You might also like